Текст
                    УДК 519.216.2
ББК 22.171
Б 90
Булинский А. В., Ширяев А. Н. Теория случайных процессов. — М.:
ФИЗМАТЛИТ, 2005. - 408 с. - ISBN 5-9221-0335-0.
Книга создана на основе лекций, прочитанных авторами в разные годы на механико-
математическом ф-те МГУ им. М. В. Ломоносова. Материал значительно превышает рамки
учебного курса, чтобы дать более глубокое представление о разнообразных разделах теории и
ее применениях. Сложные доказательства вынесены в «Приложения». «Дополнения и упраж-
упражнения» помогают в усвоении материала.
Для профессорско-преподавательского состава, научных работников, аспирантов и студен-
студентов старших курсов университетов.
Ил. 19. Библиогр. 198 назв.
© ФИЗМАТЛИТ, 2003, 2005
ISBN 5-9221-0335-0	© А. В. Булинский, А. Н. Ширяев, 2003, 2005


Оглавление Предисловие 6 Основные обозначения 8 Глава I. Случайные процессы. Распределения случайных процессов Ц Предмет теории случайных процессов, некоторые задачи. Случайные элементы и их распределения. Теорема о монотонных классах. Пополнение вероятностного пространства. Предел измеримых отображений. Построение семейства независимых случайных элементов с заданными распределениями. Процессы частных сумм, эмпи- эмпирические меры, процессы восстановления, модель страхования Крамера-Лундберга, пуассоновская случайная мера. Цилиндрическая сг-алгебра ?&тт Случайная функ- функция как семейство случайных элементов и как одно измеримое отображение. Конечномерные распределения случайной функции. Теорема Колмогорова о со- согласованных мерах. Характеристическая функция меры на (Rn, ?$(Rn)). Условия согласованности мер на евклидовых пространствах в терминах характеристических функций. Описание З&т для бесконечного Т. Процессы с непрерывными траек- траекториями. Согласованность проекций меры. Эквивалентные случайные функции. Измеримые процессы. Глава II. Процессы с независимыми приращениями. Пуассоновские и гауссовские процессы 46 Критерий существования процесса с независимыми приращениями. Пуассоновский процесс. Винеровский процесс (броуновское движение). Многомерное нормальное распределение. Построение действительной гауссовской случайной функции по функ- функции среднего и ковариационной функции. Комплекснозначные гауссовские процес- процессы. Неотрицательно определенные функции как ковариационные функции и как вос- воспроизводящие ядра гильбертовых пространств. Теорема Парзена. Эквивалентность двух определений броуновского движения. Функции Хаара и Шаудера. Флуктуа- Флуктуации последовательности стандартных гауссовских величин. Построение непрерыв- непрерывного винеровского процесса. Многомерное броуновское движение.
Оглавление Глава III. Броуновское движение. Свойства траекторий 76 Недифференцируемость п. н. траекторий броуновского движения (винеровского про- процесса). Марковское свойство винеровского процесса. Фильтрация. Марковские моменты, их примеры, сг-алгебра &т, состоящая из событий, наблюдаемых до мар- марковского момента т. Строго марковское свойство винеровского процесса. Принцип отражения. Закон нуля или единицы. Распределения, связанные с максимумом винеровского процесса на [0,?]. Закон повторного логарифма. Локальный закон повторного логарифма. Глава IV. Мартингалы. Дискретное и непрерывное время 108 Мартингалы, субмартингалы, супермартингалы. Примеры. Разложение Дуба. Компенсаторы. Дискретный вариант формулы Танака. Расширение фильтрации. Квадратическая характеристика. Квадратическая вариация. Теорема Дуба о свободном выборе. Применение к случайным блужданиям (задача о разорении). Максимальное и минимальное неравенство Дуба для субмартингалов. Лемма о числе пересечений. Теорема о сходимости субмартингалов. Ветвящийся процесс Гальтона-Ватсона. Сходимость мартингалов в L (П,^, Р). Теорема Леви. Фундаментальная теорема страховой математики. Некоторые неравенства для субмартингалов и мартингалов с непрерывным временем. Глава V. Слабая сходимость мер. Принцип инвариантности 147 Слабая сходимость мер в метрических пространствах. Сходимость случайных эле- элементов по распределению. Критерии слабой сходимости. Сохранение слабой схо- сходимости под действием непрерывных отображений. Слабая сходимость мер в про- пространстве С(Т, S). Относительная слабая компактность и плотность семейства мер. Теорема Прохорова. Принцип инвариантности Донскера-Прохорова. Многомерная центральная предельная теорема Линдеберга, лемма о максимуме сумм независи- независимых случайных величин. Схема доказательства критерия согласия Колмогорова. Броуновский мост как условный винеровский процесс. Метод одного вероятност- вероятностного пространства, теорема Скорохода. Метризация слабой сходимости. Метрика Леви-Прохорова. Глава VI. Марковские процессы. Дискретное и непрерывное время 180 Эквивалентные определения марковского процесса. Марковость процессов с незави- независимыми приращениями со значениями в R . Примеры. Цепи Маркова, их построение по переходным вероятностям и начальному распределению. Пуассоновский процесс как марковская цепь. Переходная функция марковского процесса. Нахождение пере- переходной функции d-мерного броуновского движения. Конечномерные распределения марковского процесса, их выражение через начальное распределение и переходную
Оглавление функцию. Однородные марковские процессы. Эргодическая теорема для однород- однородных цепей Маркова. Следствия. Инвариантная мера. Инфинитезимальная матри- матрица Q стохастической полугруппы (P(?))t>o- Обратная и прямая системы дифферен- дифференциальных уравнений Колмогорова. Стационарное распределение как собственный вектор матрицы Q . Формулы Эрланга. Модель системы массового обслуживания, приводящая к этим формулам. Глава VII. Стационарные процессы. Дискретное и непрерывное время 225 Ортогональные случайные меры и их сг-конечные структурные меры. Построение ортогональной случайной меры, отвечающей данной структурной мере. Интеграл по ортогональной случайной мере, его свойства. Теорема Карунена о факторизации ко- ковариационной функции и представлении процесса в виде интеграла по ортогональной случайной мере. Стационарные в широком смысле процессы и их ковариацион- ковариационные функции. Теорема Герглотца. Теорема Бохнера-Хинчина. Спектральное представление стационарных процессов с непрерывным и дискретным временем. Эргодичность в L (Q). Процессы скользящего среднего. Статистическое оценивание ковариационной функции и спектральной плотности. Задача линейного прогноза. Регулярные и сингулярные процессы. Разложение Вольда. Регулярные процессы как физически осуществимые фильтры. Критерий Колмогорова регулярности процесса. Теорема Колмогорова-Сегё. Глава VIII. Интеграл Ито. Стохастические дифференциальные уравнения 276 Стохастический интеграл для простых случайных функций по винеровскому процес- процессу. Конструкция Ито стохастического интеграла для неупреждающих случайных функций. Свойства стохастического интеграла. Формула замены переменных Ито. Уравнение Ланжевена. Процесс Орнштейна-Уленбека. Теорема существования и единственности сильного решения стохастического дифференциального уравнения. Марковость решения стохастического дифференциального уравнения. Приложение 1. Доказательство теоремы Колмогорова 317 Приложение 2. Доказательство теоремы Прохорова 323 Приложение 3. Доказательства теорем Линдеберга и Дуба 327 Приложение 4. Доказательство теоремы Бохнера—Хинчина 337 Приложение 5. Доказательство теоремы Колмогорова—Сегё 340 Приложение 6. Доказательство строго марковского свойства семейства броуновских движений 344 Приложение 7. Вероятностное решение задачи Дирихле 354 Приложение 8. Большие уклонения 364 Заключительные замечания 383 Список литературы 385 Указатель 393
Предисловие Настоящая книга возникла на основе лекций, прочитанных авторами в разные го- годы на механико-математическом факультете Московского государственного универ- университета им. М.В. Ломоносова. Теория случайных процессов изучается в б-м семестре на базе общих курсов тео- теории вероятностей D-й семестр) и математической статистики E-й семестр). Чита- Читатели заметят, что предлагаемый текст значительно выходит за рамки семестрового курса. Цель проведенного нами расширения — дать более глубокое представление о разнообразных ветвях теории и ее применениях. Стандартная программа перекры- перекрывается основным содержанием восьми глав. Технически сложные доказательства не- некоторых важных результатов отнесены в "Приложения" Кроме того, каждая глава завершается разделом "Дополнения и упражнения" Этот материал может быть ис- использован для проведения семинарских занятий и подготовки специальных курсов. Фундаментом всей теории случайных процессов явилась теорема Колмогорова о существовании процесса с заданной системой конечномерных распределений. В его классической монографии [34] она названа "основной теоремой" До ее появления ис- исследование случайных процессов как семейств случайных величин велось, главным образом, с точки зрения свойств их конечномерных распределений (например, пря- прямые и обратные уравнения для марковских систем). "Основная теорема" дала воз- возможность потраекторного анализа случайных процессов, заложив тем самым основы того направления, которое именуется ныне стохастическим анализом. К особенностям книги относится, в частности, то, что теорема Колмогорова о согласованных распределениях доказывается в максимальной общности и об- обсуждается с различных точек зрения. Даны детальные доказательства принципа инвариантности Донскера—Прохорова, функционального закона повторного лога- логарифма в форме Штрассена (с привлечением общей теории больших уклонений семейства мер), теоремы Скорохода о вложении случайного блуждания в винеровс- кий процесс, различных форм строго марковского свойства броуновского движения и других глубоких результатов. К интересным применениям излагаемой теории относятся основная теорема страховой математики, устанавливаемая с помощью мартингальной техники, вывод известных в теории массового обслуживания формул Эрланга с учетом общих свойств марковских процессов, исследование уравнения Ланжевена, приводящего к процессу Орнштейна-Уленбека, основанное на теории стохастических дифференциальных уравнений, и т. д. Отметим, что рассматривают- рассматриваются и проблемы, связанные с результатами и методами других областей математики, например, вероятностный подход к решению классической задачи Дирихле, теория прогнозирования, использующая аппарат гильбертовых пространств, доказатель-
Предисловие 7 ство формулы Колмогорова-Сегё, опирающееся на методы теории функций. Дается также начальное представление о проблематике стохастической финансовой мате- математики. Теория случайных процессов — обширный бурно развивающийся раздел совре- современной теории вероятностей, имеющий многочисленные приложения. Поэтому даже сжатый список литературы содержит почти 200 наименований. При ссылках на ма- материал курса теории вероятностей, как правило, используется учебное пособие [85]. Остановимся подробнее на структуре книги. В главе I даются определения основных понятий, используемых далее, а также устанавливается ряд вспомогательных результатов. В главе II изучается класс процессов с независимыми приращениями, важнейшим представителем которого является броуновское движение. Глава III полностью посвящена свойствам этого замечательного процесса. В IV, VI и VII главах излагается теория мартингалов, марковских процессов и стационарных процессов соответственно. Глава V содержит результаты, относящиеся к слабой сходимости вероятностных мер, в том числе мер в функциональных пространствах траекторий случайных процессов. В главе VIII рассматриваются вопросы стохастического интегрирования (главным образом, по броуновскому движению) и стохастические дифференциальные уравнения. Все главы разбиты на отдельные параграфы. Нумерация определений, утвержде- утверждений, примеров и формул в каждой главе начинается заново, причем отдельно зану- занумерованы определения, отдельно — теоремы и т. п. Сказанное относится и к "При- "Приложениям" При ссылках на материал другой главы (приложения) указывается ее (его) номер, например, A1.10) — это формула A0) главы II, а F.2) - формула B) приложения 6. Конец доказательства обозначается значком П. В разделах "Допол- "Дополнения и упражнения" каждой из глав и приложений упражнения отмечены только номерами, а их текст сдвинут вправо по отношению к основному тексту. Следует отметить, что курсы вероятностно-статистического цикла на механико- математическом факультете МГУ формировались под непосредственным влиянием А. Н. Колмогорова и Б. В. Гнеденко, долгие годы заведовавших кафедрой теории ве- вероятностей. При работе над рукописью нам были также полезны обсуждения ее со- содержания с сотрудниками и аспирантами кафедры. Выражаем им свою искреннюю признательность. Авторы благодарны Российскому фонду фундаментальных исследований за фи- финансовую поддержку издания книги. Грант № 02-01-14077. А. В. Булинский, Кафедра теории вероятностей А.Н. Ширяев механико-математического факультета Московского государственного университета им. М.В. Ломоносова
Основные обозначения := "положить по определению" N — множество натуральных чисел, N = N U {оо}, Q — множество рациональных чисел, Z — множество целых чисел, Z_|_ — множество целых неотрицательных чисел, R — множество действительных чисел, R = R U {оо}, С — множество комплексных чисел, если а, Ъ Е R, то а Л Ъ = min{a, 6}, а V Ъ = тах{а, 6}, а~*~ = тах{а, 0}, а~ = тах{—а, 0}, [а] — целая часть числа a, sgn a — знак числа, А — дополнение к множеству А, 1д — индикатор множества А, [А] — замыкание множества А (в метрическом пространстве), дА — граница множества А (в метрическом пространстве), (S, р) — метрическое пространство S с метрикой р, Вг (х) — замкнутый шар радиуса г с центром в точке ж, С(Т, S) — пространство непрерывных функций на метрическом пространстве Т со значениями в метрическом пространстве S, Cq° (Rm) — пространство бесконечно дифференцируемых функций с компактным носителем, || • || ц —норма в банаховом (гильбертовом) пространстве Н, (П, ^, Р) —вероятностное пространство, Е — символ математического ожидания, D — символ дисперсии, cov(X, Y) — ковариация случайных величин X и У, Е (X | &4) — условное математическое ожидание X относительно сг-алгебры sd, X Е & | ?& означает, что отображение X измеримо относительно сг-алгебр §* и @&, а {Ж} — наименьшая а- алгебра, порожденная системой Ж подмножеств некоторого простран- пространства S (с единицей S), — сг-алгебра борелевских подмножеств топологического пространства S, — цилиндрическая сг-алгебра в произведении измеримых пространств (Sj, ?$t), t E T, Рх (РХ~ или Law(X)) — распределение случайного элемента X, cr{Xt, t Е Т} — наименьшая сг-алгебра, порожденная случайными элементами Xt, t Е Т (в(П,^,Р)), Qn =^ Q — слабая сходимость мер Qn к мере Q, Хп —ьХ — сходимость Хп по распределению к X, Р Хп —>- X — сходимость Хп по вероятности к X, mes — мера Лебега, N(a, С) — нормальное распределение со средним а и ковариационной матрицей С, W = {W(t), t ^ 0} —винеровский процесс (броуновское движение), F = (&t)t?T — фильтрация, Wx = {&*)t?rT — естественная фильтрация процесса X = {Xt, t Е Т}, P(s,x,t,B) — переходная функция марковского процесса, Pij (t) — переходные вероятности однородной марковской цепи.
Нашим родителям
Глава I Случайные процессы. Распределения случайных процессов Предмет теории случайных процессов, некоторые задачи. Случайные элементы и их распределения. Теорема о монотонных классах. Пополнение вероятностного пространства. Предел измеримых отображений. Построение семейства независимых случайных элементов с заданными распределениями. Процессы частных сумм, эмпи- эмпирические меры, процессы восстановления, модель страхования Крамера-Лундберга, пуассоновская случайная мера. Цилиндрическая сг-алгебра ?&т. Случайная функция как семейство случайных элементов и как одно измеримое отображение. Конечно- Конечномерные распределения случайной функции. Теорема Колмогорова о согласованных мерах. Характеристическая функция меры на (Rn, @& (Rn)). Условия согласованности мер на евклидовых пространствах в терминах характеристических функций. Описание ?&Т для бесконечного Т. Процессы с непрерывными траекториями. Согласованность проекций меры. Эквивалентные случайные функции. Измеримые процессы. § 1. Важнейшей особенностью современной теории вероятностей является то, что ее методы и результаты представляют не только самостоятельный математический интерес, но и находят разнообразные приложения в других научных дисциплинах, таких как физика, химия, биология, финансовая математика и др., а также в технике. В чем же специфика того раздела теории вероятностей, который называется "случайные процессы"! Вначале теория вероятностей имела дело со случайными экспериментами (под- (подбрасывание монеты, игральной кости и т.п.), для которых подсчитывались вероят- вероятности, с которыми может произойти то или иное событие. Затем возникло поня- понятие случайной величины, позволившее количественно описывать результаты про- проводимых экспериментов, например, размер выигрыша в лотерее. Наконец, в случай- случайные эксперименты был явно введен фактор времени, что дало возможность стро- строить стохастические модели, в основу которых легло понятие случайного процесса, описывающего динамику развития изучаемого случайного явления. В 1827 году ботаник Р. Броун (R. Brown) обнаружил под микроскопом хаотичес- хаотическое движение частиц цветочной пыльцы в воде. Природа этого движения, названного броуновским, долго оставалась невыясненной. Только в конце 19 — начале 20 века было осознано, что оно представляет собой одно из проявлений теплового движения атомов и молекул вещества. Оказалось, что для описания процессов подобного рода требуются вероятностно-статистические подходы. Математические и физические модели броуновского движения и более общих процессов диффузии были построе-
12 А. В. Булинский, А. Н. Ширяев ны Л. Башелье, А. Эйнштейном, М. Смолуховским, С. Орнштейном, М. Планком, П. Ланжевеном, А. Фоккером, Н. Винером, П. Леви, А.Н. Колмогоровым, М. А. Ле- онтовичем и другими учеными. В 1997 году исследования Р. Мертона и М. Шоулса, в которых броуновское движение применялось в экономических моделях, были от- отмечены Нобелевской премией. Прежде, чем переходить к систематическому изложению курса случайных про- процессов, остановимся на некоторых конкретных, но принципиально важных вопросах и задачах, большинство из которых будет рассмотрено в этой книге. 1°. Мы покажем, как изучение функционалов от броуновского движения позволя- позволяет, например, доказать знаменитый критерий согласия Колмогорова, являющийся одним из фундаментальных результатов математической статистики. Будет также объяснено, что с помощью броуновского движения могут быть решены очень важные "неслучайные" задачи, например, задача Дирихле: найти гармоническую в области G С Ж функцию, которая принимает заданные значения на границе этой области (далее мы уточним формулировку). 2°. Упомянутые выше модели диффузии стимулировали развитие теории сто- стохастических дифференциальных уравнений, требующей аппарата особого "стохас- "стохастического исчисления" Особенность связана, в частности, с тем, что траектории броуновского движения являются непрерывными, но, тем не менее, не дифференци- дифференцируемыми ни в одной точке. Где могут пригодиться подобные уравнения? Например, с их помощью можно дать эффективный способ, применяемый на практике (восста- (восстановление изображения, теле- и радиовещание, распознавание образов и др.), выде- выделения полезного сигнала, скрытого в помехах (шуме). 3°. Для многих практических целей очевидна важность задач прогноза: на осно- основании наблюдений случайного процесса до момента времени t требуется предсказать его поведение в моменты t+s (s > 0). Такие задачи возникают, например, при анали- анализе эволюции курсов ценных бумаг на финансовом рынке, в связи с которым большой интерес представляет задача хеджирования в опционах (моделях купли-продажи акций в некоторый момент в будущем по заранее установленной цене, когда истинная цена самих акций подвержена случайным колебаниям). 4°. Представим себе деятельность страховой компании, которая, располагая опре- определенным капиталом (пополняемым поступающими взносами), осуществляет в слу- чайые моменты времени выплаты, размер которых также носит случайный харак- характер. Каким образом описывать эти случайные моменты? Оказалось, что в подоб- подобных ситуациях возникает пуассоновский процесс, подробно изучаемый далее. Будет показано, как должны быть связаны между собой параметры в модели страхования Крамера—Лундберга для того, чтобы вероятность разорения компании не превосхо- превосходила заданного (малого) числа е. 5°. Наконец, упомянем разнообразные задачи асимптотического анализа слу- случайных объектов. В качестве примера укажем на формулы Эрланга, позво- позволяющие представить, что будет происходить "при больших временах" в модели, описывающей функционирование, скажем, телефонной станции, когда вызовы посту- поступают в случайные моменты времени и длительности разговоров абонентов также случайны. Подчеркнем, что в пунктах 3°-5° мы сталкиваемся с проблемами двух типов. Во-первых, требуется построить содержательную математическую модель
Гл. I. Случайные процессы. Распределения случайных процессов 13 изучаемого явления, а во-вторых, точно поставить задачу и затем решить ее. Для всего этого требуется, разумеется, весьма существенное развитие теории. При знакомстве с основами этой теории мы увидим, сколь многочисленны привлекаемые к рассмотрению классы случайных процессов и сколь разнообразны методы их описания и исследования. Первая глава носит во многом технический, вспомогательный характер. К ней желательно возвращаться для разъяснений при чтении последующих глав. При пер- первом чтении можно ограничиться усвоением понятий случайного элемента (§2), его распределения (§6), определениями случайной функции (§ 7) и примерами из § 9, ос- основанными на последовательности независимых случайных элементов, которые рас- рассматриваются в §8. Требуется также осознать, что имеется возможность строить на основе теоремы Колмогорова (§12) семейства зависимых случайных величин, от- отправляясь от должным образом согласованных распределений их всевозможных ко- конечных наборов. Для построения процессов с независимыми приращениями и дей- действительных гауссовских процессов в главе 2 используются упомянутые выше усло- условия согласованности в терминах характеристических функций случайных векторов (§15)- § 2. Все вероятностные рассмотрения будут далее предполагать заданным, со- согласно аксиоматике Колмогорова, некоторое вероятностное пространство (П, 3, Р), т.е. измеримое пространство (П, 3) с введенной на нем вероятностной мерой Р. Напомним, что измеримое пространство — это пара, состоящая из некоторого мно- множества и сг-алгебры его подмножеств (обычно обозначаемой рукописной буквой). Мерой называется неотрицательная счетно-аддитивная функция множеств, опреде- определенная на некоторой сг-алгебре измеримого пространства. В тех случаях, когда надо подчеркнуть особые свойства меры, например, ее конечность или сг-конечность, это всегда будет особо оговариваться. Вероятностная мера Р, или просто вероят- вероятность, — это такая мера, что Р(П) = 1. Для краткости вероятностную меру часто называют просто мерой. Прежде всего, нам потребуется понятие случайной величины со значениями в абстрактном множестве S. Пусть (П, 3) и (S, ё%) — измеримые пространства (D^0hS/0). Определение 1. Отображение X: П —> S называется 3* \ S&-измеримым (пишем X е &\ё%),естлХ-1(ё%) С 3, т.е. Х~1(В) := {и: Х{ш) Е В] Е 3 для любого В е@. В теории вероятностей 3* | ^-измеримые отображения принято называть случай- случайными элементами или случайными величинами со значениями в S. Для семейства Ж подмножеств S обозначим а{Ж} наименьшую сг-алгебру с еди- единицей S, содержащую Ж. Если S — топологическое, в частности, метрическое про- пространство, то, по определению, борелевская а-алгебра 3e(S) есть сг-алгебра а {Ж}, где Ж — совокупность открытых множеств в S. В случае S = Ж (с евклидовой метрикой), Й§ = ?$(Жк) и к > 1 случайный элемент X C \ ^-измеримый) называет- называется случайным вектором, а если к = 1, то (действительной) случайной величиной. Когда X — действительная случайная величина на (П, 3), то вместо 3 \ Й§(М)-из- Й§(М)-измеримости X, как правило, говорят просто об ^-измеримости.
14 А. В. Булинский, А. Н. Ширяев Лемма 1. Пусть X: П —у S (отображение X не предполагается измеримым относительно каких-либо а-алгебр) и Ж — некоторая совокупность подмно- подмножеств S. Тогда X G d \ а{Ж}, где si := а{Х~1(Ж)}. При этом а{Х-\Ж)}=Х-\а{Ж}). Доказательство. Легко видеть, что семейство множеств 3) = {D С SiX^iD) G d} есть сг-алгебра, так как si является сг-алгеброй и взятие прообраза сохраняет теоре- теоретико-множественные операции. По построению Ж С @. Следовательно, а{Ж} С @ и, значит, X G ^\а{Ж},т.е.Х~1(а{Ж}) С d. Далее, Х^) С Х~1(а{Ж}), причем Х~1(а{Ж}) есть сг-алгебра. Поэтому «с/ = а{Х~1(Ж)} С Х~1(а{Ж}). Итак, d = Х-1(а{Ж}).В Следствие 1. Пусть в дополнение к условиям доказанной леммы дана а-ал- а-алгебра & в п. Если Х~1(Ж) С &, то X G & \ а{Ж}. Доказательство состоит в применении леммы 1 с учетом соотношения \^. П Смысл следствия заключается в том, что в случае Зё = а {Ж} для доказательст- доказательства 3* | Й§-измеримости X необязательно рассматривать прообразы всех множеств из Й§, а достаточно убедиться, что Х~1(В) G 3 лишь для множеств В из Ж. Именно поэтому в определении случайного вектора X со значениями в Жк достаточно толь- только (объясните, почему это так) требовать, чтобы для любого z = (zi,..., Zk) G Шк множество {и;: Х{и) ^ z} := {и;: Хг{ш) ^ zu ..., Хк{и) ^ zk} G 3. Замечание 1. Полезно иметь в виду, что а-алгебра может порождаться не только множествами, но и функциями. Запись si — a{Xt, t G Т},гдеХ^: Q —> St, (St, Sfot) — измеримые пространства, t G T, означает, что si — наименьшая сг-ал- сг-алгебра подмножеств ft такая, что Xt G si\S&t Для каждого t G Т. Легко видеть, что d = crjx-1^): Bt e@ute T} и если Xt € 3\8&t Для t G Г, то, очевидно, d С 3. §3. Наряду с сг-алгебрами нам придется иметь дело с более узкими классами множеств. Назовем тг-системой совокупность *$ подмножеств S, замкнутую отно- относительно конечных пересечений, т.е. если А, В G ?>, то А П В G *€ (к ^ всегда до- добавляем само множество S). Определим Х-систему @ как класс подмножеств S со следующими свойствами: S G @ и если А, В G ®, А С В, то В \ A G ®, кроме / °° \ того, если in G ®,nGN,Hin |i (т.е. Ап С Ап+Ь п G N, и А = (J Ап ), v n=i у то i G 9. Для любого класса JT подмножеств S существуют (объясните) наимень- наименьшая тг-система тг{Ж} и наименьшая Л-система Л{ JT} подмножеств S, содержащие J#\ Система Ж подмножеств S является сг-алгеброй в том и только том случае, когда Ж есть одновременно тг- и Л-система.
Гл. I. Случайные процессы. Распределения случайных процессов 15 Теорема 1 (о монотонных классах). Пусть тг-система Чо и Х-система @ под- подмножеств пространства S таковы, что Ч? С @. Тогда cj^} = А{^} с @. Доказательство. Можно сразу считать, что @ = А{^}. Достаточно прове- проверить, что @ является тг-системой, так как в этом случае @ будет сг-алгеброй, содер- содержащей S^3. Итак, покажем, что АП В Е ®, если А, В Е @. Фиксируем произвольное В Е ^ и введем класс множеств j?\b = {-4 С S: АП В Е @}. Тогда &в является А-системой и Ч> С 3*в- Поэтому @ = А{^} с ^в- Следовательно, А П В ? *3 для любых i G ® и Б G ^. Теперь возьмем произвольное A Е @ и рассмотрим класс множеств ^ = {5 С S: АП В Е @}. Аналогично убеждаемся, что @ С ^а для каждого i G i Таким образом, система @ замкнута относительно конечных пересечений и сг{^} С А{^}. Очевидно, А{^} С о{Щ. П Теорема 1 доказана В. Серпинским. Широкое применение этого результата в тео- теории вероятностей связано с работами Е. Б. Дынкина. Поэтому А-системы называют также системами Дынкина или ©-системами. Полезность приведенной выше теоремы о монотонных классах раскрывают, на- например, следующие утверждения. Лемма 2. Пусть на измеримом пространстве @,,&) заданы меры Р и Q, причем 3< — сг^}, где Чо есть тг-система подмножеств П. Для совпадения мер Р и Q на 3< необходимо и достаточно, чтобы Р = Q на Чо. Доказательство . Необходимость утверждения очевидна. Достаточность сле- следует из того, что совокупность множеств A G ^, для которых Р(А) = Q(-A), образует А-систему. ? Лемма 3. Пусть (П, ^, Р) — вероятностное пространство и si — некото- некоторая алгебра подмножеств из §<'. Тогда для любого события С G a{si} имеет место следующее свойство inf P(C AA) =0, A) где С А А = (С\А)и(А\С). Тем самым, для каждого С G a{si} и любого г > 0 существует А? G si такое, что Р(С А А?) < г, откуда |Р(С) — P(^U)| < s. Доказательство. Любое событие С из алгебры si, являющейся тг-системой, удовлетворяет соотношению A). Легко проверить, что совокупность событий, для которых верно A), есть А-система. Требуемое утверждение следует из теоремы 1. ? Лемма 4 (свойство регулярности меры). Пусть S — метрическое простран- пространство и Q — мера на ?&{S). Тогда для каждого борелевского множества В Q(B)=supQ(F)=infQ(G), B) где верхняя грань берется по всем замкнутым множествам F С В, а нижняя — по всем открытым множествам G D В.
16 А. В. Булинский, А. Н. Ширяев Доказательство. Пусть р — метрика в S и р(х, D) = inf{р(х,у): у Е D}, где х Е S, D С S. Для 5 > 0 и замкнутого множества F обозначим F5 = {х Е S: р(х, F) < < 5}. Легко видеть, что F5 — открытые множества и F5 I F при 5 I 0. Поэто- Поэтому B) справедливо для любого замкнутого множества В. Читателю предлагается доказать самостоятельно (или см. [2, с. 16]), что борелевские множества В, для ко- которых верно B), образуют сг-алгебру. Тогда утверждение леммы вытекает из того, что 8&(Ъ) — наименьшая сг-алгебра, содержащая все замкнутые подмножества S. § 4. В дальнейшем нам часто придется использовать операцию пополнения ис- исходного вероятностного пространства (?1,3, Р). Пусть Л — класс подмножеств п таких, что С е Jf <^> 3D е 3: С С Dn P(D) = 0. Пусть Ж — совокупность множеств вида A U С, где A G 3* и С G J/. Легко видеть, что 3 есть сг-алгебра (называемая пополнением 3 по мере Р). Продолжение меры Р на сг-алгебру 3 задается формулой Р(А U С) = Р(А), где А е &, С G Я C) (тем самым множествам из J/ приписана мера нуль). Нетрудно видеть, что Р является корректно определенной мерой на 3. Корректность означает, что если F = AUC = ВиВ,где_А,В е&пС,В е ^,toP(F) = Р(А) = Р(В). Пространство (П, 3, Р) называется пополнением (по мере Р) вероятностного про- пространства (П, 3, Р). Как правило, мы будем считать, что эта операция пополнения произведена изначально и не станем писать черту над 3 и над Р (при этом JV ока- оказывается классом всех событий нулевой вероятности). Если сг-алгебра sd С 3*, то обычно к ней также добавляют класс Р-нулевых множеств J{, получая при этом рас- расширенную сг-алгебру d^ — d U Л (и продолжая меру на d^ так же, как в C)). Легко видеть, что d^ = a{d, Jf}. Заметим, что если взять сужение Р|^ меры Р с сг-алгебры 3 на сг-алгебру d С 3 и пополнить d по мере Р ^, то, вообще гово- говоря, получится сг-алгебра, содержащаяся в d^. Когда вероятностное пространство (П, 3, Р) фиксировано, то вместо d^ используется обозначение d. § 5. При различного рода аппроксимациях случайных элементов ключевую роль играют следующие утверждения. Лемма 5. Пусть (М, ^) — измеримое пространство, (S,p) — метрическое пространство с метрикой р и борелевской а-алгеброй S$(S). Пусть hn: M —у S, hn e ^|^(S) при п = 1,2,... . Пусть hn{x) -^ h{x) для каждого х е М, т. е. p(hn(x),h(x)) —У 0 при п —у оо. Тогда h: М —у S есть <? \ S$(S)-измеримое отображение. Доказательство. Для любого замкнутого множества В е 88(S) оо оо {x:h(x)eB}=f) U f| {х: hn(x) k = l m=l где В^ = {х G S: р(х,В) < г}, р(х,В) = inf{p(x,y): у G В}. Поэтому требуемое утверждение вытекает из следствия 1. ?
Гл. I. Случайные процессы. Распределения случайных процессов 17 Лемма 6. Пусть в отличие от леммы 5 на (М, ^) задана мера Р и hn(x) —У h(x) при п —у оо для Р-п.в. ж Е М {т.е. Р(ж Е М: hn(x) -^ h{x), п —У оо) = 0). Тогда h является *& \Se(S)-из- \Se(S)-измеримым отображением, где <$ — пополнение <$ по мере Р. Доказательство. Пусть hn(x) -У h(x), п -у оо, для ж е Мо, где Р(М0) = 1. Выберем точку zq Е S и определим hn (x) = hn (х) для х G Мо и /in (ж) = zq для ж G М \ Мо, n G N. Тогда hn(x) —У h(x) для всех х G М, где /г(ж) = h(x) при ж G Мо и /г(ж) = zo при ж G М \ Мо. Очевидно, hn Е *& \$(S). Из леммы 5 вытекает, что h является ^ | ^(S)-измеримым отображением. Теперь учтем, что для любого В G SS(S) прообраз Л^) = {Мо П Л^)} U {(М \ Мо) П h-\B)} G W, поскольку Мо е ^Д^) е cSn(M\M0)nh-1(B) С М\М0,Р(М\М0) = 0. ? Лемма 7. Пусть вероятностное пространство (П, ^, Р) полно и а-алгеб- ра si С 3*. Действительная случайная величина Y(uS) является si -измеримой тогда и только тогда, когда найдется si -измеримая действительная случай- случайная величина Z{uj) такая, что У — Z (Р-п.в.), т. е. Р(и; G п: Y(u) ф Z(u)) = 0. Доказательство . Пусть Y e si \ 0В(Ж). Возьмем "простые функции" {uj: УМеНмнЦпг^И) п е ^ D) к = -2п где, как обычно, 1в(') ~ индикатор множества В, в. (Вместо 1в(х) часто используется запись 1{В}.) Очевидно, Yn(uj) —у Y(uS) при п —У оо для всех и; G П, причем Yn G & п G N. По определению сг-алгебры si имеем {и: Y(u) е (кп2~п, (к + 1)п2-п}} = Ап,к U Сп,к, где Ап^к € d, a Cnjfe € JV (к = -2п,..., 2П - 1, n G N). Положим оо 2п-1 = U U А^- п=1 А; = -2 Тогда Ае^и Р(А) = 1. Определим ^ П1А^Н, иеп, пек k = -2n
18 А. В. Булинский, А. Н. Ширяев Ясно, что Zn е d | SB (Ж), Zn = 0 для и ? А (п е N) и Zn(cj) -»> Y(u) при п -»> оо для оо Е А. По лемме 5 величина Z = 1а^ ? si \ ?ё(Ж). Кроме того, Z = Y (Р-п.в.). Таким образом, искомая величина Z найдена. Пусть теперь Z Е d \ S3 (Ж) и Y = Z п.н. (т.е. Р-п.в.). Положим Zn = Z, n Е N. Тогда Zn —>- Y п. н. при п —у оо, и по лемме б величина Y G si \ Зё(Ж), что и требова- требовалось показать. П § 6. Измеримые отображения позволяют переносить меру с исходного простран- пространства на другие. Пусть (П, &) и (S, 9S) — измеримые пространства, причем на (П, &) задана неко- некоторая вероятностная мера Р. Пусть X: П —у S является 3* \ ^-измеримым отобра- отображением. Определение 2. Распределением, или распределением вероятностей, слу- случайного элемента X называется мера на Й§, обозначаемая Рх и задаваемая форму- формулой Рх(В) := Р(Х-\В)), Вей. F) Распределение X обозначается также РХ, Law(X) или Law(X | P), если важ- важно подчеркнуть, о какой конкретно мере Р идет речь. Меру Рх называют образом меры Р при (измеримом) отображении X. Рис. 1 Отметим, что изучение распределений 3 \ ^-измеримых случайных элементов, заданных на (П, ^, Р) со значениями в S, и изучение мер на (S, 9&), по сути дела, одно и то же. В самом деле, любой случайный элемент X индуцирует меру Рх • Обратное утверждение содержит элементарная Лемма 8. Любую вероятностную меру Q на (S, Й§) можно рассматривать как распределение некоторого случайного элемента X. Для доказательства достаточно взять П = S, 3 = S3, Р = Qh X(uj) = uj (тож- (тождественное отображение). ? § 7. Введем важное во всем дальнейшем понятие случайной функции и связанное с ним понятие случайного процесса. Определение 3. Семейство X = {X(t), t G Т} случайных элементов X(t), за- заданных при каждом t G Т на некотором измеримом пространстве (П, 3), принято называть случайной функцией.
Гл. I. Случайные процессы. Распределения случайных процессов 19 Отметим здесь важные детали, уточняя и терминологию, и соответствующие обо- обозначения. В данном определении предполагается, что все случайные элементы X(t),t G Т, задаются на одном и том же измеримом пространстве (П, ЗР). Однако для многих целей полезно считать, что области значений случайных элементов X(t), t G Т, вообще говоря, различны. Точнее говоря, будем считать заданным семейство изме- измеримых пространств (St,?$t)teT- Тогда на введенное выше семейство X = {X(t), t G Т} можно смотреть также как на функцию X = X(t, ио), определенную на Т х п, принимающую при каждом t G Т значение из St и являющуюся 3* | ^-измеримым отображением (X(t, -):Q —у St). В этом уточненном смысле функцию X = X(t, uS) естественно называть случайной функцией, ассоциированной с измеримыми прост- пространствами (п, 3) и (St, e$t)teT- Элементарный пример случайной функции X = {X(t), t G Т}, принимающей при каждом t G Т действительные значения, получим, положив где ? — действительная случайная величина, заданная на некотором вероятностном пространстве (П, 3, Р), а детерминированная функция /: Т —у Ж. В следующей гла- главе мы увидим, что ряд, составленный из такого рода случайных функций, может быть использован для описания броуновского движения. Определение 4. При фиксированном и; функцию Х( •, о;), или X(t,u), t G T, называют траекторией, реализацией или выборочной функцией. Аргумент uj в записи случайной функции обычно опускают и пишут просто X (t) или Xt, мы используем оба обозначения в зависимости от контекста. Если ГсМ = = (—оо, оо), то параметр t G Т интерпретируется как время и случайную функцию называют случайным процессом. Когда множество Т — прямая, полупрямая, отре- отрезок, интервал или полуинтервал, то говорят о случайном процессе с непрерывным временем, а при Г С Ъ — {...,-1,0,1,...}-о процессе с дискретным време- временем или о случайной последовательности. Если Т С Ж , d > 1, то X называют случайным полем. Часто применяется процедура дискретизации, когда случайный процесс (поле) X = {Xt, t G Т} рассматривается лишь на конечном или счетном подмножестве Т' С Т. Например, вместо X = {Xt, t ^ 0} изучается процесс с шагом А > 0, т.е. процесс Y = {Хап, п = 0,1, 2,... }. Здесь же заметим, что исследование случайных полей, определенных на множест- множестве Т С Md (d > 1), обычно не сводится к изучению случайных процессов путем пе- перепараметризации (даже если между Т и подмножеством прямой установлено вза- взаимно-однозначное соответствие). Дело в том, что при описании зависимости систем случайных величин {Xt, t G U} w {Xt, t G V} в ряде задач существенную роль играет геометрическое расположение множеств U,V С Т. Термин случайный процесс используют и для обозначения любой случайной функции (не только при Г С 1). Вообще, классификация случайных функций, основанная на структуре параметрического множества Т или на типах про- пространств (St, S$t)teT, в произведении которых лежат траектории этих функций, не является всеобъемлющей. Дело в том, что при описании тех или иных явлений
20 А. В. Булинский, А. Н. Ширяев на первый план могут выдвигаться некоторые специальные требования к свойствам используемых (изучаемых) процессов. Примером могут служить весьма обширные (и вообще говоря, пересекающиеся) классы случайных функций, такие как процессы с независимыми приращениями, гауссовские процессы, мартингалы и т. д., подробно изучаемые в последующих главах. § 8. Сейчас мы напомним, предполагая заданным некоторое вероятностное про- пространство (П, ^, Р), понятие независимости сг-алгебр событий (и случайных элемен- элементов), чтобы привести ряд примеров случайных процессов. Определение 5. Системы множеств (содержащие П, в частности, сг-алгебры) «c/i,..., sin С 3* (п ^ 2) называются независимыми (в совокупности), если P(Ai---An) = P(Ai)---P(An) G) для любых Ak G ?//с5 к = 1,..., п. Бесконечное семейство систем множеств sit С 3 (Q, G s#t), t G Т, называется независимым, если набор sitx,..., sitn независим при всех п ^ 2 и любых несовпадающих друг с другом точках t\,..., tn G Т. Лемма 9. Пусть независимы и-системы Ж\,...,Жп С 3 (п ^ 2). Тогда независимы и-алгебры сг{Ж\},... ,(т{Жп}. Если независимы а-алгебры, то не- независимы и их расширения классом Л нулевых событий. Доказательство. Пусть ®i — система таких подмножеств ^, что G) справед- справедливо для любых А\ G @i и всех А^ G М^, к = 2,..., п. Очевидно, ®i есть А-система, причем Ж\ С @i. По теореме 1 получаем, что а{Ж\} С ®ь Аналогично рас- рассматривается система ®2, состоящая из событий ^2, для которых верно G) при всех А\ G ст{Ж\} и4 G ^/с, где /с = 3,... ,п. Действуя таким образом, приходим к первому утверждению леммы. Второе утверждение очевидно, поскольку любое событие А из сг-алгебры ^, т. е. из расширения сг-алгебры ^ классом Jf, имеет вид А = CUD, где Cg^hDg^.D Независимость (в совокупности) семейства случайных величин {Xt, t G Т}, заданных на некотором вероятностном пространстве (П, ^, Р) и принимающих при каждом t G Т значения в каком-либо измеримом пространстве (St,?$t), означает независимость порожденных ими сг-алгебр cr{Xt} = X~1(S%t), t G Т. Таким образом, величины Х$, ? G Т (множество Т содержит не менее двух эле- элементов), независимы в том и только том случае, если P(xtl eBu...,xtne вп) = Д p(xtk g вк) (8) k=l для всех п ^ 2, любых несовпадающих точек ti,..., tn G Т и произвольных мно- множеств Б/, G 3StfcJ fe = 1,.. .,n. В связи с данным определением независимых случайных элементов представляет интерес следующий результат.
Гл. I. Случайные процессы. Распределения случайных процессов 21 Теорема 2 (Ломницкий-Улам, [161]). Пусть (St,e$t)teT — произвольное семейство измеримых пространств и пусть при любом t Е Т на (St,$t) за- задана мера Qt- Тогда существует вероятностное пространство (П,^, Р) и семейство независимых случайных элементов Xt: Q —> St, Xt Е 3* \ ?$t, таких, что Рxt — Qt ма S&t пРи каждом t Е Т. Иначе говоря, всегда можно построить семейство независимых случайных эле- элементов с любыми наперед заданными распределениями. Заметим, что бывают ситуа- ситуации, когда действительно удобно рассматривать не только одно и то же пространство (S, Й§) в качестве (St, $&t) ПРИ всех t Е Т (см. пример 6). Ниже в § 12 результат этой теоремы комментируется в связи с теоремой Колмого- Колмогорова (теорема 4), см. также упражнение 1. § 9. Далее мы неоднократно увидим, что многие важные процессы удается стро- строить, отправляясь от последовательности независимых случайных элементов. Приведем ряд примеров такого рода. Отметим, что для построения последователь- последовательности независимых действительных случайных величин достаточно совершенно эле- элементарных средств, как показывают упражнения 2 и 3. Пример 1. Пусть ?о ? ?i ? • • • — независимые случайные векторы, заданные на не- некотором вероятностном пространстве (П, 3, Р) и принимающие значения в простран- пространстве Ж171 (т ^ 1). Случайный процесс (с дискретным временем) п Sn = Y,tj, neZ+ = {0,l,...}, (9) 3=0 принято называть случайным блужданием. Если представить себе частицу, которая в нулевой момент времени находится в точке ?о и в каждый (дискретный) момент времени п G N смещается на величину ?п, то тогда вектор (п,5п) будет давать координаты этой частицы во времени и пространстве. Пример 2. Для последовательности ^i, ^2, • • • независимых одинаково распреде- распределенных невырожденных неотрицательных случайных величин процесс восстанов- восстановления определяется формулой = supjn: ^ ^(и) ^ Л, Х0(и) = 0, Xt(u) = supjn: ^ ^(и) ^ Л, t> 0. A0) Всюду считаем сумму по пустому множеству равной нулю, поэтому Xt(uf) = 0, если ?i(kO > t. Вообще говоря, так определенные величины Xt(uS) принимают значения в! = Ми{оо}; при этом сг-алгебра Й§(М) считается состоящей из множеств В и В U {оо}, где В е 38(R). Название "процесс восстановления" объясняет следующая интерпретация. Пусть, начиная с to = 0, в моменты, отделенные друг от друга промежутками ?j, происходят поломки некоторого устройства (например, перегорает электрическая лампочка), и вышедшее из строя устройство или его неисправный блок мгновенно заменя- заменяется идентичным новым. Тогда Xt(uS) есть число замен ("восстановлений") на промежутке @, t]. Траекторию процесса X = {Xt(uj), t G M+ = [0, оо)} дает рис. 2.
22 А. В. Булинский, А. Н. Ширяев з-- 2 1 У У Рис. 2 Пример 3. Рассмотрим на некотором вероятностном пространстве (П, ^, Р) две последовательности неотрицательных случайных величин {?j, j G N} и {?7j, j G N} таких, что {?j,?7j, J G N} независимы в совокупности и Law(^j) = Law(?i), Law(?7j) = Law G71), j G N. Для положительных констант уонс определим процесс Yt(u) = ct - где Х^(а;) задается формулой A0). Процессы вида A1) используются в модели страхования Крамера-Лундберга (см., например, [86; с. 99]), в которой у0 — начальный капитал компании, с — скорость поступления взносов, r]j — размер j выплаты в случайный момент tj = 5^ ?г (j' ? N), а 1^ интерпретируется как капитал компании в момент t. г=1 Пример 4. Пусть ^i, ^2, • • • — независимые одинаково распределенные случай- случайные векторы, заданные на вероятностном пространстве (П, ^, Р) и принимающие значения в Мт (т ^ 1). Введем эмпирические меры n G A2) Пользуясь определением 3, легко видеть, что формула A2) при каждом п G N задает случайный процесс, индексированный борелевскими множествами В, т.е. Вместо индикатора 1^ в A2) можно было бы взять функцию / из некоторого клас- класса Ж. Тогда при должной измеримости / получится случайный процесс, индекси- индексированный семейством функции. Пример 5. Пусть {(j, j G Zd} — случайное поле, образованное независимыми одинаково распределенными векторами со значениями в Ж171. Пусть \± есть сг-конечная мера > на ), d ^ 1. Определим процесс частных сумм Sn(B,co) = Y, Сз("МпВ П В G Щ0, l]d), n G A3) *^/i есть сг-конечная мера на (S, ?$), если S = (J Sn, где Sn G ^ и //(Sn) < 00 для любого п G N. ™=!
Гл. I. Случайные процессы. Распределения случайных процессов 23 где Cj = (j — 1, j] = (ji — 1, ji] x • • • x (jd — l,jd] - единичный куб с верхней вершиной в точке j G Zd, nB = {х = п?/, 2/ G 5}. Другими словами, в отличие от (9) здесь берутся определенным образом взвешен- взвешенные частные суммы. Пример 6. Пусть Л есть конечная мера (А ^ 0), заданная на некотором измери- измеримом пространстве (S, ?$). Пусть У, Xi, X2,... — независимые случайные элемен- элементы на вероятностном пространстве (П, ^, Р) такие, что Y — действительная пу- ассоновская величина с параметром A(S), a Xi, Х2,... — независимые одинаково распределенные величины, являющиеся 3* | ^-измеримыми, для которых Рхх (В) = \(В)/\(S) при В G Й§. Возможность такого построения вытекает из теоремы 2. (Об- (Обратите внимание, что здесь мы имеем дело с величинами Y и Х& со значениями в разных пространствах.) Введем на Зё х П функцию Z(B, и), положив Z{B,u)= Y, M*») A4) (Z(B,uj) = 0, когда Y(со) = 0). Формула A4) определяет так называемую пуассо- новскую случайную меру с мерой интенсивности (или ведущей мерой) Л. Слу- Случай о -конечной меры Л обсуждается в дополнении к этой главе. § 10. Рассмотрим более пристально данное ранее определение случайной функ- функции. Покажем, что случайную функцию X = {Х(?, о;), t G Т}, понимаемую как семей- семейство случайных элементов X(t, •), t G Т, удобно представлять себе и как один слу- случайный элемент на (П, 3) со значениями в соответствующем измеримом простран- пространстве функций, заданных на Т. Идея этого соответствия проста. Каждому ио G П сопоставляется целая траек- траектория Х( •,uj), т.е. вводится отображение X формулой Х(ш):=Х(-,ш). A5) Поскольку нам все время приходится иметь дело с измеримыми объектами, то сле- следует разобраться с вопросом измеримости введенного отображения A5). С этой целью рассмотрим декартово произведение St = П ^t (обозначаемое также как X St или ST, когда St = S при всех t G Т), состоящее из функций у = y(t), T заданных на Т и таких, что y(t) G St для t G Т. Каждая траектория случайного процесса X = {X(t), teT} является элементом пространства St- Выясним, для какого класса Ж множеств В С St заведомо можно утверждать, что {uj: X(cj) G В} G 3. Тогда следствие 1 обеспечит, что отображение X будет 3 | а {Ж} -измеримым. Возьмем в качестве Ж совокупность элементарных цилиндров, т.е. множеств вида
24 А. В. Булинский, А. Н. Ширяев В- Рис. 3 где Bt G Set, t G Г (здесь варьируются и ? и ??t). Образно говоря, Cr(t, Bt) состоит из тех функций у G St, которые в точке t проходят через "ворота" Bt (см. рис. 3). Заметим, что {ио: X(cj) G CT(t,Bt)} = {cj:X(t,cj) G ??t} G ^, поскольку X(t, •) e&\3Bt при любом ? G Г. Введем следующее определение. Определение 6. сг-алгебра ?$т в пространстве St, порожденная совокупностью Ж элементарных цилиндров, называется цилиндрической а-алгеброй. Для ё%т используется также обозначение 0 S&t, называемое произведением сг-алгебр Set, t е Т. Если St = S и Set = также = S8 при всех t G Г, то вместо ?$т пишут По следствию 1 отображение X, введенное в A5), является 3* \ ^т-измеримым. Весьма примечательно, что справедлив и обратный результат. Для пояснения этого определим координатные отображения тгт,г: St —> St, положив где t G Г, yeST- A6) Легко видеть, что тгт,г G ^т | ^t для любого t G Т, поскольку прообраз любого множества ??t G ^t есть элементарный цилиндр в пространстве St- В соответствии с замечанием 1 можно сказать, что а-алгебра ё%т порождается семейством коор- координатных отображений тгт,г, t G Т, т.е. где : Bt G A7) Ясно, что X(t, о;) = 7TT,tX(o;) при любых t G T и о; G П. Суперпозиция измери- измеримых отображений дает измеримое (относительно соответствующих сг-алгебр) ото- отображение. Тем самым, имеет место следующая теорема. Теорема 3. Эквивалентное определение случайной функции X = {X(t), t ?T} состоит в том, что отображение A5) является 3 \ ё%т-измеримым. Иначе говоря, семейство X = {X(t), t G Г} есть семейство 3 \ <$t-измеримых слу- случайных элементов X(t), t G Т, в том и только в том случае, когда отобра- отображение A5) является 3 \ ё$т-измеримым. Отметим, что в этой теореме несущественно, снабжено или нет измеримое про- пространство (П, 3) какой-либо вероятностной мерой Р.
Гл. I. Случайные процессы. Распределения случайных процессов 25 Очевидно, ё$т порождается также тг-системой, состоящей из всевозможных ко- конечных пересечений элементарных цилиндров. Из теоремы 3 вытекает, что каждая случайная функция X = X(t,u), рассмат- рассматриваемая как случайный элемент X, индуцирует распределение вероятностей Рх на (St, S%t) (далее пишем просто Рх)- Таким образом, всегда можно говорить о ве- вероятности Р(и: Х( •, и) G В), если множество В принадлежит сг-алгебре §11. Пусть (St, ?&t)t€T- семейство измеримых пространств. Для каждого п G N и любых не совпадающих друг с другом точек ti,..., tn G Т введем $&ti,...,tn как наименьшую сг-алгебру пространства Stb...,tn = S^ x • • • х Stn, порожденную "пря- "прямоугольниками" вида Btx х • • • х Btn, где Btk G Setk •> к = 1,..., п. Пусть X = {Х(?), t G T} — случайная функция, заданная на вероятност- вероятностном пространстве (П, 3, Р) и ассоциированная с семейством измеримых про- пространств (St,?$t)teT- При всех п G N и ti,...,tn G Т случайный вектор ? = (X(ti),... ,X(tn)) будет 3* | $ti,...,tn-измеримым отображением по следст- следствию 1, поскольку Г\вч х ... х Btn) = k=l Определение 7. Для п G N и t\,..., tn G Т меры Ptx,... ,tn ? являющиеся распре- распределениями на Зёц,...,гп случайных векторов (X(ti),..., X(tn)), называются конеч- конечномерными распределениями случайной функции X = {Х(?), t G Т}. Итак, для "прямоугольника" С = Bt± x • • • х Btn Из A8) сразу вытекает, что для каждого п ^ 2, всех t^ G TwBtk ?S$tk (fc = l,...,n), а также любой перестановки (ii,..., гп) набора A,..., n) выполняются следующие условия симметрии и согласованности: 1°. Ptl,...,tn(Btl x---xBtn) = Ptil,...,tin(Btil x ••• xBtJ, 2°. Ptl,...,tn(Btl x ¦ • ¦ x Btn_1 x Stn) = Ptb...,*^!^ x ¦ ¦ ¦ x Btn_J. Действительно, 1° справедливо, поскольку правая часть A8) не меняется от того, в каком порядке пересекаются события, а 2° верно, так как {X(tn) G Stn} = ft- Заметим, что условия 1° и 2° равносильны условиям 1° и 3°, где 3° означает, что взятие Btm = Stm для любого т = 1,... ,п в Ptb...,tn(^ti x • • • х Btn) приводит в записи этой функции к одновременному "выкидыванию" tm и Btm: 3°. Pt!,...,^,...,^^! x -.. х Stm x ... х BtJ = Замечание 2. В наборах (ti,..., tn), индексирующих меры Ptb...,tn5 имеет смысл рассматривать we совпадающие между собой точки ti,..., tn. Дело в том, что в противном случае можно было бы осуществить редукцию к более "короткому" вектору, состоящему из различных точек ?& G Т, например, Р±АВ' х в") = p(xt ? в'> xt е в") = p(xt ев'п в") = pt(B' n в").
26 А. В. Булинский, А. Н. Ширяев §12. Теорема 2 (Л омницкий-Улам) утверждала, что по любому семейству мер Qt, t G Г, определенных на измеримых пространствах (St,?$t), t G Т, можно всегда построить (единое) вероятностное пространство (П, &<, Р) и семейство независимых (по мере Р) случайных элементов Xt = Xt(oo), t G T, таких, что Pjq = Qt, т.е. Law(Xt | P) = Qt. Важно отметить здесь следующие два обстоятельства. С одной стороны, в этой теореме нет никаких требований топологического характера относительно приро- природы пространств (St, S$t), t G Т. Предполагается только, что это — измеримые про- пространства. С другой стороны, эта теорема не отвечает на вопрос о том, можно ли построить вероятностное пространство и семейство случайных элементов Xt, t G T, на нем такие, чтобы совместные распределения случайных элементов Xtx,..., Xtn совпадали бы с a priori заданными распределениями вероятностей Qt1,...,tn • Теоре- Теорема Ломницкого-Улама утверждает лишь то, что требуемые построения можно осу- осуществить, когда меры Qtx,... ,tn являются прямым произведением "одномерных" рас- распределений Qt:,..., Qtn (п ^ 1 и ti,..., tn — несовпадающие друг с другом точки из множества Т). В этой связи замечательна теорема Колмогорова (впервые опубликованная в 1933 году в его монографии "Основные понятия теории вероятностей" на немецком языке; см. также на русском языке [34]), для формулировки которой понадобится следую- следующее определение. Определение 8. Измеримые пространства (S, ё%) и (V, d) называются изоморф- изоморфными (пишем (S, ?$) ~ (V, &/)), если существует взаимно-однозначное отображение h: S ->• V такое, что h G <$\dvih~x G d\$&. Если V — борелевское подмножество отрезка[0,1] иdестьсг-алгебраборелевскихподмножеств V (т.е. d — Vn^([0,1])), то изоморфное ему пространство (S, Й§) называется борелевским пространством. Напомним, что топологическое пространство (S, Й§) называется польским, если имеется метрика, превращающая S в полное сепарабельное метрическое простран- пространство. Польским, например, будет пространство Ж171 с евклидовой метрикой. Извест- Известно (см., например, [41; том 1]), что любое борелевское подмножество польского про- пространства с сг-алгеброй своих борелевских подмножеств является борелевским про- пространством. Теорема 4 (Колмогоров). Пусть (St,8$t)teT — семейство борелевских про- пространств. Пусть на пространствах (Stb...,tn, ^tb...,tn), яде п G N и несовпа- несовпадающие друг с другом точки ti,..., tn G Т, заданы меры Ptb...,tn5 удовлетворя- удовлетворяющие условиям симметрии и согласованности 1° и 2° (см. § 11). Тогда сущест- существуют вероятностное пространство (П, ^, Р) и определенная на нем случайная функция X = {X(t), t G Т} такие, что конечномерными распределениями X являются меры Pt1,...,tn- Доказательство этой теоремы вместе с ее эквивалентными формулировками дается в приложении 1. Первоначальный вариант теоремы Колмогорова относился к построению се- семейств действительных случайных величин Xt, t G Т (индексированных точками произвольного множества Т), по системе согласованных совместных функций распределения конечных наборов этих величин. Доказательство, данное Кол- Колмогоровым, охватывает (с должными изменениями) и более общую ситуацию,
Гл. I. Случайные процессы. Распределения случайных процессов 27 рассмотренную нами выше. В этой связи отметим, что в случае Т = N сформулиро- сформулированная теорема Колмогорова превращается в теорему Даниеля (см. [112]). Поэтому в случае произвольного множества Т и любого семейства борелевских пространств (St, S$t)teT говорят также о теореме Даниеля-Колмогорова. Замечание 3. Пусть случайная функция X = {X(t), t G Т} определена на множестве Т С I Тогда в силу условия 1° можно рассматривать конечномерные распределения только для t\ < • • • < tn. С другой стороны, пусть на простран- пространствах (Stb...,tn, ^tb...,tn) заданы меры Ptb...,tn, где h < • • • < tn, tk G Г С М, к = l,...,n,n G N. Если эти меры удовлетворяют условию 3°, то по теореме Кол- Колмогорова найдется случайная функция X = {X(t), t G Т} такая, что ее конечномер- конечномерными распределениями, индексированными векторами (ti,..., tn) с t\ < • • • < tn, будут меры Ptb...,tn. § 13. Дадим еще одну форму условий симметрии и согласованности мер Pti,... ,tn •> которую используем далее. Для п ^ 2, ti,..., tn G Т и перестановки (ii,..., гп) набора A,..., п) определим отображения грп: Тп ->• Гп и Фп: Stx х • • • х Stn ->• S^ x • • • х St-п , положив ^п(*1,...,*п) = (^i,-..,^n), ФП(Ж1,...,ЖП) = (я:^,... ,^гп). A9) Введем отображениявп:Тп ^Тп~х и 0П: Stx х • • • х Stn ->• Stx x • • • х Stn_1: On{ti,...,tn) = (tb...,tn_i), 0п(жь...,жп) = Oi,...,xn_i). B0) Большие и малые буквы для одинаковых по своей сути отображений подчеркива- подчеркивают их действие на разных множествах. Индекс п у отображений A9) и B0) будем опускать для упрощения записи, а также не будем отмечать их зависимость от точек Лемма 10. Для каждого п ^ 2 и всех т — (ti,..., tn), где ti,..., tn — не сов- совпадающие друг с другом точки множества Т, условия согласованности 1° и 2° (или 1° и 3°) мер Рт = Ptx tn эквивалентны следующим двум условиям: (А) Р^т = РТФ"! и (В)Рвт = Рхв. Доказательство . Следует рассмотреть тт-систему, состоящую из "прямоуголь- "прямоугольников" Btl х • • • х Btn в пространстве Stl,... ,tn, где ti,..., tn G Г (n G N), и восполь- воспользоваться леммой 2. ? § 14. При построении случайных процессов, принимающих действительные зна- значения, часто удобно использовать взаимно-однозначное соответствие между мера- мерами на евклидовом пространстве W1 (с борелевской сг-алгеброй SfelW1)) и их харак- характеристическими функциями. Определение 9. Характеристической функцией меры Q на (Mn, SfelW1)) на- называется функция / exp{i(A,o;>}Q(da;)) АеГ, B1) п где (Л,х) = X) ^k%k,i2 = -I-
28 А. В. Булинский, А. Н. Ширяев Хорошо известно (см., например, [85; гл. II, § 3]), что мера Q на (Mn, SfelW1)) пол- полностью определяется заданием функции распределения F(x) = Q((—оо,ж]), где (-оо,х] = (-оо,xi] х • • • х (-сю,хп], х = (xi,... ,хп) G Г. В каждой точке х, в которой эта функция распределения F = F(x) непрерывна, ее значение определя- определяется по характеристической функции формулой обращения: F(x) = B7r)-n lim / dy f d\exp{-i(\,y)-a2\\\2/2}ipQ(\), B2) где | A|2 = (A, A), dX и dy обозначают интегрирование по мере Лебега (ср. с теоремой 3 § 12 гл. II в [85]). Зная функцию F = F(x) в точках ее непрерывности, однозначно восстанавливаем F всюду. Значит, по характеристической функции однозначно восстанавливается и мера Q на SfelW1). Заметим, что если (/?q Е L1 (Ж71, SfelW1), d\), то в B2) можно положить а = 0 и не брать предел по а. Напомним также формулу замены переменных в интеграле Лебега. Пусть (S, S&), (V, si) — измеримые пространства, g: S —> V является ё$ \ ^/-измеримым отображе- отображением nh'.V —>-Mn,/iE?/| SfelW1). Тогда в предположении существования приводи- приводимых ниже интегралов [ h(g(x)) Q(dx) = f Ky^Qg-1) (dy), B3) is Jv где интеграл от вектор-функции берется покомпонентно, а мера Q определена на (S, 9S). Оба интеграла в B3) существуют или не существуют одновременно, а если существуют, то, тем самым, и равны (см. [85; гл. II, § 6]). Пусть на измеримом пространстве (S, S&) заданы сг-конечные меры \iviv. Мера \± называется абсолютно непрерывной относительно меры v (пишем /л ^ и), если равенство is (А) = 0 влечет /л(А) = 0. По теореме Радона-Никодима (см., например, [35; с. 405]) \i ^C is тогда и только тогда, когда существует функция / G I/1(S, Зё, is), называемая плотностью меры \± по мере is и обозначаемая dfi/dis такая, что = / f{x) is(dx), AeS%. B4) JA Известно, что в случае, когда /i ^ is, для функции h: S ->• W1, h G 38 \ SfeiW1), спра- справедлива формула / h(x) /i(dx) = / h(x)f(x) is(dx); B5) Js Js оба интеграла здесь существуют лишь одновременно (и в этом случае они, следова- следовательно, совпадают). Определение 10. Пусть (П, 3<, Р) — вероятностное пространство. Характе- Характеристической функцией случайного вектора Y: П —у Mn (Y G 3* \ S^iW1)) называет- называется функция tpY{\) = Е exp{i(A,F)}, A G Жп, B6) где Е — усреднение по мере Р.
Гл. I. Случайные процессы. Распределения случайных процессов 29 Из формул B1) и B3) видим, что <pY(\) = f exp{i(\,Y(u))}P(du) = Jn ехр{г(Л,г)}РУ-1(^)=^Ру(Л), B7) т. е. характеристическая функция вектора Y совпадает с характеристической функ- функцией его распределения вероятностей. § 15. Пусть семейство мер Рт, т = (ti,..., tn), на пространствах (W1, ё%(Жп)) ин- индексируется не совпадающими друг с другом точками t\,..., tn (варьируются nGN и ti,..., tn из Т). Обозначим срт = (fT{\) характеристическую функцию меры Рт. Если меры Рт — конечномерные распределения процесса X = {X(t), t G Т}, то тогда ipT может быть представлена в виде: ipT(\) = Е expji J2 Х(гк)\Л, А = (Аь ..., An) G Rn. B8) Отсюда видно, что характеристическая функция ц>т(\) не должна меняться при од- одновременной перестановке координат векторов г и А, и характеристическая функ- функция, индексированная "укороченным" вектором т, получается из характеристичес- характеристической функции ipT подстановкой в ее аргументы Ai,..., Ап нулей на места "выкидыва- "выкидываемых" в векторе т координат. Оказывается, эти простые условия в точности эквивалентны условиям симмет- симметрии и согласованности мер 1° и 2° (см. § 11). Теорема 5. Для симметрии и согласованности мер РТ, т = (ti,..., tn) G Tn, заданных на пространствах (Mn, ^(Mn))n^i, необходимо и достаточно, чтобы для любых A G Mn, r G Тп и п ^ 2, одновременно выполнялись следующие два условия: где отображения ф, Ф, в и О определены в § 13 (с St = M, t G Т), а (ОА, 0) = = (Ai,..., An_i, 0) дл^ А = (Ai,..., An) G Mn. Доказательство. В силу взаимно-однозначного соответствия между мерами на (Mn, SfelW1)) и характеристическими функциями условия (А) и (В) леммы 10 равно- равносильны тому, что при п ^ 2 ?>0ТМ =<pPr0_i(/x), /iGM71 C0) (напомним, что <рт = срр ). По лемме 8 найдется случайный вектор УТ со значениями в Мп, такой что Рт = Рут • Тогда Р^-Ф есть распределение вектора ^YTJ поскольку = Р(ФУТ G Б). Далее, = Е ехр{г(ФГт,А>} = Е ехр{г(Гт, Ф*А)} = ^(Ф^А), C1)
30 А. В. Булинский, А. Н. Ширяев где мы отождествили Ф с ортогональной матрицей, задающей это отображение (тог- (тогда Ф* = Ф, Ф* — транспонированная матрица Ф). Согласно C1) условие B9) принимает вид (^ , (А) = (^Т(Ф~1А), A Е Жп, что эквивалентно условию (а). Аналогично, Е exp{i@FT,/i)} = Е exp{i(FT, ( C2) Таким образом, C0) равносильно условию (Ь). П Замечание 4. Пусть X = {(Xi(t),... ,Xm(t)), ? Е Т} — (векторный) случай- случайный процесс со значениями в рассмотрим вектор 6b...,tn = щий на Й§ (Mmn) распределение Ptx,... ,tn c характеристической функцией (^,... ,tn (A), где А = (Ai,..., An), Xj = (A^- ,..., X™ ) G Mm, j = 1,..., п. Теорема 5 верна и для векторного случая (для мер Рт, заданных на пространствах (Mmn, ^(Mmn))n^i)- В этом (векторном) случае в условии (а) одновременно переставляются ti,..., tn и Ai,..., Ап, а в условии (Ь) вектор Ап приравнивается нулю в Ж171. 171. Для произвольного п Е N и любых t\,..., tn G T №(*i), • • • ,Xm(ti),... ,Xi(?n),... ,Xm(tn)), имею- имею§ 16. Рассмотрим подробнее структуру сг-алгебры 88т- Нам понадобятся некото- некоторые новые понятия. Для подмножества U С Т (включение С не предполагается, вообще говоря, строгим) аналогично пространству St и сг-алгебре 8$т определяют- определяются пространство Sjj и сг-алгебра 8&и. Заметим, что сг-алгебра 8&и для конечного множества U = {t\,..., tn } близка по смыслу к сг-алгебре 8&ц,... ,tn •> введенной в § 11. Отличие состоит в том, что, когда оперируют функциями (а не векторами), неважно, в каком порядке занумерованы точки множества U. Если V С U С Т, то определим отображение ("проектирование") тг^ v: Sjj —> Sy формулой = У C3) где у v есть сужение на множество V функции у, заданной на множестве U (см. рис. 4). Когда V состоит из одной точки ?, то вместо тгт rt| пишем, как в A6), тгт t. V и Рис. 4
Гл. I. Случайные процессы. Распределения случайных процессов 31 Из следствия 1 вытекает, что 7r^vSSv C@u для V С U С Г, C4) т.е. тг^у у Е S%u \?%v (прообраз элементарного цилиндра из Sy есть элементарный цилиндр в S[/). Отметим также, что тгт у = тг^ утгт и для V С U С Т. Поэтому 7г-\{Ву) = тг-^ (тг-V(BV)), C5) где By — произвольное множество из S&y. Для любого множества U С Т существует естественное вложение сг-алгебры Зёц в сг-алгебру ё%т- С этой целью определим ] U d Т. Нетрудно видеть, что эта система множеств ё$т,и является сг-алгеброй, содержа- содержащейся в S&t как прообраз сг-алгебры при измеримом отображении (см. C4)). Между множествами из сг-алгебр ё&и и ?$т,и можно установить взаимно-одно- взаимно-однозначное соответствие: @и Э Ви ^ Ви = Ви х Д Ste 0&т,и, C6) ter\u причем Вц П B'v = 0 <^^> Б[/ П Б^ = 0. Кроме того, для V С U С Т, согласно C5) и C4), ^ j^ ^ . C7) Обозначим где F(T) — совокупность конечных подмножеств Г (т.е. С G ^т ^=> 3 J G F(T): С G ^T,j). Учитывая C7), видим, что Чот есть ("цилиндрическая") алгебра, состоящая из всевозможных цилиндров С = тг^ jBj, где основание цилиндра Bj E ?$j, J E F(T) (разумеется, С можно записать также в виде С = Bj х П St). ter\j Для того, чтобы пояснить происхождение названия цилиндр в данном контексте возьмем в плоскости жз = 0 пространства М3 множество В и все его точки "умно- "умножим" на прямую Ж. Полученное таким образом множество В xR = {х = (х1,х2,х3) е М3: (жьж2) е В, х3 е Щ, является, очевидно, цилиндром в обычном геометрическом смысле. § 17. Известно, что одна и та же сг-алгебра может порождаться разными систе- системами множеств. Применительно к сг-алгебре ё%т имеет место следующая теорема.
32 А. В. Булинский, А. Н. Ширяев Теорема 6. Пусть Т — бесконечное множество и N(T) — совокупность счетных подмножеств Т. Тогда для а-алгебры 3%т справедливы следующие представления: сг{^т} и 3Вт = U ^т,и- C8) Доказательство. Поскольку 38т,и С 38т для каждого U С Г, то Чот С 38т и а^т} С З&т- С другой стороны, 7r^\^t = ^T,{t} С ^т, поэтому ?$т С ctJ^t} в силу A7). Первое соотношение C8) установлено. Покажем, что d — (J 38т,и есть сг-алгебра. Пусть Ап G «с/, т. е. Ап G 38т,ип для некоторого C/n Е AT"(Г) (n E IN). Из C7) получаем, что 3%т,ип С 38т,и, гае оо оо С/ = U C/n G N(T). Следовательно, (J An G 3$т,и С «с/. Очевидно, система 71=1 71=1 множеств «с/ имеет и остальные свойства сг-алгебры. В то же время для любого t G Т можно взять (так как Т бесконечно) множество U(t) G N(T) такое, что t G U(t). Тогда S%T,{t} С 3&T,u(t) С «с/ при каждом ? G Т. В итоге 38т С «с/. Кроме того, d С ?$т, так как 38т,и С ?$т для любого U С Г. Значит, ?$т = ^, что доказывает второе соотношение C8). ? Наглядно теорема б утверждает, что принадлежность функции у [у из St) мно- множеству В цилиндрической а-алгебры 3$т определяется только ее значениями на некотором счетном множестве U С Т (выбор U зависит лишь от множест- множества В] если само Т счетно, то C8) сводится к тривиальному равенству 38т = 38т,т)- Пусть Т = [0,1] и St = М для t G Т. Тогда из сказанного следует, например, что С[0,1] ^ З&Тч гДе С[0,1] — множество непрерывных действительных функций на отрезке [0,1]. Множество Ма = \ у е St '• sup y(t) ^ a > при любом а е R есть ^ ter J несчетное пересечение (по t G [0,1]) элементарных цилиндров {у G St: y(t) ^ a}, поэтому, исходя из свойств сг-алгебры, нельзя утверждать, что Ма G 38т- Однако теперь мы можем сказать больше: иМа заведомо не входит в 38т при каждом a G К Итак, в случае несчетного Т возникают серьезные трудности в связи с тем, что многие интересные множества в пространстве траекторий не входят в цилиндри- цилиндрическую сг-алгебру. Поэтому, вообще говоря, неизвестно, можно ли говорить о соот- соответствующей им вероятности содержать траектории процесса (см. упражнение 22). §18. Если (п.в.) траектории процесса X = {X(t), t G Г} лежат в "хорошем" подмножестве пространства всех траекторий St, to ситуация упрощается. Для ил- иллюстрации этого положения потребуются вспомогательные утверждения. Лемма 11. Пусть St, t G T, — сепарабельные метрические пространст- пространства с метриками pt и борелевскими и-алгебрами 381 = 38(St)', по-прежнему, F(T) — совокупность конечных подмножеств Т. Тогда пространство Sj, J G F(T), функций у = y(t), заданных на J и таких, что y(t) G St, снабженное метрикой pj(x,y) = maxpt(x(t),y(t)), x,y G Sj, C9) является сепарабельным метрическим пространством. При этом 38 j= т. е. цилиндрическая а-алгебра совпадает с борелевской. Если, кроме того,
Гл. I. Случайные процессы. Распределения случайных процессов 33 пространства St, t G Т, полны (тем самым являются польскими), то Sj — польское пространство. Доказательство . Очевидно, функция pj обладает всеми свойствами метрики. Сходимость в этой метрике функций уп(') G S j равносильна тому, что yn(t) сходят- сходятся в (St, pt) при п —у оо для каждого t G J. Легко видеть, что (Sj,pj) — сепара- бельное пространство. "Проектирование" ttj^'-Sj —у St, определяемое формулой тгj^y = y(t), есть непрерывное отображение (S j, pj) в (St, pt) для каждого t G J. Поэтому TrjJ(Gt) G G ^S(Sj) для любого открытого множества Gt G Se(St). По следствию 1 имеем Trjj^t G 38(Sj) для любого Bt G 3§(St). Из A7) видим, что S%j С 38(Sj). Подчеркнем, что для последнего включения не требуется сепарабельность St, t G Г. Пусть теперь G — любое открытое множество в (S j, /? j). В силу сепарабельности метрического пространства (Sj,pj) множество G можно представить в виде счет- счетного объединения открытых шаров (проверьте это в качестве упражнения или см. [35; гл. II, § 5, п. 3]). Шар В?(у) = {z G Sj: pj(z,y) < s} является "прямоугольни- "прямоугольником" в Sj. Следовательно, любое открытое множество G представимо в виде счет- счетного объединения "прямоугольников" из 3S j. Поэтому S$(Sj) С S&j. Если пространства St, t G J, полны, то полно и S j. ? Замечание 5. Мы установили, что в сепарабельном метрическом пространстве борелевская сг-алгебра совпадает с сг-алгеброй, порожденной открытыми (значит, и замкнутыми) шарами. В несепарабельном пространстве упомянутые сг-алгебры могут, вообще говоря, не совпадать. Пусть Т — компакт в некотором метрическом пространстве V, a S — некоторое польское пространство с метрикой р. Введем пространство С(Т, S) непрерывных функций х = x(t), заданных на множестве Т и принимающих при каждом t значе- значения в S. Снабдим это пространство равномерной метрикой рс(х,у) = supp(x(t),y(t)), x,ye C(T,S). D0) Цилиндрическую а-алгебру в пространстве С(Т, S) определим как сг-алгебру, порожденную системой ^т Г\С(Т, S). Иначе говоря, в каждом цилиндре пространст- пространства St оставляем лишь непрерывные функции, а затем берем наименьшую сг-алгебру в С(Т, S), содержащую эти "непрерывные цилиндры" Лемма 12. Введенное пространство С(Т, S) непрерывных функций является польским. В этом пространстве борелевская о-алгебра совпадает с цилинд- цилиндрической. Доказательство. Первое утверждение элементарно и не обсуждается. Ана- Аналогично доказательству леммы 11 убеждаемся, что цилиндрическая сг-алгебра про- пространства С(Т, S) содержится в его борелевской сг-алгебре. Для проверки обратного включения возьмем счетное всюду плотное в Т множество М. Тогда замкнутый шар Вг(х) = {у G C(T, S): рс(х,у) ^ г}, где х G C(T, S) иг ^ 0, можно представить в виде Br(x)= f]{y&C(T,S):p(x(t),y(t))^r}, D1)
34 А. В. Булинский, А. Н. Ширяев т. е. как счетное пересечение цилиндрических множеств в С(Т, S). Искомое утверж- утверждение вытекает из замечания 5. ? С теоремой 3 интересно сопоставить следующий результат. Теорема 7. Пусть X = {X(t), t G Т} такая случайная функция, заданная на некотором вероятностном пространстве (П,^, Р), что п. в. ее траектории лежат в описанном выше пространстве С(Т, S). Тогда случайный элемент X, введенный в A5), является 3 \3ё(С(T,S))-измеримым отображением. Доказательство. Пусть п обозначает множество тех^ точек ио Е П, для кото- которых траектория Х{ • ,ио) непрерывна на Т (по условию Р(Й) = 1). Пользуясь D1), получаем {и:Х(и) еВг(х)}= р| {uefi:p(x(t),X(t,u)) ^ г} U {и ^ п: Х(и) еВг(х)}. teM Учитывая непрерывность функции р, имеем {и;: p(x(t), X(t, и)) ^ г} Е 3* при всех х(-) е C(T,S),t еГиг^О. Множество^ ^ п:Х(ио) е Вг(х)} С п\п, а веро- вероятностное пространство (как было оговорено ранее) всегда предполагается полным, значит, это множество также принадлежит 3. Требуемая измеримость X следует из замечания 5. ? Пример 7. Рассмотрим Т = [0,1] и S = Ж (с евклидовой метрикой р). Пока- Покажем, что для процесса X = {Х(?), t G [0,1]} с п.н. непрерывными траекториями множество \ sup X(t) ^ a \ G 3 при любом aGl. lte[oi] J Функционал h(x( •)) := sup ж(t), где х G С([0,1]), есть непрерывное отобра- [] жение пространства (С([0,1]),рс) в №р), следовательно, ft G Й8(С([0,1])) Кроме того, X G ^| Й§(С([0,1])) по теореме 7. Очевидно, <cj: sup X(t,cj) ^ a > = {cj: ft(X(o;)) 1 tG[O,l] J при каждом a G 1 Суперпозиция измеримых отображений дает измеримое (отно- (относительно соответствующих сг-алгебр) отображение. Требуемое утверждение полу- получено. § 19. Если на измеримом пространстве (St, $&t) заданамераО, то в соответствии с леммой 8 случайная функция X = {X(t), t G Г}, имеющая своим распределением вероятностей меру Q (т.е. Рх = Q), может быть определена следующим образом: X(t,u)=u(t) для u(-)eST, teT. D2) Формулой D2) задается (см. A6)) семейство координатных отображений прост- пространства St; при этом также говорят, что рассматриваемый процесс X непосредст- непосредственно задан или является координатным.
Гл. I. Случайные процессы. Распределения случайных процессов 35 ? Qv= Qu*u]v Рис. 5 Пусть на (St,^t) имеется мера Q. Тогда на "суженных" пространствах (S[/, Sfou), U С Т, возникают меры Qc/ := Qtt^ ^ ("проекции" или образы меры Q). Для V С С/ С Г, учитывая F) и C5), имеем Qtt"^ = QGr^c/7r^v) = (Q^t.u^u.v Тем самым получены условия, называемые условиями согласованности проекции меры Q на "суженных" пространствах: Qv = Quirky на ё$у при всех V С С/ С Г. D3) Дадим простое условие, эквивалентное D3) для конечных множеств У, U (обо- (обозначаемых далее / и J соответственно) из Т. Из леммы 2 вытекает, что семейство мер Q j на (S j, Й§ j), где J G F(T), согласовано тогда и только тогда, когда Qi(Bi) = Qj7r7J(Bj) D4) для любого "прямоугольника" Bj = {^/ G S/: ?/(?) G ??t, ? G /} (??t G S&f> t G /), всех J G F(T) и / С J. Удобство обращения с мерами, индексированными множествами (а не упорядо- упорядоченными наборами точек) состоит в том, что вместо двух типов условий симметрии и согласованости, введенных в § 11, возникает одна система условий согласованности. При этом формула D3) позволяет лучше понять смысл согласованности условных распределений, используемых для построения гиббсовских случайных полей (см., например, [15]). § 20. Обратимся к вопросу о том, какие случайные функции и в каком смысле могут считаться "неотличимыми" Определение 11. Процессы X = {Xt, t G Т} и Y = {Y^, t G T}, заданные на вероятностном пространстве (П, &, Р) идля? G Т принимающие значения в (Sj, ?^t), называются эквивалентными (или стохастически эквивалентными), если Р(сс?: Xt(o;) ф Yt(u)) = 0 для каждого t G Г. При этом предполагается, что множества {cj: Xt(uS) ф Yt(uS)} G ^. Очевидно, для процессов X и Y со значениями в пространстве Мт последнее предположение выполнено автоматически. Любой процесс У, эквивалентный X, называется моди- модификацией или версией процесса X (тогда и X является модификацией Y). Традиционный пример эквивалентных процессов (X и У), но в то же самое время с "сильно отличающимися" траекториями, строится следующим образом: Т = П = [0,1],с? = ^([0,1]), Р — мера Лебега на [0,1], Xt(u) = l{t}(uj),Yt(uj) = 0, t G T, cj G П. Отметим, что все траектории X разрывны, а все траектории У —
36 А. В. Булинский, А. Н. Ширяев Определение 12. Процессы X и Y (возможно, заданные на разных вероятност- вероятностных пространствах, но при каждом t G Т принимающие значения в одном и том же пространстве (St,?$t)), называются стохастически эквивалентными в широком смысле, если у них одни и те же распределения вероятностей, т. е. когда Law(X) = = Law(F) (на^т). Пусть X = {Х±, t G Т} и Y = {Y±,1 G Т} — случайные процессы с траекториями в пространстве St • Тогда из леммы 2 вытекает, что Law(X) = Law (У) в том и только том случае, когда Law(Xtl,...,XtJ =Law(Ftl,...,FtJ, tu ..., tn G Г, nGN, D5) т. е. когда совпадают все конечномерные распределения этих процессов. Легко видеть, что эквивалентые процессы являются стохастически эквивалент- эквивалентными в широком смысле. Определение 13. Процессы X = {Xt, t G Т} и Y = {Yt, t G T}, определенные на вероятностном пространстве (П, ^, Р) и принимающие при каждом t G Т значе- значения в (St, S%t)i называются (стохастически) неразличимыми, если Р(и: Xt{uo) Ф Yt{uo) для некоторого t G Т) = О, т.е. с вероятностью 1 совпадают траектории этих процессов. Заметим, что в данном определении предполагается, что Р-мера множества А = {oj'.Xt(oj) ф Yt(u;) для некоторого t G Г} определена (и более того, равна нулю), т.е. A G 3*. Если вероятностное пространство (П, ^, Р) является полным, то из верного при каждом s G Т включения {и: Xs(uo) ф Ys(u)} С {и: Xt{uo) Ф Yt{uo) для некоторого t G Т} вытекает, что неразличимые процессы являются эквивалентными. § 21. В связи с теоремой Колмогорова и построениями процессов с заданными конечномерными распределениями отметим ряд обстоятельств. Теорема Колмогорова утверждает, что, отправляясь от системы согласованных конечномерных распределений вероятностей, можно построить случайную функ- функцию с теми же самыми конечномерными распределениями. Важно при этом подчерк- подчеркнуть, что в теореме Колмогорова случайная функция X = {X(t, uo),t G Т} строится координатным образом: X(t,uj) = oo(t), t G Т. Это означает, что траектория- траекториями X могут быть любые функции uj = co(t), t G T, принимающие при каждом t G Т значения в (St, S%t)- Тем самым, в этой теореме остается открытым вопрос о том, насколько же "хороши", так сказать, "типичные" траектории того или иного процес- процесса X, построенного по заданным конечномерным распределениям. Далее многие утверждения содержат следующие слова: "Пусть X = {Xt, t G T} — случайный процесс на вероятностном пространстве (П, 3<, Р)" При этом структу- структура самого вероятностного пространства вовсе не уточняется, особенно тогда, когда основной интерес связан с распределениями вероятностей процесса X. В то же са- самое время часто возникает необходимость рассмотрения на заданном вероятностном пространстве (П, &<, Р) других, помимо X, случайных объектов. В этой ситуации надо, чтобы вероятностное пространство было достаточно "богато" и на нем можно было бы определить новые объекты.
Гл. I. Случайные процессы. Распределения случайных процессов 37 Одна из возможных процедур, позволяющая учесть такое требование, состоит в рассмотрении наряду с (П, 3, Р) его расширения. Именно, пусть (П', 3\ Р1) — некоторое иное вероятностное пространство. Опре- Определим новое вероятностное пространство (П, 3, Р), полагая п = пхп\ & = &&&', P = P(g)P/. Здесь 3 0 3' есть сг-алгебра в п х П', порожденная "прямоугольниками" вида АхА',гдеАе&иА'е 3',Р(АхА') := Р(А)Р'(А')дпяАе 3nAf e 3', а затем однозначно продолжается до меры Р на 3. Если X = {Xt(uo),t Е Т} — случайный процесс на (П, 3, Р), то его можно естественным образом считать также заданным и на (П, i?, Р). Действительно, для а; = (а;, а/) положим Xt(?)=Xt(u>), *еТ. D6) Очевидно, для так определенного процесса X = {Xt, ? Е Т}) справедливо сле- следующее свойство: _ Law(X|P) = Law(X|P), D7) т.е. распределения вероятностей Р^ и Рх на ^т совпадают. Конечно, вероятностное пространство ((l,3, P) "богаче", нежели пространство (П, 3, Р). Поэтому очень часто в вероятностных рассуждениях сразу подразумева- подразумевают, что исходное вероятностное пространство "достаточно богато" В том случае, когда (П7, 3\ Р') = ([0,1], Й§([0,1]), Л), где Л — лебеговская мера и Й§[0,1] — лебеговское пополнение борелевской сг-алгебры Й§([0,1]), расширение (П, ^, Р) принято называть стандартным расширением пространства (П, 3, Р). §22. Как правило, наиболее важен случай процессах = {Xt,t G Т}, длякоторо- го величины Xt при каждом ? принимают значения в одном и том же измеримом про- пространстве (S, 9S). В этом случае часто возникает необходимость в дополнительных условиях на измеримость процесса. Первым шагом в этом направлении является следующее Определение 14. Пусть (Т, si) — измеримое пространство. Случайный про- процесс X = {Xt,t G Т}, определенный на (П, 3, Р) и принимающий при каждом t G Т значения в измеримом пространстве (S, Й§), называется измеримым, если отображе- отображение (t, из) н-> xt(w) e S, (*, и)етхо, D8) является si 0 ^ | ^-измеримым. В связи с этим определением важно подчеркнуть следующее принципиально важ- важное обстоятельство: здесь мы рассматриваем случайный процесс X = {Xt (и), t G Т} не как функцию от ? при заданном и или как функцию от и при заданном ?, а как из- измеримое отображение (?, и) \—У X-t(uS). Иначе говоря, мы рассматриваем процесс X как (измеримую) функцию пары (?, ш). Чаще всего S — метрическое пространство и Й§ = 5S(S). Если Т — дискретное множество (конечное или счетное) и si — сг-алгебра всех подмножеств Т, то про- процесс X, очевидно, измерим.
38 А. В. Булинский, А. Н. Ширяев Основной смысл введения этого понятия измеримости процесса состоит в том, что, рассматривая случайное время т = т(и) (т Е 3 \&4), мы всегда будем получать (см. упражнение 27) случайную величину Хт^(ии). Пусть X, Х\, Х2,... — последовательность независимых одинаково распределен- распределенных величин, принимающих значения в измеримом пространстве (S,^). В следу- следующем примере показано, что наличие случайного индекса т приводит к тому, что распределение Хт может отличаться от распределения Хп (п Е N). Пример 8. Для описанной выше последовательности X, Xi, X2,... и некоторого множества В Е ё% с Рх(В) > 0 определим (расширенную, т.е. со значениями в Ж) случайную величину т: т{и) = inf{n Е N: Хп(ио) G В} D9) (t(uj) := 00, если Xn(uj) (? В для всех п G N). Покажем, что т < оо п.н. и найдем распределение Хт. (Если т(ии) = 00, то пола- полагаем, например, Хт(ио) = Х(ш), т.е. считаем Х^ио) = Х(ио).) Заметим, что {г = 1} = {Х1 G В}, {т = п} = {Xi ? В, ..., Xn_i ? В,Хп е В} для n ^ 2. Поэтому Р(г = п) = Р(Х G В)A - Р(Х G В)O1, n ^ 1. Следовательно, г < оо п.н. Для любого множества С G Й§ получаем оо р(хт € с) = х; п=1 G ВС) + X] Р(Х- еВСХг^В,..., Хп_! ^ В) = оо ™^2 = ^ Р(Х G ВС)A - Р(Х G В)O1'1 = п=1 = Р(х е вс)/Р(х еВ) = Рх(С\ в). Интересно отметить, что распределение Хт будет совпадать с распределением X, ес- если только В не зависит от любого С G Й§ (в вероятностном пространстве (S, Й§, Рх)), что возможно лишь для 5 с Рх (В) = 1. Дополнения и упражнения 1. С помощью теоремы Колмогорова (теорема 4) получите частный случай теоремы Ломницкого-Улама (теорема 2) для семейства борелевских прост- пространств. Интересно отметить, что в элементарном случае, когда требуется построить последовательность независимых действительных случайных величин с заданными распределениями, это легко сделать непосредственно (без теоремы Колмогорова) следующим образом.
Гл. I. Случайные процессы. Распределения случайных процессов 39 Пусть п = [0,1], & — ^([0,1])иР = mes (мера Лебега). Рассмотрим двоичное разложение числа ио Е П: ^k(ujJ-\ E0) к=1 где коэффициенты а&(о;) равны 0 или 1. Представление E0) неоднозначно. Пред- Предполагается, что в неоднозначных случаях рассматриваются только представления с бесконечным количеством нулей среди коэффициентов а^(ио). 2. Проверьте, что величины а\ (ио), a<i (uj) ,..., введенные в E0), образуют после- последовательность независимых случайных величин на указанном вероятностном пространстве (П, ^, Р), причем такую, что Р(ак = 1) = Р(ак = 0) = 1/2, к Е N. E1) Докажите, что если на некотором вероятностном пространстве (П, &, Р) имеется последовательность независимых случайных величин, удовлетво- оо ряющих условию E1), то случайная величина ?(а;) = ^2 пк(шJ~к имеет равномерное распределение на отрезке [0,1]. к=1 3. Докажите, что если F — некоторая функция распределения и U — равномер- равномерно распределенная на отрезке [0,1] случайная величина (заданная на некото- некотором вероятностном пространстве), то X(uj) = F1uv(U(uj)) является случай- случайной величиной с функцией распределения F (обобщенная обратная функция Finv(x) = inf{у :F(y) ^ ж}, х е [0,1]; если F(y) < 1 для всех у е М, то Finv(l) := +оо;если^(^/) > 0 для всех у е М, ToFinv@) := -оо). Расположим величины ai, a2,..., фигурирующие в E0), в виде таблицы (например, Ьц = а\,Ъ\2 = о<2, ^21 = &з, Ьз1 = «4, ^22 = о>5, ^13 = «6, • • •) иопределим оо величины Un(uj) = J^ Ьп^(ооJ~к, n G N. Из упражнений 2 и 3 следует, что для любых заданных функций распределения Fi, F2,... величины (со значениямив Л = [-оо, +оо]) Хп = F™(Un), n G N, будут независимый Р(ХП ^ ж) = Fn(x), ж G М. 4 (сравните с теоремой 6). Приведите пример семейства сг-алгебр d\, A G Л, состоящих из некоторых подмножеств п таких, что (J ^л не является сг-ал- геброй. ЛеЛ 5. Получите обобщение леммы 7 на случайные элементы со значениями в боре- левском пространстве. 6. Пусть множество G состоит из функций д: Ж —у М, которые не убывают по каждому аргументу. Покажите, что G ? ё%т (St = M, t G Г = Rd). Тем не менее, найдите Px(G), где X = {Xt = ^A+ /(t)/y), t G Md},/:Md -^ Rявля- Rявляется невозрастающей по каждой переменной функцией, а ? и 77 — независимые величины, равномерно распределенные на отрезке [—1,2].
40 А. В. Булинский, А. Н. Ширяев 7 (сравните с упражнением 2). Докажите, что на (П, ^, Р) = ([0,1],Й§([0,1]), Р), где Р — мера Лебега, невозможно построить (континуальное) семейство {Xt, t G 1} независимых бернуллиевских величин, т.е. таких случайных величин xt,t e м, что P(Xt = 0) = P(Xt = 1) = 1/2 при каждом t. 8. Докажите, что ни на каком вероятностном пространстве невозможно за- задать семейство независимых одинаково распределенных действительных не- невырожденных случайных величин X(t, и), t Е Ж, и; Е П, так, чтобы с вероят- вероятностью 1 траектории Х( •, и) были непрерывны на Ж. 9 (сравните с теоремой 4). Пусть С (Ж) — пространство действительных функ- функций, непрерывных на Ж. Зададим на цилиндрических множествах вида Dj = = {х G С(Щ: x(t) e Bt,t e J}, где Bt G 0&(Ж), t G J n J G F(R) (F(R) — совокупность конечных подмножеств Ж), функции Qj(Dj) := Д здесь Qo — некоторая мера на Й§(М). Можно ли ввести меру Р на сг-алгебре цилиндрических множеств пространства С (Ж) так, чтобы P(Dj) = Qj(Dj) для всех J G F(M) и любых введенных выше цилиндров D j? 10. Как выглядят траектории процесса A1)? 11. Пусть т = 1 в определении A3), т.е. {(j} — независимые одинаково распре- распределенные скалярные случайные величины, и пусть d = 1. Положим xn(t,u) = ^=sn([o,t],u), *e [0,1]. Постройте график траектории этого процесса в следующих случаях: a) \i — мера Лебега на Ж; б) \i — считающая мера, т. е. О процессах, возникающих в пунктах а) и б) упражнения 11 (соответственно с тра- траекториями в пространстве С[0,1] ив пространстве Скорохода D[0,1], вводимом далее), рекомендуется прочитать главы 2 и 3 монографии [2]. 12. Пусть т = 1 в определении A2). Положим F*(x,uj) = Рп((-оо,х],и), х G Ж. Постройте график траектории процесса F?(x,uj), который принято называть эмпирической функцией распределения. В чем будет отличие случаев, когда ?j равномерно распределены на [0,1] и когда ?j — бернуллиевские величины с Р(& = 1) =р, Pfe = 0) = 1 -р, 0 < р < 1?
Гл. I. Случайные процессы. Распределения случайных процессов 41 Введем следующее определение. Определение 15. Класс множеств Ж С ?$(Мт) называется конечно-аппрокси- конечно-аппроксимируемым относительно меры Q, заданной на S^iW71), если при любом е > 0 можно указать множества S[?\ ..., S^ Е ?$(Mm), где N = N(e), такие, что для каждого В еЖ найдутся S^ и sf со свойствами S^ С В С SJe) и Q(SJe) \ S^) < е. Имеет место следующая теорема. Теорема 8 (см. [3; с. 421]). Пусть Ж — класс множеств, являющийся ко- конечно-аппроксимируемым относительно меры Q. Пусть Рп(В,со) — процесс, определенный формулой A2), и соответствующие векторы имеют закон рас- распределения Р^ = Q. Тогда sup \Pn(B,u)-Q(B)\->0 при п-^оо E2) веж для всех uj Е П, где О \ О С Но и Р(^о) — 0- Иначе говоря, если вероятност- вероятностное пространство (П,^, Р) полно, то в E2) имеет место сходимость почти наверное. 13. Докажите, что класс Ж = {(—оо,ж], х G Mm}, т.е. совокупность множеств вида (-оо, xi] х- • -х (-оо, хш], (х\,... ,жт) G Мт, является конечно-аппрок- конечно-аппроксимируемым относительно любой (вероятностной) меры Q. Тем самым справедлив многомерный вариант теоремы Гливенко-Кантелли о п.н. равномерной сходимости при п —У оо эмпирических функций распределения F?(x, uS) := Рп((—оо, ж], о;) к функции распределения i7^ (ж): *(o;,a;)-ib(aO|-X) (Р-п.н.). 14. Докажите, что если мера Q = Р^ является абсолютно непрерывной от- относительно меры Лебега в Ж171, то утверждение E2) также справедливо для класса Ж, являющегося классом выпуклых множеств. (Это утверждение, вытекающее из упражнения 13 при т = 1, становится нетривиальным и не вытекающим из него, если т ^ 2.) 15. Покажите, что утверждение, содержащееся в упражнении 14, может стать неверным, если отказаться от условия абсолютной непрерывности Q относи- относительно меры Лебега (при т ^ 2). Об исследовании эмпирических мер, а также эмпирических процессов, индекси- индексированных семейством функций, см., например, [120]. Пусть d С Й8([0, l]d), d^l. Для функции F: d ->> Ж обозначим = sup \F(A)\. Положим F6(A) = mes(A^)), A e d, 5 > Ои# = {x e Rd:p(x,dA) < 5}, p(x, dA) = inf p(x, y), p — евклидова метрика и mes — мера Лебега в Rd. уедА
42 А. В. Булинский, А. Н. Ширяев Предположим, что класс множеств si С Й§([0, l]d) таков, что ->0, при ?^0 + . E3) 16. Докажите, что свойство E3) выполнено, если si — класс прямоугольников вида \а\, Ь\] х • • • х [a^, bd] С [0, l]d. Справедлив следующий равномерный усиленный закон больших чисел. Теорема 9 ([96]). Пусть {Xj, j G Nd} — случайное поле, состоящее из не- независимых одинаково распределенных действительных величин со средним с$. Пусть класс множеств si С Й§([0, l]d) удовлетворяет условию E3). Тогда i~dSn( •) — со mes( •) |^ —у 0 п.н. при п —У оо, здесь Sn(A) = Yl Xj, nA = {х = пу, у е A}, n е N, т. е. с нормировкой n~d рассматривается процесс частных сумм A3), где \± — считающая мера. О центральной предельной теореме и законе повторного логарифма для процессов, индексированных множествами, см., например, [90, 94]. Процессы восстановления A0) играют важную роль в прикладных исследова- исследованиях. Процесс восстановления называется дискретным, если распределение ?i — решетчатое, т.е. сосредоточено на множестве вида кХ, где к Е Z, а А > 0 — пара- параметр, называемый "шагом распределения" (если^1 —неотрицательная величина, то к = 0,1, 2,...). В противном случае процесс восстановления называется непрерыв- непрерывным. Определим функцию восстановления u(t), t > 0, как среднее число восста- восстановлений на промежутке @, t], т. е. u(t) = E Xt. 17. Докажите, что u(t) < оо для всех t > 0. Имеет место следующая основная теорема теории восстановления (см., например, [78; т. 2, с. 408]). Теорема 10 (Блэкуэлл). Для непрерывного процесса восстановления u(t + s) - u(t) -у s/c при t —У оо для любого фиксированного s > 0, где с = E?i (считаем формаль- формально s/oo = 0). Для дискретного процесса восстановления утверждение верно при s, кратных шагу X распределения ?i. Наглядно "почти очевидно", что если среднее время жизни электрических лампо- лампочек есть с, то за время s "в среднем" требуется s/c замен. Однако получить утверж- утверждение теоремы 10 довольно сложно. Вероятностное доказательство этой теоремы можно прочитать, например, в [1; с. 46-50] или [190; п. 8 гл. 2]. О процессах восста- восстановления см. также [4; гл. 9], [78; том 2, гл. 11], [125; гл. 5], [131; гл. 25], [146; гл. 8]. 18. Объясните, почему функция Z(B, •), определенная согласно A4), является при каждом В G S$(S) случайной величиной со значениями в пространстве (Z+, ?/), raeZ+ = {0,1,... } = {0}UN, si — сг-алгебра всех подмножеств Z+. (Отсюда вытекает, что Z = {Z(B), В G ^S(S)} — действительный случай- случайный процесс, индексированный множествами ^
Гл. I. Случайные процессы. Распределения случайных процессов 43 Пусть Л есть сг-конечная мера на некотором измеримом пространстве (S, Й§) , оо (S = U Sm, где Sm е $, Sm П S/c = 0 при т Ф к и 0 < A(Sm) < оо, т, к Е NJ. В этом случае конструкцию пуассоновской случайной меры A4) легко модифицировать следующим образом. Построим на некотором вероятностном пространстве (П, ^, Р) счетную совокуп- совокупность независимых величин таких, что y(m) имеет распределение Пуассона с параметром Am = A(Sm) и Law(XJm)) = A(-)/A(Sm)Ha^m = ^nSm;m,j G N. Пусть Zm{B,u) дляВ е @>т определяется согласно A4) с заменой Y на У (т) и Xj на Х^\ Положим оо Z(B,oo)=Y,zrn(BnSm,oo), Вей, ш G П. E4) т=1 Отметим, что Z(B, со) может принимать и бесконечные значения. 19. Докажите, что если Х(В) = оо для некоторого В G Й§, то Z(i?) = оо п.н., а если АE) < оо, то ^(??) < оо п.н. Теорема 11 (см. [12; с. 23]). Процесс E4), называемый пуассоновской случай- случайной мерой, обладает следующими свойствами: 1. Для любого п G N и любых непересекающихся множеств В\,..., Вп G Й§ величины Z(Bi, •),..., Z(Bn, •) независимы. 2. Дл^ каждого В Е ?$ величина Z(B, •) имеет распределение Пуассона с параметром Х(В). 3. Траектории Z(-,о;) являются целочисленными мерами на о-алгебре S&. Уточним, что пуассоновскую величину с параметром А = оо мы понимаем как величину, равную бесконечности с вероятностью единица. Величину Z(B) можно интерпретировать как число "случайных точек", попавших во множество В G ё$. В этой связи интерес представляет следующее упражнение. 20 (см. [32; с. 370]). Пусть В G SS таково, что 0 < Х(В) < оо, где А есть п сг-конечная мера на (S,^). Пусть В — (J Bq, где Bq G Зё, Bj П Bq = 0 g=l при g т^ J (^? J = 1? • • • 5n)- Докажите, что для любых &i,..., kn G Z+ и n k = ^2 kq верно равенство q=l P[Z(B1) = fei,..., Z(Bn) = &n
44 А. В. Булинский, А. Н. Ширяев Таким образом, если Й§ = ?$(Rd), d ^ 1, то при фиксированном числе "пуассонов- ских точек", попавших во множество В (конечного и ненулевого объема) их место- местоположение можно рассматривать как такое, какое имеют к независимых случайных величин, равномерно распределенных в В. Случайным точечным процессам и полям (не только пуассоновским), как важным моделям, используемым в физике, технике и других областях, посвящена обширная литература (см., например, [10, 13,111,145,151,156,178]). Обратимся к вопросам, связанным с отождествлением (см. §20) случайных функций. 21. Приведите пример действительных процессов X = {Xt, t G Г}иУ = {Yt, t G T}, заданных на одном вероятностном пространстве (П, 3, Р), таких, что Рх — Ру на ё%т, но X и Y (стохастически) неэквивалентны. Приведите пример, когда Рх ф Ру, но распределения Xt и Yt совпадают при каждом t G Т. 22. Приведите пример (действительного) процесса X = {Xt, t G Т} и множеств D,L ^ $т таких, что {и;: Х( • ,и) G D] G ^и {и\ Х( • ,и) G L} ? 3. 23. Приведите пример эквивалентных процессов X — {Xt, t G Т} и Y = {Y^, ? G Т}, а также множества D ^ ^т таких, что А = {uj: Х( •,и) G D} G ^ иВ = {u;:Y(- ,uj) e D} е 3, ноР(А) ^ Р(#). Упражнения 22 и 23 показывают, что не всегда можно говорить о вероятнос- вероятности попадания процесса во множество, не входящее в цилиндрическую сг-алгебру, но если это и возможно, то вероятность попадания в такое множество не обязана опре- определяться конечномерными распределениями процесса. 24. Приведите пример (стохастически) эквивалентных процессов таких, что они не являются неразличимыми. В связи с тем, что многие важные для теории и приложений множества не вхо- входят в сг-алгебру ё%т (см. теорему 6), Дж. Дубом было предложено понятие сепа- сепарабельности процесса, которое позволяет в определенных случаях строить версию процесса, поведение которой, по сути дела, задается значениями на счетном пара- параметрическом множестве. Определение 16. Пусть на вероятностном пространстве (П, 3, Р) имеется случайный процесс X — {Xt, t G Т}, где Т — топологическое пространство, принимающий при каждом t G Т значения в польском пространстве (S, р). Процесс X = {Xt, t G Т} называется сепарабельным по отношению к множеству сепарабельности То С Т, если То — счетное всюду плотное в Т множество и существует событие N G 3 такое, что P(N) = 0 и для всех t G T ,uj G П\ N E5) [ П {хв(ш), seenт0}], здесь [В] обозначает замыкание множества В, а $ — совокупность открытых мно- множеств в (S,p). Процесс X называют сепарабельным, если в качестве множества сепарабельнос- сепарабельности То может быть взято любое счетное всюду плотное подмножество Т.
Гл. I. Случайные процессы. Распределения случайных процессов 45 Теорема 12 (см. [153; с. 128]). Пусть (S,p) — компактное метрическое про- пространство, и (Т, S) — псевдометрическое пространство. Тогда случайный процесс X = {Xtj t G Т}, определенный на (П,^, Р) и при каждом t G Т при- принимающий значения в (S,p), имеет сепарабелъную модификацию. Если S — ло- локально компактное пространство, то теорема действует при одноточечной компактификации S. В качестве упражнения можно сравнить определение 16 с определениями, приве- приведенными в [101; с. 111], где рассмотрен случай Г = [0,1], S = Ж. О сепарабельных процессах см. также [12, 25]. Теорема 13 ([79]). Пусть выполнены условия теоремы 12 и процесс X = {Xt, t G Т} является стохастически непрерывным на Т, т. е. для каждого t G Т и любого е > 0 hmP(p(X3,Xt) >e)=Q. E6) Тогда у процесса X существует модификация, являющаяся сепарабельной и измеримой. 25. Докажите, что если S — сепарабельное метрическое пространство, то под знаком вероятности в E6) стоит событие, т. е. множество, принадлежащее 3* (указание: воспользуйтесь леммой 11). 26. Докажите, что если процесс измерим (см. определение 14), то каждая его тра- траектория является si | ^-измеримой функцией. Верно ли обратное утвержде- утверждение? Приведите пример процесса, не имеющего измеримой модификации. 27. Пусть X — {Xt, t G T} — измеримый процесс иг:0ч T,r G 3*\si. Дока- Докажите, что тогда Y(uj) = ХТ^(ш) есть 3 \ ^-измеримаявеличина. Покажи- Покажите, что если отказаться от условия измеримости процесса X, то сформулиро- сформулированное утверждение не обязано выполняться. Случайные величины Хт, порожденные, скажем, "случайным числом случайных величин" (Хт = ^2 ?д.), где т — целочисленная случайная величина, использу- ются в многочисленных приложениях, например, в теории страхования. При этом для таких случайно индексированных сумм возникают эффекты, которые не наблю- наблюдаются для сумм детерминированного числа слагаемых (см., например, [133]). 28. Пусть почти все траектории процесса X = {X(t), t ^ 0} являются непре- непрерывными справа на [0, оо). Докажите, что такой процесс имеет измеримую модификацию. Различные постановки задач о построении процессов с заданными свойствами траекторий изложены, например, в [12] на с. 120-130. В дополнении к главе II мы также затронем некоторые вопросы, относящиеся к этой проблематике.
Глава II Процессы с независимыми приращениями. Пуассоновские и гауссовские процессы Критерий существования процесса с независимыми приращениями. Пуассоновский процесс. Винеровский процесс (броуновское движение). Многомерное нормальное распределение. Построение действительной гауссовской случайной функции по функ- функции среднего и ковариационной функции. Комплекснозначные гауссовские процессы. Неотрицательно определенные функции как ковариационные функции и как воспроиз- воспроизводящие ядра гильбертовых пространств. Теорема Парзена. Эквивалентность двух определений броуновского движения. Функции Хаара и Шаудера. Флуктуации после- последовательности стандартных гауссовских величин. Построение непрерывного винеров- ского процесса. Многомерное броуновское движение. §1. В этой главе изучается специальный, но обширный и важный класс случайных процессов, имеющих независимые приращения. При первом чтении можно опустить § б о комплекснозначных гауссовских процессах и § 7, в котором рассматриваются случайные процессы, индексированные семействами функций. Особое внимание желательно обратить на пуассоновский и винеровский процессы (вводимые в параграфах 2 и 3), широко используемые в последующих главах. Определение 1. Действительный случайный процесс X = {Xt, t ^ 0} назы- называется процессом с независимыми приращениями, если для любого п Е N и всех to, t\,..., tn таких, что 0 = to < h < ••• < tn, величины Xto, Xtl - Xto, ..., Xtn — Xtn_1 независимы в совокупности. Теорема 1. Пусть {(p(s,t; •)}, где 0 ^ s < t < оо, — семейство характерис- характеристических функций, отвечающих некоторому семейству Qs,t, 0 ^ s < t < оо, ве- вероятностных мер на Зё(Ж). Для существования случайного процесса X = {Xt, t ^ 0} с независимыми приращениями такого, что характеристическая функ- функция случайной величины Xt — Xs есть (f(s,t; •) при любых 0 ^ s < t < оо, необходимо и достаточно, чтобы (p(s, t; v) = (p(s, щ v) if {и, t; v) A) для всех 0^s<u<t< oo; i/El. При этом распределение вероятностей Qo величины Xq может быть выбрано каким угодно.
Гл. П. Процессы с независимыми приращениями 47 Доказательство . Необходимость условия A) очевидна, поскольку характерис- характеристическая функция суммы независимых случайных величин равна произведению ха- характеристических функций слагаемых. Пусть теперь выполнено условие A). Допустим, что удалось построить некоторое вероятностное пространство (П, ^, Р) и искомый процесс X, причем Law(Xo | P) = = Qo- Тогда характеристическая функция величины Xq равна <?qo(A), а характе- характеристическая функция вектора ? = (Xto,Xt1 — Xt0,X X) О = to < t\ < • • • < tn есть ,..., An) = Заметим, что X *2 /1 1 1 0 1 1 0 .. 0 .. 1 .. . 0^ . 0 . 0 \1 1 1 1/ / х, Xtn - Xtn-l) ДЛЯП ? ^И p(tn-i,tn', An). Для любого случайного вектора r\ € afc,m ё t (/:, m = 1,..., ?), и всех А е К любой матрицы А = B) =1, где то=1 = Е ехр{г(Л, Аг])} = Е ехр{г(А*Л,7?)} = C) где А* — транспонированная матрица А, (•,•)- скалярное произведение в W. Следовательно, при п?1ЧиО = ?о <t\ < • • • <^п конечномерные распределения самого процесса X должны задаваться характеристическими функциями ^to,ti,...,tn(Ао, Аь ..., An) = у>?(А*\) = (^Qo(/io)^@, ti; /ii) • • • <p(tn-i,tn] /in), где /i = A*\vl A — треугольная матрица, фигурирующая в B): /io = Ао + • • • + An, Mi = Ai H h An, ..., fjLn = An. Кроме того, мы должны иметь y?to(Ao) = ^qo(Ao), M(Pti,...,tn(Ab---5An) =^to5tb...5tn@,Ai,...,An)npHn e NhO < ti < ••• < tn. Итак, предположив существование искомого процесса X, мы выяснили, какие характеристические функции должны быть у его конечномерных распределений. Отправляясь от заданных функций ^qo( •) и ip(s, ?; •), где 0 ^ s < t < оо, вве- введем теперь описанным выше способом характеристические функции cptQ, ^to,tb...,tn и ipti,...,tn (® = to < ti < • • • < tn, n G N) и воспользуемся теоремой 5 главы I. Условие (а) этой теоремы не требует проверки в силу замечания 3 главы I, а усло- условие (Ь), точнее, условие (V), означающее подстановку 0 в (рт(Х) вместо любого ар- аргумента Ат, также выполнено, поскольку согласно A) для 1 ^ m ^ п '-, 0 Ап) = Ап). Тем самым, выполнены условия согласованности и, значит, существование требуе- требуемого процесса с независимыми приращениями вытекает из теоремы 5 главы I. Оче- Очевидно, что распределение Xq будет искомым распределением Qq. ?
48 А. В. Булинский, А. Н. Ширяев Замечание 1. Определение действительного процесса с независимыми прира- приращениями распространяется и на процессы со значениями в W71 (т ^ 1). При этом теорема 1 остается в силе с незначительной модификацией в ее доказательстве. Так, в матрицу, фигурирующую в B), вместо единиц следует вписать 1Ш — единичные матрицы ттг-го порядка, и рассматривать А& Е Мт, к = 0,1,..., т. Замечание 2. Процесс с независимыми приращениями может быть определен не только на полупрямой [0, оо). Если Т = N и процесс X = {Xt, t E N} имеет независимые приращения (т.е. Xt0, Xtx — Xt0, ..., Х±п — Xtn_1 независимы для 1 = to < t\ < • • • < tn, где ti E N при г = 1,..., n и n E N), то процесс Х будет иметь простую структуру: Xt =?]. + ••• + &?? ? N, где {f;j}j?=1 — последовательность независимых величин. Тем самым, процессы с независимыми приращениями — ес- естественное обобщение случайных последовательностей, являющихся суммами не- независимых случайных величин. § 2. Напомним, что неотрицательная счетно-аддитивная функция m на Й§(М) (или на Й§(М_|_)) называется локально конечной мерой, если m([a,b]) < оо для любых — оо < а ^ Ъ < оо. Определение 2. Пуассоновским процессом с ведущей мерой m (где m — ло- локально конечная мера, т(Ж) = оо) называется случайный процесс TV = {N(t),t ^ 0} такой, что 1) jV(O) = 0п.н.; 2) процесс N имеет независимые приращения; 3) величины N(t) - N(s), где 0 ^ s < t < оо, распределены по закону Пуассона с параметром m((s,i\). В частности, если m((s, t]) = (t — s)X, 0 ^ s < t < оо, А > О, то говорят о стан- стандартном пуассоновском процессе с постоянной интенсивностью А. Условимся считать, что пуассоновское распределение с нулевым параметром име- имеет случайная величина, тождественно равная нулю. Существование пуассоновского процесса следует из теоремы 1. Действительно, если бы такой процесс имелся, то согласно 3) мы получили бы Для так определенной функции ip(s,t;u), 0 ^ s < t < оо, z/EM, условие A) вы- выполнено, поскольку m((s,i\) = m((s, и]) + т((и, ?]), что позволяет воспользоваться теоремой 1. В качестве начального распределения Qo надо взять меру, сосредото- сосредоточенную в точке 0. Следующий результат интересен тем, что объясняет как устроены траектории пуассоновского процесса. Теорема 2 (явная конструкция пуассоновского процесса). Пусть ?i, ?2 ? • • • — независимые случайные величины, имеющие экспоненциальное распределение с плотностью Хе~^х, х ^ О, О, х < О, где А — положительный параметр, г Е N. Тогда соответствующий процесс восстановления A.10) является пуассоновским процессом интенсивности А.
Гл. П. Процессы с независимыми приращениями 49 Эта теорема не доказывается, поскольку она есть частный случай общего пост- построения марковских цепей с помощью инфинитезималъной матрицы (см. упражне- упражнение 33 главы VI). Пример 1. Пусть процесс У = {Yt,t ^ 0} определяется согласно формуле A.11), где величины ?i, ?2? • • • имеют показательное распределение с параметром Л > 0. Покажем, что процесс Y имеет независимые приращения. Заметим, что если Z = {Zt, t ^ 0} — действительный процесс с независимы- независимыми приращениями и h = h(t) - неслучайная действительная функция на [0, оо), то {Zt + h(t), t ^ 0} — также процесс с независимыми приращениями. Поэтому про- процесс Y (см. A.11)) будет иметь независимые приращения, если этим свойством обла- xt дает S = {St, t ^ 0}, где St = ^2 Vj- В силу теоремы 2 процесс X = {Xt, t ^ 0}, задаваемый формулой A.10), является пуассоновским процессом интенсивности Л. Положим ф(у) = Е elUTl1, v G ffiL Учитывая независимость приращений пуассо- новского процесса, а также независимость последовательностей {?j}j^n и {l (т. е. независимость порожденных ими сг-алгебр), имеем для 0 ^ s < ^иг/ G оо Eeiu(st-ss) = ^ Eeiu{St-Ssh{X k,rn=0 00 , к+т k,m=0 Y, P(xs = k) ? k=0 rn=0 Аналогично вычисляется совместная характеристическая функция набора прираще- приращений процесса {St, t ^ 0}, откуда следует требуемое утверждение. § 3. Введем один из самых важных случайных процессов. Определение 3. Винеровским процессом, или. броуновским движением, на- называется случайный процесс W = {W{t), t ^ 0} такой, что 1) W@) = 0п.н.; 2) процесс W имеет независимые приращения; 3) величины W(t) — W(s) ~ N(O,t — s) при всех 0 ^ s < t < 00, т. е. величины W(t) — W(s) имеют гауссовское (нормальное) распределение с параметрами 0nt-s. Существование процесса, обладающего свойствами 1)-3), вытекает из теоремы 1, поскольку (t-s)u2 (u-s)u2 (t-u)u2 е 2 =е 2 е 2 , 0 ^ s < и < t, г/Gl, что обеспечивает выполнение условия A).
50 А. В. Булинский, А. Н. Ширяев Для винеровского процесса W(t) = W(t) - W@) ~ N@, t), поэтому EW(t) = 0 при всех t ^ 0. Если 0 ^ s ^ ?, то ковариация cov(W(s),W(t)) = coy(W(s),W(t) - W(s) + W(s)) = D(W(s) -W@)) = s, где D обозначает дисперсию. Тем самым, для винеровского процесса W EW(t)=0, cov(W(s),W(t)) = min{s,?} при s,t e[0,oo). F) Замечание 3. Обычно в определение винеровского процесса включают еще тре- требование непрерывности п. н. (т. е. непрерывности с вероятностью единица) его тра- траекторий. Далее мы увидим, что это свойство действительно всегда можно считать выполненным наряду со свойствами 1)—3). Определение 4. Многомерным (m-мерным) броуновским движением W = = {W(t) = (Wi (t),..., Wm(t)),t ^ 0} называется процесс со значениями в R171, со- составленный из т независимых (непрерывных) броуновских движений {Wk(t), t^O}, к = 1,... ,т. Независимость процессов в этом определении понимается как независимость по- порожденных ими сг-алгебр (см. замечание 1 главы I). Такой процесс W легко получим, задав на некоторых вероятностных пространствах (П&, ^, P&), к = 1,..., т, соот- соответствующие броуновские движения Wk = {Wk(t), t ^ 0}. Далее берем к=1 и доопределяем Wk на П аналогично формуле A.46). § 4. Перейдем к изучению гауссовских случайных функций. С этой целью, преж- прежде всего, напомним, что случайный вектор Y вЖп называется нормальным или га- уссовским (обозначение: Y ~ 1\1(а,С)), если для A G 1П его характеристическая функция ipY(А) = Е ехр{г(А, Y)} имеет вид 1 ] ( " а5 А) --(СА, А) > = ехр< г ^ а^А/с - - ^ c/cmA/cAm [>, G) к^ 1 1 п 1 - J^ ckmXkXm>, k,rn=l * где a G Мп, С = (ckmO^ ш=1 — симметричная неотрицательно определенная матрица с действительными элементами, (•, •) — скалярное произведение в Жп, за- задающее евклидову норму (с векторами мы оперируем как со столбцами). Неотрица- Неотрицательная определенность матрицы С (запись С ^ 0) означает, что (СА, А) ^ 0 для всех A G 1П. Легко проверить, что требуемые условия С ^ 0 и С = С* равносильны выполнению лишь одного условия: 0 (8) к,1=1 для любых комплексных z\,..., zn (z = и — iv для z = и + iv, где u,v G Ж).
Гл. П. Процессы с независимыми приращениями 51 Известно (см., например, [85; т. 1, с. 383]), что для описанных выше матриц С и любого вектора a Е W1 функция, стоящая в правой части G), является характерис- характеристической функцией некоторого случайного вектора Y. Если С > 0, т.е. (СА, Л) > О для любого Л ф О Е Мп, то случайный вектор У имеет плотность (по мере Лебега) распределения вероятностей Плг(т\ — (Oqr\~n 2\ГЛ~1 2 Wnf — (A~1(T — п) Т — п\\ где |С| — определитель С. Для У ~ |\|(а, С) нетрудно установить, что ak = EYk, ckrn = cov(F/c,Fm), fc,m = 1,... ,n (n ^ 1). (9) Определение 5. Действительная случайная функция X = {Х(?),? Е Т}, задан- заданная на вероятностном пространстве (П, ^, Р), называется гауссовской, если все ее конечномерные распределения являются гауссовскими. Другими словами, X — гауссовский процесс, если (X(ti),..., X(tn)) есть гаус- совский вектор для каждого п Е N и любого конечного набора точек t\,..., tn E Т. Это свойство достаточно потребовать лишь для наборов (ti,..., tn), состоящих из различных точек, что, в частности, вытекает из следующей элементарной леммы, которая нам понадобится и далее. Лемма 1. Вектор Y = (Yi,..., Yn) является гауссовским в Жп тогда и толъ- п ко тогда, когда (г, Y) = ^2 ТкУк есть гауссовская случайная величина для всех k = l Доказательство. Пусть Y ~ Ы(а, С). Тогда в силу G) для любого у Е Ж г- iu(rY) г iLvr Y) [•/ \ 1 /п \\ Ее ^' ' = Еек ' ; = ехр<г(а,Ут}--(Сут,Ут) > = т.е. (г, Y) rsj |\|((а,т), (Ст,т)) (как обычно, i/r = {ут\,..., утп)). Обратно, пусть скалярная случайная величина (г, Y) ~ Ы(ат, а*2.). Тогда (см. (9) при п = 1) ат = Е(т,?) = ^ГткЕ?к = (т,Е?), k=l A0) Kk = l п п -/crmcov(F/c,Fm) = ^2 TkTmCkm = (Ст,т); k,m=l k,m=l здесь мы учли, что EYf < oo, j = 1,..., п (выбрав вектор т с компонентами тj = 1 и Y? = 0 при к ф 2-> видим, что Yj — гауссовская величина). Для любого у Е A1)
52 А. В. Булинский, А. Н. Ширяев Подставив в A1) v — 1 и взяв ат, и\ из A0), получим формулу G), означающую, что вектор Y является гауссовским. ? §5. Покажем, как описываются действительные гауссовские процессы. Для этого нам потребуется следующее определение. Определение 6. Действительная функция г = r(s, t), заданная на Т х Т, назы- называется неотрицательно определенной, если для каждого п G N и любого набора точек ti,..., tn G Т неотрицательно определены матрицы (г(?&, ?т))& m=i- Теорема 3. Пусть имеются произвольная действительная функция а = а(?) г/ симметричная неотрицательно определенная действительная функция г = r(s,t), где s,t G Т. Тогда найдутся вероятностное пространство (П,^, Р) и заданная на нем гауссовская случайная функция X = {X(t), t G Т}, такие, что EX(t) = a(t) и co\/(X(s),X(t)) = r(s,t) при всех s, t G Т. Доказательство. Для любого п е N и любого г = (ti,..., tn) e Tn введем на (W1, ^(W1)) меры QT, отвечающие характеристическим функциям вида G), где а = (a(ti),..., a{tn)) и с^ш — ^(tkjtrn)- Для мер QT очевидным образом выполнены условия (а) и (Ь) теоремы 5 главы I, обеспечивающие свойство согласованности этих мер. Поэтому по теореме Колмогорова (теорема 4 главы I) найдутся вероятност- вероятностное пространство и на нем случайная функция X, имеющая своими конечномерными распределениями гауссовские меры QT. Понятно из построения, что Е X(t) = a(t) и cov(X(s),X(t))=r(s,t).O Отметим, что для любой действительной случайной функции X = {Х(?), t G Т} с конечными моментами ЕХ2(?), где t G Т, при всех п G N, ti,...,tn G Т и Ai,..., An G Ж выполнено неравенство П , П П ч J2 cov(X(tk),X(tm))\k\m = cov (J2 xkX(tk), J2 AmX(tm) J ^ 0. A2) k,m=l ^k = l rn=l ' Ясно также, что cov(X(s),X(t)) = cov(X(t),X(s)). Поэтому условия, налагаемые в теореме 3 на функцию г = r(s, t), являются we только достаточными, но и weo^- ходимыми для существования действительной гауссовской случайной функции с ко- ковариационной функцией г = r(s,t). Следовательно, класс неотрицательно опре- определенных действительных функций совпадает с классом ковариационных функ- функций (действительных) гауссовских процессов. §6. Обратимся к рассмотрению комплекснозначных (или С-значных) гауссов- гауссовских процессов. Для этого дадим несколько определений. Определение 7. Комплекснозначная функцияR = R(s, t),s,t G Т, где Т — не- некоторое непустое множество, называется неотрицательно определенной, если для каждого nGN, любых t\,..., tn G Т и всех z\,..., zn G С п J2 zkzmR(tk,tm) ^0. A3) k,m=l
Гл. П. Процессы с независимыми приращениями 53 Определение 8. Случайная функция X = {X(t), t G Т} (действительная или комплекснозначная) называется L2-процессом, если Е |Х(?)|2 < оо для t G Т. Ко- Ковариационная функция г = r(s, t) этого процесса вводится формулой r(s,t) = E (X(s) - EX(s)) (X(t) - EX(t)), s,teT. A4) Определение 9. Пусть ?(?), f](t) — действительные процессы. Случайная функ- функция X = {X(t) = ?(?) + *^(t), ? G Т} со значениями в С называется гауссовской, если {(?(t),rj(t)), t е Т} — гауссовский процесс в R2, т.е. для любого п G N и всех ti,..., tn G Т вектор (?(ti),r)(ti),... ,?(tn),r)(tn)) является гауссовским. Теорема 4. Для всех s,t G Т класс неотрицательно определенных комп- лекснозначных функций R = R(s,t) совпадает с классом ковариационных функ- функций г = r(s,t) L2-процессов X = {X(t), t G Г} и, более того, совпадает с классом ковариационных функций комплекснозначных гауссовских процессов Доказательство. Если X = {X(t),t G Г} есть L2-nponecc, то для каждого GN, любых t\,..., tn G Т и всех z\,..., zn G С имеем к,тп=1 k=l 2 TzkX(tk) т. е. ковариационная функция г = r(s,i) является неотрицательно определенной. Пусть теперь R = R(s,t), s,t G T, — неотрицательно определенная функция. Положим Ri(s,t) = KeR(s, t) и R2(s,t) = Imi?(s, ?), s,t € Т. Для 2:^=1/^+ гг'/е, & = 1,... ,п, неравенство A3) после выделения действительной и мнимой частей примет вид /c,m=l те 1 m) + ^ R2(tk,trn)(ukUrn + УкУш) ^ 0. A5) Из A3) при п = 1 видим, что i?(t, t) ^ 0 для всех t ? Т. При п = 2, любых ti, ?2 ? Т и 2i, 22 G С согласно A3) имеем z1z2R(t1,t2) + z^Rfah) + Ы2Я(?2,?2) ^ 0. Следовательно, 2122^(^1,^2) + ^i^2^(^2, ?1) — действительное число. В частности, при 2i = 22 = 1 получаем, что i^(ti,t2) + R(t2,ti) G М для всех ti,t2 G Г. Вы- Выбрав 2i = 1, z2 = г, приходим к тому, что i^(ti,t2) — R(t2,ti) есть чисто мнимое число. Таким образом, R\(s, t) = Ri(t,s) ni?2(s,t) = -i?2(t,s) npns,t G Г. Иначе говоря, R(s,t) = R(t,s) для любых s,? G T, т.е. имеет место свойство эрмито- эрмитовой сопряженности функции R(s,t). Поэтому неравенство A5) можно переписать в виде (СА, А) ^ 0, где А = (и\,..., ип, v\,..., vn), (•, •) — скалярное произведение
54 А. В. Булинский, А. Н. Ширяев в М2п, а матрица С — симметричная неотрицательно определенная с действитель- действительными элементами, состоящая из четырех блоков: С = A6) к,т=1,...,п В силу замечания 4 главы I существует гауссовский процесс {(?(?), ??(?)), t G Т} со значениями в М2 при каждом t такой, что для всех п G N и ti,..., tn G Т век- вектор (?(?i),..., ?(?n), 77(^1)? • • • 5 v(tn)) ~ N@, С), где матрица ковариаций С задается формулой A6). Возьмем Тогда для s,t ? Т (принимая во внимание, что —R2(t, s) = ^2(^5 t)i sit ? Т) имеем 1 EI(s)l(t) = -(i?i(s,t) -iR2(t,s) +iR2(s,t) -\- R1(s,t)) = R(s,t). ? Пример 2. Пусть /(t) — комплекснозначная (в частности, действительная) функция, определенная на некотором множестве Т. Тогда функция R(s,t)=f(s)f(t), s.teT, является неотрицательно определенной. Действительно, справедливость условия A3) очевидна. Замечание 4. Если функция R = R(s, t) является неотрицательно определенной на множестве Т х Т, то при любой константе с ^ 0 этим же свойством обладает функция cR(s,t). Пусть Rn = Rn(s,t) — неотрицательно определенные функции N на Т х Т при п = 1,..., N. Тогда функция ^2 Rn(s, t) неотрицательно определена 71=1 на Г х Г. Если Rn(s,t) ->• R(s,t) при п ->• оо для каждых s,t G Г, то функция R = R(s, t) также является неотрицательно определенной (на Т х Т). Определение 10. Функция ip = ip(t), где t G Т С М, называется неотрица- неотрицательно определенной, если s — t ? Т при s,? G Т и функция R(s,t) = <p(s — t) неотрицательно определена. Пусть ^ — действительная случайная величина с характеристической функцией ср = ip(t),t G Ш. Легко видеть, что ср является неотрицательно определенной функ- функцией на Ж. Таким образом, получаем Пример 3. Следующие функции являются ковариационными функциями L2-npo- цессов, заданных на Ж: r(s, t) = cos(s - t), r(s, t) = exp{ia(s - t) — (s - tJcr2/2}, где aGl, a ^ 0; r(s, t) = exp{-A|s - t|}, r(s, t) = exp{A(exp{i(s - t)} — 1)}, где А > О; r(s,t) = (a-b\s-t\)l[_a/bia/b](s-t), где a ^ 0, 6 > 0.
Гл. П. Процессы с независимыми приращениями 55 Рассмотренный выше пример позволяет в силу теоремы 4 немедленно вводить гауссовские процессы с указанными ковариационными функциями. В главе VII мы вернемся к рассмотрению случайных процессов, ковариационные функции которых зависят от разности аргументов. § 7. Покажем, что свойство неотрицательной определенности функций тесно свя- связано с природой важного класса гильбертовых пространств. Определение 11. Функция К: Т хТ —>¦ С называется воспроизводящим ядром гильбертова пространства Н, состоящего из некоторых комплекснозначных функ- функций, определенных на множестве Т, если 1°. K(t, •) G Н для любого t G Г, 2°. (f,K(t, •)) = /(t) для любых/ G Яи? G Г, где (•, •) — скалярное произведение в Н. Теорема 5 (Ароншайн). Для того чтобы комплекснозначная функция К была воспроизводящим ядром некоторого гильбертова пространства, необхо- необходимо и достаточно, чтобы она была неотрицательно определена. Доказательство . Необходимость. Пусть К — воспроизводящее ядро. Для лю- любых п G N, t\,..., tn G Г, z\,..., zn G С имеем здесь |H|2 = (и,и), и G H. Достаточность. Пусть К — неотрицательно определенная функция на Т х Т. Рас- Рассмотрим Lin(K) — линейную оболочку функций K(t, •), t G Т. Для /, д G Lin(K), т. е. для функций вида п m f = / ajK(tji ' M 9 = / bqK(sq, • ), A8) где aj, 6g G С, tj, Sg ? T (j = 1,..., n; g = 1,..., m; n, ттг G N), положим В частности, n (f,K(sq, ¦)) = YajK(tj,sq) = f(sq). B0) Поскольку Sg — произвольная точка из Т, получаем, что для любой функции п f G Lin(K) справедливо свойство 2°. Кроме того, (/, /) = ^2 f(tj)uj, поэтому (/,/>= 0 для/ = 0.
56 А. В. Булинский, А. Н. Ширяев Рассмотрев для aGl дискриминант квадратного трехчлена (af + g,af + g) ^ 0, стандартным способом получаем неравенство Коши-Буняковского-Шварца: B1) где \\h\\ := (ft, ftI/2, а элементы /, g, ft G L'm(K). Заметим, что функция (•, •) обладает на Lin (if) всеми свойствами скалярного произведения. Пусть (/, /) = О для / Е Lin (if), тогда (/, д) = 0 при любых д Е Lin (if) в силу B1). Из B0) видим, что f(sq) = 0 для произвольной точки sqbT. Поэтому равенство (/, /) = 0 влечет соотношение / = 0. Остальные свойства скалярного произведения очевидны. Подчеркнем, что его задание формулой A9) корректно, т. е. (/, д) не зависит от способа представления / и д в виде A8). В самом деле, если для элементов Lin (if) имеем /(?) = /(?) и #(?) = #(?) при всех t Е Т, то !</,<?> - G,9)\ = К/ -7,9) + A,9- 9)\ ^ II/ - /II \Ы + 11/11 \\9 - 9 II, а||/ — /|| = ||# — # || = 0, как отмечалось ранее. Пополним Lin(if) по норме || • ||, рассматривая, как обычно (см., например, [35; гл. 2, § 3, п. 4]), классы "эквивалентных" фундаментальных последовательностей. Пусть {/п}п^1 — фундаментальная последовательность функций в Lin(if), т.е. ||/п — /т|| —> 0 при п, т —> оо. Используя B1), при каждом ? G Т имеем l/n(*)-/m(*)| = \(fn-fm,K(t, -)>K ||/n-/m||(#( где учтено, что \\К(?, • )||2 = (if (?, •), if (?, •)) = К(?, ?) по доказанному свойству 2° (для функций из Lin (if)). Таким образом, последовательность {fn(t)}n^i опреде- определяет единственную функцию /(?) = lim fn(t) на Т. п—>-оо Если фундаментальные последовательности {/п( • )}n^i и {#п( * )}n^i функций из Lin(if) определяют соответственно функции /(•) и д{ •), то полагаем Легко проверить, что этот предел существует и не зависит от выбора фундаменталь- фундаментальных последовательностей, задающих / и д. Пополнение Lin (if) обозначим Н', при этом функции / G Lin (if) естественно отождествляются с последовательностями {/п}п^1 такими, что /п = / при каждом п G N. Теперь нетрудно убедиться, что ii" является гильбертовым пространством, имеющим функцию if воспроизводящим ядром. ? Согласно теореме 4 класс неотрицательно определенных функций совпадает с классом ковариационных функций. Поэтому любую ковариационную функцию г = r(s,?); s,? G Т, можно рассматривать как воспроизводящее ядро некоторого гильбертова пространства Н со скалярным произведением (•,•). Следовательно, r(s,t) = (r(s, -),r(t, ¦)). B2) Из доказательства теоремы 5 мы даже знаем, каково "минимальное" Н, полученное пополнением линейной оболочки функций г(?, •), где t G Т. Для L2-nponecca X = {Х(?), ? G Т} обозначим Ь2[Х] замыкание в среднем квад- ратическом Lin(X), т. е. замыкание линейной оболочки величин X(t),t G Т.
Гл. П. Процессы с независимыми приращениями 57 Теорема 6 (Парзен). Пусть X = {X(t), t Е Т} есть центрирований L2-про- L2-процесс, заданный на вероятностном пространстве (П,^, Р), с нулевым средним и ковариационной функцией г = r(s,t), s,t Е Т. Пусть Н — "минимальное" гильбертово пространство с воспроизводящим ядром г = r(s,t) и скалярным произведением (•,•)• Тогда на (П, ^, Р) существует центрированный L2-про- L2-процесс Y = {Y(h), ft Е Н} такой, что X(t) = Y(r(t, •)) для каждого t G Г, (Y(h),Y(g)) = (h,g) для любых h,g G Н, где равенство случайных величин понимается (как всегда) п. н. и (^rj) = для ?,?7 G 1/2(П, i^, Р). При этом пространства L2[X] и Н изометричны. Доказательство. Для ft = Yl ckr(tk, •) E H,Ck E C, tj~ E T, & = 1,... ,n, n ^ 1, положим k=1 n У (ft) = ^cfcX(tfc). B3) m Для указанной функции ft и функции ^ = ^ dqr(sq, •), где dq EC, sg Е Т, q=l g = l,...,m, ?тг^1, очевидно, (Y(h),Y(g)) = fc=lq=l k=lq=l Легко видеть, что определение B3) корректно и продолжение изометрического ото- отображения ft ь->- У (ft) с Lin(X) на Н обладает искомыми свойствами. ? § 8. Винеровский процесс входит в класс гауссовских процессов, как показывает Теорема 7. Определение 3 винеровского процесса W = {W(?), t ^ 0} экви- эквивалентно следующему: 1°. W = {W(t), t ^ 0} — гауссовский процесс; 2°. EVF(t) = 0, t Е [0,оо); 3°. cov(VF(?), W(s)) = min{t, s}, где t,se [0, oo). Доказательство. Прежде всего заметим, что по теореме 3 процесс со свойства- свойствами 1°-3° действительно существует. В самом деле, из формулы F) видим, что вине- винеровский процесс имеет ковариационную функцию г (s,t) = min{s, ?},гдез, t E [0, оо). Поэтому свойство неотрицательной определенности этой функции следует из A2). Пусть для процесса W = {W(t), t ^ 0} справедливы условия 1°-3°. Проверим, что выполнены требования 1)—3) определения 3. Свойство 1) очевидно, так как DH^@) = cov(H/@), W@)) = min{0,0} = 0. Для установления 2) воспользуемся тем, что в Жп для гауссовского вектора Y ~ N(a, С) и матрицы А = (a/c?m)^ m=1 с действительными (неслучайными) элементами из G) и C) имеем AY ~ П(Аа,АСА*). B4)
58 А. В. Булинский, А. Н. Ширяев Пусть 0 ^ t\ < ¦¦¦ < tn. Выбрав соответствующую матрицу Л, преобразуем гауссовский случайный вектор (W@),W(h),..., W(tn)) ? Жи+1: W@) W(t!)-W(O) W(t2)-W(h) \ / 1 0 0 -1 10 0 -1 1 0 0 \ 0 0 0 -1 1/ W@) W(t2) \W(tn)J Согласно B4) вектор (VF(O), W(t\) — W@),..., W(tn) — W(tn-i)) являетсягауссов- ским. Напомним, что компоненты гауссовского вектора независимы тогда и только тог- тогда, когда матрица ковариаций диагоналъна. (Это следует из того, что вектор имеет независимые компоненты тогда и только тогда, когда его характеристическая функ- функция распадается в произведение характеристических функций компонент; для га- гауссовского вектора надо воспользоваться формулой G).) В рассматриваемом случае это свойство "диагональное™" выполнено, поскольку соу(W(tk+1) - W(tk),W(tm+1) - W(tm)) = = min{?fc+i,?m+i} - min{tk+i,tm}- - min{tfe,tm+i} + min{tfe,tm} = 0 при к Ф т. B5) Следовательно, у построенного процесса приращения независимы, т.е. справедли- справедливо свойство 2). Величина W{t) — W(s) имеет нормальное распределение (вектор, составленный из части компонент гауссовского вектора, является гауссовским) с ну- нулевым средним, и аналогично B5) находим, что D(W(t) — W(s)) = t — s при t ^ s. Таким образом, свойство 3) также установлено. Обратно. Пусть для процесса!^ = {W(t),t ^ 0} выполнено определение 3. Тогда справедливо F), т.е. условия 2° и 3°. Гауссовость процесса W, т.е. 1°, получаем, снова воспользовавшись B4). ? § 9. Покажем, как можно дать явную конструкцию винеровского процесса, кото- который будет иметь с вероятностью 1 непрерывные траектории. Вначале построим требуемый процесс на отрезке [0,1]. С этой целью введем функ- функции XaapaHk(t),t е [0,1], к = 1, 2 ...: -1 A/2,1] Hk(t) = где п ] ) — П — 1 = 2-п(к-2п -1) (см. рис. б для к > 2).
Гл. П. Процессы с независимыми приращениями 59 7п/2 нк (t) W2 Рис. 6 Система функций {Н^} является полной и ортонормированной в пространстве L2[0,1] с мерой Лебега и скалярным произведением f,9)= Г Jo (f,9)= / №g(t)dt, f,geL2[Q,l]. Jo Действительно, ортонормированность системы {Н^} очевидна, а ее полнота выте- вытекает из того, что с помощью линейных комбинаций функций Н^ можно записать ин- индикаторы промежутков с двоично рациональными концами. Так, например, Я2)/2, 2)/2, l - Я2)/2, )/2, ..., ДЛЯ 2" < к Следовательно, любую функцию / G Ь2[0,1] можно представить в виде B6) к=1 где ряд в правой части B6) сходится в L2 [0,1]. Отметим также, что согласно равен- равенству Парсеваля оо ?*>• B7) k = l По функциям Хаара определим теперь функции Шаудера (см. рис. 7): Sk(t) = [ Hk(y)dy = (l[0,t],Hk), t G [0,1], k G N. Jo y-n/2 - 1 1 t Рис. 7
60 А. В. Булинский, А. Н. Ширяев Нам понадобятся далее следующие две леммы. Лемма 2. Пусть числовая последовательность {о>к\^=\ такова, что а& = оо = О(к?) при к —у оо для некоторого г < 1/2. Тогда ряд ^2 ctkSk(t) сходится k=i равномерно на [О,1] и, следовательно, задает непрерывную на [О,1] функцию. Доказательство. Достаточно показать, что Rm := sup У2 \ak\Sk(t)-> 0, га-^ оо (Sk (t) ^ 0 при любых k е N и t ^ 0). Имеем \а,к\ ^ ск? для каждого к ^ 1 и некоторого с > 0. Поэтому для всех t е [0,1]ип ^ 1 мы учли здесь, что t попадает только в один из непересекающихся носителей функций Sk, 2П < к ^ 2n+1, и то, что 0 ^ Sk(t) ^ 2~п/2~1 для рассматриваемых /с. По условию г < 1/2. Следовательно, Rm ^ c2? Y^ 2"nA/2-?) ^0 при т -+ оо. ? Лемма 3. Пусть на некотором вероятностном пространстве (П, ^, Р) за- (необязательно независимые) величины ?& ~ N@,1), /с G N. Тогда для любого с > 21/2 и Р-почти всех со G П можно указать Щ = 7Vo(^5c) G N такое, что |^fc(^)| < c(lnA^I/2 при всех k^N0. Доказательство. Для ? ~ N@,1) и любого ж > О ж) = B7Г)-1/2 / ехр{-у2/2} dy = B7т)-1/2 / (-1/у) <1{е~У /2) = •/Ж «/Ж B8) Следовательно, при любом х > О Р(|?| ^ х) ^ х-1B/пI/2е-х2/2. B9) Поэтому для с > 21/2 j: Р(\Ы 2 c{\nkf'2) < c-1B/7rI/2 53 fc-^^anfc)/2 < оо. /с>2 k>2
Гл. П. Процессы с независимыми приращениями 61 Отсюда требуемое утверждение следует по лемме Бореля-Кантелли (см. [85; т. 1, с. 327]), утверждающей, что если YlPi^k) < оо, то бесконечное число событий Ак произойдет лишь с нулевой вероятностью, т.е.Рп] |J Ак) = 0. ? Полезно отметить, что наряду с B8) верно соотношение Р(?^я) -жBтг)/2е"ж2/2 при х -юо. C0) §10. Перейдем непосредственно к описанию явной конструкции винеровского про- процесса (на отрезке [0,1]) с помощью последовательности независимых стандартных гауссовских величин. Теорема 8. Пусть {^к}к^1 — последовательность независимых случайных величин, имеющих стандартное нормальное распределение N@,1) и заданных на некотором вероятностном пространстве (П, ^, Р). Положим дляг Е [0,1], uj Е П оо ~W(t,Ld) = у ^ky^J^kitj- (^1) k = l Тогда W = {W{t), t E [0,1]} есть винеровский процесс на [0,1], к тому же имеющий с вероятностью единица непрерывные траектории. Доказательство . Равномерная на [0,1] сходимость п. н. ряда в C1) и непрерыв- непрерывность п.н. траекторий процесса W = {W(t),t E [0,1]}, построенного с помощью C1), следует из лемм 2 и 3. Проверим теперь выполнение требований 1°, 2° и 3°, содер- содержащихся в теореме 7. Для этого покажем, что ряд C1) не только п.н. равномерно сходится на [0,1], но и при каждом t E [0,1] сходится в среднем квадратическом. п С этой целью обозначим Zn(t) = J^ ^kSk(t)- Для n, m E N и t E [0,1] k = l п+т E\Zn+m(t) - Zn(t)\2 = Е k=n+l п+т п+т п+т к=п+1 1=п+1 к=п+1 оо оо оо Имеем Yl Sk(t) ^ S B~fc/2~1J < оо, поэтому полнота пространства Ь2(п) = = L2(Q, ^, Р) обеспечивает при каждом t G [0,1] наличие 1/2-предела Z(t) у после- последовательности {Zn(t)}, т.е. Е \Zn(t) — Z(t)\2 —>¦ 0 при п —>- оо. Из C1) мы знаем, что Zn(t) —>¦ W(t) п.н. при п —>- оо для каждого t G [0,1]. Отсюда вытекает, что Z(t) = W{t) п.н., поскольку (см., например, [85; т. 1, с. 328]) как сходимость п.н., так и сходимость в среднем квадратическом влекут сходимость по вероятности, а пре- предел по вероятности последовательности случайных величин определен однозначно с точностью до эквивалентности. Таким образом, Zn(t) —У W(t),n —У оо, для t G [0,1].
62 А. В. Булинский, А. Н. Ширяев Найдем среднее и ковариационную функцию процесса W, определенного в C1). Поскольку Е Zn(t) = 0 для всех п Е N и t E [0,1], то \EW(t)\ = \EW(t) - EZn(t)\ sC (E\W(t) - Zn(t)\2I/2 -Ю, n -»• оо. Следовательно, Е W(?) = 0 при всех t E [0,1]. Принимая во внимание непрерывность скалярного произведения, получаем, что для любых s, t E [0,1] В силу независимости элементов последовательности {?,k}k^i и равенства Е ^ = 1, к Е N, находим, что п оо (Zn(s),Zn(t))L2(n) = EZn(s)Zn(t) = Согласно B7) оо Поэтому cov(H/(s), W(t)) = Покажем, наконец, что построенный процесс W является гауссовским, для чего воспользуемся леммой 1. Возьмем т — [т\,..., тп) G W1, t\,..., tn G [0,1] и рассмотрим случайную вели- п чину Y = J2 rrnW(trn). Ясно, что Y = m=l fc = l fc = l п где 6fc = bfc(ri,..., rn; ?ь ..., tn) = J] rmSk{tm), а ряд в правой части C2) схо- 771=1 дится как п.н., так и в L2(Q) (как конечная сумма рядов, обладающих этими свой- N N ствами). Заметив, что Удг = J2 ^к^к ~ N@, сг^), где сг^ = J2 Ь|, получим, что к=1 к=1 EYjy = a^j —У EY2 = а2 (сходимость элементов в Ь2(п) влечет сходимость их норм). Учитывая сходимость по распределению случайных величин Удг к случайной величине Y (обозначения: Уд/- —У Y или Уд/- —У У) при N —у оо, вытекающую как из сходимости п. н., так и из сходимости в L2 (Q), находим, что <PYn(\) = ехр{-а%\2/2} -* ехр{-а2\2/2} = у>у(А), A G Ж, т.е. У- N(O,a2). Итак, построенный процесс W является гауссовским, что завершает доказатель- доказательство теоремы 8. ? Из теоремы 7 главы I вытекает, что W = {W(?), t G [0,1]} (и любая его модифи- модификация) является 3* | Й§(С([0,1]))-измеримым случайным элементом.
Гл. П. Процессы с независимыми приращениями 63 Определение 12. Распределение вероятностей на сг-алгебре Й§(С[0,1]) постро- построенного случайного элемента W = {W{t), t G [0,1]} называется мерой Винера, или винеровской мерой, и обозначается W. §11. Перейдем к построению винеровского процесса W на [0, оо). Пользуясь теоремой 2 главы I, зададим на некотором (полном) вероятностном пространстве (П, j^, P) последовательность независимых случайных элементов Wn = {Wn(t), t G [0, l]},n G N, таких, что Wn(-,u) G С[0,1]длясс; G unPwn = WHa3S(C[0,1]). Требуемый процесс W на [0, оо) определим теперь с помощью непрерывного "скле- "склеивания" процессов Wn, т.е. положим ш) для t? [0,1), для [k,k t 2 Рис. 8 Теорема 9. Процесс W, определяемый формулой C3), есть винеровский про- процесс на [0, оо). Бее траектории этого процесса непрерывны. Доказательство . Непрерывность процесса W очевидна по построению. Ясно также, что W@) = 0. Покажем, что W есть процесс с независимыми приращениями, у которого W(t) - W(s) ~ N(O,t - s) приО ^ s < t < оо. Если s, t G [fc, fc + 1) для некоторого fc ^ 0, то P7(t) - W(s) = Wk+i(t -k)- Wk+1(s - k) ~ N@, t - s). Если же s G [k, k + 1) и t G [m, ттг + 1), где k < m, то k<l<m (см. рис. 9). Поэтому VF(?) - W(s) = VF(A: + 1) - W(s) + J^ ( k<l<m k<l<m
64 А. В. Булинский, А. Н. Ширяев т га + 1 Рис. 9 Заметим, что величины Ofe> • • • > Cm независимы и Q ~ N@, сг2), г = &,..., т, поэ- тому С/с + * * * + Cm ~ N ( 0, ^ ст|). Тем самым, распределение W(t) — W(s) является v i=k J нормальным N@, t — s). Аналогичные рассуждения показывают, что процесс W имеет независимые при- приращения. Для доказательства надо учесть, что измеримые функции от непересека- непересекающихся наборов независимых величин будут независимы. ? Далее считаем, что все рассматриваемые винеровские процессы имеют п.н. не- непрерывные траектории. Дополнения и упражнения Обратимся к несколько более подробному рассмотрению введеного класса про- процессов с независимыми приращениями. Два важнейших представителя этого класса — броуновское движение и пуассоновский процесс — уже были определены в гла- главе П. 1. Найдите ковариационную функцию пуассоновского процесса, имеющего по- постоянную интенсивность Л. Сопоставьте полученный ответ с ковариационной функцией винеровского процесса. 2. Пусть г = r(s,t) — неотрицательно определенная функция (s,t Е Т, где Т — некоторое множество). Существует ли негауссовский процесс X = {Xt, t еТ} такой, что cov(Xs, Xt) = r(s, t) при всех s,t G Г? 3. Докажите, что если Рп(^ъ • • • •> zm) — многочлен степени п (от m перемен- переменных) с положительными коэффициентами и r^ (s, t) (k = l,...,m;s,iGT) — ковариационные функции, то Рп (г\ (s, t),..., rm (s,t)) также является ковари- ковариационной функцией. 4. Для L2-nponecca X = {Xt, t G Г}, заданного на (П, ^, Р), обозначим L2[X] замыкание в пространстве L2(Q, ^, P) линейной оболочки всех величин Xt, t G Г. Докажите, что если X — гауссовский процесс, то Ь2[Х] есть гаус- совская система, т. е. для любого п G N и любых Y\,..., Yn G Ь2[Х] вектор (Yi,..., Yn) имеет гауссовское распределение. 5 (свойство автомодельности). Пусть W = {W(?), t ^ 0} — винеровский про- процесс и с — положительная константа. Докажите, что процесс X = {X(t) = = -k^W(ct), t ^ 0} является винеровским процессом. 6. Докажите теорему 8, проверив исходное определение винеровского процесса, т.е. докажите, что процесс, задаваемый формулой C1), имеет независимые приращения. В силу теоремы Дуба (см. приложение 5) этот процесс с не- непрерывными траекториями автоматически будет гауссовским.
Гл. П. Процессы с независимыми приращениями 65 Рассмотрим случайное блуждание в Ж171 из примера 1 главы I, т. е. процесс част- частных сумм S = {Sn, n ^ 0}, где Sn = So + ?i + • • • + ?n, n ^ 1, и ?i, ?2> • • • — неза- независимые одинаково распределенные векторы. Будем считать So = 0. Во многих ситуациях бывает интересно выяснить, какую часть пространства Ж171 заполняют траектории блуждания, с какой вероятностью достигается то или иное множество и т. д. Классический пример проблем такого рода, изучаемый в курсе теории веро- вероятностей, — это задача о разорении игрока (см. [85; т. 1, с. 109], а также пример 7 главы IV). К излагаемым ниже результатам о случайных блужданиях будет полезно вернуться после прочтения главы VI о марковских процессах. Для описания интересующих нас случайных блужданий понадобится ряд новых понятий. Определение 13. (Случайной) мерой пребывания процесса S = {Sn, n ^ 0} в борелевском множестве В из Ж171 называется величина Соответствующая этой мере \± — /i( •) мера интенсивности v — i/( •) определяется формулой ~ ев), ве Определение 14. Обозначив V?(x) открытый шар в Ж171 с центром в точке х и радиусом г > 0, введем следующие множества: R = (| {х G Ж171: /jl(V?(x)) = оо} — множество возвратности, е>0 М — С\ {х G Ж171: i/(V?(x)) = оо} — множество возвратности в среднем, А— (| {х G Ж171:1/A4 (ж)) > 0} — множество достижимости. Теорема 10 (о дихотомии; см., например, [146; с. 137]). Для случайного блуж- блуждания S в Мт, определенного согласно A.9), реализуется только одна из следу- следующих двух возможностей: 1) R — M — Au это множество является замкнутой подгруппой в Мт, 2) R = М = 0 и \Sn\ —У оо п. н. при п —> оо. Определение 15. Случайное блуждание называется возвратным, если в при- приведенной теореме 10 имеет место свойство 1) и невозвратным (или транзиент- ным), если выполнено свойство 2). 7. Докажите, что если S = {Sn, п ^ 0} — возвратное случайное блуждание в Ж171, то при каждом k G N таким же будет и блуждание S^ = {Snk,n ^ 0}. Приведем критерий возвратности случайного блуждания, полученный Чжу- ном и Фуксом.
66 А. В. Булинский, А. Н. Ширяев Теорема 11 (о возвратности; см., например, [146; с. 139]). Случайное блужда- блуждание S = {Snj п ^ 0} в Ж171 с So = 0, Sn = ?i + • • • + ?n, n ^ 1, возвратно тогда и только тогда, когда для любого г > 0 SUP / Re(^ -77TV W = cx), где /(?), t G Mm, — характеристическая функция вектора ?i. 8. Пусть случайная величина ?i имеет симметричное устойчивое распределе- распределение в Ж с показателем a Е @, 2], т. е. пусть /(*) = Ее^1=е"с1*1а, tGM, О 0. C5) Докажите, что такое а-устойчивое случайное блуждание возвратно при а ^ 1 и транзиентно при а < 1. 9. Пусть S = {5n, n ^ 0} — случайное блуждание в Ж171 (т ^ 2) такое, что компоненты вектора^! независимы и имеют а-устойчивое распределение (см. упражнение 8). Исследуйте, при каких значениях а случайное блуждание возвратно и при каких — транзиентно. Симметризацией случайного блуждания S = {5П, п ^ 0} в Ж171 называется по- последовательность S = {5n, n ^ 0} с таким же распределением, как у {Sn — S'n, п ^ 0}, где S'n = {5^,п ^ 0} — независимая копия последовательности {Sn} (можно считать, что для этого строится независимая копия {^, п ^ 0} последо- последовательности {?п, п ^ 0} и S? = 0, 5^ = Е ^-, n G N). 10. Докажите, что если возвратно случайное блуждание S = {5n, n ^ 0}, то возвратно и симметризованное случайное блуждание S = {5n, n ^ 0}. 11. Докажите, что случайное блуждание 5 = {5n, n ^ 0} в Мт возвратно, если a) n~1Sn —У 0 при п —У оо, когда т = 1; b) E ^i = 0 и Е |?i |2 < оо, когда т = 2. Постройте примеры, показывающие, что эти достаточные условия возврат- возвратности не являются необходимыми. Определение 16. Носителем произвольной меры Л на борелевской сг-алгебре топологического пространства S называется наименьшее замкнутое множество F С S такое, что A(S \F)=Q. 12. Пусть S = {5n,n ^ 0} — случайное блуждание в Ж, причем ?i имеет сим- симметричное невырожденное распределение с ограниченным носителем. Дока- Докажите, что такое случайное блуждание возвратно. Определение 17. Эффективной размерностью случайного блуждания A.9) называется размерность подпространства Мт, порожденного линейной оболочкой носителя меры Р^. 13. Докажите, что если эффективная размерность случайного блуждания не меньше трех, то это случайное блуждание транзиентно.
Гл. П. Процессы с независимыми приращениями 67 Назовем простейшим симметричным случайным блужданием в Ж171 такое блуждание, для которого P(fi = е») = Р(& = -е{) = Bm)-\ г = 1,..., m, где i-я координата вектора е^ равна 1, а остальные равны 0. Это блуждание может рассматриваться как имитация движения "частицы", с равной вероятностью смеща- смещающейся на единицу в любую сторону вдоль любой координатной оси. Из результата упражнения 13 вытекает хорошо известный критерий Пойа (см. [78; т. 1, с. 374]) о том, что простейшее симметричное случайное блуждание в Ж171 невозвратно при т ^ 3. Там же ([78]) доказано, что при т = 1 и т = 2 такое блуждание возвратно (в этой связи В. Феллер замечал, что "все дороги ведут в Рим"). 14 (сравните с приведенным выше результатом о случайном блуждании). До- Докажите, что траектории частицы, совершающей простейшее симметричное случайное блуждание в Мт, с вероятностью 1 имеют бесконечное число само- самопересечений, т. е. с вероятностью 1 частица будет возвращаться в положения, которые она занимала ранее. Важную область применений случайные блуждания находят в статистическом последовательном анализе Валъда. Рассмотрим Х\, X<i,... —независимые одина- одинаково распределенные случайные векторы в Мт, относительно которых имеются две гипотезы: Но: плотность распределения вероятностей Х\ равна ро (х), х Е Ж171, Hi: плотность распределения вероятностей Х\ равна р\ [х), х Е Ж171. Требуется по наблюдениям Xj, j = 1,..., различить эти гипотезы. Считая плотнос- плотности pi (ж), г = 0,1, положительными функциями на Ж171, введем логарифм отношения правдоподобия Суть подхода Вальда к проблеме различения рассматриваемых гипотез состоит в следующем. Выбираются две "границы" а, 6, где а < 0 < Ъ. До тех пор, пока точка (п, 5П) лежит в полосе {(х,у): х ^ 0, а < у < 6}, решение о различении гипотез не принимается (в отличие от статической схемы Неймана-Пирсона допускается воз- возможность продолжения наблюдений). Решение в пользу гипотезы Hi принимается в тот момент, когда случайное блуждание впервые покидает полосу через верхнюю границу (т.е. в тот момент, когда Sn ^ Ь), а в пользу гипотезы Но — тогда, когда выход происходит через нижнюю границу. Мы не останавливаемся здесь на том, из каких соображений выбираются эти границы, а также на обобщениях рассмотрен- рассмотренного примера, отсылая, например, к монографии [84]. 15. Докажите, что если P(?i ф 0) > 0, то одномерное случайное блуждание с вероятностью единица выходит из всякой полосы (а, Ь) с границами а < 0 < Ъ (напомним, что So = 0).
68 А. В. Булинский, А. Н. Ширяев Случайным блужданиям посвящена обширная литература, см., например, [74], [78; т. 1, гл. 14; т. 2, гл. 12], [123; гл. 3], [146; гл. 8]. О применениях случайных блуж- блужданий в математической теории запасов см., например, [1]. Упомянем также интен- интенсивно развивающееся направление в теории случайных процессов — исследование случайных блужданий в случайной среде. Главный смысл моделей такого рода со- состоит в том, что сами вероятности переходов частицы в новые положения случайны и зависят от того, где находится изучаемая частица (см., например, [141]). Имеются и более сложные модели, относящиеся к случайным средам, связанные, например, с магнитным полем (см. [166] и там же библиографию). Напомним, что стохастическая непрерывность процесса в точке определяется со- согласно A.56), а стохастическая непрерывность на множестве означает стохастичес- стохастическую непрерывность в каждой точке этого множества. Аналогично известной теореме о разложении меры Лебега—Стилтьеса в сумму дискретной, абсолютно непрерывной и сингулярной компонент (см., например, [35; с. 410]) имеет место следующая теорема (см., например, [73; с. 163]). Теорема 12 (П. Леви). Всякий случайный процесс X = {X(t), t ^ 0} с неза- независимыми приращениями допускает представление X(t) = m(t) + Y(t) + Z(t), t ^ 0, где m(t) — неслучайная функция, Y = {Y(t), t ^ 0} и Z = {Z(t), t ^ 0} — независимые процессы с независимыми приращениями, причем Y — стохас- стохастически непрерывный процесс на [0,оо), a Z — чисто дискретный процесс, т. е. процесс вида ~ и** ^ *> + Е^+1{** < *>. здесь Т = {?&, k G N} — некоторое счетное подмножество [0,оо), а {?к ,^, к G N} — совокупность независимых случайных величин. Обратимся к специальному классу стохастически непрерывных действительных процессов, имеющих независимые приращения. Начнем с трех простых упражнений. 16. Пусть X = {Xt, t G [а, Ь]} — действительный процесс, стохастически не- непрерывный в каждой точке t G [а, Ь]. Тогда процесс X является равномер- равномерно стохастически непрерывным на [а,Ь], т.е. для любых e,j > 0 найдется А = А (г, 7) > 0 такое, что e)^j, если |s - t\ ^ A, s,te[a,b]. C6) Кроме того, процесс X ограничен по вероятности, т. е. lim sup P(\Xt\ ^c) = 0. C7)
Гл. П. Процессы с независимыми приращениями 69 17 (неравенство Оттавиани). Пусть Yi,..., Yn — независимые действительные величины такие, что для некоторых a Е [0,1), г ^ О P(\Sn-Sk\ >r) ^апри* = 1,...,п, C8) C9) к где Sk = ^2 Yj. Докажите, что при всех с ^ О max 1 — а 18. Пусть процесс X = {Xt, ? Е [0, оо)} имеет независимые приращения и сто- стохастически непрерывен на [0, оо). Докажите, что Р( sup |Xt|<oo)=l, D0) 4t?Mn[0,oo) У где М — множество двоично-рациональных точек прямой. Определение 18. Назовем пространством Скорохода D[0,oo) совокупность действительных функций, заданных на [0, оо), непрерывных справа и имеющих ко- конечные пределы слева в каждой точке t Е @, оо). Это пространство можно превратить в польское с помощью введения специаль- специальной метрики (см. [2, § 14]). Функции из этого пространства часто называют cadlag функциями (от французского continue a droite, limite a gauche — непрерывный спра- справа, имеющий предел слева; используется также обозначение rcll, происходящее от аналогичного английского термина: right continuous, left-hand limits). С помощью упражнений 16-18, а также доказываемого далее неравенства Дуба (лемма б главы IV) можно установить (см. [39; ч. 1, с. 70-72]), что верна Теорема 13. Стохастически непрерывный на [0, оо) процесс с независимыми приращениями имеет модификацию с траекториями из пространства D[0, оо). 19. Докажите, что пуассоновский процесс стохастически непрерывен на [0, оо). Конечномерные распределения стохастически непрерывных действительных про- процессов с независимыми приращениями описываются с помощью следующих двух теорем, принадлежащих соответственно А. Я. Хинчину и П. Леви. Теорема 14 (формула Хинчина; см., например, [39; ч. 1, §16]). Пусть стохас- стохастически непрерывный действительный процесс X = {Xt, t ^ 0} имеет незави- независимые приращения. Тогда для любого t ^ 0 и всех A G 1 характеристическая функция Е exp{i\Xt} = exp< t ( ia\ + / g(X,x) u(dx) ) >, D1) I V J J где a G M, v — конечная мера на ё%(Ж) и (ехр{гЛж}- 1 - iAsinx)(l + ж2)/ж2, х ф 0, , = 0. При этом представление D1) единственно, т. е. в этой формуле число а и мера v определены однозначно.
70 А. В. Булинский, А. Н. Ширяев Теорема 15 (формула Леви; см., например, [39; ч. 1, §16]). Если выполнены условия предыдущей теоремы, то для любого t ^ 0 и всех A G 1 exp{iXXt} = expltlibX + / (eiXx - 1 - iXsinx) u(dx)\ \, IV 2 J-oo ) J D3) где b G Ж, a ^ 0, v — мера на ё$(Ж) (принимающая, вообще говоря, бесконечные значения) такая, что 2 / J — х2 —2*W<oo. D4) При этом Ъ,о uv, фигурирующие в представлении D4), определены однозначно. 20. Покажите, как из формулы Хинчина выводится формула Леви и наоборот. Какова при этом связь между параметрами a, v шЪ, и2, и? 21. Как выглядит формула Леви для процесса Wx(t) = х + W(t), t ^ 0, где W = \W(t), t ^ 0} — винеровский процесс, а х G Ш Обратимся к рассмотрению процессов с независимыми приращениями, заданных наМ_|_ = [0, оо) (и некотором вероятностном пространстве (П, 3*, Р)) ипринимающих значения в Ж171. По аналогии со случаем т = 1 вводится понятие cadlag-функции /: М+ —> Ж171. Определение 19. Говорят, что случайный процесс X = {Xt, t ^ 0} со значени- значениями в Мт является cadlag-процессом, если его траектории (п. в.) являются cadlag- функциями. Процесс X имеет скачок в фиксированной точке t > 0, если P(Xt ф XtJ) > 0, где Xt-((j) = ]ипХ3(ш). stt 22. Пусть X = {Xt, t ^ 0} — стохастически непрерывный процесс с незави- независимыми приращениями, принимающий значения в Ж171. Докажите, что у X есть cadlag-модификация, не имеющая скачков в фиксированные моменты (ср. с теоремой 13). Определение 20. Процессом Леви X = {Xt, t ^ 0} со значениями в Ж171 назы- называется cadlag-процесс с независимыми приращениями такой, что Xq = 0 п. н. и при каждом h > 0 распределения величин Xt+h — Xt не зависят от значения t G M+. Процессы Леви с неубывающими покоординатно траекториями (Хо = 0, Xs ^ Xt для s ^ t) принято называть субординаторами. Имеются и другие, родственные, определения процессов Леви. Например, требу- требуют стохастическую непрерывность процесса X, которая (см. упражнение 22) обеспе- обеспечивает, что X — cadlag-процесс. Процессом Леви называют и процесс Y = {Xt + Уо, t ^ 0}, где Уо — случайный вектор, не зависящий от описанного выше процесса {Xt, t ^ 0} со значением Xq = 0. Полностью описаны все непрерывные по вероятности сас?/ш;-процессы с незави- независимыми приращениями (со значениями в Ж171), в частности, процессы Леви. Эти ре- результаты используют интегрирование по пуассоновской случайной мере (см., напри- например, [146; гл. 11]). Здесь же сформулируем лишь следующий результат о распреде- распределениях приращений процессов Леви.
Гл. П. Процессы с независимыми приращениями 71 Теорема 16 (формула Леви-Хинчина; см., например, [86; с. 238]). Пусть про- процесс Леей X = {Xt, t ^ 0} принимает значения в Ж171. Тогда для любых t ^ 0, и Е Ж171 справедливо представление Е ехрЩщXt)} = ехр{?Ф(^)}, D5) где Щи) = i(b, и) - {-^^ + [ (е^'ж> - 1 - i(u, x)l{]xK1}) v(dx). D6) Однозначно определенные характеристики (b,C,iy) таковы: Ъ — вектор в Мт, С — симметричная неотрицательно определенная матрица порядка т, v-мера на ?$(Мт), называемая мерой Леей. При этом ^({0}) = 0 и / (\х\2 Al)v(dx) < оо, D7) где знак "Л" обозначает взятие минимума (имеются случаи, когда г/(Мт) < оо ии(Жт) = оо). Формула D5) возможна с функциями ^(и), допускающими представления, отлич- отличные от D6) (см., например, [86; с. 240]). П. Леви и А. Я. Хинчин, как мы видели, предложили разные формулы для харак- характеристических функций безгранично делимых распределений, но каждая из них мо- может быть выведена из другой. Этим формулам предшествовала формула, установ- установленная А. Н. Колмогоровым для характеристической функции безгранично делимой случайной величины, имеющей конечную дисперсию. Интересно отметить, что пер- первоначально устойчивые и безгранично делимые распределения возникли в теории суммирования независимых действительных случайных величин. Фундаменталь- Фундаментальный вклад в эту область внесли П. Леви, Д. Пойа, А.Я. Хинчин, Б. де Финетти, А. Н. Колмогоров, Б. В. Гнеденко. Ознакомиться с этим направлением исследований можно по книгам [20, 27, 50, 78, 158]. Для броуновского движения W = {W(t), t ^ 0} при любом с > 0 п.н. верно следующее равенство lim sup \W(t + c\nT)-W(t)\/\nT = V2c~, D8) являющееся аналогом закона Эрдёша-Реньи для сумм независимых одинаково рас- распределенных слагаемых. Соотношение D8) допускает уточнение. Для этого зафиксируем некоторое с > О и при Т ^ с In T определим процесс f (Г) = sup (W(t + chiT)-W(t)), D9) O^t^T-clnT где W = {W(t),t ^ 0} — одномерное броуновское движение.
72 А. В. Булинский, А. Н. Ширяев Теорема 17 ([170, 181]). Для процесса D9) с вероятностью единица выпол- выполняются соотношения lim sup (f (T) - л/2с In T) / In In T = у/ф, E0) Т—>>оо liminf (f(T) - л/2с In Г)/in In Г = -у/с/8. E1) Соотношение E0) получили Й. Ортегаи М. Вщебор (см. [170; теорема 1]), а соот- соотношение E1) — П. Ревес (см. [181; теорема 2.1]). Случайным процессам с независимыми приращениями, в том числе процессам Леви, посвящена обширная литература (см., например, [73], [99] и там же биб- библиографию). Отметим также, что имеются обобщения процессов с независимыми приращениями на случай, когда значения процессов лежат в более общих линейных пространствах, нежели Ж171 (см., например, [16; т. 2, гл. 4]). Теперь обсудим некоторые вопросы, касающиеся непрерывности траекторий слу- случайных функций. Начнем со следующего классического результата Колмогорова. Теорема 18 (см., например, [12; с. 124]). Пусть X = {X(t), t G [а, Ь]} — действительный случайный процесс такой, что для некоторых а,е > 0 и С = = С(а,е) >0 E\X(t)-X(s)\a ^C\t-s\1+? при s,te[a,b\. E2) Тогда у процесса X существует непрерывная модификация. 23. Покажите, что если в условии E2) положить е = 0, то утверждение теоре- теоремы 18, вообще говоря, не обязано выполняться (указание: рассмотрите пуас- соновский процесс). Теорема Колмогорова о непрерывной модификации получила многочисленные обобщения. Рассмотрим некоторые из них. Напомним, что для множества U С Т (U может совпадать с Т), где (Т, S) — псев- псевдометрическое пространство, е-сетью называется множество S? С Т такое, что для любого t G U найдется элемент s G S?, для которого S(s, t) ^ e. Другими словами, U покрывается замкнутыми шарами B?(s) = {teT: S(s,t) ^ г}, se S?. E3) Если U обладает конечной г-сетью, то назовем минимальной е-сетью любую та- такую его г-сеть 5™Ш(С/), что #EJQm(t/)) минимально, где #(V) обозначает число элементов конечного множества У. Положим N§(г, U) = #E'^Qm(t/)), подчеркивая зависимость и от псевдометрики 5. Число H6(e,U)=]nN6(e,U) E4) называется г-энтропией множества U (часто логарифм в E4) берется по основа- основанию 2).
Гл. П. Процессы с независимыми приращениями 73 Следуя [153], введем два условия на пространство (Т, S). Пусть существуют такие d G N и а > О, что для каждого U С Т с диаметром Djj := sup{?(s, t)\ s,t G U} < < oo и любого г > О N6(e, U) ^ mzx{a(Du/s)d, 1}. E5) Пусть существует такое Ъ > О, что для любой минимальной г-сети 5^Qm(T) #(S?min(T) П Bb?(t)) ^ b при всех t G S?min(T), E6) где B$?(t) определяется согласно E3). Теорема 19 (см. [153; с. 134]). Пусть X = {X(t), t G T} — случайная функция со значениями в польском пространстве (S,p) при каждом t G Т, заданная на некотором (П,^, Р) г/ псевдометрическом пространстве (T,S), удовлетво- удовлетворяющем условиям E5) г/ E6). Предположим, что для некоторых 7 ? @,1), а > d/7 и /? > О E(p(X(t),X(s)))a ^/3(S(t,s))ai при всех s,teT. E7) Тогда у функции X существует модификация Y = {Y(t), t G Г}, имеющая непрерывные траектории, такая, что при любом Л G @,7 — d/a) lim sup p(Y(t),Y(s))/5x(t,s) =0 n. w. E8) каждого to G T } d/a - l)a. E9) Замечание 5. Если U — замкнутый параллелепипед в Rd или сфера в Md+1 (мет- (метрики евклидовы), то условия E5) и E6) выполнены. Поэтому из теоремы 19 выте- вытекает теорема 18, причем с дополнительными утверждениями о локальных свойствах траекторий модификации. Для того, чтобы ввести более общие моментные условия на случайную функ- функцию X, рассмотрим положительную строго возрастающую выпуклую функцию ф, определенную на М+ = [0, оо) и такую, что ip@) ^ 1. Обратную к ф функцию обозначим Ф. Теорема 20 (см. [79]). Пусть X = {X(t), t G T} — случайная функция со значениями в польском пространстве (S,p) при каждом t G Т, заданная на некотором (П,^, Р) и псевдометрическом пространстве (Т,5), такая, что для некоторого М > О Ei/>(p(X(t),X(s))/5(t,s)) ^M при s,teT, если S(t,s)^O, F0) и p(X(t),X(s)) = 0 п. н., если S(t, s) = 0. Пусть также f <S>(Ns(T,s))ds <oo, F1) где интеграл берется по положительной части некоторой окрестности нуля. Тогда у случайной функции X существует непрерывная модификация.
74 А. В. Булинский, А. Н. Ширяев 24. Пусть случайное поле X = {X(t), t G [0, l]d} со значениями при каждом t Е Т в польском пространстве (S,p) удовлетворяет для некоторой неубыва- неубывающей положительной непрерывной функции / на М+ и р ^ 1 условиям E(p(X(t),X(s)))p^F(\\t-s\\), F2) f(x~p/d)dx < +oo, F3) / + ОО где интеграл берется в окрестности +оо. Докажите, что тогда у случайного поля X существует непрерывная модификация (воспользуйтесь теоремой 20). Замечание 6 (см. [79]). Интегральное условие F3) является оптимальным, т. е. можно построить поле, удовлетворяющее всем условиям упражнения 24, кроме ус- условия F3), такое, что оно не имеет непрерывной модификации. Из теоремы 20 выводится также известный результат Дадли: Теорема 21. Пусть X = {X(t), t G T} — действительный гауссовский про- процесс на произвольном множестве Т, снабженном псевдометрикой 5(s,t)=(E(Xs-XtJI/2. Пусть L y/ln Ns(T,e)d? < +оо. F4) Тогда существует модификация X, имеющая непрерывные траектории. При исследовании свойств траекторий случайных функций наряду с метрической энтропией весьма плодотворным оказался подход, основанный на понятии мажори- мажорирующей меры. Это понятие позволяет в определенном смысле лучше учесть струк- структуру параметрического множества Т. Дело в том, что метрическая энтропия как бы не различает "почти пустые" и "заполненные" шары данного радиуса. Определение 21. Вероятностная мера /i, заданная на борелевской сг-алгебре пространства (Т, S), называется мажорирующей, если гоо sup / \\пц(В?(Щ1/2 de < оо, F5) где B?(t) определяется согласно E3). Поскольку n(B?(t)) = 1 для г ^ Dt, то в F5) интегрирование фактически ведет- ведется от 0 до Dt, a условие конечности интеграла F5) равносильно конечности этого интеграла по некоторой положительной окрестности 0. 25 (см. [48; с. 182]). Докажите, что определение мажорирующей меры равно- равносильно следующему. Существуют q > 1 и последовательность *€ = {^ь k G N} измельчающихся разбиений Т на множества положительной /i-меры такая, что sup Dc ^ 2q~k и sup V q~k\ Inyu(Cfc(*))|1/2 < оо, F6) где Ck (t) — то (единственное) множество разбиения ^, которое содержит точку t.
Гл. П. Процессы с независимыми приращениями 75 Теорема 22 (см. [48; с. 193]). Пусть X = {X(t), t е Т} — действительная га- уссовская случайная функция на некотором компактном пространстве (T,S). Тогда необходимым и достаточным условием ее ограниченности будет су- существование мажорирующей меры, а необходимым и достаточным условием непрерывности будет условие limsup Г |ln/i(??(?))|1/2 de = 0. F7) Jo Разумеется, в этой теореме речь идет о наличии модификации с соответствующи- соответствующими траекториями. Достаточность приведенных условий была доказана К. Ферником, а необходи- необходимость — М. Талаграном. Заметим, что имеется также определение мажорирующей меры, использующее конечность интеграла вида для функции Ф определенного класса, см., например, [153]. Свойства гауссовских случайных функций изучаются в [29, 48, 52, 79, 81]. В этих книгах имеются обширные библиографические указания. Некоторые результаты, связанные с траекториями броуновского движения, излагаются в последующих гла- главах и дополнениях к ним.
Глава III Броуновское движение. Свойства траекторий Недифференцируемость п.н. траекторий броуновского движения (винеровского процесса). Марковское свойство винеровского процесса. Фильтрация. Марковские моменты, их примеры, сг-алгебра &т, состоящая из событий, наблюдаемых до марковского момента т. Строго марковское свойство винеровского процесса. Принцип отражения. Закон нуля или единицы. Распределения, связанные с максимумом винеровского процесса на [0,t]. Закон повторного логарифма. Локальный закон повторного логарифма. § 1. При первом чтении этой главы можно ограничиться знакомством с до- доказательством марковского и строго марковского свойств броуновского движения (§2 и §6). Для этого важно овладеть понятиями марковского момента т (§3) и сг-алгебры событий §<т (§5). Желательно разобрать доказательство закона повторного логарифма для винеровского процесса (§11), для чего требуется усво- усвоить формулировку результата следствия 1 из § 10, описывающего распределение максимума винеровского процесса на отрезке [0, Т]. Следующий результат показывает, сколь нерегулярно устроены траектории бро- броуновского движения. Теорема 1 (Пэли-Винер-Зигмунд). С вероятностью единица траектории броуновского движения W = {W(t), t ^ 0} не дифференцируемы ни в одной точке t полуоси [0, оо). Доказательство. Рассмотрим промежуток [k, k + 1), где к Е {0,1,...}. Для ио Е О, из дифференцируемое™ W{ •, и) хотя бы в одной точке sG [к, к + 1) вытекала бы дифференцируемость справа в s, а это влекло бы существование g,! G N (q = q(uj, s), I = 1(uj, s, q(s,uj))) таких, что \W(t,u)-W(s,u)\^l(t-s) при всех t e[s,s + q'1) c[k,k+ 1). A) Для l,n,i G N (и фиксированного к) введем события W (к+-,(Л - W \ n J - при j = i + l,i + 2,г + з n Рассмотрим l и q, фигурирующие в A). Если 4/n < \jq и г — i(s, n) (i G {1,..., n}) выбрано так, что к + (i — l)/n ^ s < к + i/n, то для j = г + 1,г + 2,г + 3иа;,
Гл. III. Броуновское движение. Свойства траекторий 77 п к+1 Рис. 10 удовлетворяющих A) (см. рис. 10), получим )-w 41 (>¦ w ( 4 n j - n + 1- 1 i n 3 n ) < _ 71 n W(s, + -W[k 3-1 п Поэтому, если для и справедливо свойство A), то о; G П (J ^,п,г- Введем мно- n>4q г=1 жество Dfc = {cj : W( •, о;) дифференцируема хотя бы в одной точке s G [к, к + 1)} (если s = fc, то подразумевается дифференцируемость справа). Из проведенных рас- рассуждений следует, что л* с и U П U *.«,<• д=1 1 = 1 n>4q г=1 Заметим, что для любой последовательности событий ??n G <?, n G N, B) lim inf n—^oo Поэтому, принимая во внимание независимость приращений процесса W, для каж- каждых q, I G N получим, что , П Р П UA^ xn>4gt=l г=1 ^ lim inf n ( P n—^oo i=l 3 = lim inf n ( P n—>>oo liminf n -j -I A J\ 3 • —= ) = = 0, где мы воспользовались тем, что W{t) ~ N@, t) при t ^ 0, а для z > 0 1 Г _^ , . 1 в 2 BЖ $
78 А. В. Булинский, А. Н. Ширяев (Теперь ясно, почему при определении А\^п^ надо было рассмотреть приращения процесса W на трех промежутках: [(j — l)/n, j/n), j = i + 1, i + 2, i + 3.) Учитывая, что объединение счетного числа множеств меры нуль имеет меру нуль и что, как было оговорено ранее, вероятностное пространство (П, 3, Р) полно, полу- получаем, что P(-Dfc) = 0 для любого к = 0,1,2,... . Если D — множество точек и, для оо которых W( •, ш) дифференцируема хотя бы в одной точке s G [0, оо), то D = (J D&. fc=o Следовательно, P(-D) = 0. П Приведенное выше простое доказательство дано А. Дворецким, П. Эрдёшем и С. Какутани. § 2. Покажем, что приращения винеровского процесса снова приводят к винеров- скому процессу. Определение 1. Случайная функция X = {X(t), t G Т}, заданная на вероят- вероятностном пространстве (П,^, Р), называется не зависящей от а-алгебры 8 С 3, если независимы сг-алгебры &{Xt, t G Т} и 8. Теорема 2 (марковское свойство винеровского процесса). Дл^ каждого числа а ^ 0 процесс X = {Х(?) = W(? + а) — VF(a), t ^ 0} является винеровским процессом, не зависящим от а-алгебры 3*^ — a{W(s): 0 ^ s ^ а}. Доказательство. Очевидно, Х@) = 0, процесс X = {X(t), t ^ 0} имеет непрерывные траектории, независимые приращения и I(t) - I(s) = VF(t + a) — —VF(s + a) ~ N@, t — s), 0 ^ s ^ t. Следовательно, процесс Х является винеров- винеровским. Покажем теперь справедливость последнего утверждения теоремы. В силу леммы 9 главы I достаточно проверить, что при любых n, m G N и всех 0 = to < t\ < • • • < tnj 0 = so < si < • • • < Sm ^ а независимы события A1 = {x(t0) e Bo, x(h) gBi,..., x(tn) e вп], A2 = {W(so) e Co, W(Sl) G Ci, ..., W(sm) G Cm}, где Bi, С'j G 3&(R) (i = 0, ...,n; j = 0, ...,ш). Векторы ? = (X(t0),... ,X(tn)) и ту = (W(so), • • •, W(sm)) получаются аналогично (П.2) с помощью линейных преоб- преобразований соответственно векторов ? = (Х(?о), X(?i)— X(to), ..., X(tn)—X(tn-{j) vi rj = (W(so), W'(si) — W(so), • • •, W(srn) — W(sm-i)). Независимость ? и ^вы- ^вытекает из независимости приращений процесса W = {VF(?),? ^ 0}. Поэтому (как функции от независимых векторов) независимы ? и 77. Следовательно, независимы события Ai и ^2. ? §3. Естественно возникает вопрос: можно ли обобщить теорему 2, взяв вмес- вместо константы а случайную величину? Оказывается, это можно сделать для опре- определенного класса случайных величин. Определение 2. Пусть (П, ЗР) — измеримое пространство, ТсЖ и Ft = (&t)teT — некоторый поток сг-алгебр в 3* ("фильтрация"), т.е. 3's С 3t для s ^ t (s, t G T) n3t С i^ при всех ? G Т. Естественной фильтрацией процессах = {Х(?),? G Т} называется поток сг-алгебр ^^ = a{X(s), s ^ t, s G T}, t ? Т. Иначе говоря, сг-алгебра ^^ при каждом t € Т порождается течением процесса на промежутке (-оо,?]ПТ.
Гл. III. Броуновское движение. Свойства траекторий 79 Определение 3. Отображение т: П —> Т U {оо} называется марковским мо- моментом относительно фильтрации Ft, если {со: г (со) ^ ?} Е 3*t для любого t G Г. Моментом остановки называется марковский момент т, для которого г (со) < оо п. н. Замечание 1. Как и ранее, все рассмотрения проходят на полном вероятност- вероятностном пространстве (П, ^, Р). Будет также предполагаться, что все сг-алгебры ^, t G Т, расширены классом J/ множеств нулевой Р-вероятности. В качестве упражнения убедитесь, что если т\, Т2,... — марковские моменты от- относительно фильтрации Ft , где Т = Z+, то следующие величины г (о;) = a G Т (а = const), min rn, max тп, V^ rn (ттг G N), C) inf rn, sup rn, lim inf rn, lim sup rn D) также являются марковскими моментами относительно Ft . Если т — марковский момент относительно фильтрации Ft, где Т = [0, оо), то для любого t ^ О T<t}G &t, E) поскольку для t > 0 имеем {г < ?} = (J {r ^ t — к~г} G (J ^t_^-i С ^t (если t - А: < 0, то {г ^ t - к'1} = 0). Отметим, что в дискретном случае, когдаF = (^п)п^о^ определение марковского момента г, очевидно, равносильно тому, что {г = п) G &п ПРИ каждом п G Z+. Замечание 2. Пусть г — марковский момент относительно фильтрации Ft, a сг:П —>¦ Ти{оо}, причем т = сг п.н. Тогда в силу замечания 1 величина а также является марковским моментом относительно фильтрации Ft . Пусть X = {Хп, п ^ 0} — действительный процесс с дискретным временем и ?х = (ЗРп)п^ъ — естественная фильтрация. Тогда для любого борелевского множества В г (со) = inf{n ^ 0: Хп(со) е В} F) будет марковским моментом, поскольку {г = 0} = {Xq Е В} G 3*§~ •> а для п ^ 1 {г = п} = {х0 $ в,..., xn_i ^5ДпеБ}е ^. § 4. Аналогичный F), но более сложный пример марковского момента для про- процессов с непрерывным временем дает следующая теорема. Теорема 3. Пусть X = {X(t), t ^ 0} — случайная функция со значениями (при каждом t ^ 0) в метрическом пространстве (S,p). Пусть траектории X почти наверное непрерывны. Тогда для любого замкнутого множества F С S момент rF(co) = inf{t ^ 0: X(t,u) e F} G) есть марковский момент относительно естественной фильтрации ?х про- процесса X (считается, что тр(со) = оо, если X(t,u) ? F для всех t ^ 0).
80 А. В. Булинский, А. Н. Ширяев _ Доказательство. Пусть траектории X непрерывны для точек ио G П, где п С П и Р(П) = 1. Взяв произвольную точку жо G S, определим процесс / X(*,w), если?^0, ио ей, X(t,uo) = < ~ [ жо, если ? ^ 0, cj G П \ П. В силу полноты вероятностного пространства процесс X есть модификация процес- процесса X, имеющая непрерывные траектории при всех ио G П. Пользуясь замечаниями 1 и 2, видим, что без потери общности можем считать все траектории X непрерывны- непрерывными на [0, оо). Далее схема доказательства теоремы такова. Непрерывность траекторий процес- процесса X и замкнутость множества F позволят нам установить, что тр(ио) = lim тп(ио) при всех и, (8) п—>>оо где тп :— топ — inf{t ^ 0: X(t, ио) G Gn} (для упрощения записи), а открытые мно- множества Gn = {х G S: p(x, F) < 1/п} представляют собой сжимающиеся с ростом п окрестности F. Надо иметь в виду, что для доказательства марковости момента тр мы не можем непосредственно воспользоваться свойствами D) и (8), поскольку будет показано лишь то, что {тп < ?} G 3*^ для каждого п G N и любого t > 0 (сравни- (сравните с определением марковского момента). Тем не менее этого будет достаточно для нашей цели, так как мы увидим, что оо {uj\ tf{uj) ^ i) = P| {uj: rn{uj) < t} при О < t < oo, (9) 71=1 а то, что {<jj\ tf ^ 0} = {-^@, uS) G F} (и, значит, входит в 3^) вытекает из непре- непрерывности справа траекторий X и замкнутости множества F. Перейдем теперь к подробному доказательству. Прежде всего, покажем, что ес- если G — произвольное открытое подмножество S и tq определяется согласно G) с заменой F на G, то {со: tq{oo) < t} G 3^. Для этого проверим вначале, что {rG<t}= U {X(r)eG}, A0) где Q+ — множество неотрицательных рациональных чисел. Действительно, если X(r,uj) G G для некоторого г < ?, г G Q+, то tq{^) ^ т < t. С другой стороны, пусть tg(^) < ?. Тогда найдется такое s = s(uo) < t, что X(s,uo) G G. Поскольку G- открытое множество, а траектории X непрерывны справа, то X (и, ио) G G для всех и, достаточно близких к точке s справа. Среди таких и есть рациональные точки г < t. Итак, A0) доказано. Теперь заметим, что {X(r) G G} G &* С &* для г < ?, а в правой части форму- формулы A0) берется счетное объединение событий из З*^. Следовательно, как утверж- утверждалось, {ио: tq(oo) < t} G З^ и, в частности, {ио: тп(ио) < t} G ^^ для каждого n G N и любого t > 0.
Гл. III. Броуновское движение. Свойства траекторий 81 Теперь докажем, что —У оо, tf(uo) G uo G П. [0,oo]. при всех п GN, (И) Ct^ G u L . при n Очевидно, F С Gn+i С Gn, откудатп(ио) ^ ту Поэтому для каждого ио существует lim rn(uo) = Too(cj) ^ n—>-oo Отсюда видно, что если Гоо(с^) = оо, то тр(ио) = оо. Покажем, что Тоо(и;) = тр(ио) для ио G {тоо < оо}. Непрерывность справа траекторий X влечет, что Х(тп(ио), ио) G G [Gn], n G N, где [•] обозначает замыкание множества. Учитывая непрерывность слева траекторий X и то, что [Gn] С [G&] при n ^ &, получаем для cj G {гоо < оо} оо Таким образом, тр(ио) ^ тЪо^) ПРИ всех ио G П. Сопоставляя это с доказанным неравенством Too(cj) ^ т~р(оо) дляоо G П, приходим к A1). Проверим, наконец, свойство (9). Если тп(ио) < ?, п G N, то г^(с^) ^ t в силу A1). С другой стороны, пусть тр(ио) ^ ?, где t > 0. Если г^(с^) = 0, то, согласно A1), имеем тп(ио) = 0 < t при всех nGN. Для оо ? п' = {оо:0 < тр(оо) ^ ?} докажем, что тп(оо) < тр(оо), п G N. Пусть cj G П', тогда rn(cj) —У тр(оо) > 0 (п —У оо). Поэтому при всех достаточно больших п получаем, что тп(оо) > 0 и Х(тп(оо), оо) G <9Gn (dGn — граница множест- множества Gn). Последнее утверждение справедливо, поскольку в любой окрестности точки X (тп (оо),оо), где 0 < тп (оо) < оо, найдутся как точки из Gn, так и точки, не входящие в Gn (имеются tп,к(^) I тп(и) при к —У оо, для которых X(tn^(u),oo) G Оп,втоже время X(t,oo) ? Gn дляг < тп(оо)). Учитывая, что dGn С {х G S: p(x,F) = 1/п}, получаем неравенства тп (cj) < rn+i (cj) для cj G П' и всех достаточно больших п (ес- ли р(Х(тп(оо), о;), F) = l/nH^(X(rn+i(cj),cj), F) = 1/(п + 1), то rn(cj) 7^ тп+\(оо)). Следовательно, (9) установлено, что и завершает доказательство теоремы, посколь- поскольку, как было показано, {оо: тп(оо) < t} G &*, п G N, t ^ 0. ? § 5. Пусть т — марковский момент относительно потока сг-алгебр Определение 4. Положим &т = {Ае &:Ап{т ^t} e &t Для любого t G Г}. A2) Система множеств §<т является (как легко проверить) сг-алгеброй, называемой сг-ал- геброй событий, наблюдаемых до случайного момента т (включительно). Следующее простое свойство марковских моментов обобщает соотношение E). Если а — марковский момент относительно потока (&t)teT и A G i^v, то для каж- каждого t еТ А П {а < t} = U (А П {а <: t - q-1}) G ^t, A3) q=l An {a = t} = (An {a ^ t}) \ (An {a < t}) G &t- (Щ Определение 5. Назовем сг-алгебры^, Ж (содержащиеся в &) совпадающими на событии А, если A G У П Ж и А П У = А П Ж (А П У = {АС: С G Щ). Аналогично, будем говорить, что ^ С Ж на А, если Ае^ПЖиАП^сАП Ж.
82 А. В. Булинский, А. Н. Ширяев Положим (г Л ct)(uj) = min{r(cj), a(uj)}, uj G П. Лемма 1. Пусть г и а — марковские моменты относительно фильтрации (&t)teT, TcM. Тогда 1°. $Т П {г <С а} С frAG = Jv П J^. 2°. §<т = &О- на множестве А (из &<Т П^а), если т = а на А, в частности, &<Т = 3*t на множестве {г = ?}, t G Т. 3°. г является &<Т-измеримой величиной. Доказательство. Покажем, что ^П{г^}с^, A5) т. е. проверим, что для любого A Е 3*т и каждого t Е Т множество С = Ап{т ^ сг} П П{сг ^ ?} G c^t- Очевидно, С = Ап{т ^ t}n{cr ^ t}n{rAt ^ a At}. ВеличиныгЛ^ и сг Л ?, как легко видеть, ^~измеРимы- Поэтому {г Л t ^ а Л t} G 3*t- Кроме того, А П {г ^ t} G ^t и {сг ^ t} G 3t- Следовательно, С е Зц что доказывает A5). Подставим т и г Л а соответственно вместо г и сг в A5). Тогда, замечая, что {г ^ т Л а} = {г ^ а}, приходим к первому соотношению в 1°. Теперь, заменив в A5) пару (г, а) сначала на (г А а, г), а затем на (г Л сг,сг), получим ЗтА(Т С«?ти 3*т/\(у С За (поскольку {т А а ^ г} = п и {г Л а ^ а} = п). Таким образом, ^тЛG С 3*т П За. Чтобы доказать обратное включение, возьмем любое событие A G 3<т П &о- и убедимся, что D = А П {г Л а ^ ?} G ^t- Учитывая, что {г Л сг ^ t) = {г ^ i) U {сг ^ t}, имеем D = (An {т ^ t}) U (An {a ^ ?}) G ^t. Соотношение 1° установлено. Из A5) вытекает, что &т П {т = а} = &т П {т ^ а} П {т = а} С &а П {т = а}. Меняя местами г и сг, видим, что За П {г = а} С &*т П {т = а}, что обеспечивает равенство 3ТП {т = а} = ЗаП{т = а}. Кроме того, множество {г = a} G ^П^, поскольку П G ^т и J^T п {г = сг} = &т П {г ^ сг} П {сг ^ г} С Jv П {сг ^ г} С &т. Теперь заметим, что &т П А = &т П {т = а} П А = &а П {т = а} П А = &а П А. Следовательно, 2° выполнено. Для доказательства 3° достаточно убедиться, что {г ^ х A t} G 3*t ПРИ любых х G Ж и t G Т. Это верно при всех х ^ ?, а также при ж < ?, если ж G Т. Пусть далее х ^ Т, х < t. Если множество Д^ = {s G T: s < ж} = 0, то {г ^ х} = 0 G ^t- При Вх ф 0 положим х' — sup{s: s G Вх}. В случае ж' G Т имеем {г ^ ж} = {г ^ ж'} G оо G Jv С c^t- Если ж' ^ Г, то {г ^ ж} = |J {r ^ tn},Tj\etn G T,tn t^'npnn ->> оо, J поэтому {r^i}G^fD n=1 § 6. Перейдем к изложению одного важного обобщения марковского свойства ви- неровского процесса (броуновского движения).
Гл. III. Броуновское движение. Свойства траекторий 83 Теорема 4 (строго марковское свойство). Пусть т — момент остановки от- относительно естественной фильтрации ?w = (^t^)t^o винеровского процесса W = {W(t), t ^ 0}. Тогда процесс Y = {Y(t) = W(t + r) - W{t), t ^ 0}, бу- будет винеровским, притом не зависящим от а-алгебры ^^ {построенной по семейству (^t^)t^o согласно A2)). Доказательство. Вначале уточним, как процесс Y задается для ио Е {т(и) = = оо}. Положим Y(t u) = l ^(* + гИ^)^(гИ^) Для^еПт {г<оо}, \ 0 длясс? ^ пт. То, что Y — случайный процесс (т.е. семейство случайных величин), вытекает из упражнений 27 и 28 главы I (процесс W измерим как процесс с непрерывными п.н. траекториями; ниже мы увидим и непосредственно, что Y(t) G 3*\ Й§(М) при каждом Построим случайные величины тп = тп(ш), аппроксимирующие т = г (и) по сле- следующим формулам: оо TnM = 5>2-nlAfc5n, neN, A7) k=i где А1:П = {г ^ 2"n}, Akjn = {(к - 1J"п < г (о;) ^ к 2~п} для к ^ 2. Очевидно, что тп(сс?) ^ т(о;) при п —> оо для всех w G От. Кроме того, для каждого п G N ве- величина тп есть марковский момент относительно потока З*^•> поскольку для любого t ^ О {тп ^t} = {T^k2-n}e&™,-n C&F, где /с = В силу непрерывности п. н. траекторий броуновского движения W(t + тп(и),и) ->• W(t + t(oj),oj) п.н. при п ->• оо A8) для каждого ? ^ 0. При всех n G N, t ^ 0, 2: Е 1, w E 1), учитывая E), имеем z] = |J {cj: W(t + ^2"n,^) ^ z, rn(cj) = ^2"n} G ^. A9) Из A8) и A9) с помощью леммы б главы I и замечания 1 выводим, что Y(i) есть случайная величина при каждом t ^ 0. Очевидно также, что траектории процесса Y непрерывны п.н. Докажем, что процесс Y не зависит от сг-алгебры 3<т и одновременно убедимся, что Y — винеровский процесс. Как и при доказательстве теоремы 2, достаточно проверить, что для любых А е &r, m е N, о ^ h < • • • < tm, в е ё%(жт) В), B0)
84 А. В. Булинский, А.Н. Ширяев где ? = (Y(ti),..., Y(tm)). В B0) можно рассматривать лишь замкнутые множест- множества В (по лемме 4 главы I для любого В и любого г > 0 существует замкнутое F? С В такое, что Р^(В \ F?) < е). Перепишем B0) в виде B1) Покажем, что соотношение B1) будет выполнено, если для каждой непрерывной ограниченной функции /: Ж171 —У Ж имеет место равенство B2) Для этого возьмем Д(ж) = ip(kp(x, В)), где {1 при t ^ 0, 1-t npnte [0,1], B3) 0 при t ^ 1 и р(х, В) = inf{p(x, у): у G В}, р- евклидово расстояние. Суперпозиция непрерыв- непрерывной и непрерывной ограниченной функций дает непрерывную ограниченную функ- функцию (р( •, В) непрерывна для замкнутого В). Поэтому, учитывая, что Д (ж) —у 1в(х) для х G Mm при к —у оо, требуемое утверждение получаем из теоремы Лебега о ма- мажорируемой сходимости. Установим теперь справедливость свойства B2). По той же самой теореме Лебега находим, что для любого A G ^т EW@=limElA/(?n), B4) где ?n = (W(ti + тп) - W(rn),..., W(tm + тп) - W(rn)) -У ? п.н. при п -у оо. Пользуясь счетной аддитивностью интеграла Лебега, видим, что {п }к), B5) к=1 к=1 где ?n,fc = (W(h + fe2"n) - W(k2~n),..., W(tm + k2~n) - W(k2~n)). В силу A7) An {тп = &2~n} = А П Afcjri G ^^-n Для A e &т- По теореме 2 сг-алгебра ^fc2-n не зависит от €п,к- Кроме того, распределение ?П)д. такое же, как у вектора (W(ti),..., W(?m)). Таким образом, правая часть B5) равна следующему выраже- выражению: оо Е f{W(h),..., W(tm)) Е Е 1лп{гп=Л2-»} = Е /(^(ti),..., W(tm)) E 1А. B6) Взяв А = П, получим из B4) и B6), что для любой непрерывной ограниченной функ- функции /, всех mENnO^ti <---<tm, справедливо равенство Ef(Y(h),...,Y(tm)) =Ef(W(h),...,W(tm)), B7) что завершает доказательство B2).
Гл. III. Броуновское движение. Свойства траекторий 85 Из B7), совершив предельный переход к индикаторным функциям, получим, что конечномерные распределения Y wW совпадают. Следовательно, Y — винеровский процесс. ? Отметим, что иная (и более сложная форма) строго марковского свойства рас- рассматривается в приложении 6. § 7. Результат предыдущего пункта о строго марковском свойстве мы использу- используем ниже для нахождения распределений некоторых функционалов от броуновского движения. Начнем с установления так называемого принципа отражения. Теорема 4 показывает, что если г = г (со) — момент остановки относительно фильтрации ?w = (^tV)t^o, то процесс W ведет себя после момента г (со) так же, как если бы из точки (г(о;), W(t(co), со)) "выпускался" новый винеровский процесс. Очевидно, процесс — W = {—W(t), t ^ 0} также является винеровским. Тем самым, "отражение" процесса W относительно оси абсцисс снова приводит к винеровскому процессу. Поэтому естественно ожидать, что мы также получим винеровский процесс, если из точки (г(о;), W(г(со), со)) выпустим для t ^ т(со) отраженную относительно прямой у = W(г(со),со) траекторию W(t,co). Реализация этих наводящих соображений, однако, требует определенных усилий. Пусть т = г (со) — момент остановки относительно естественной фильтрации про- процесса W. Длясс? G 1)т = {т < оо} (Р(пт) = 1) введем "отраженныйпроцесс" Z(t,u,) = I Wit>U)> O^t<r(a,), V ; \2W(T(w),w)-W(t,w), t>r(w), и пусть Z(t,w) = W(t,w) для a; ? п\пт (см. рис. 11). B8) V/ X(t,u>) т(ш) Рис. 11 Теорема 5 (принцип отражения). Процесс Z = {Z(t), t ^ 0}, определенный в B8), есть винеровский процесс. Доказательство. Для всех t ^ 0 и со е П Z(t,u) = W(t,u)l{T>t} + BW(r(co),co) - W(t,u))l{T<t}, следовательно, Z(t, •) есть случайная величина при каждом t ^ 0. Траектории Z( • ,со) непрерывны п.н., поэтому аналогично доказательству лем- леммы 12 главы I нетрудно показать, что Z( •, со) есть случайный элемент со значениями
А. В. Булинский, А. Н. Ширяев в польском пространстве Со [0, оо) = {/ Е С[0, оо): /@) = 0}, снабженном метрикой равномерной сходимости на компактах: = S l + Bup|/(t)fl(t)r /'^[°'^ <29> Пусть У = {Y(t, и), t ^ 0} — процесс, определенный формулой A6). Введем еще "остановленный процесс" X = {X(t,u) = VF(? Л т(о;),а;), t ^ 0} (X(?,?j) = W(t,u) длясс? Е П\ПТ). Учитывая непрерывность п. н. траекторий процесса W, убеждаемся, что Х( •, и) — случайный элемент со значениями в пространстве Со[0, оо). Определим отображение ("склеивание" в точке Ъ функций g и /) h:\/ —У Со[0, оо), где V = [0, оо) х Со[0, оо) х Со[0, оо), положив Заметим, что для всех со G Пт h(r(L0),X(-,L0),Y(-,L0))=W(-,L0), Принимая во внимание замечание 2, далее без потери общности можем считать Пт п Q, = П, где п С О, состоит из тех точек и, для которых траектории W{ •, и) непрерывны на [0, оо). Теорема будет доказана, если мы убедимся, что случайные элементы (г, X, Y) и (г, X, —Y) имеют одинаковые распределения на пространстве (V, ^(V)) (в произве- произведении трех польских пространств берется метрика, равная максимуму из метрик этих пространств). Этого будет достаточно, поскольку h является непрерывным отображением пространства V в пространство Со[0, оо) (с введенными метриками), а значит, h G 38(V) | ^(Со[0, оо)). Схема дальнейших рассуждений такова. Мы увидим, что случайный вектор (г, X) будет §<т | 3§([0, оо) х 3§(Со[0, оо))-измерим. По строго марковскому свойству FJLi^v, где JL обозначает независимость. Следовательно, (г, X)A.Y. Поэтому Law((r, X, У)) = Law((r, X)) 0 Law(F) = Law((r, X)) 0 Law(VF). Очевидно, процесс (-Y) также не зависит от ^т, а так как (-Y) снова является винеровским процессом, то Law((r, X, -Y)) = Law((r, X)) 0 Law(-F) = Law((r, X)) 0 Law(VF). Согласно следствию 1 и лемме 11 главы I для проверки требуемой измеримости вектора (г, X) достаточно установить &*т-измеримость каждой его компоненты. В силу утверждения 3° леммы 1 т есть &*т-измеримая величина. Доказательство ^-измеримости X основано на построении ^-измеримых ап- аппроксимирующих случайных элементов Хп, сходящихся к X при п —у оо. С этой целью введем величины к=0 т(и;) ПРИ п ~* °° для всех
Гл. III. Броуновское движение. Свойства траекторий 87 ПоложимXn(uS) = W(t/\an(uS),uS),T№t ^ 0фиксировано, an Е N, и; Е П. Проверка ^-измеримости величин Хп полностью аналогична рассуждениям, применявшим- применявшимся при доказательстве теоремы 3, и поэтому опускается. ? § 8. Принцип отражения является одним из мощных средств стохастического ана- анализа, что будет видно, в частности, из доказательств, приводимых ниже в § 10. Нам понадобится также следующий известный результат Колмогорова. Теорема 6 (закон нуля или единицы). Пусть на вероятностном пространст- пространстве (П,^, Р) задано семейство {Xt, t G Т}, ТсМ, независимых случайных эле- элементов со значениями в измеримых пространствах (St,?$t), т. е. Xt: П —у St, Xte&\3&t,teT. Обозначим &*t = a{XSj s e T П [t, oo)}. Тогда "хвостовая" а-алгебра, т. е. а-алгебра <$ = f] &>t, вырождена. Иначе teT ^ говоря, для любого A Е ^ вероятность Р(А) равна нулю или единице (если Т П [t, oo) = 0, mo &^t := 0). Доказательство. Возьмем произвольное событие Ag^h покажем, что А не зависит от А. Имеем A G ^yt при каждом t G Т. В силу леммы 3 главы I для любого ? > 0 существует множество А? из алгебры, порождающей ^>^, т.е. множество А? вида {(Xtl,... ,Xtn) е В}, где В е ^8tb...,tn, t ^ ti < • • • < tn (все точки из Г), п G N, такое, что Р(А А А?) < е. Следовательно, \С(А,А) -С(А,А?)\ ^2Р(ААА?) <2е, гдеС(А,Б) = P(AD) — Р( A) P(-D) для событий Аи D. Из леммы 9 главы I вытекает, что сг-алгебры cr{Xt1,..., Xtn} и ^^w независимы при и > tn. Поэтому независимы события АиА?,т.е. С (А, А?) =0. Отсюда С (А, А) = 0 и, значит, -Р(АJ = С(А,А) =0. Итак, Р(А) = 0 или Р(А) = 1. ? Лемма 2. Пусть а > 0. Тогда момент первого достижения уровня а вине- ровским процессом, т. е. момент та(и) = inf{? ^ 0: W(t,u) = a}, есть момент остановки относительно естественной фильтрации ?w. Доказательство. Из теоремы 3 следует, что та — марковский момент. Дока- Докажем, что та < оо п.н. Для этого проверим, что для действительных случайных ве- величин Yn, n G N, и константы с G Ж PnimsupFn > с) ^ HmsupP(Fn > с). ^ п,—vc>C) ' п,—Vein
А. В. Булинский, А. Н. Ширяев Действительно, p(limsupFn > с) = р( П И {Ym > с}) = lim р( I) {Ym > с}) и достаточно лишь заметить, что Р( (J {Уш > с}) ^ P(^n > с) при каждом n Е N. Теперь убедимся, что та < оо. Для всякого cGl где "б.ч." означает "бесконечно часто (по п)" а хвостовая сг-алгебра ^ строится по последовательности независимых величин Х^ = W(k) — W(k — 1), к G N. Следо- Следовательно, по закону нуля или единицы Колмогорова (теорема 6) p{limsupW(n)/v/ri> с) е {0,1}. Для а > 0 имеем Р(та < оо) ^ р( sup VF(t) > a) ^ P(W(n) > ал/п б.ч.) ^ VtG[0,oo) У ^ p(limsupn/2VF(n) > a) ^ limsup P(n/2VF(n) > а) = P(f > а) > 0, где ^ ~ N@,1). Таким образом, Р(та < оо) = 1. ? § 9. Применим принцип отражения для нахождения совместного распределения величин W(t) и M(t,uj) = sup W(s,o;), где ? ^ 0. sG[O,t] Прежде всего заметим, что величина M(t, •) при каждом t ^ 0 есть действитель- действительная случайная величина. Следует это непосредственно из того, что Gtf= sup /(s), /gC0[0,oo), C0) [] является непрерывным отображением Со[0, оо) в М. Теорема 7. ТТрг/ всеж t, ж, у ^ 0 совместное распределение случайных вели- величин W(t) и M(t) подчиняется соотношению P{W(t)<y-x, M(t)^y)=P{W(t)>y + x). C1)
Гл. III. Броуновское движение. Свойства траекторий 89 Доказательство. Если у = 0, то C1) превращается в тривиальное равенство P(W(t) < -х) = P(W(t) > х). Пусть теперь у > 0. По лемме 2 Ту =inf{O0: W(s) =y} есть момент остановки относительно фильтрации F^. Пусть Z = {Z(t), t ^ 0} — процесс, введенный формулой B8), где г = ту. Тогда а у (и) = inf {s ^ 0: Z(s, uj) = у} — также момент остановки относительно фильтра- фильтрации ?z, при этом (Ту (и) = Ту (и) для всех у ^ 0, и; G {т^ < оо}. Заметим, что {ту ^ t} = {M(t) ^ ^/} для любых t,^y ^ 0. Поэтому для всех В eSS(C[0,oo)),t ^0 р^ ^ t, vr g Б) = р( sup w(s) ^y,weB) = P(w евпв), где 5 = G^1([^/, oo)) G Й§(С[0, оо)), см. C0). Применив теорему 5, видим, что Р((ту ^ t, z е в) = P(z е впБ) = p(vf евпв). Итак, случайные элементы (т^, W) и (cry, Z) имеют одинаковые распределения. Следовательно, для всех х G М, ?, 2/ ^ 0 Р(^ ^?, Z(t) <2/-ж) = Р^ ^ t, W(t) <y-x). C2) В силу непрерывности W имеем \?(ту(ш),ш) = у при ^/ ^ 0, cj G Пт. Поэтому для t ^ (ту{ио) получаем Z(t,u) = 21^(^@;),о;) — W(t,o;) = 2у — W(t,u). Таким образом, при всех |/^0ихЕ1из C2) имеем P(M(t) > у, W(t) <у-х)= Р(ау <: t, Z(t) <у-х) = = Р{<Ту^ t, W(t) > у + х) = Р(ту <: t, W(t) > у + х) = y, W(t)>y + x). C3) Если ж ^ 0, ToP(M(t) ^ у, W(t) > у + х) = P(W(t) > у + х) и C3) влечет C1). П § 10. Из теоремы 7 получим результаты о распределении максимума броуновско- броуновского движения на отрезке [0, t]. Следствие 1 (Башелье). При всех t,y ^ 0 P(M(i) ^ у) = 2P(W{t) 2v)= P(\W(t)\ 2 У), C4) m. e. Law(M(f)) = Law(|H/(i)|) ^л^г каждого t ^ 0.
90 А. В. Булинский, А. Н. Ширяев Доказательство . Возьмем х = 0 в формуле C1). Тогда, поскольку P(W(t) < у, M(t) 2v) = P(W(t) > у), имеем P(M(t) >y)= P(M(t) > У, W(t) <y)+ P(M(t) > У, W(t) >y) = = P(W(t) >y)+ P(W(t) >У)= 2Р(W(t) > У) (учли, что P(W(t) = у) = 0 при любых у е Ж и t ^ 0). ? Полезно отметить, что наряду со свойством Law(M(t)) =Law(|VF(?)|), верным при каждом t ^ 0, справедливо следующее соотношение, полученное П. Леви (см., например, [182, с. 230]: Law(M- W,M) = Law(|VF|,L), здесь L = {Lt(O),t ^0} — локальное время (в нуле) броуновского движения (см. далее гл. VIII, упражнение 19). Равенство в последней выключной формуле понима- понимается как совпадение распределений двумерных процессов. Следствие 2. Для всех у^0и0^а<Ь<оо Р( sup \W(t)-W(a)\^y)^4P(W(b-a)^y)=2P(\W(b-a)\^y). C5) Доказательство. В силу теоремы 2 Р( sup \W(t) - W(a)\ ^у) ^ Р( sup W(s)^y)+P( inf W(s)^-y). Принимая во внимание, что sup (-W(s)) = - inf W(s) sG[O,t] sG[O,t] для всех t ^ 0 и —W = {—W(t),t ^ 0} есть броуновское движение, требуемое утверждение получаем из C4). ? §11. Следующий результат, принадлежащий А. Я. Хинчину, является одним из самых замечательных вероятностных законов, раскрывающих структуру поведения траекторий винеровского процесса (броуновского движения). Отметим, что первым, кто установил закон повторного логарифма для сумм независимых бернуллиев- ских величин, был также А. Я. Хинчин. В дополнении к этой главе и в главе V пока- показывается, что между процессом частных сумм, построенным по независимым слага- слагаемым, и броуновским движением имеются глубокие связи.
Гл. III. Броуновское движение. Свойства траекторий 91 Рис. 12 Теорема 8 (закон повторного логарифма). С вероятностью единица w{t) = 1 и lim inf ¦ = -1, C6) BtLogLogtI/2 где Logt = ln(t V e) и V обозначает взятие максимума. Согласно этой теореме почти все траектории винеровского процесса остаются внутри расширяющейся "трубы" между кривыми ± A + е) yJ2b Log Log t (для любого е > 0, начиная с некоторого to(e,u)). В то же время с вероятностью 1 они бесконечно часто "выскакивают" из "трубы" с границей =ЬA — s)^/2tLogLogt (см. рис. 12). Доказательство закона повторного логарифма разобьем на две части. А. Положим = W(t)/h(t), t^O, 1. Возьмем произвольные е > 0, с > 1 и убедимся, что событие {|V(cn)| > 1 + е для бесконечно многих п} имеет вероятность 0. Применив оценку A1.29), получим оо оо 1)| > (l + ?)BLogLogc-I/2) ^ C7) и проверим, что с вероятностью единица limsup|V(?)| t п=1 п=1 1
92 А. В. Булинский, А. Н. Ширяев поскольку при п ^ по (с), т.е. таких, что сп ^ ее, имеем Log Log cn = lnlncn и e-(i+e) Log Log с _ (ninc)-A+?) ф Из оценки C7) выводим, воспользовавшись леммой Боре ля-Канте л ли, что Р (\V(cn) | > 1 + е бесконечно часто по п) = 0. Обозначим An = L: sup \V(t,u) - V(cn^)\ > Л и покажем, что вероятность того, что происходит бесконечное число событий Ап, равна нулю. Поскольку \V(t) - V(cn)\ < \V(t) - W(cn)/h(t)\ + \W(cn)/h(t) - V(cn)\, получим P(An) ^ P( sup \W(t) - W(cn)\ > ?-h{cn)) + - п + a (^R) * \nyu--j иуи ~ ¦ -;/ / По следствию 2 и в силу A1.29) Y^ Рп <: 2 ^ P(V(cn+1 ~СП)\ > |/1(сП) cn+l _ сп \ 1/2 eV^ J^ \2cnlnlnc' ( c-1 X1/2 V21nlnc" f /e\ cnlnlncnl exp< — - —n r = (nlnc)~? /D(c~1)) < oo, если выбрать с = c(e) > 1 так, чтобы г2/D(с — 1)) > 1. Далее, для у ^ ее 2x/2 v Поэтому, пользуясь формулой Лагранжа, при п ^ по (с) имеем с U(c") Mc^1); ^ (mine) (с с) - (c_i) Таким образом, Е *¦ Z (lnlncnI/2 если с = с(е) > 1 таково, что г2/B(с — IJ) > 1. < ос, D0)
Гл. III. Броуновское движение. Свойства траекторий 93 Следовательно, для каждого г > 0 по лемме Боре ля-Канте л ли из C7)-D0) вы- вытекает, что для п.в. uj Е П найдется to = to(e, uj) такое, что \V(t,uj)\ <l + 2s при всех t^to(e,uj). D1) Итак, limsup \V(t)\ ^ 1п.н. В. Теперь проверим, что почти наверное limsupV(t) ^ 1. Для этого достаточно показать, что при любом е Е @,1/2) для п. в. uj среди чле- членов последовательности {тк, ^ G N}, где т = т(е) достаточно велико, найдется подпоследовательность {ткэ} (здесь kj = kj(uj)) такая, что V(mkJ,u) > 1 - 2е, если j > jo(w,e,m(e)). D2) Введем события Вк = {uj: (W(mk) - W(mk~1))/h(mk - ш^) > 1 - г}, к G N. Согласно A1.30) имеем Р(Вк) = P(WA) > (l-s)BLogLog(m/c - ш^ (] Поэтому для всех к ^ ко(е, т) получим Р(Вк) ^ (\п(тк - 7пк-1)) оо Таким образом, ^2 Р(Вк) = оо. События Вк независимы в силу независимости к=1 приращений броуновского движения. По лемме Боре ля—Канте л ли с вероятностью 1 произойдет бесконечное число событий Вк, т.е. для п.в. о; G Q можно указать под- подпоследовательность kj = kj(uj,s,m) —у оо такую, что для всех j G N (W(mki)- VF(m^-1))//i(m^ -т^'1) > 1 - г, то есть, V(mki)h(mki)lh(mki -m^-^-V^m^-1^^ > 1-е. D3) Заметим, что при j —У оо h(mkJ) ( т \1/2 ЩтЪ-1) ( 1 \1/2 h(mki - mki г) \m-lj ' h(mki - Поэтому, выбрав достаточно большое т = т(г) и учитывая D1), D3) и D4), приходим к D2). Неравенства D1) и D2) обеспечивают выполнение первого соотношения C6). Для получения второго соотношения достаточно рассмотреть {-W(t),t^O}.n § 12. Рассмотрим характер флуктуации броуновского движения в окрестности нуля. Следствие 3 (локальный закон повторного логарифма). С вероятностью еди- единица У W(*) 1 Г ¦ W(*) limsup —. =1 и hmint —. = — 1. /2* L L(l/t) *^o+ /2t\\{llt)
94 А. В. Булинский, А. Н. Ширяев Доказательство. Положим t = 1/s, где s > 0, и заметим, что i при s = 0, W(l/s) при s > 0, есть процесс броуновского движения. Непрерывность траекторий В в точке 0 следу- следует из теоремы 8, а остальные свойства броуновского движения проверяются легко. Искомые утверждения получаются теперь из теоремы 8 при s —> оо. ? Заметим, что в приложении 8 рассматриваются дальнейшие обобщения закона повторного логарифма. Дополнения и упражнения Известны разнообразные конструкции винеровского процесса с непрерывными п.н. траекториями. В дополнение к представлению A1.31), данному в теореме 8 гла- главы II, приведем еще одну конструкцию, предложенную П. Леви. Возьмем множества Тп = {fc2~n+1, к = 0,..., 2n~1}, n E N. Пусть на некото- некотором вероятностном пространстве (П, ^, Р) задан массив независимых стандартных гауссовских величин {Хп^, к Е Tn, n E N}. Определим индуктивно процессы Вп = {Bn(t), t E [0,1]}, п Е N. Положим B\(t) = tXi5i, t E [0,1]. Тем самым на множестве Т\ имеем Bi@) = 0 и Bi(l) = Xi5i, а для t E @,1) осуществляем линейную интерполяцию значений В\ в концах отрезка [0,1]. Считая процесс Вп заданным, введем процесс Вп+\ следующим образом. Пусть Bn+1(t) = Bn(t) для t?Tn. Если t = к 2~п Е Tn_|_i \ Тп, то положим ТЭ /j.\ I'D (+ r>~n\ I D (+ i r>~n\\ I O~(n + l)/2 V /^ /g c^ ^v %+l(t) = -\r>n{t - z J+,Dn(t + ^ ^)^ —I— ^^ v > vx An+i5/c. D5J Итак, процесс 5n+i задан на множестве Tn+i. Для остальных точек отрезка [0,1] определим Вп+\ с помощью линейной интерполяции по узлам (?, 5n+i(t)), где t E Tn+i. Таким образом, процессы Вп непрерывны на [0,1] при всех п Е N и uj E П. Заметим, что часть величин Хп^ не использовалась в построениях, но их наличие позволило упростить запись формулы D5). Очевидно, sup |??i(?)| = |Х1д|, априп ^ 1 sup \Bn+1(t) - Bn(t)\ = 2-^n+1)/2 max \Xn+lik [] ^ /2^GT\T Поэтому V sup где ^ ~ N@,1). Пользуясь оценкой A1.28) и леммой Бореля—Кантелли, получаем, что ряд n=l
Гл. III. Броуновское движение. Свойства траекторий 95 для п. в. uj сходится равномерно на отрезке [0,1]. Согласно теореме Вейерштрасса этот ряд задает непрерывную п.н. на [0,1] функцию B(t). С помощью леммы 1 главы II легко установить, что при каждом п ^ 0 процесс Вп = {Bn(t), 0 ^ t ^ 1} является гауссовским. 1. Найдите EBn(t) и cov(Bn(s),Bn(t)) для s,t G [0,1] и п ^ 0. Докажите, что B(t)—гауссовский процесс с EB(t) = 0 и cov(B(s),B(t)) = mm{s,t} для s,te [0,1]. Упражнение 1 показывает, что В = {!?(?), 0 ^ t ^ 1} ~ винеровский процесс на отрезке [0,1]. Среди других явных построений винеровского процесса отметим результат Вине- Винера и Пэ ли, которые установили, что ряд &* + ? Е Ыг-^-1-, D6) п=1 к=2п~1 где ^о ? ^1 ? • • • — независимые одинаково распределенные стандартные нормальные величины, сходится п. н. на [0,1] равномерно и задает непрерывную случайную функ- функцию, являющуюся винеровским процессом на [0,1]. Интересно сопоставить это построение с результатом Харди, который доказал (задача была поставлена Риманом), что функция 71=1 непрерывна, но ни в одной точке не имеет производной. Частный случай теорем вложения классов Бесова позволяет доказать (см. [40]), что функции для некоторой подпоследовательности {nm} будут равномерно п. н. сходиться к бро- броуновскому движению на отрезке [0, тг] (см. также формулы (VII.68) и (VII.81)). Принципиальная возможность построения непрерывного на [0,1] винеровского процесса сразу вытекает из теоремы 1 главы II и теоремы Колмогорова о существо- существовании непрерывной модификации (теорема 18 главы II). Действительно, т. е. оценка A1.52) выполнена при а = 4 и ? = 1. 2. Пусть {?n}n^i — последовательность положительных чисел такая, что _1/2 ' tn < оо. Докажите, что тогда |W(?n)| —У оо п.н. при п —> оо. оо 71=1
96 А. В. Булинский, А. Н. Ширяев В дополнение к локальному закону повторного логарифма (см. следствие 3) сфор- сформулируем следующий полезный результат о локальном поведении траекторий вине- ровского процесса. 3. Рассмотрим последовательность Пп, п Е N, измельчающихся разбиений (ргп ) отрезка [0, t] точками специального вида tm = ?m2~n, m = 0,..., 2П. Тогда У^ (^(^m+l) "" ^(^m^)) ~~^ t п*н- ПРИ ^—^00. D7) 4. Покажите, что утверждение D7) о сходимости п.н. не обязано выполняться, если брать измельчающиеся произвольные разбиения Пп, п Е N, отрезка [0,1] точками tm ? тп = 0,..., Nn (n E N), для которых max (г™\л — tffl) —>¦ 0 при п —>- оо (но тем не менее будет иметь место сходимость по вероятности). О дальнейших обобщениях соотношения D7) см. [75; п. 3 гл. 24]. 5. Докажите, что почти все траектории винеровского процесса являются ге'ль- деровскими функциями с показателем 7 < 1/2, т.е. на каждом временном отрезке [а, Ь] С [0, оо) \W(t) -W(s)\ ^ C7|?-s|7, s,t Е [а, Ь], С1 = const > 0. (Доказательство этого утверждения можно найти, например, в [12], теоре- теорема 3 на с. 127.) Можно ли в приведенном результате взять константу G7 не зависящей от промежутка [а, Ь\1 6. Докажите, что с вероятностью единица HmsupVF(?)/?1/2 = оо (тем самым условие Гёльдера порядка 1/2 не выполняется в точке 0). 7. Пользуясь рассуждениями, проведенными при доказательстве теоремы 1, до- докажите, что винеровский процесс W = {W(?), 0 ^ t ^ 1} имеет траектории, не удовлетворяющие п. н. ни в одной точке условию Гёльдера с показателем 7 > 1/2- 8. Выведите результат упражнения 5 из теоремы 19 главы II и того свойства, что для gGN, 0^?<?<оо Е (W(t) - W(s)Jq = Bq - 1)!! (t - s)q. D8)
Гл. III. Броуновское движение. Свойства траекторий 97 Обозначим H1(uj) множество тех точек t G [0, оо), в которых траектория винеровского процесса, отвечающая элементарному исходу uj G П (можно считать П = С [О, оо)), удовлетворяет условию Гёльдера с показателем 7. Упражнения 5 и 8 показывают, что Р(Н1 = [0, оо)) = 1 для 7 < 1/2. Упражнение 7 обеспечивает соотношение Р(Н1 = 0) = 1 при 7 > 1/2. Из упражнения б следует, что Р(? G Н1/2) = 0 при каждом t ^ 0, однако, как показал Б. Дэвис, Р(Н1/2 ф 0) = 1- Следующий результат (см. [73; с. 313]) интересно сопоставить со следствием 3. Для этого напомним, что процесс X является однородным, если распределение X(t + h) — X(t) не зависит от t. Теорема 9 (Хинчин). Если X = {X(t), t ^ 0} — однородный процесс с неза- независимыми приращениями, не содержащий гауссовской компоненты, то lim —, = =0 п.н. UO ^/tlnlnil/t) 9. Постройте пример действительного процесса X = {X(t), t ^ 0} типа cadlag (но не являющегося непрерывным) и замкнутого множества F С М таких, что ^оо < т~р п.н., где Too и tf определены при доказательстве теоремы 3. 10. Пусть X = \Хп, п G N} — последовательность действительных случай- случайных величин и множество В G Й§(М). Пусть г = т(о;) — момент первого достижения множества В, определяемый формулой A.49). Мы уже видели (см. § 3), что т — марковский момент относительно естественной фильтрации процесса X. Введем теперь а(ии) = inf{n G N: Xn(uj) ^ В} — момент первого выхода из В, 7(cj) = sup{n G N: Xn(uj) G В} — момент последнего пребывания в В (в определении моментов т и а предполагается, что если множество, стоящее в фигурных скобках, для некоторой точки uj пусто, то в этой точке значе- значение рассматриваемой величины считается равным +оо). Докажите, что а — марковский момент относительно естественной фильтрации процесса X. Постройте пример последовательности X и множество В, для которых мо- момент 7 является марковским моментом относительно естественной фильт- фильтрации, а также пример X и В, для которых 7 ме является таким моментом. 11. Пусть X = {X(t), t G Т}, Т С М, — действительный случайный процесс и Ft — его естественная фильтрация. Пусть т — т(ии) — марковский момент относительно этой фильтрации; если т(uj) = 00, полагаем X(г(uj),uj) = 0. Можно ли утверждать, что X(t(uj), uj) является &*т \ ё%(Ж) — измеримой ве- величиной? Что можно сказать в случае Т = № 12. Положим та = inf{? > 0: W(t) = а}, где W = {W(t), t ^ 0} — винеровский процесс, a G Ж. Докажите, что та = a2ri, т.е. распределения величин та и а2т\ совпадают: Law(ra) = Law(a2ri). 13. Пусть U — это наибольший нуль процесса W на отрезке [0, t]. Докажите, что для распределения величины U справедлив закон арксинуса: 2 Гх РШ ^ х) = — arcsin W— , х G [0,?]. 7Г У t
98 А. В. Булинский, А. Н. Ширяев Напомним следующее важное понятие. Определение 6. Семейство действительных случайных величин {?а, ot Е А} называется равномерно интегрируемым, если lim sup / \?a\dP = O. D9) Из этого свойства сразу следует, что для таких семейств sup E|fa| < oo. E0) Л Хорошо известно (см., например, [85; гл. II, §6]), что если семейство {?n, n ^ 1} равномерно интегрируемо и ?п —у ? почти наверное (или только по вероятнос- вероятности), то ? будет интегрируемой величиной и E?n^E?, E|fn-f|->0, п^оо. E1) В случае неотрицательных интегрируемых случайных величин ?n, n ^ 0, та- таких, что ?п —у ? (п.н. или по вероятности), где Е? < оо, имеет место и обратная импликация Е?п —>¦ Е? => семейство {?n, n ^ 1} равномерно интегрируемо. Важно также подчеркнуть следующее обстоятельство: во всех приведенных утверждениях, кроме второго соотношения в E1), сходимость почти наверное (?п ~^ ? п.н.) может быть заменена сходимостью лишь по распределению (?п —у ?). Этот важный факт следует из вышеприведенных результатов и доказываемой далее теоремы Скорохода (теорема 11 главы V). Свойство равномерной интегрируемости выявляет Теорема 10 (Де ла Балле Пуссен; см., например, [101; с. 10]). Семейство слу- случайных величин {?со & ? ^} является равномерно интегрируемым тогда и только тогда, когда найдется измеримая функция G: М+ —У М+, т.е. G G ), такая, что lim G(t)/t = оо г/ sup EG(|fa|) < oo. E2) Из этой теоремы вытекает, что условие sup Е|?а|7 < оо для некоторого ^ > 1 E3) Л является достаточным для равномерной интегрируемости семейства {?а, a G Л}. Далее мы покажем, как случайные величины могут быть "вложены" в броунов- броуновское движение, и как исследование сумм независимых слагаемых (с конечными дис- дисперсиями) сводится к изучению поведения броуновского движения в случайные мо- моменты времени. Для этого нам потребуются тождества Вальда.
Гл. III. Броуновское движение. Свойства траекторий 99 Теорема 11 (первое тождество Вальда). Пусть ?i, ?г? • • • — независимые оди- п наково распределенные величины, Sn = J^ ?& и г — марковский момент от- к = 1 носителъно потока а-алгебр ^п = cr{?i,... ,?n}, n Е N, причем E|?i| < оо г/ Ег < оо. Тогда Е5т = ЕгЕ^, E4) т(о;) где »SfT(a;)(a;) = J^ ?fc(^O дл^т ^ G {r < оо}, г/ полагаем Sr^(u) = 0 дл^ cj G G {r = оо}. Доказательство. По определению введенных величин ST находим, что п. н. оо к = 1 Заметим, что п-1 {Г^П} = П\ U {т = к} е &п-!. к = 1 Следовательно, ^ и 1 {т^/с} ~ независимые величины при любом /с G N. Поэтому Е5Т ОО = Е y^?fel{T: к=1 оо — 2^ E?fcEl{ оо :t 1 - s/c -1- oo L J^ k=l ^ Jb) = где законность смены порядка суммирования и интегрирования следует из того, что ; оо. ? к = 1 Из проведенного доказательства видно, что результат обобщается на любую по- последовательность (в том числе зависимых) случайных величин ?i, ?2 ? • • • ? имеющих одинаковое математическое ожидание a G М, и интегрируемую случайную величи- величину г, принимающую натуральные значения, если ?& не зависит от l^T^fcj при каж- каждом fc G N (в частности, если ?& не зависит от г при любом fc G N). 14. Докажите второе тождество Валъда: пусть D?i < 00 дополнительно к ус- условиям теоремы 11, тогда 2 = EtD?i. E6)
100 А. В. Булинский, А. Н. Ширяев Теорема 12. Пусть X — действительная случайная величина, заданная на вероятностном пространстве (П,^,Р) и имеющая Е\Х\ < оо. Тогда сущест- существуют {быть может, на расширении исходного вероятностного пространст- пространства) броуновское движение W = {W(t), t ^ 0} и случайная величина г такие, что X = EX + W(t). E7) Если ЕХ2 < оо, то г может быть построено так, что Ет < оо и, более того, Er=DX. E8) Прежде чем доказывать эту теорему, заметим, что основная трудность связана с одновременным выполнением сотношений E7) и E8), как показывает упражнение 15. Пусть X = X(uS) —центрированная случайная величина на (П,^,Р)сЕХ2< < оо. Возьмем на ((У, 3*', Р') броуновское движение W = {W(t,ujf),t^O}. До- Доопределим X и W на (П, &, Р) ® ((У, &1, Р') согласно A.46). Введем т{ш, ш') = = inf{t ^ 1: W(t,uf) = Х(о;)}. Тогда W(r) = X. Докажите, что при этом Е т — оо, где Е — усреднение по мере Р (g) P'. Доказательство теоремы 12 . Без ограничения общности сразу можем счи- считать, что на исходном вероятностном пространстве (П, 3<, Р) определена не только случайная величина X, но и некоторый винеровский процесс VF = {W(t), t ^ 0}. При этом достаточно рассмотреть лишь случай, когда ЕХ = 0 и X — невырож- невырожденная величина (иначе надо было бы просто положить т = 0). Пусть сначала X принимает лишь два значения а и Ъ (а < 0 < Ъ, поскольку ЕХ = О). Если = а)=р, Р(Х = Ь) = 1-р, E9) то условие Е X = 0 влечет равенства Р=-^-, l-p=f^. F0) о — а о — а В силу теоремы 3 величина та,ъ = inf{t ^ O:W(t) e {а,Ъ}} есть марковский момент относительно естественной фильтрации ?w винеровского процесса. Докажем, что та?ь < оо п.н. При любом т G N {та,ъ ^ т} С {\W(n) - W(n -l)\^b-a,n = l,.. .,m}, откуда Р{та,ь ^ т} ^ P(\W(n) - W(n - 1)\ ^Ъ-а,п = 1,...,т) = [P(|f| ^6-a)]m, где ^ ~ N@,1). Таким образом, P(rfljb = оо) = 0 и Ет?ь<оо при всех fe G N. F1)
Гл. III. Броуновское движение. Свойства траекторий 101 Учитывая непрерывность броуновского движения, получаем, что с вероятнос- вероятностью 1 или W(rajb) = CL4 или W(rajb) = Ь. Следовательно, если ра?0 таково, что P(W(ra,b) = а)= ра<ь, РAУ(та<ь) = Ь) = 1 - ра,ь, F2) и если справедливо равенство EVF(ra,b) = 0, F3) то из F2) найдем, что ра^ — Ъ/{Ъ — а) и, согласно E9), F0), для двузначных вели- величин X соотношение E7) будет установлено. Заметим, что с помощью мартингальной техники (гл. IV) соотношение F3) будет легко получить, воспользовавшись для непрерывного времени аналогом следствия 2 главы IV. Однако F3) нетрудно установить непосредственно следующим образом. Определим величины ^n?m = W(m/n) — W((m — l)/n), m,n G N, и положим TS =ini{m:?n,i + ... + ?n,m g (a, b)}. F4) В силу упражнения 10 при каждом п G N величина т^ есть марковский момент относительно потока сг-алгебр ^ = сг{?п,ь • • • ^п,к}^ к G N. Аналогично F1) убеждаемся, что Ег? < оо. Заметив, что ?n i + ••• + ? О) = W(r^h\ /n) и ' а,Ь Е^пд = 0, согласно E4) находим, что EW(r^/n) = 0. F5) Относительно величин т^ можно утверждать следующее. 16. Докажите, что имеет место свойство: т^ ^/п —> та^ь п.н. при п —> оо. Поскольку траектории винеровского процесса непрерывны п.н., то в силу этого свойства W(r^/п) —> W{ra^) п.н. при п —> оо. Выведем отсюда, что Для этого потребуется следующее утверждение. 17. Докажите, что семейство случайных величин {W(r^nb,/n), n G N} является равномерно интегрируемым. Соотношение F6) следует из результата упражнения 17, а F3) вытекает из F5) и F6). Итак, для центрированных двузначных величин вида E9) доказано, что X = W(Ta,b). F7) Рассмотрим теперь общий случай. Обозначим F(х) = Р(Х ^ х). Учитывая, что Е X = 0 и что случайная величина X невырождена, имеем с= [ {-y)dF{y)= f zdF(z)^0. F8) ./(-оо,0] -/(О.оо)
102 А. В. Булинский, А. Н. Ширяев Пусть /: Ж —у М_|_ — непрерывная и ограниченная функция. Тогда cEf(X) = c Г f(x)dF(x) = J — ОО [ f(z)dF(z) f (-y)dF(y) + [ f(y)dF(y) [ zdF{z) = @,oo) ./(-00,0] J(-oo,0] •'(O.oo) Отсюда / dF{z) [ (zf(y)-yf(z))dF(y). F9) / dF(z) [ (z-y)\f(y)^ + f(z)^^)dF(y). G0) @,oo) J(-oo,0] l %-У Z-y) На некотором вероятностном пространстве ((У, 3*\ Р') построим случайный век- вектор (У, Z) со значениями в М2 такой, что P'((Y, Z)eB) = с-1 ff (z - у) dF{y) dF(z), В е ®{Ж2). G1) ^^ВП{(-оо,0]х@,оо)} Возможность такого построения следует из того, что правая часть формулы G1) есть неотрицательная счетно аддитивная функция множества В. То, что это вероятност- вероятностная мера, сразу вытекает из формулы G0), в которой полагаем / = 1. Если у viz таковы, что у < 0 < z, то из F2) с pVjZ = z/(z — у) и F7) найдем, что Эта формула будет верна и для у ^ 0 < z, если положить то,ь = 0 при Ъ > 0. Учитывая G1) и A.25), видим, что Ef(X)= [ Ef(W(Ty,z))dP{YiZ)(y,z). G3) В силу теоремы Фубини и A.23) равенство G3) можно переписать в виде Е f(X) = ЕЕ'f(W(TY,z)), G4) где ту5^(сс?,сс/) = Ту(ш'),г(ш')(и) дляс^ G П, о;' G (У, а Е' означает интегрирование по мере Р'. _ _ _ Возьмем теперь вероятностное пространство (П, ^, Р) = (П, ^, Р) (8) (П', ^', Р') и для uj = (о;, cj') G li,t ^ 0 положим Х(и) = X(cj), W(t,^) = W(t,u). 18. Покажите, что на (П, ^, Р) r(o5) :=ry(w/)jZ(w/)(a;) G5) является случайной величиной, a VF = {W'(t), t ^ 0} — броуновским движе- движением.
Гл. III. Броуновское движение. Свойства траекторий 103 Учитывая это упражнение и формулу G4), видим, что (X) = Ef(W(r)). Поэтому соотношение E7), требуемое в теореме 12, следует из того, что Law(X | P) = = Law(X | Р), и простого факта, относящегося к теории меры, который вытекает из следующего упражнения. 19. Пусть Р и Q — вероятностные меры на метрическом пространстве (S, < Пусть для любой непрерывной и ограниченной функции /: S —> М_|_ f(x)P(dx) = f f(x)Q(dx). s Js Тогда Р = QH Итак, (П, i^, P) есть то расширенное вероятностное пространство, о котором и говорится в формулировке теоремы 12. Для завершения доказательства теоремы осталось лишь проверить, что при усло- условии ЕХ2 < оо имеет место равенство E8). 20. Используя второе тождество Вальда (упражнение 14), докажите, что Era^ = -ah. G6) Воспользовавшись равенством G6), теоремой Фубинии формулой G5), имеем Et=E'E(ty,z) = E'(-YZ). G7) Отсюда, принимая во внимание G1) и F8), находим: Er = E\-YZ) = f dF(y)(-y) f dF(z)z(z-y)c-1 = ./(-oo,0] J@,oo) dF(y)(-y){-y+ [ dF(z)c-1z2\ = o,0] I -/@,oo) = f y2dF(y)+ f z2dF(z) = ./(-oo,0] ./@,oo) f () = EX2. @,oo) Тем самым, требуемое свойство E8) (с соответствующими переобозначениями для расширенного вероятностного пространства) установлено. ? Теперь нам понадобится следующее обобщение понятия броуновского движения. Определение 7. Процесс W = {W(t), t ^ 0} называется действительным броуновским движением относительно фильтрации (^)t^o? если 1) W{t) является ^~измеРим°й величиной при каждом t ^ 0; 2) W(t) - W(s)±&s при 0 ^ s < t (т.е. W(t) - W(s) не зависит от &s); 3) Wo = Оп.н. nW(t) -W(s) - N(O,t- з)приО ^ s < t; 4) траектории процесса W непрерывны п. н.
104 А. В. Булинский, А. Н. Ширяев Данное определение очевидным образом распространяется и на m-мерное броу- броуновское движение: тогда в условии 3) требуется, чтобы W(t) — W(s) ~ N @, (t — s)I) при 0 ^ s < ?, где / — единичная матрица порядка т. Заметим, что если W будет броуновским движением относительно фильтрации (^t)t^o, т0 — и относительно естественной фильтрации (&t^)t^o- Когда гово- говорится о броуновском движении без упоминания фильтрации, то имеется в виду ес- естественная фильтрация. Пусть JV — класс событий нулевой вероятности (полного) вероятностного про- пространства (П, ^ Р). Без потери общности будем считать, что в определении 7 каж- каждая сг-алгебра ^ (t ^ 0) содержит все события из JV. Теорема 13 (Скороход; см. [71]). Пусть Xi,X2,... — последовательность центрированных независимых случайных величин, определенных на некотором вероятностном пространстве. Тогда существует такое вероятностное пространство, на котором найдутся последовательность случайных величин {Tkjk^i и броуновское движение W = {W(t), t ^ 0} такие, что {хк, кеЩ = {W(Tk) - W(Tfc_i), к е N}, G8) где неотрицательные величины Тк — T^-i, к Е N (То = 0), независимы. Если ЕХ| < оо, то в дополнение к указанным свойствам последовательности можно обеспечить соотношение Е (Т& — T^-i) = EX|; k G N. Доказательство. Пусть {(Yn, Zn)}n^i — последовательность независимых случайных векторов со значениями в М2, причем {(Yn, Zn)}n^iJLW, где W = = {W(t), t ^ 0} — броуновское движение (относительно фильтрации (^t^)t^o)- Легко построить вероятностное пространство и задать на нем все указанные случайные элементы. Для t ^ 0 положим J% = а{^^, {(Уп, Zn)}n^i}, т.е. рассмотрим сг-алгебру, порожденую величинами W(s) при s G [0, t] и (Уп, Zn)n^i. Будем считать ее расширенной классом J/ нулевых событий. Воспользуемся следующим упражнением. 21. Покажите, что W — броуновское движение относительно введенной фильт- фильтрации («#?t)t^o- Продолжим доказательство теоремы 13. Пусть при каждом п G N распределение вектора (Уп, Zn) строится по распределению величины Хп так, как это делалось для (Y, Z) и X в формуле F4). Определим теперь последовательность случайных величин 0 = То ^ Т\ ^ • • •, положив Тп = inf{t > Гп_!: W(t) - W(Tn_i) G {Yn, Zn}}, n G N. Нетрудно проверить, что Тп — конечные п.н. марковские моменты относительно фильтрации (J^i)t^o- Небольшая модификация строго марковского свойства броу- броуновского движения (теорема 4) показывает, что при каждом п = 0,1,... процесс #(™) = {в[п) = W(Tn + t)- W(Tn), t ^ 0} есть броуновское движение, не завися- зависящее от а-алгебры Жтп• Кроме того, dn := a{Tk, W(Tk); к ^ п) С №тп- Поэтому В^±^п,п = 0,1,... . Теперь заметим, что (Yn+1, Zn+1)±a{a{B^},dn}. Сле- Следовательно, ((Уп+1, Zn+i), B^^JLs^n. Отсюда заключаем, что независимы в сово- совокупности пары величин (Tn+i — Tn, W(Tn+i) — W(Tn)), n = 0,1,.... Согласно
Гл. III. Броуновское движение. Свойства траекторий 105 теореме 12 и в силу построения величин (Уп, Zn) распределение W(Tn+i) — W(Tn) совпадает с распределением Хп (п Е N). ? По поводу разных конструкций случайных величин Ti, T2,..., участвующих в G8) и обладающих некоторыми свойствами "минимальности", см. [182]. В упомянутой книге строится такое броуновское движение W, что величины Т& (к Е N), фигури- фигурирующие в теореме 13, являются моментами остановки относительно естественной фильтрации (^tV)t^o- Обратимся к дальнейшим обобщениям введенного в главе II броуновского дви- движения. Определение 8. Фрактальное (или дробное) броуновское движение с пока- показателем Н Е @,1] — это центрированный гауссовский процесс В^н^ = {B^H\t), t ^ 0} с ковариационной функцией t) = ±(s2H + t2H-\t-s\2H), s,t^O. G9) При Н = 1/2 получаем стандартное броуновское движение. Константа Н носит название параметра Харста. 22. Докажите, что функция, фигурирующая в правой части G9), является кова- ковариационной в том и только том случае, когда Н Е @,1]. Определение 9. Процесс X = {X(t), t ^ 0} называется процессом со ста- стационарными приращениями, если для каждого п ^ 1, любых 0 ^ t\ < • • • < tn и произвольного h > 0 (X(t2) - X(h),..., X(tn) - X(tn-!)) = i (x(t2 + h) - x(h + л),...,x(tn + h) - x(tn-! + h)), где = означает равенство по распределению. Из приводимых ниже двух упражнений вытекает, что фрактальное броуновское движение — это процесс со стационарными приращениями. 23. Пусть X = {X(t), t ^ 0} — гауссовский процесс такой, что для всех 0 ^ s ^ ^ t < 00 Е (X(t) - X(s)) = (t- s)c, D(X(t) - X(s)) = f(t - s), (80) где с G M, а функция /: M+ —У IR+. Тогда X — процесс со стационарными прир ащениями. 24. Покажите, что для фрактального броуновского движения с показателем Я G @,1] J = \s-t\2H, 8,t>0. (81)
106 А. В. Булинский, А. Н. Ширяев Пусть (Т, р) — метрическое пространство с некоторой фиксированной точкой в. Определение 10. Броуновской (действительной) функцией Ле в и на множест- множестве Т называется действительный гауссовский процесс BL = {BL(t), t G T} с нуле- нулевым средним и ковариационной функцией RL(s,t) = ±(p(s,e)+p(t,9)-p(s,t)). (82) Данное определение предполагает, что метрика р такова, что формула (82) задает неотрицательно определенную функцию. Вообще говоря, для произвольной метри- метрики р это может оказаться не выполненным. 25. Гауссовская случайная функция X = {X(t), t G Т} является броуновской функцией Леей на Т тогда и только тогда, когда Х(в) = 0 п. н. и E(X(s)-X(t)J=p(s,t) (при этом предполагается неотрицательная определенность функции, введен- введенной в (82)). Определение 11. Если (Т, || • ||) — нормированное пространство (в частности, ~Rd с евклидовой нормой | • |), то броуновским движением Леей называется гаус- гауссовская случайная величина с нулевым средним и ковариационной функцией Д(М) = ^(Nl + ||t||-II*-*||), s,t&T. (83) В многомерном случае аналог формулы G9) приводит к определению случайного поля Леви-Шемберга как центрированной гауссовской случайной функции V^ ' = = {V(H\t), t G W± = [0, oo)d}, H G @,1], для которой ковариационная функция д(")(М) = 1(|в|2" + |г|2"-|г-в|2"), s,tGRd+. (84) Определение 12. Случайным полем Винера-Ченцова называется действи- действительная гауссовская функция X = {Х(?), t G ffi^J.} такая, что d EX(t) = 0 и cov(X(s),X(t)) = Д mm{sk,tk}, (85) k=l где? = (?i,...,?d),s = (sb... ,sd) G M+. Покажем, что такое поле существует. Пусть Wk = {Wk(t), t ^ 0}, к = 1,..., d, — независимые броуновские движения. Положим Y(t) = VFi(ti) • • • Wd(td) для teW[= [0, oo)d. Тогда EY(t) = 0 и cov(F(s),F(t)) = Д min{^,tfc}. (86)
Гл. III. Броуновское движение. Свойства траекторий 107 d Следовательно, Yl ти1{8& •> tk } является симметричной неотрицательно определен- к=1 ной функцией и требуемое утверждение о существовании случайного поля Винера- Ченцова следует из теоремы 3 главы П. Заметим, что X(t) = 0 п.н. для точек ?, лежащих на координатных плоскостях. Случайную функцию X = {X(t), t Е ffi^J.} можно считать индексированной "прямо- "прямоугольниками", т.е. X(t) = Х(@, ?]), где Построение броуновского движения, индексированного множествами, с помощью функций Хаара проводится в работе [174]. Назовем приращениями случайного поля X = {Xt, t G ld} на классе прямо- прямоугольников (параллелепипедов) В = (a, b] = (ai, bi] х • • • х (а^, Ь^] С Md случайные функции вида Х(В) = ^(-l)lle"A:(ei0i + A - eiNi,... ,?dad + A - ed)bd), (87) где сумма берется по всем векторам е — (si,..., г^), имеющим компоненты 0 или 1, 26. Пусть Х- поле Винера-Ченцова. Докажите, что {Х(В), В G П}, где П — совокупность "прямоугольников" В = (а, Ь] С М+, есть гауссовская слу- случайная функция. Найдите ее среднее значение и ковариационную функцию. Докажите, что для любого п ^ 2 и непересекающихся прямоугольников i?i,..., 5n G П величины X(i?i),..., Х(Вп) — независимы. Дайте определе- определение поля Винера-Ченцова в терминах величин Х(В), В G П. Определение 13. Процессом Орнштейна-Уленбека с параметрами а, E > 0 называется процесс, задаваемый формулой Vt = e-CtW{ae2Ct), t e M, (88) где VF = {W(t),t ^ 0} — некоторое стандартное броуновское движение. 27. Докажите, что У = {Vt,t G М}—гауссовский процесс и найдите его ковариа- ковариационную функцию. Со многими свойствами броуновского движения (и более общих процессов) можно ознакомиться, например, по книгам [31, 43, 48, 81, 182].
Глава IV Мартингалы. Дискретное и непрерывное время Мартингалы, субмартингалы, супермартингалы. Примеры. Разложение Дуба. Компенсаторы. Дискретный вариант формулы Танака. Расширение фильтра- фильтрации. Квадратическая характеристика. Квадратическая вариация. Теорема Дуба о свободном выборе. Применение к случайным блужданиям (задача о разорении). Максимальное и минимальное неравенство Дуба для субмартингалов. Лемма о числе пересечений. Теорема о сходимости субмартингалов. Ветвящийся процесс Гальтона- Ватсона. Сходимость мартингалов в L (П, ^, Р). Теорема Леви. Фундаменталь- Фундаментальная теорема страховой математики. Некоторые неравенства для субмартингалов и мартингалов с непрерывным временем. § 1. Цель данной главы — дать изложение ряда фундаментальных результатов теории мартингалов, имеющей разнообразные приложения. При первом чтении главы IV желательно ознакомиться с определениями из § 1 и примерами из § 2. Строе- Строение субмартингалов с дискретным временем выявляет разложение Дуба (§ 5). Прос- Простое применение этого разложения дает § 6. Мотивы расширения фильтрации клас- классом нулевых событий раскрывает § 7. Важные результаты о сохранении свойств мар- тингальности (субмартингальности) при случайной замене времени содержит §9, их иллюстрирует классическая задача о разорении (§11). Мартингальными метода- методами (с использованием леммы из § 18) в § 19 доказывается фундаментальная теорема страховой математики. К остальным результатам этой главы можно вернуться при повторном чтении. Прежде всего напомним, что условное математическое ожидание Е (? | «с/) дейст- действительной случайной величины ? относительно сг-алгебры si С 3* определяется как такая функция (: П —у М, что 1) является si | ?$(Ж)-измеримой; 2) для каждого С Е si математические ожидания Е Aс и Е ?1с определены и Из теоремы Радона-Никодима вытекает (см., например, [35; гл. 6, §5]), что если Е |?| < оо, то такая величина ? существует и определена однозначно с точностью до эквивалентности. (Величины (д и B эквивалентны, или стохастически эквивалентны ((д ~ ?2), если Р((д = B) = 1.) В векторном случае, когда
Гл. IV. Мартингалы. Дискретное и непрерывное время 109 ? = (?ъ • • • 5^п): ^ —> ^п и существуют математические ожидания компонент, по определению полагаем Отметим, что взятие условного математического ожидания представляет собой обобщение операции ортогонального проектирования (см. упражнение 1). Пусть задано вероятностное пространство (П, 3, Р) и некоторая фильтрация (^t)ter, где Гс1, т.е. неубывающее семейство сг-алгебр 3t, t ? Г, таких, что f s С f t С f при s ^ t, s, ? G Г. Определение 1. Процесс X = {Х^,? Е Т},гдеХ^: П —>- М, называется мартин- мартингалом (относительно фильтрации (<^t)tGT), если выполнены следующие условия: 1) случайный процесс X = {Xt, t Е Т} согласован с фильтрацией (&t)teT, т.е. Х^ является j^t-измеримой величиной при каждом ? Е Т; 2) E|Xt| < оо, ? GT; 3) Е (Xt | J^s) = Xs (п.н.), s,teT,s^t. Часто, чтобы подчеркнуть роль согласованности в данном определении, про- процесс X обозначают (Xt, ^t)teT- Если вместо свойства 3) требуется, чтобы Е (Xt \^s) ^ Xs п.н., то говорят, что процесс (Xt)t?T является субмартингалом. Замена 3) условием Е (Xt \ ^s) ^ ^ Xs п.н. приводит к определению супермартингала. Очевидно, что процесс (Xt, ^t)teT является супермартингалом тогда и толь- только тогда, когда (—Xt^t)teT — субмартингал. Это объясняет, почему будет говориться, в основном, только о мартингалах и субмартингалах. Мартингал, разу- разумеется, является одновременно субмартингалом и супермартингалом. Заметим, что в данном определении условие 3) равносильно тому, что при всех s,? G T,s ^ ?,и любом A G 3*s имеет место равенство f XsdP= f Xt dP. A) J A J A Свойство субмартингальности переформулируется аналогично A) с заменой знака равенства "=" на знак "^" Условие 3) влечет, очевидно, соотношение EXt = EXS для всех s,t G Т. Если (Xt,^t)teT — мартингал (субмартингал) и имеется фильтрация (^)гет такая, что ^t С i^t, при этом Xt E ^t \ $(Щ при t G Т, то (Xt, ^t)teT — также мартингал (субмартингал). Это вытекает (см. [85; т. 1, с. 270-271]) из телескопи- телескопического свойства условного математического ожидания: для сг-алгебр si\ С &$2 С 3* и интегрируемой случайной величины ? E(E(e|«/i)|^2)=E(E(^|^2)|«/i)=E(^|«/i) п.н. B) В частности, если в качестве (^)гет взять естественную фильтрацию &* = a{Xs, s ^ ?, s G Г}, t e Г,
ПО А. В. Булинский, А. Н. Ширяев то (Xt,^t<:)teT останется мартингалом (субмартингалом). Однако относительно более "бедных" (нежели &?¦) сг-алгебр % процесс (Xt,cSt)teT не будет мартинга- мартингалом, поскольку в силу свойства 1) необходимым образом &* С ^ Для t Е Т. Когда в пространстве (П, ^, Р) явно не указывается фильтрация (&t)teTiT0 обыч- обычно процесс X = {Xt, t G Т} называют мартингалом, подразумевая использование естественной фильтрации (&*)гет- Однако часто фильтрацию (&t)teT указыва- указывают сразу непосредственно в записи исходного вероятностного пространства (п,3,Р), т.е. пишут (П, 3, (&)teTi P)? и говорят о фильтрованном вероятностном про- пространстве, или о стохастическом базисе. Кроме того, если на одном и том же измеримом пространстве (П, ЗР) приходится иметь дело с семейством SP — {Р} ве- вероятностных мер Р, то говорят о фильтрованном стохастическом эксперимен- эксперименте (П, 3, (&t)teTi &)• Если оперируют с конкретной мерой Р из семейства ?Р, то вместо записи "(Xt, &t)teT — мартингал" иногда отмечают, что "(Xt, ^ ? Р)*ет — мартингал", и пишут, что свойство 3) выполняется Р-п. н. Определение мартингала распространяется и на векторные случайные процессы со значениями в Ж171, при этом требуется, чтобы условия 1), 2) и 3) выполнялись для каждой компоненты. Для Т = Z+ = {О,1,...}в условии 3) достаточно рассматривать лишь значения t = s + 1. Иначе говоря, достаточно (и необходимо) требовать, чтобы E(AXn|^n_i) =0 п.н, где АХп = Хп - Хп_ь п е К C) Определение 2. Последовательность (^п^п)п^о^ гДе случайные величины ?п — интегрируемы и согласованы с потоком сг-алгебр (^п)п^о, называется мартингал-разностью, если E(fn|^n-i) = 0 п.н., n^O. Таким образом, свойство C) показывает, что процесс (Xn, ^n)n^i — мартингал тогда и только тогда, когда (АХП, ^п)п^о — мартингал-разность (АХо := 0). § 2. Приведем ряд примеров. Пример 1. Пусть X = {Х^, ? G Т} — процесс в Мт с независимыми приращени- приращениями, где Г С М+, Xt = (X/,..., Xtm), причем Е Xt = a G Жт для всех t G Г. Тогда (Xt)t^T - мартингал (относительно естественной фильтрации). Действительно, при s,tGT,s^t,ii естественной фильтрации (^t^)teT имеем Е (Xt | ^f) = Е (Xt - Xs + Xs | &X) = E(Xt- Xs) +XS=XS п.н. D) В частности, m-мерный винеровский процесс является мартингалом. Процесс {Nt — Е А^, t ^ 0}, где N = {Nt, t ^ 0} — пуассоновский процесс, также явля- является мартингалом. Последовательность Sn = ?1 + • • • + ?n, п G N, где ?1, ?2 ? • • • — независимые векторы в Mm, образует мартингал тогда и только тогда, когда Е?п = 0, п G N. Заметим, что a {Si,..., Sn} = cr{?i,... ,?n}, n G N. Соотношение D) показывает, что вместо естественной фильтрации можно было бы взять любую фильтрацию (&t)teTi лишь бы Xt были ^~измеРИМЬ15 Xt — Xs не зависели от 3е' s при ?^з,иЕХг=аЕ Ж171 для всех t G Т.
Гл. IV. Мартингалы. Дискретное и непрерывное время 111 Пример 2. Пусть ?n, n Е N, — независимые действительные величины и п Е (п = 1 при всех п. Положим Хп = YI Сь &п — 0"{Съ • • • Хп}, п ? N. Тогда, fc=i очевидно, (Хп, ^n)nGN — мартингал. Определение 3. Пусть вероятностные меры Р и Q заданы на некотором фильт- фильтрованном измеримом пространстве (П, ^, (&t)teT) (т-е- измеримом пространстве, снабженном фильтрацией). Говорят, что мера Q локально абсолютно непрерыв- 1ос wa относительно меры Р (обозначение: Q < Р), если при каждом t Е Т сужение Qt = QI меры Q на ^ абсолютно непрерывно (см. § 14 главы I) относительно сужения Pt = Р | д. меры Р на ^. Пример 3. Пусть Q < Р и #t = dQt/dPu t G Г. Тогда процесс (gt,&t,P)teT является мартингалом. Действительно, для s ^ t и В G ^s C^t видим, что в силу ^-измеримости ^s (и, соответственно, = [ gs dPs = QS(B) = Q(B) = Qt(B) = [ gt dPt = [ gt dP, в Jb Jb Jb что и доказывает требуемое утверждение (см. A)). Пример 4 (мартингальноепреобразование). Пусть (П, ^, (&п)п^о, Р)) -фильт- -фильтрованное вероятностное пространство, ^о = {0,П}иМ = (Mn, ^n)n>o ~ мартин- мартингал. Процесс X = (Хп, ^п)п^о, полученный из М по формуле п Xn=X0 + Y,<PkbMk, n^l, E) k=i где if = {ifn^n-i)n^i — предсказуемая последовательность, т.е. срп являются 3*п-1 -измеримыми, причем (для простоты) \ipn\ ^ С = const, носит название мартингалъного преобразования. Процессы подобного типа часто используют при описании эволюции капитала в моделях азартных игр (см. подробнее [85; т. 1, с. 653]). Заметим, что если срп ^ 0 и М — субмартингал (супермартингал), то таковым же будет и процесс X. п Выражения ^2 сркАМ^ с предсказуемыми ср = ((pk)k^i иногда называют "дис- к=1 кретными стохастическими интегралами функции ip по процессу М" Они возни- возникают естественным образом при определении понятия "стохастического интеграла / cps dMs" (см. далее главу VIII). </О Пример 5. Пусть (&t)teT — некоторая фильтрация и ? — интегрируемая случайная величина. Положим Xt = E(?| i^), t G Т. Из "телескопического" свойства B) вытекает, что (Xtj^t)teT — мартингал. Этот процесс называют мартингалом Леей. § 3. Пусть Y = {Yt, ? ^ 0} — действительный процесс с независимыми прираще- приращениями такой, что при некотором aGln всех s, t ^ 0 EeQY* < оо, Ееа(у*-у«> < оо. F)
112 А. В. Булинский, А. Н. Ширяев Рассмотрим Ясно, что если ^t = ? s , T0 (8) Отсюда понятно, что E(Zt — ^s ПРИ s ^ ? в том и только том случае, когда EeaYt aYs eaYs (9) Таким образом, справедливо следующее утверждение. Лемма 1. Пусть Y = {Yt, t ^ 0} — действительный процесс с независи- независимыми приращениями, удовлетворяющий условию F). Тогда процесс Z = {Zt, t ^ 0}, где Zt определено в G), является мартингалом тогда и только тогда, когда справедливо соотношение (9). § 4. Широкую возможность строить примеры субмартингалов предоставляет Лемма 2. Пусть (Xt,^t)teT — мартингал, h: Ж —у Ж — выпуклая функция и величина Yt = h(Xt) интегрируема при каждом t Е Т. Тогда (Yt,^t)teT — субмартингал. Если функция h не убывает, то утверждение леммы сохра- сохранится и в случае, когда (Xt,^t)teT — субмартингал. Доказательство . Неравенство Иенсена для условных математических ожида- ожиданий (см. [85; т. 1, с. 297]) дает, что с вероятностью единица при s ^ t (s,t G T) Z-./V"\ Z-. (С ( ~у I 67: \\ S* U. (У* ( V \ \ <Ж \ (Л Г\\ ILyYv g} — /ML ^У\. t <-s s) j ^ '— I \ ?/ s I ' \ ) Если [Xt ,&t)teT — субмартингал и h не убывает, то равенство в A0) заменяется на неравенство h(Xs) ^ h(E (Xt \ &s)) п-н- П §5. Следующий результат, называемый р а зло жен и ем Дуба, является ключе- ключевым при использовании мартингальных методов для анализа стохастических по- последовательностей. Соответствующий результат для случая непрерывного време- времени, п&зыв&емый разложением Дуба-Мейера, приведен в дополнении к главе VIII. Теорема 1 (Дуб). Пусть на вероятностном пространстве (П,^, Р) задан интегрируемый (при каждом п) случайный процесс X = {Хп, п ^ 0}, согласо- согласованный с фильтрацией (^n)n^o- Тогда существует единственное (п.н.) раз- разложение вида X = М + А такое, что Хп = Мп + Ап для п ^ 0, где М = (Мп, ^п)п^о — мартингал и А — (An,^n-i)n^o — предсказуемый процесс с Ао = 0 и 3*-\ = {0,Щ. В частности, в представлении X = М + А процесс (Хп,^п)п^о является субмартингалом тогда и только тогда, когда процесс А не убывает, т. е. ААп ^ 0 п. н. при п ^ 1.
Гл. IV. Мартингалы. Дискретное и непрерывное время 113 Доказательство. Если имеет место указанное представление для X, то в си- луC) ААп = Е (АХп | J^n_i) при всех га ^ 1 и, следовательно, п ^^-i), n^l, A1) к=1 что доказывает п. п.-единственность разложения X = М + А с предсказуемым процессом А = (Ап, &n-i)n^0i Для которого Aq = 0. Пусть теперь X есть процесс, удовлетворяющий условиям леммы. Определим предсказуемый процесс А для га ^ 1 формулой A1) и положим Ао = 0. Тогда М = X — А есть мартингал, поскольку условия 1), 2) в определении 1, очевидно, выполнены и E(AMn|^n_i) = E(AXn|^n_i)-AAn=0 п.н., п > 1. A2) Тем самым, требуемое разложение X = М + А построено. Утверждение теоремы, относящееся к субмартингалам, сразу следует из формул A1) и A2). ? § 6. Покажем, что даже в очень простых случаях разложение Дуба приводит к ин- интересным результатам (как, например, соотношение A9) ниже, которое обычно уста- устанавливается с привлечением комбинаторных методов). Пример 6. Пусть So = 0, Sn = S\ + • • • + еп, где ?i, ?2? • • • — независимые бер- нуллиевские величины такие, что P(sn = 1) = P(sn = —1) = 1/2, п G N. В силу леммы 2 процесс X = {Хп, п ^ 0} с Хп = \Sn\ является субмартингалом (отно- (относительно фильтрации (^n)n^o с ^п — &{?ij • • • ,?п} Для п ^ 1 и ^о = {05 ^})- Найдем его разложение Дуба. Имеем АХп = |5П| - |5n_i|, га ^ 1. Тогда АМП = АХП - ААп = АХп - Е (АХп \ ^п_г) = = |5n|-E(|5n||^n_i), n^l. A3) Заметим, что \Sn\ = |5n_i + еп| = (Sn-г + епЩБп-! > 0} + l{Sn_i = 0}- i<0}, A4) где 1{.В} — индикатор множества В. Поэтому = Е ((Sn-г + en)l{Sn_i > 0} | &n-i) + E (l{5n_i = 0} | ^n_i) - - E ((Sn-! + еиI{5п_1 < 0} | &n-i) = 1<0}, n^l, A5)
114 А. В. Булинский, А. Н. Ширяев здесь мы учли, что Е (еп \ 3*п-\) = Егп = 0. Из A3)—A5) следует, что к=1 где Согласно A1) Ап = п ^2 Е (АХк к = 1 sgnx f = I n = E fc=i 1, o, -1, (E(| X X X > = < _1 o, o, 0. A6) Из A5) видно, что Е (ДХ& | ^-i) = ^-{sk_1=o} Для & ^ 1? поэтому An = Ln@), где т.е. I/n@) — это число нулей последовательности (Sk)o^k^n-i- Таким образом, п \Sn\ = ^(sgn5fc_!)A5fc +Ln@), A7) что есть дискретный аналог известной формулы Танака для модуля броуновского движения (см., например, [86; т. 2, с. 375]). Из A7) вытекает, что ELn@) = E|Sn|. A8) Пользуясь центральной предельной теоремой (о слабой сходимости распределений случайных величин Sn/y/n к стандартному нормальному закону N@,1)) из A8) на- находим, что ELn@) ~ \ —п при п ->• оо. A9) V 7Г Общим вопросам теории слабой сходимости посвящена следующая глава и допол- дополнение к ней; доказательство A9) отнесено в пример 2 главы V. § 7. На используемую фильтрацию, как правило, налагаются некоторые ограни- ограничения, что поясняет Замечание 1. В теореме 1 мы требовали, чтобы Aq = 0 вместо Aq = 0 п.н., хотя обычно все равенства и неравенства со случайными величинами понимаются выпол- выполненными почти наверное. Дело в том, что последняя запись не гарантировала бы, что А\ — Е (AXi | ^о) + ^.о является i^o-измеримой величиной. Чтобы избежать подоб- подобных неприятностей, всегда предполагают, что исходное вероятностное пространство (П,^, Р) полно и фильтрация (&t)teT расширена, т.е. каждая сг-алгебра SPt со- содержит класс Р-нулевых множеств /cf (для простоты рассматриваем одну меру Следующее предложение показывает, почему переход к расширенной фильтрации не только удобен, но и не снижает общности рассуждений.
Гл. IV. Мартингалы. Дискретное и непрерывное время 115 Лемма 3. Пусть X = (Xt,3t)teT — мартингал (субмартингал), Г С I. Тогда процесс X = (Xt,3t)teTi яде 3<t — a{3t,^}, t G T, также является мартингалом (субмартингалом). Доказательство. Очевидно, (Xt, &t)teT удовлетворяет условиям 1) и 2) опре- определения 1 (учитываем, что 3*t С 3t, t G Т). Выберем s ^ t (s, t G T) и покажем, что E (Xt | ^s) = Xs п.н. Действительно, Xs e 3< s \ ?%(Щ и для любого А е 3< s найдутся В Е 3* s и С Е сЖ такие, что А = BUC (при этом в силу полноты ^ можно считать В П С = 0). Применяя A), имеем f XsdP = f Xs dP = [ XtdP= [ Xt dP, B0) ^A ^B ^B ^A где мы воспользовались тем, что если существует Е?иР(С) = 0, то Е ?1с = 0. Для субмартингала второе равенство в B0) заменится на неравенство "^" П § 8. Введем ряд важных характеристик мартингалов и субмартингалов. Определение 4. Пусть X = (Xnj3n)n^o — субмартингал. Неубывающая предсказуемая последовательность А = (An,3n-i)n^o, фигурирующая в разло- разложении Дуба (Ао = 0, 3*-\ — {0,П}), называется компенсатором процесса X ("компенсация", т. е. переход от X к X — А, превращает субмартингал в мартингал). Пусть М = (Мп,Зп)п^о — квадратично интегрируемый мартингал, т.е. 2 = (М^ З < оо, п ^ 0. Тогда в силу леммы 2 получаем, что М2 = (М ^, субмартингал. По теореме Дуба М% = тп + (М)п, где т = (mn, З^ мартингал, а (М) = ((M)n, 3n-i)n^o — компенсатор, называемый в рассматрива- рассматриваемом случае квадраттической характеристикой мартингала М. Из A1) для п ^ О находим, что (М)п = 53 Е (АМ| | ^fc_!) = 53 Е fe=i fe=i B1) где мы воспользовались тем, что Е ( для любого к ^ 1. Квадратическую характеристику естественно рассматривать как аналог диспер- дисперсии. В том случае, когда Mq = 0 и Мп = ?i + • • • + ?п, п ^ 1, где ?i, ?2? • • • — неза- независимые центрированные величины с Е^ < оо (к ^ 1), формула B1) показывает, что (М)п = Е D& = DMn, fe ^ 1. Определение 5. Квадратической вариациейпроцессах = {Xn, n ^ 0} назы- называется процесс [X] = ([Х]п)п^о5 задаваемый формулой п ^ 1 и [Х]о = 0. B2) fc=i
116 А. В. Булинский, А. Н. Ширяев § 9. Обратимся к некоторым весьма полезным результатам относительно сохране- сохранения свойства мартингальности (субмартингальности) при случайной замене време- времени. Для этого нам понадобится еще одно простое свойство условных математических ожиданий (напомним, что совпадение сг-алгебр на некотором событии определялось в § 5 главы III). Лемма 4 (свойство локальности условного математического ожидания). Пусть даны а-подалгебры ^,Ж С 3* и случайные величины ^,Т] Е L1(Q,^r, P), причем на некотором событии А (из <? П Ж) *& = Ж и ? = rj (п.н. на А). Тогда Е (? | <S) = Е (ту | Ж) п. н. на А. B3) Доказательство . Функции l^E (? | Щ и l^E (т\ \ Ж) являются ^ П ^-измери- ^-измеримыми. Следовательно, D = А П {Е (? | Щ > Е (ту | Ж)} G ^ П Ж Далее, | »)) = Е A^0 = Е A^ту) = Е Если С ^ 0 п.н. иЕ( = 0, тоС = 0 п.н. Поэтому Е (? | ^) ^ Е(г1\Ж) п.н. на Аналогично устанавливается, что Е (? | ^) ^ Е G71 J^) на А. П Пусть X = (Xnj3n)n^o — стохастическая последовательность, т.е. стохас- стохастический процесс {Хп, п ^ 0}, согласованный с некоторой фильтрацией (i^n)n^o- Марковские моменты мы будем рассматривать относительно этой фильтрации. При этом марковский момент т назовем ограниченным, если т ^ к п.н. для некоторо- некоторого натурального к. Положим Хт^(ш) = 0, если г (о;) = оо (Р(т = оо) = 0 для ограниченных г). Теорема 2 (о свободном выборе или об остановке; Дуб). Пусть стохастичес- стохастическая последовательность X = (Xnj3n)n^o такова, что Е|ХП| < оо, п ^ 0. Тогда следующие условия эквивалентны: 1) X = (Хп, &п)п^о — мартингал (субмартингал); 2) Е(ХТ\3(Т) = (^)ХтАа для любого ограниченного марковского момен- момента г и любого марковского момента а; 3) ЕХТ = (^)ЕХа для любых ограниченных марковских моментов г и а таких, что т ^ а п.н. В данной формулировке и приводимом ниже доказательстве знак "(^)" от- относится к рассмотрению субмартингалов. Доказательство. 1) => 2). Пусть г ^ к п.н., где число к G N. Тогда п=0 и, следовательно, условное математическое ожидание Е (Хт \ &?) существует для лю- любой сг-подалгебры sd С 3.
Гл. IV. Мартингалы. Дискретное и непрерывное время 117 Докажем, что Е (Хт \ ^а) = (^)Ха на множестве {г ^ а}. Достаточно убедить- убедиться, что ЕХт1Вп{т^т} = (^)ЕХт1Вп{г^т},гдет ^ О, В = An {a = m} n А — произвольное событие из 3*G. Имеем = Е ХШ1 Bn^T=rny + EXmlBnjT>mj. E(E(Xm+i | ^гоIвп{т>го}) B4) По лемме 4 на множестве {г ^ а}, входящем согласно лемме 1 главы III в <^v, полу- получаем, что Е{Хт\&1Г) = Е(ХтЛ<т\&1Г) = ХтЛ<т, B5) где мы воспользовались тем, что ХтА(Т является j^rA о--измеримой величиной, а ^тАсг с &*&. Из B4) и B5) следует утверждение 2). 2) => 3). Очевидно, ЕХТ = (^)ЕХтАа = ЕХа, поскольку ХтАа = Ха п.н. при г ^ а п.н. 3) => 1). Пусть г и сг — марковские моменты, причем т ^ к п. н., где & есть некото- некоторое натуральное число. Положим а = г Л а и возьмем произвольное A G ^а- Легко видеть, что ал = «1а + 00I34, т~А = т~1а + 00I34, /? = «А Л к, S = та /\к есть марковские моменты и /3 ^ ? ^ к для всех cj G П (считаем оо-0 = 0,оо-1 = оо). Поэтому ЕХ$ = (^)ЕХ^. Следовательно, Таким образом, Учитывая, что ХтАа является ^до-измеримой величиной, а также замечание пос- после формулы A), приходим к соотношению Е(Хт\&тАG) = (^)ХтАG. B6) Возьмем т = п и а = ш, где 0 ^ ш ^ п (ш, n G Z+). Тогда 3<тАG — &ш по лемме 1 главы III, и A6) превращается в утверждение 1). ? Из доказанной теоремы немедленно вытекает Следствие 1. Стохастическая последовательность X = (Хп, ^п)п^1 явля- является мартингалом (субмартингалом) тогда и только тогда, когда все вели- величины Хп интегрируемы и для любой неубывающей последовательности мар- марковских моментов тп, каждый из которых ограничен (т. е. тп ^ rn+i п. н. для п ^ 1 и тп ^ кп п.н. для некоторых натуральных чисел /сп), последователь- последовательность (ХТп,^Тп)п^о есть мартингал (субмартингал).
118 А. В. Булинский, А. Н. Ширяев Следствие 2. Пусть X = (Хп, ^п)п^о — мартингал. Пусть т — момент остановки такой, что |Хтлп| ^ с п. н. для некоторой неотрицательной кон- константы с и всех п ^ 0. Тогда ЕХТ = ЕХо. Действительно, Хт/\п —У Хт п.н. при п —У оо (как обычно, полагаем Хт = 0 на множестве {г = оо}, имеющем вероятность нуль). По теореме Лебега о мажори- мажорируемой сходимости величина Хт интегрируема и Е ХтАп —у Е Хт при п —У оо. В силу утверждения 3) теоремы 2 имеем Е1тЛп = ЕХ$. Отсюда получаем требуемый результат. § 10. Наиболее полезным утверждением доказанной теоремы 2 является, конеч- конечно, импликация 1) => 2). Приведем некоторые условия, гарантирующие выполнение этой импликации и для неограниченных моментов. Следствие 3. Пусть (Хп,^п)п^о — мартингал (субмартингал), т — мо- момент остановки такой, что Е|Хт|<оо B7) и liminfE|Xn|l{T>n} =0. B8) Тогда для любого марковского момента а Е(ХТ\^(Т) = (^)ХТА<Т. B9) В частности, если а — момент остановки такой, что а ^ г (п. н.) и Е \Ха\ < оо, то ЕХТ = (^)ЕХа. C0) Доказательство. По теореме 2 для ограниченных марковских моментов гдг = = т Л N для N ^ 1 имеем Очевидно, Хт-ллгло- —> ХтАа п.н. при N —У оо. Поэтому достаточно убедиться, что для некоторой подпоследовательности {п^}, п^ —У оо (к —У оо) справедливо соотношение Е (ХтАПк \&а)^Е (Хт | &а) п.н. при к -> оо. C1) Условие B8) равносильно тому, что существует последовательность {ttij}, rrij —у оо (j —у оо), для которой Нт Е|Хт |1{Т> }=0. C2) Тогда в силу C2) Е Хт/\тп. — Хт\ ^ Е \Xjn- 1{т>ш,-} + Е |XT|l{r>mj} -> 0 C3) при j —у оо, поскольку Е |Хт|1{т>т.} —У 0 (j —У оо) по теореме Лебега о мажори- мажорируемой сходимости (г < оо п.н., поэтому 1{т>т} —У 0 п.н. при rrij —у оо). Выберем теперь из последовательности {tuj} подпоследовательность {п^} так, чтобы обеспечить C1) (из последовательности, сходящейся в пространстве L1 всегда можно выделить подпоследовательность, сходящуюся п.н.). Интегрируемость Ха и установленная формула B9) гарантируют справедли- справедливость C0). ?
Гл. IV. Мартингалы. Дискретное и непрерывное время 119 Лемма 5. Если семейство случайных величин {Хп, п ^ 0}, образующих суб- субмартингал X = (Хп, ^n)n^0i равномерно интегрируемо, то для любого мо- момента остановки г (т. е. марковского момента со свойством Р(т < оо) = 1) выполнены условия B7) и B8). Доказательство. Поскольку Р(т > п) —> Оприп —у оо, то условие B8) следует из леммы 2 в [85; т. 1, с. 236]. Кроме того, равномерная интегрируемость семейства {Хп, n ^ 0} влечет соотношение supE|Xn| < оо. C4) п Для ограниченных марковских моментов гдг = т A N, где N Е N, по теореме 2 (пункт 3)) имеем EXTN ^ EXq. Поэтому Е \XTN | = 2ЕX+N - ЕXTJV sC 2ЕX+w - Е Хо. C5) В силу леммы 2 последовательность Х~*~ = (-Х"^", ^n)n^o является субмартингалом. Снова применяя теорему 2 (пункт 3)), получаем, что Е X+N ^ Е Х~^ ^ Е |Хдг |. Заметим, что tn(uj) = т(ш) для п. в. cj при N > г (со). Следовательно, XTN = Хт п.н. при N > т(о;). Учитывая, что |-X"TiV| ^ 0, и воспользовавшись C4), C5), по лемме Фату (см. [85; т. 1, с. 233]) заключаем, что Е|Хг| = Е liminf \ХТ \ ^ liminf Е |ХТ | ^ sup Е \ХТN\ ^ 3supE|Xiv| < оо. П N N N N §11. В качестве иллюстрации изложенных результатов о сохранении свойства мартингальности при случайной замене времени рассмотрим классическую задачу о разорении. Пример 7. Пусть ?i, ?2? • • • — независимые одинаково распределенные бернул- лиевские случайные величины: Положим Sn = х+ ^i + • • • + ?п, где ж — фиксированное целое число, п ^ 1. Поль- Пользуясь игровой интерпретацией, будем считать, что х — это начальный капитал не- некоторого игрока, a Sn — его капитал в момент времени п. Возьмем целые числа а и Ъ ("границы") а < 6, такие, что х G (а, Ь). Спрашивается, с какой вероятностью случайное блуждание 5 = {5n,n ^ 0} выйдет раньше на границу а, чем на границу Ъ (выход на уровень а интерпретируется как разорение игрока, а выход на границу Ъ — как выигрыш игрока). Пусть та = inf{n ^ 1: Sn = а}, ть = inf{n ^ 1: Sn = Ь} и т = inf{n ^ 1: 5n ^ (а, Ь)}. Величины та,тъ,т — марковские моменты относительно фильтрации (^п)п^ъ гДе ^n = 0"{?i, • • •, ?п}? причем г есть момент остановки согласно упражнению 15 гла- главы П.
120 А. В. Булинский, А. Н. Ширяев Теперь заметим, что Хп = (q/p)Sn, п ^ 0, является мартингалом относительно введенной фильтрации (^n)n^o (здесь ^о = {0,Щ), поскольку свойства 1) и 2) определения 1, очевидно, выполнены, а для п Е N Е(Хп |^„_!) = («/^-^((«/р)*») = Xn_i. При любом п ^ 0 значения величины ST/\n лежат между анЬ. Поэтому |Хтлп для всех п ^ 0, где константа с = с(а, Ь,р). Пользуясь следствием 2, имеем ЕХ0 = (q/p)x = EXT = (q/p)aP(ST = а) + (q/p)bP(ST = Ъ). Принимая во внимание, что РEТ = а) + РEТ = Ъ) = 1 (так как т < оо п. н.), приходим при р ф q к ответу (q/рГ ~ (q/p)b РEТ = а) = Р(га < ть) = (q/p)a ~ (q/p) b ' Если р = g = 1/2, то решение еще проще. Очевидно, можно вместо а и Ъ взять а — х и 6 — х и считать тогда начальный капитал нулевым. Первое тождество В а льда (см. теорему 11 главы III) дает для т = inf{n G N: Sn fi (a — ж, Ъ — х)} Следовательно, (а - x)P(ST =a-x) + (b- x)P(ST =b-x)=0, откуда с учетом равенства РEТ = а — х) + РEТ = Ъ — х) = 1 находим РEТ = а-ж) = (Ь- Таким образом, для блуждания, начинающегося в точке xGZb момент п = 0, имеем прир = q = 1/2 и границах а < Ъ (а, 6 G Z, ж G (а, 6)) ответ Р(та <?ъ) = (Ъ-х)/(Ъ-а). § 12. Теорема 2 оказывается весьма эффективной при установлении ряда важных неравенств для мартингалов и родственных им процессов. Теорема 3 (максимальные и минимальные вероятностные неравенства; Дуб). Пусть X = (Хп, &п)п^о — субмартингал. Тогда для любого N G N и всех и>0 иР( max Хп ^ и) ^ EXjylj max Хп ^ и] ^ ЕХ+, C6) min Xn < —и) ^ — EXo + ЕХдг1< min Xn < —и \ ^ ^-EX0 + EX+. C7)
Гл. IV. Мартингалы. Дискретное и непрерывное время 121 Доказательство. Определим марковский момент г = min{n ^ 0: Хп ^ и} AN. Тогда EXN ;> ЕХТ = ЕХТ1А + ЕХТ% ^ иР(А) + где А = \ max Xn > и \. Следовательно, иР(А) ^ EXN - ЕХ,у1^ = EX^Ia ^ ЕХ. Доказательство C7) проводится аналогично с использованием марковского момента г = min{n ^ 0: Хп ^ -?/} Л TV. D Приводимая ниже теорема обобщает классический результат Колмогорова (см., например, [85; т. 2, с. 535]) об оценке сверху вероятности Р( max |5n| > и) для частичных сумм Sn = ^о + ^1 + * * * + ?n (n ^ N) независимых центрированных случайных величин ?о5 • • • ? ?,n, имеющих конечные вторые моменты. Теорема 4 (Дуб). Пусть X = (Хп, &п)п^о — мартингал и Е \Хп\р < оо для некоторого р ^ 1 и любого п ^ 0. Тогда для всех и > 0 и натуральных N max C8) Доказательство. К субмартингалу (\Хп\р,^п)п^о надо применить неравен- неравенство C6) и учесть, что \ max \Хп > и> = \ max Хп\р > ир >. П Помимо приведенных "максимальных вероятностных неравенств" C6) и C8) в стохастическом анализе важную роль играют следующие "максимальные L^-не- равенства" Теорема 5 (максимальные ^-неравенства; Дуб). Пусть X = (Хп,^п)п^о — мартингал или неотрицательный субмартингал. Пусть Е\Хп\р < оо, где р G A, оо) и п ^ N. Тогда max \Хп\\\ ^(p/(p-l))\\XN\\p, C9) ^n^iV \\p где, как обычно, \\?\\р = (Е^I^. Доказательство. Без ограничения общности (учитывая лемму 2) можно счи- считать Хп ^ 0 п.н. для п ^ N. Положим Мдг = max Xn. Применяя последо- вательно формулу для вычисления момента порядка р неотрицательной случайной величины (см., например, [85; т. 1, с. 258]), неравенство C6), теорему Фубини и не- неравенство Гёльдера, находим, что гоо гоо Е(М?) =р vP'^Mn >u)du^p up-2E(XNl{MN^u})du = Jo Jo N Р-2Л p c up аи I = E . ?
122 А. В. Булинский, А. Н. Ширяев Подчеркнем, что максимальные неравенства C9) справедливы для 1 < р < оо. Случай р = 1 требует специальных рассмотрений (см. упражнение 23). § 13. При изучении вопросов сходимости субмартингалов (§ 14) нам понадобит- понадобится оценка числа/Здг (а, Ъ) пересечений "снизу вверх" субмартингалом полосы (а, Ъ) на временном интервале [О, N]. Введем необходимые обозначения. Пусть X = (Хп, ^п)п^о — субмартингал и (а, Ь) —некоторый интервал, а < Ъ. Определим марковские моменты т^, k ^ О, положив го = 0 и Т2т-1 = min{n: п > т2т-2, Хп ^ a}, r2m = min{n: n > r2m_i, Xn ^Ъ], те N. Как обычно, считаем т\~ и tj, j > к, равными бесконечности, если соответствующее множество в фигурных скобках пусто. Пусть g (а Ь\ - { °' еслиг2 > N, \ тах{?тг: r2m ^ 7V}, еслиг2 ^ N. По своему смыслу эта величина есть, как нетрудно видеть, "число пересечений" (в смысле, вполне ясном из рис. 13) процессом X снизу вверх полосы (а, Ъ) за вре- время от 0 до N. Рис. 13 Лемма 6 (о числе пересечений; Дуб). Пусть (Хп,^п)п^о — субмартингал и даны числа а, Ъ (а < Ь). Тогда для среднего числа пересечений справедливы следующие неравенства: E/3N(a,b) -a)+ Е (XN - a) Ъ — а ¦yvN ~ Ъ — а D0)
Гл. IV. Мартингалы. Дискретное и непрерывное время 123 Доказательство. Величина Д/у(а,6) для последовательности (Хп, совпадает с величиной Д/у@,6 — а) для последовательности ((Хп — a) + ,^n)n^05 поэтому мы будем далее считать, что а = 0 и Хп ^ О, п ^ 0. Положим Хо = 0, ^о = {&>•> Щ и пусть фг — 1{тт < г ^ rm+i для некоторого нечетного т}, где г ^ 0. Тогда, очевидным образом, iV г=1 Заметим, что {фг = 1} = (J ({rm < г} \ {rm+i < г}) G ^г-ъ * ^ 1- т — нечетно Поэтому N N г=1 г=1 iV iV г=1 г=1 что доказывает неравенство D0). ? § 14. Следующий результат о сходимости субмартингалов является естествен- естественным аналогом теоремы классического анализа о существовании предела у ограни- ограниченной монотонной числовой последовательности. Теорема 6 (Дуб). Пусть X = (Хп, ^п)п^1 — субмартингал такой, что sup E \Хп\ < оо. п Тогда с вероятностью единица существует lim Xn = Хоо, причем Е \XqqI < оо. Доказательство будем вести от противного. Пусть Х_ = lim inf Xn, X = lim sup Xn. Допустим, что Р(Х_ < X) > 0. Поскольку {Х<Х}= IJ {Х<а Q, а<Ъ где Q — множество рациональных чисел, то для некоторых рациональных а < Ъ Р{Х_ < а < Ъ < ~Х] > 0. D1)
124 А. В. Булинский, А. Н. Ширяев В силу леммы б для любого натурального N Тогда Обозначим /Зоо(а,6) = lim C^(а,Ь) (последовательность C^(а,Ь) не убывает). ^ (supEX++ \а\)/(Ь-а). iV Теперь заметим, что поскольку для субмартингала X = (Хп, ^п)п^1 справедливы соотношения ЕХ+ ^ Е \Хп\ = 2ЕХ+ - ЕХп ^ 2ЕХ+ - ЕХЬ то sup EX+ < оо «=> sup E |Xn| < оо. D2) Таким образом, Е C^ (а, Ь) < оо п. н. Однако по сделанному допущению Р(Х < X) > 0 и, как следствие, для неко- некоторых а и b, a < b, выполнено свойство D1), которое означает, что на множестве {со: Х_ < а < Ъ < X} число пересечений /^(а, Ь) равно бесконечности. Поскольку вероятность этого множества положительна, то Е /Зоо (а, Ь) = оо. Итак, из предполо- предположения supn Е \Хп\ < оо следует, что Р{Х_ < X} = 0, то есть Р(Х = X) = 1. Тем самым, с вероятностью единица существует lim Xn (= Xqq). п Наконец, утверждение Е |Хоо | < оо вытекает из того, что по лемме Фату Е|Хоо| = Efliminf \Xn\] ^ liminf E|Xn| ^ supE|Xn| < оо. ? V п—юо / п—^оо п Понятно, что теорема б применима и к мартингалам и к супермартингалам (как уже отмечалось, мартингал является субмартингалом, а (—Хп, ^n)n^i — субмар- субмартингал, если (Хп, &п)п^1 — супермартингал). Полезно также отметить, что усло- условие sup E \Хп\ < оо автоматически выполняется для неотрицательных супермар- супермартингалов. Следствие 4. Пусть X = (Хп, ^n)n^i ~ неотрицательный субмартингал такой, что sup ЕХ^ < оо для некоторого р G A, оо). п Тогда предел Х^ = lim Xn существует как с вероятностью единица, так п—юо и в смысле сходимости в Lp. Доказательство . Поскольку (Х%, ^п)п^1 является субмартингалом (см. лем- лемму 2), то существование с вероятностью единица предела Х%, а значит, и преде- предела Хп (в силу неотрицательности величин Хп, п ^ 1) следует из предположения sup E Х% < оо и теоремы б. п Далее, из теоремы 5 оо. D3) п Поэтому, если Х^ = lim Xn, то п 1 D4) п
Гл. IV. Мартингалы. Дискретное и непрерывное время 125 так что величины \Хп — Хоо|р, п ^ 1, мажорируются интегрируемой величиной 2Р supX^. В силу сходимости с вероятностью единица Хп —> Хоо находим, что по п теореме Лебега о мажорируемой сходимости Е \Хп — Хоо\р ~~^ О, т-е- имеет место сходимость в смысле Lp (Хп —У Х^). ? § 15. Нижеследующий пример иллюстрирует возможность применения теоре- теоремы б к исследованию асимптотического поведения ветвящихся случайных процессов. Пример 8 (ветвящийся процесс Га ль тона-Ватсона). Пусть имеется массив {Q, ; /c,n ^ 1}, состоящий из независимых одинаково распределенных случайных вели- величин, принимающих целые неотрицательные значения, и таких, что Е^[ ' = \± > 0. A) Sn~1 Ы) ( ° (п) \ Положим So = 1, Si = ^ и Sn = ^2 ?>к ПРИ п ^ ^ ( ^2 ?>к :— 0, п ^ 11. Так к=1 ^к=1 ' образованную последовательность S = {Sn, n ^ 0} принято называть ветвящим- ветвящимся случайным процессом, описывающим эволюцию численности некоторой попу- популяции, развивающейся следующим образом. В начальный момент п = 0 имеется один представитель популяции, который порождает случайное число ^ потомков, где ^ ^ может принимать значения 0,1, 2,... . Каждый представитель этого поко- поколения порождает, в свою очередь, независимо от других, случайное число потомков, распределение которых такое же, как у ^ . Этот процесс размножения происходит и в последующих поколениях в предположении, разумеется, что еще не произошло вырождение популяции. Введем величины Ап = оп//1 , 77/ ^ 1, и покажем, что они образуют мартингал относительно естественной фильтрации ?Jj В силу независимости величин {?Jj. , к ^ 1} от "истории" (So, Si,..., Sn_i) на- находим, ,Sn-l E(Xn\&n-1)=ii-nE(^E ¦ An) X о? • • ¦ -i = Sn-1 У" f(n) / ^ ^к С С \ — ^05 • • • 1 ^п-1 — 3=0 {Sn-l=J} 3 k=l что и доказывает требуемое свойство мартингальности.
126 А. В. Булинский, А. Н. Ширяев Нетрудно видеть, что supE|Xn| = supEXn = 1. п п Поэтому по теореме б получаем, что Хп —у Х^ п.н. при п —У оо и ЕХ^ < оо. Отсюда вытекает, что если /л < 1, то Sn —У 0 п.н. при п —У оо. Иначе говоря, при \± < 1 с вероятностью единица популяция вырождается. Методом производящих функций (см., например, [68; § 36]) доказывается, что lim P(Sn = 0) = 1 при fi = 1 и lim P(Sn = 0) < 1 при ц > 1. п—^оо п—^оо Ветвящимся процессам посвящены, например, монографии [67, 139]. В качестве введения в эту обширную область можно прочитать § 4 в [61]. § 16. Следствие 4 является одним из результатов о 1/р-сходимости (неотрицатель- (неотрицательных субмартингалов и, в частности, мартингалов) при р > 1. В следующей теореме для мартингалов будет рассмотрен и случай р = 1. Теорема 7. Пусть X = (Хп, ^п)п^1 — мартингал. Приводимые ниже условия эквивалентны: 1) Хп = Е(?|«^п), п ^ 1, где ? — некоторая интегрируемая случайная величина; 2) семейство {Хп, п ^ 1} равномерно интегрируемо] 3) существует интегрируемая случайная величина Х^ такая, что имеет место сходимость в L1: Е |Хоо - Хп\ -у 0 при п -у оо; 4) sup Е \Хп\ < оо и Xk = Е (Хоо | ^) с Х^ = lim Xn (предел существует п п—^оо п. н. и интегрируем в силу теоремы 6), k ^ 1. Доказательство. 1) => 2). Для п ^ 1 и любых а > 0, 6 > 0 имеем Е |ХП|1{|ХП| > аК Е |?|1{|ХП| > а} = = Е \(\1{\Хп\ > а, |?| ^ 6} + Е |?|1{|Х„| > а, |?| > 6} < CL Поэтому lim sup E \Xn\\{\Xn\ > a} ^ E |?|1{|?| > 6}. a>oo Заметим, что 6 > 0 выбиралось произвольным образом и можно перейти к пределу Ъ —У оо. Тогда в силу того, что Е |?| < оо, правая часть в предыдущем неравенстве стремится к нулю, что и доказывает свойство 2). 2) => 3). Из равномерной интегрируемости семейства {Хп, п ^ 1} получаем, что sup Е \Хп\ < оо. По теореме 6 п.н. существует Хоо = lim Xn. Вновь используя п п—юо
Гл. IV. Мартингалы. Дискретное и непрерывное время 127 равномерную интегрируемость семейства {Хп, п ^ 1}, находим (см. A11.51)), что Е \Хп — Xqq | —у 0, т. е. имеет место сходимость в L1. 3) => 4). Из условия 3) следует, что sup Е \Хп\ < оо. Поэтому опять же по тео- п реме б п.н. существует lim Хп = Y, причем E\Y < оо. Условие 3) показывает, п—>>оо что очевидным образом Y = Х^ п.н., где Х^ — случайная величина, входящая в условие 3). Осталось лишь проверить, что Х^ = Е (Х^ | &k)i к ^ 1. Для любых п ^ 1, к ^ 1 имеем Поскольку Е \Хп — Хоо| —у 0, п —У оо, и Е (Хп | ^/с) = Х^ (п.н.) при n ^ /с, то 4) => 1). Эта импликация очевидна. П Теорема 7 дает возможность построить простой пример неравномерно интегри- интегрируемого мартингала. Пример 9. Пусть ?i, ?2? • • • — независимые одинаково распределенные величи- величины, P(fi = 0) = P(fi = 2) = 1/2. Положим к=1 Последовательность X = {Xn, n ^ 1} является неравномерно интегрируемым мар- мартингалом. Действительно, X — мартингал, поскольку это частный случай примера 2. За- Заметим, что {Хп = 0} С {Xn+i = 0}, Р(Хп ф 0) = 2~n, n ^ 1, и, следовательно, Хп —У 0 (п.н.) при п —У оо. Если бы семейство {Хп, п ^ 1} было равномерно ин- интегрируемым, то тогда бы в силу импликации 2) => 3) нашли бы, что Е Хп —у 0 при п —У оо. Но ЕХп = 1, п ^ 1. Тем самым, семейство {Хп, п ^ 1} не является равномерно интегрируемым. § 17. Предположим, что на (полном) фильтрованном вероятностном пространст- пространстве (П, ^, (^п)п^ъ Р) задана случайная величина ? такая, что Е |?| < оо. Эта слу- случайная величина "порождает" равномерно интегрируемый мартингал Леей Х = (Хп,(&п)п>1,Р) с Хп = Е(?\&п). Согласно теореме 7 с вероятностью единица, а также и в смысле сходимости в L1, существует предел lim Е (? I &п). Оказывается, этот предел допускает весьма ес- п тественное представление. Соответствующий приводимый ниже результат, обычно называемый теоремой Леви, явился одним из самых первых результатов о сходимос- сходимости мартингалов, полученных еще в тридцатых годах прошлого столетия (В. Jessen A934), P. Levy A935)), когда, как правило, рассматривались сг-алгебры j^n, порож- порожденные некоторыми случайными величинами Y\,..., Yn, а в качестве случайной ве- величины ? брались индикаторы \в множеств SGf. 00 Пусть i^oo = \j ЗРп обозначает наименьшую сг-алгебру, содержащую все сг-ал- 71=1 гебры^п, п G N.
128 А. В. Булинский, А. Н. Ширяев Теорема 8 (П. Леви). С вероятностью единица Е(^|^п)^Е(^|^0О) при п^оо. D5) Сходимость в D5) имеет место также и в смысле сходимости в L1. Доказательство. По теореме 7 с вероятностью единица существует lim E(^|^n)=Xoo, п—ь-оо причем Е (? | &п) = Е (Хоо | &п) для всех п ^ 1. D6) Покажем, что Х^ = Е (? | J^oo) п.н. Очевидно, Хп = Е (? | J^n) G J^ п ^ 1. Поскольку ЗРп С ^оо5 величины Хп являются ^оо -измеримыми (n ^ 1). По лемме б главы I Х^ G ^оо |Й§(М). В силу леммы 7 главы I найдется ^оо-измеримая случайная величина Zoo такая, что Zoo = Хоо п.н. Таким образом, достаточно установить, что Z^ = Е (? | ^оо) п-н- Для этого надо лишь проверить, что при любом A G ^оо Так как Zqo = Хоо п.н., последнее утверждение равносильно следующему: D7) для каждого A G ^оо • Согласно D6) соотношение D7) выполнено для событий А, входящих в алгебру оо д? — у j^n (и, следовательно, образующих тг-систему). Легко видеть, что совокуп- п=1 ность событий i G ^, для которых верно D7), есть А-система. По теореме о моно- тоных классах a{s^} содержится в упомянутой А-системе. Очевидно, a{s^} = ^оо5 поэтому соотношение D7) справедливо для всех A G ^оо • П § 18. Исследование мартингалов и субмартингалов в случае непрерывного вре- времени, как правило, сложнее, чем в случае дискретного времени. Однако ряд резуль- результатов переносится (при определенных условиях) на случай непрерывного времени весьма просто. В силу леммы 3, изучая мартингал или субмартингал (Xt, &t)t^Oi MbI будем счи- считать фильтрацию F = (^t)t^o расширенной. Определение 6. Опциональным моментом (относительно фильтрации F) на- называется такая функция т: П —у [0, оо], что {и: г (и) < t} G 3*t для каждого t ^ 0. Лемма 7. Пусть X = (Xt, &t)t^o — мартингал, имеющий п. н. непрерывные справа траектории. Пусть а и г — опциональные моменты (относительно фильтрации F) такие, что а ^ т ^ с п. н., где с — положительная константа. Тогда ЕХТ = ЕХа D8) (на множествах {а = оо} и {г = оо}, имеющих нулевую вероятность, соот- соответственно полагаем Ха = 0 и ХТ = 0).
Гл. IV. Мартингалы. Дискретное и непрерывное время 129 Доказательство. Введем множестваТп = 2~nZ+ = {к2~п, к е Z+}, п G N. Ясно, что (Xu,^u)U?Tn - мартингал. Определим т(п) = 2~п[2пт + 1], сг(п) = = 2~п[2па + 1], где п Е N, [•] —целая часть числа. Очевидно, т(п),а(п) Е Тп и т(п) I т, (т{п) I а при п —> оо для п.в. u; Е П. Кроме того, сг(п) ^ т(п) п.н., n E N. Заметим, что г(п), сг(п) — марковские моменты относительно фильтрации (^и)иетп ? поскольку для каждого /с Е Z+ имеем {г(п) ^ А:2-п} = {т< k2~n} e &к2-п (аналогично и для а(п), п G N). При всех n G N верна оценка т(п) ^ с + 1 п.н. Поэтому в силу теоремы 2 (несущественно, что в этой теореме рассматривалось па- параметрическое множество Z+, а не Тп) Е(*т(п)|^(п))=*<т(п) п.н., nGN. D9) В силу той же теоремы для каждого m G Ъ _|_ имеем Е (^c(m) I ^V(n)) = ^(п) п'н- ПРИ п ^ ш^ где с{т) = 2~гп[2гпс + 1]. Рассуждения, проведенные при доказательстве теоре- теоремы 7, показывают, что совокупность величин вида Е (? | ?/а) равномерно интегрируе- интегрируема для любого семейства сг-алгебр d^ С <?, a G Л (( — интегрируемая случайная величина). Учитывая непрерывность справа траекторий Xt, t G M+, а также (выте- (вытекающую из E0)) равномерную интегрируемость семейства {Х^^}, получаем, что Хсг(п) ~^ Ха п.н. ив L1(Q, ^, Р) при п —У оо. Таким образом, Ха — интегриру- интегрируемая случайная величина. Поскольку Е (Хс(т) | ^Т(п)) = -Х"т(п) п-н- ПРИ п ^ т, аналогичным образом заключаем, что ХТ^ —> Хт п.н. и в L1(Q, ^, P) при п —у оо. По лемме 1 главы III 3*^{п) С За(т) Для п ^ т. Следовательно, для каждого т G N величина Ха^ является ^сг(т)-измеримой. В силу леммы б главы I видим, что Хст есть 3(j^rn)-измеримая величина при любом т G N, а, значит, Ха является ^-измеримой величиной, где 71=1 Докажем, что Ха п.н. E2) Для этого нужно лишь установить, что ЕХТ1А = ЕХСТ1А E3) при каждом A G ^. Поскольку ^ С 3* G(n) для n G N, из D9) и леммы 7 главы I заключаем, что ЕХт^п>Iа = EX^^Ia при всех nGN. Устремляя п к бесконеч- бесконечности в обеих частях этого равенства и воспользовавшись доказанной L1 -сходимос- -сходимостью Хт(п) к Хт и Хр^ к Хсг, приходим к E3). Соотношение E2), очевидно, вле- влечет D8). ?
130 А. В. Булинский, А. Н. Ширяев § 19. Мартингальные методы позволяют достаточно просто получить фундамен- фундаментальный результат (теорема 9) в модели страхования Крамера-Лундберга (при- (пример 3 главы I). Для этого предположим, что ^(v) = Eevm < оо при v > 0. E4) Это условие заведомо выполнено для ограниченных величин "выплат" r]j, что яв- является весьма реалистическим. Ясно, что поскольку 771 ^ 0 п.н., то E4) справед- справедливо для v ^ 0. Вычисления, совершенно аналогичные (П.5), показывают, что при о ^ s <tnveж Ee-v(Yt-Ys) =e(t-s)g(v)^ где 0(v) = A(^(u) - 1) - VC. E5) Из E5), принимая во внимание, что Yo = у0, получаем Е e~vYt = et9^~vyo, t G Следовательно, соотношение (9) выполнено, и для каждого uGl процесс z = {zt = e-vYt-t9^v\ t е является мартингалом относительно фильтрации &t = a{Zs, 0 ^ s ^ i) = cr{Fs, 0 ^ s ^ t}, t e M+. Введем момент "разорения" г = inf{? > 0: Yt < 0} = inf{t >0:Yte (-oo,0)}. E6) Поскольку {Yt, t ^ 0} имеет п.н. непрерывные справа траектории, а множество (—оо, 0) — открытое, аналогично теореме 3 главы III получаем (проверьте в качест- качестве упражнения), что т есть опциональный момент, т.е. {г < t} G ^t для любого t G M+. В силу леммы 7 (а = 0 ^ т A t ^ ?; очевидно, 0 и t А т — ограниченные опциональные моменты) для ?, v G M+ имеем с учетом неравенства YT ^ 0 п.н., что EZ0 = EZtAT ^ E exp{-«lW - (t Ar)g Е exp{-vYT - Tg(v)}l{T^t} ^ Ee-T»<">l{T<t} ^ inf е-^")р(г ^ «). E7) Итак, для v ^ 0 и t ^ О P(r ^t) ^e"^° sup es^. E8) Предположим, что с > Ла, где а = Е771 > 0. E9) Тогда g'(v) = \ф'(v) - cngf@) = Xa - с < 0. Кроме того, g"(v) = Хф"(v) дляу ^ 0, поскольку ф"(у) = Ej]fevr]1 (условие Ещ > 0 влечет, что Ej]f > 0). Сле- Следовательно, существует единственное значение г?о > 0 такое, что g(vo) = 0. Выбрав v = vo, из E8) получаем Р(т ^ t) ^ e~v°yo при всех t G M+. Отсюда заключаем, что Р(т <оо) ^e-vo^0. F0)
Гл. IV. Мартингалы. Дискретное и непрерывное время 131 Итак, доказан следующий фундаментальный результат математической теории страхования: Теорема 9. Пусть модель Крамера-Лундберга A.11) описывается пуассо- новским процессом A.10) интенсивности Л > 0; величины г}^ j Е N, удов- удовлетворяют условию E4) и выполнено неравенство E9). Тогда вероятность "разорения" Р(т < оо) оценивается с помощью формулы F0), где уо — началь- начальный капитал, vo > 0 есть (единственный) корень уравнения g(v) = 0, v G M+, а функция g введена в E5). §20. Далее нам потребуются также следующие утверждения (ср., например, с C6)). В этом параграфе Т обозначает некоторое положительное число. Следствие 5. Пусть (Xs^s)q^s^t — субмартингал с непрерывными тра- траекториями. Тогда для любого с > 0 Р( sup Xs ^ с) <С ЕХ+/с. F1) Доказательство . Учитывая непрерывность траекторий Xt, 0 ^ t ^ T, имеем оо { sup Xs > и] = I I II {Xk2-n > и} U {ХТ > и}. U^^ 1 =1 к: Рассуждения, проведенные при доказательстве C6) (вместо Xq, ..., Xn рассматри- рассматриваем XSI, ¦ ¦ ¦, XSm, si < • • • < sm), дают неравенство / N \ P( U U {Xk2-n>u}U{XT>u}UEX+/u. ^n=l 0<k2~n<T ( \ ( \ Требуемое неравенство F1) следует из свойства: Р( (J Вп)^Р[ (J Bn), и предельного перехода по и. ? Следствие 6 (Дуб). Пусть (Xt,^t)o^t^T — квадратично интегрируемый мартингал (т. е. ЕХ^ < оо дл^ ? G [0,Т]), имеющий п.н. непрерывные траек- траектории. Тогда при всех с > 0 sup \Xt\ ^cWc-2EX|. F2) Доказательство непосредственно следует из неравенства F1) с учетом того, что ^, ^s)se[o,T] является субмартингалом. ?
132 А. В. Булинский, А. Н. Ширяев Дополнения и упражнения 1. Пусть случайная величина ? G Ь2(п, &, Р) и сг-алгебра d С 3*. Обозначим Рг# — оператор ортогонального проектирования на подпространство Н = = L2(ft, d, Р) С L2(ft, с?, Р). Покажите, что Е (f | ^) = Рг# f. Заметим, что поскольку пространство 1/2(П,^, Р) плотно в простран- пространстве Ь1(П,^Г, Р), оператор Рг# можно продолжить по непрерывности на 2. Пусть (^t)t^o — некоторая фильтрация. Докажите, что т — опциональный момент относительно фильтрации (^t)t^o тогда и только тогда, когда т — марковский момент относительно фильтрации (^+)t^o? где 3t+ — П ^s5 Введем сг-алгебру J^T+ = {iG^:ifl{r<t}Gft при каждом t ^ О}. 3. Можно ли утверждать, что в соотношении E1) сг-алгебра ^ = i^v+? 4. Пусть {?n}n^i — последовательность интегрируемых случайных величин, п &п = сг{?ь • • • ,fn}, ^п = Е ^/с, n G N. Докажите, что (Xn, ^ мартингал тогда и только тогда, когда для любой ограниченной борелевской функции /п: Мп —у Ж и всех n ^ 1. Интересно сопоставить это утверждение со следующим просто проверяемым фак- фактом: действительные случайные величины (необязательно интегрируемые) ?&, к ^ 1, независимы тогда и только тогда, когда cov((?(?n+i),/„(&,•••,&»))= О всеж п г/ любых ограниченных борелевских функций g: М —у Ж и fn : Жп —у Ж. 5. Докажите, что если (?n, ^n)n^i — последовательность независимых цент- центрированных величин и &п — сг{^1,... ,^n},n ^ 1, то процесс En,m, вида Ь = / / ^ является мартингалом с нулевым средним для каждого т ^ 1. 6. Пусть урна содержит а белых и Ъ черных шаров. Из урны последовательно наудачу вынимается один шар, который возвращается обратно с добавлением d шаров того же цвета, что и у вынутого. Обозначим Хп (п ^ 1) — отношение числа белых шаров к общему числу шаров в урне после n-го вынимания шара и возвращения его в урну вместе с d шарами соответствующего цвета (Xq := а/(а + Ь)). Докажите, что X = {Хп, п ^ 0} — мартингал.
Гл. IV. Мартингалы. Дискретное и непрерывное время 133 7. Пусть(П,^,Р) = ([0,1], ^[0,1], mes),mes —мера Лебега. Пусть {Tn,n ^ 1} — последовательность измельчающихся разбиений отрезка [0,1], т. е. Тп со- состоит из точек 0 = ?п,0 < • • • < tn^rrin = 1 и Тп С Тп+Ь п ^ 1, mn ^ 2. Введем сг-алгебры j^n на [0,1], порожденные множествами Апд = [0,?П)о]> An,/с = (*n,fc-ij*ri,fc]j 2 ^ к ^ шп,п ^ 1. Для функции/: [0,1] -»> R поло- положим (tn,k) — f(tnk-i)., / ч . -, Докажите, что (Xn, 3n)n^i — мартингал. 8. Пусть в условиях предыдущего упражнения функция / удовлетворяет усло- условию Липшица, т.е. \f(x) — f(y)\ ^ L\x — у\ для ж, у Е [0,1] и некоторой кон- константы L > 0. Пусть разбиения Тп таковы, что max (tn^ —tn,k-i) ~^ 0? п -у оо. Докажите, что V &п (:= сг{Зп, п е N}) = Й§([0,1]) и семей- п=1 ство {Xn, n ^ 1} равномерно интегрируемо. Тогда по теореме 7 величины Хп —у Хоо п.н. и в L1 при п —> оо, где Хоо G Й§([0,1]) | 88(Ж). Следователь- Следовательно, для любого [а, Ь] С [0,1] гЬ гЬ \ Xndt = EXnl[a:b]^EXool[cL:b]= Xoodt, n-^oo. F3) J a J a Поскольку также / Xn dt —> f(b) — f(a) при n —> оо (объясните, почему), J a то вместе с F3) отсюда заключаем, что функция / является абсолютно не- непрерывной и ее плотность (по мере Лебега) может быть найдена как предел величин Хп, п —у оо. 9. Пусть {Xn}n^i —последовательность действительных случайных величин, заданных на вероятностном пространстве (П, 3*, Р), и пусть /^(xi,..., хп) и /^(xi,... ,хп) — некоторые плотности вероятностей относительно меры Лебегав (Мп, ^(Мп)), п ^ 1. Рассмотрим отношение правдоподобия считая, что /^B:) = 0, если f%(z) = 0 (тогда LnB:) := 0), z G Mn. Покажите, что (I/n,^rn)n^i —мартингал, где &п — о{Х\,... ,Xn}, n ^ 1. 10. Пусть Т С М+ и (Xt,&t)t?T - комплекснозначный мартингал, т.е. Im Xt), 3*t)t€.T ~ мартингал со значениями в М2, каждая компонента которо- которого (по определению) есть мартингал. Тогда Е [Х± \ &s) = Xs п.н. для s ^ t, s,t е Т (здесь Е (У + iZ | ^) := Е (У | d) + iE (Z | d) для интегрируемых У, Z и сг-алгебры d С ^). Пусть Е |Х^|2 < оо для t G Т. Докажите, что X = {Х^, ? G Т} -процесс с некоррелированными приращениями, т.е. Е (Х^ — XS)(XW — Xv) = 0 для г; < и ^ s < ?, v,u,s,t G Т. (Тем самым, квадратично интегрируемые мартингалы занимают промежуточное положе- положение между процессами с независимыми приращениями, имеющими постоян- постоянное среднее значение, и процессами с некоррелированными приращениями.)
134 А. В. Булинский, А. Н. Ширяев 11. Пусть {?п}п^1 — последовательность независимых действительных случай- п ных величин. Положим Sn = J^ ?&, п ^ 1. Мартингальными методами до- кажите, что следующие утверждения относительно ряда ^2 ?к эквивалент- к=1 ны: 1) ряд сходится п. п., 2) ряд сходится по вероятности, 3) ряд сходится по распределению. Заметим, что по закону "О или 1" Колмогорова (теорема б главы III) дан- данный ряд сходится или расходится с вероятностью единица. 12. Пусть Y = {It 51 G Т}, Tel, — (комплекснозначный) процесс с независи- независимыми приращениями такой, что Е \Yt\2 < oo, t G Т. Докажите, что сущест- существует неслучайная функция h: Т —У Ж такая, что Xt = \Yt\2 — h(t) есть мар- мартингал относительно фильтрации ^ = o-{YSj s ^ ?, s G Т} = а{Re Ys, Im Ys, s ^ t,s G T},t G T, тогда и только тогда, когда E\Yt -Ys\2 = h(t) -h(s), s^t, s,teT. F4) Пусть W = {Wt, t ^ 0} — броуновское движение с естественной фильтрацией F = 13. Найдите все а, /3 G М, для которых процесс Х = {exp{aWt+f3t}, t ^ 0} является мартингалом. 14. Пусть т — момент остановки (относительно фильтрации F). Докажите, что (WtAn &t)t^o — мартингал. 15. Для a,xGtHt^0 определим функцию h(a,x,t) = ехр{ах - a2t/2). Положим ^_dkh(a,x,t) Докажите, что (hk(Wt,t), &t)t^o — мартингал при каждом к ^ 1 (заметим также, что в силу упражнения 13 процесс {/i(a, Wt, t), t ^ 0} — мартингал при каждом ft G 1). Таким образом, процессы Wt, W2 — t, Wf — 3tWt, W? - 6tW? + 3t2, ... — мартингалы (t ^ 0). 16. Докажите, что Ет2 = 5a4/3 для та = inf{t ^ 0: \Wt\ = a}, a > 0. 17. Пусть Xt = jt + oWti t ^ 0, и i?a = inf{?: Xt = a}. Докажите, что если 7 > 0, а ф 0 и a < 0, то
Гл. IV. Мартингалы. Дискретное и непрерывное время 135 18. Выполнено ли условие (9), если а) Y = {Yt, t ^ 0} — винеровский процесс, б) Y = {Yt, t ^ 0} — пуассоновский процесс? 19 (см. [12; с. 166]). Пусть W = {Wt, t ^ 0} — m-мерное броуновское дви- движение и /(ж) — непрерывная супергармоническая функция в Мт, т.е. /(ж) для любого ж Е Mm не меньше интеграла от этой функции по любой сфере с центром в точке ж. Докажите, что процесс X = {Xt = f(Wt), t ^ 0} являет- является супер мартингал ом относительно естественной фильтрации броуновского движения, если Е \Xt\ < оо при всех t ^ 0. В связи с теоремой б представляет интерес следующее упражнение. 20 (разложение Крикеберга). Мартингал X = (Хп, ^п)п^о обладает свойст- свойством sup Е \Хп\ < оо тогда и только тогда, когда существуют неотрицатель- п ные мартингалы Y = (Fn, &n)n^Q и Z = (^ru^n)ri^o такие, что Хп = 21 (разложение Рисса). Пусть X = (Хп, ^п) п ^ 0 — равномерно интегрируемый супермартингал. Тогда существует представление вида Хп = Мп + Rn, где М = (Мп, ^п)п^о — равномерно интегрируемый мартингал и R = = (Rm^n)n^o — потенциал, т.е. равномерно интегрируемый неотрица- неотрицательный супермартингал такой, что Rn —> 0 п.н. при п —у оо. 22. Пусть h: Ж —У М+ — неубывающая выпуклая функция и (Хп, ^n)i^n^N — субмартингал. Покажите, что тогда для любых и G Ш. и t > О max Xn^u) ^Eh(tXN)/h(tu). F5) 23. Пусть (Xn, i^n)i^n^iv — мартингал или неотрицательный субмартингал. Докажите, что Е max |Хи|^^-A + Е|Хлг|1оё+|Х^|), F6) l^n^iv e — 1 где log+ х = log х для х ^ 1и log+ х = 0 для ж < 1. Для сумм независимых слагаемых оценка F6) может быть несколько уточнена, как показывает следующий результат. Теорема 10 (Дуб). Пусть ?i,...,?jv — независимые действительные слу- п чайные величины с Е?& = 0, к = 1,..., N. Положим Хп = ^2 ?&, п = 1,..., N. Тогда к=1 Е max \Xn\ ^8Е|Х^|. F7) Приведем несколько результатов Буркхольдера, Дэвиса и Ганди, обобщающих классические неравенства Хинчина и Марцинкевича-Зигмунда, известные для сумм независимых слагаемых. Подробнее со всеми этими неравенствами можно ознако- ознакомиться по обзору [51].
136 А. В. Булинский, А. Н. Ширяев Теорема 11 (Буркхольдер). Пусть X = (Хп, ^п)п^о — мартингал с Xq = 0. Тогда для каждого р > 1 найдутся константы Ар и Вр, не зависящие от X, такие, что Ap\\V\XU\\p <: WXn\\p <: Вр\\у/\Х]^\\р, П > 0, F8) где [Х]п — квадратическая вариация X (см. B2)). В F8) можно взять Ар = [18р^2/(Р ~ I)], Вр = 18р^(р - II/2. Кроме того (в силу теоремы 5), для Х^ = max \Xk\ при р > 1 a;\\V[xu\\p < WKWp <: в;\\у/\хп где А*р = Ар, Б* = (р/(р - 1))ВР. (См., например, [85; т. 2, с. 678]) 24. Постройте пример (см. [85; т. 2, с. 678]), показывающий, что при р = 1 двой- двойное неравенство F8) не обязано выполняться. Однако как показал Б. Дэвис, неравенство F9) остается, тем не менее, справедли- справедливым прир = 1 (с некоторыми "универсальными" константами А^, В± , не зависящими от мартингала X). Для моментов более общего вида, чем степенные, справедлива Теорема 12 (Буркхольдер-Дэвис-Ганди; см., например, [106; с. 425]). Пусть Ф: [0, оо] —у [0, оо] — неубывающая функция, конечная и выпуклая на [0,оо), такая, что ФB?) ^ сФ(?) для всех t > 0 и некоторого с > О, Ф@) =0, Ф(оо-) = Ф(оо). Тогда найдутся константы 0 < А < В < оо, зависящие только от с, такие, что для любого мартингала X = (Хп, ^п)п^1 А Е ФEоо) ^ Е Ф(Х*) ^ В Е ФEоо), / оо ч 1/2 где X* = sup \Хп\, 5^ = ( ? (АХкJ) , АХк = Хк - Хк_г, Хо = 0,кеК 25. Пусть W = {Wt, t ^ 0} — m-мерное броуновское движение. Докажите, что (||Wt||,^t)t^o — субмартингал (с п.н. непрерывными траекториями), где || • || — евклидова норма в Ж171. Используя упражнение 22 (с функцией h(x) = etx и подходящим значением t > 0), докажите, что для каждого s > О и всех х > y/ms справедливо неравенство \ / аИ \ -ш/2 о sup \\Wt\\^x)^ ^ e~x2^2s\ G0) J \ex2 J ex2 (Процесс (||Wt||, &t)t^o называется процессом Бесселя размерности т.)
Гл. IV. Мартингалы. Дискретное и непрерывное время 137 26 (см. [12; с. 182]). Пусть W = {Wt, t ^ 0} — m-мерное броуновское движение, где т ^ 3. Докажите, что ||И^|| —У оо п.н. при t —> оо. Докажите, что с вероят- вероятностью 1 процесс W не вернется в точку 0 ни за какое конечное время. Объясните, почему для т = 1 приведенные утверждения неверны. Как выглядит соответству- соответствующий результат в случае т = 2? Ряд важных результатов стохастического анализа может быть получен путем со- сочетания методов теории мартингалов и идеи обращения времени. Определение 7. Пусть (^)гет — убывающее семейство о-алгебр в §<, т.е. % С % С & при s < ?; s,t G Г С 1. Пусть X = {Xt, ? G Г} — действи- действительный случайный процесс, согласованный с фильтрацией (^)г?Т- Процесс А = (Xt^^teT называется обращенным мартингалом (обращенным суб- субмартингалом, обращенным супермартингалом), если (обращенный) процесс (Xu^uju^u при U = — Т = {—?, t G Г} является мартингалом (субмартингалом, супермартингалом). В частности, стохастическая последовательность А = (Хп, ^п)п^о — обращен- обращенный мартингал, если Е (Хп \ ^n+i) = -^n+ъ тг ^ 0. Записав в последнем равенстве вместо "=" знак "^" получим определение обращенного субмартингала (при "^" — обращенного супермартингала). Понятие обращенного мартингала распространя- распространяется и на процессы со значениями в Ж171 (гп ^ 1). 27. Пусть ?]_,...,?jy — независимые одинаково распределенные векторы в Ж171 (тп ^ 1) с E||?i|| < оо, где || • || — евклидова норма. Положим Хп = ^n,iv = cr{Xn,...,Xiv}. Докажите, что (X^^j п — обращенный мартингал. Определение 8. Последовательность действительных случайных величин ^ называется перестановочной, если (?^, • • •,б^) = (?i,...,^iv) для каж- каждого N ^ 1 и любой перестановки (ii,..., гдг) множества {1,..., N}. Пусть т ^ 1 и ^: Ж171 —у Ж есть борелевская функция. Рассмотрим последова- последовательность U-статистик вида где С™ = п\/(т\(п — т)\). Положим ^п.гп — &{Uk,rm к ^ п},п ^ m. 28. Докажите, что для последовательности ^/-статистик, введенной выше, (Un^rn^n^n^m есть обращенный мартингал. Следующий результат является аналогом теоремы 6. Теорема 13. Пусть (Хп, ^п)п^о — обращенный субмартингал. Тогда п.н. существует lim Хп = Х^ G [—оо,оо). Если, кроме того, существует п—>>оо lim EXn = c>-oo, G1) п—юо то Хоо G (-оо, оо) и Хп -у Х^ в пространстве Ьг(п,^,Р) при п -у оо. Если Хп ^ 0 п. н. при п ^ 0 и EXq7 < оо для некоторого р G A, оо), то Хп —у Х^ в ЬР(П,&,Р) прип^ оо.
138 А. В. Булинский, А. Н. Ширяев Доказательство. Для непустого интервала (а, Ь) и N ^ 1 пусть /Здг(а, Ь) обо- обозначает число пересечений "снизу вверх" полосы (а, Ь) (см. выше § 13) мартингалом (Xjv, ^jv), ..., (Хо, %). Тогда Е Д/у(а, Ь) ^ Е (Хо — а) + /F — а) по лемме 6. Следо- Следовательно, /Зоо(а,6) := lim Д/у(а,6) < оо п.н., iV—юо и так же, как при доказательстве теоремы б, заключаем, что п. н. существует Хоо = = lim Xn. Теп< п—юо галом. Поэтому = lim Хп. Теперь заметим, что и (Х+ ^п)п>о является обращенным субмартин- Е ( lim Х+) ^ limini EX+ <С ЕХ+ < оо. Отсюда видим, что Р(Хоо = оо) = 0. Первое утверждение теоремы доказано. Пусть выполнено условие G1). Поскольку при п ^ 0 выполнено Е Хп ^ Е Xn+i и + Е |ХП| = 2ЕХ+ - ЕХп <: 2ЕХ+ - с ^ 2ЕХ0+ - с, п ^ 0. Поэтому sup Е \Хп\ = со < оо. п Докажем равномерную интегрируемость семейства {Xn, n ^ 0}. Для произволь- произвольного г > 0 выберем т = ттг(г) G N так, чтобы с — е < Е Хп < с + г при п ^ т. Тогда для любого а>0ип^т(с учетом аналога A) для субмартингала), получаем Е \Хп\1{\Хп\ >а} = ЕХП1{ХП ^ а} - ЕХИ1{ХП ^ -а} = Для любого 7 > О Е \Хт\1{\Хп\ ^ а} ^ Е |Хт|1{|Хт| ^ 7} + 7P(l*n| ^ а). В силу неравенства Чебышёва и доказанной оценки sup Е \Хп\ ^ со при всех п Е имеем п Р(\Хп\ > а) ^ а-гЕ\Хп\ ^ соа-г. Выберем 7 = т(г) > 0 так5 чтобы Е |Xm|l{|Xm| ^ j} < s. Тогда sup E |Xn|l{|Xn| ^ а} ^ Зг Тем самым равномерая интегрируемость семейства {Xn, n ^ 1} установлена. Сле- Следовательно, Хп —у Xqq в L1(Q, ^, P) при п —у оо. А так как Xqq E L1, то, очевидно, |Хоо| < 00 П.Н. ЕслиХп ^ 0 п.н. дляп ^ OhEXq7 < оо для некоторого р G A, оо), то следствие 4, примененное к Xn ,..., Хо, дает оценку Е( max x Устремив N к бесконечности, получаем Е ( sup X?) < оо. Как и при доказательстве следствия 4, имеем Хп —у Х^ в Lp(Ct, 3*, Р) при п —> оо. ?
Гл. IV. Мартингалы. Дискретное и непрерывное время 139 Замечание 2. Условие G1) выполнено для любого обращенного мартингала. Обращенную версию теоремы Леви (теорема 8) содержит упражнение 29. Пусть Е|Х| < оои(^п)п^1—невозрастающаяпоследовательность сг-алгебр в (П, 3, Р). Положим ^оо = Р| ^п. Тогда при п —У оо 71=1 п.н. ив G2) Покажем, как закон 0 или 1 Колмогорова (теорема б главы III) может быть по- получен с помощью мартингальной техники. Пусть {^k}k^i — последовательность независимых векторов в Ж171, оо Пусть A Е ^оо = П ^п- Тогда, поскольку А не зависит от Зп, то Е1а = 71=1 В силу D5) имеем ЕA^ | Зп) —у ЕAа | ^оо) п-н- ПРИ п —> оо. Но ^оо С ^оо5 поэто- поэтому A G ^оо и, значит, ЕAа \ ^оо) = 1а п.н. Итак, при п —У оо P(A) = ElA = E(lA|^n)^E(lA|^oo) = lA п.н. G3) Следовательно, Р(А) равно 0 или 1. Полезное обобщение закона 0 или 1 Колмогорова для независимых одинаково рас- распределенных величин дается в приводимой ниже теореме 14. Для ее формулировки введем следующие понятия. Определение 9. Взаимно-однозначное отображение тг: N —> N называется ко- конечной перестановкой (пишем тг G ПAЧ)), если тг(п) ф п для не более чем ко- конечного числа (зависящего от тг) значений п G N. Для последовательности дейст- действительных случайных величин {?/с}/с^ъ заданных на вероятностном пространстве (П,^, Р) (т.е. для случайного элемента ? = (?1,^2? •••) ? ^^(М00)), положим 7г? = (?7Г1, ?^2,...), где тг = (tti , тг2,...) есть перестановка множества N, и опре- определим в 3* а-алгебру перестановочных событий У = {С1 (В), в G SB(ROO):C1(B) = (тгО^) для всех тг G Теорема 14 (закон 0 или 1; Хьюитт-Сэвидж). Пусть {?,k}k^i — последова- последовательность независимых одинаково распределенных величин. Тогда а-алгебра <? перестановочных событий вырождена, т. е. содержит только события веро- вероятности 0 и 1. Доказательство дано, например, в [85; т. 2, с. 533]. 30 (продолжение упражнения 28). По теореме 14 получаем, что Un,m ->• Uoo,m П.Н. при П -»> ОО. С помощью теоремы 13 докажите, что если {?/c}/c^i ~~ последовательность независимых одинаково распределенных величин, то С/оо,т — Е д(?\,..., ?т).
140 А. В. Булинский, А. Н. Ширяев 31. Пусть (Хп, ^п)п^1 — обращенный субмартингал, и пусть выполнено усло- оо вие G1). Докажите, что Хоо ^ E(Xm\(Soo), т ^ 1, где ^ = f) <Sn п=1 (напомним, что Xqq = lim Xn существует п.н. ив L^(Q,3, P) по теоре- v n—>>oo ме 13). Иначе говоря, "замкнутый" процесс (Xn, ^n)i^n^oo —обращенный субмартингал. Покажите, что если (Xn, ^n)n^i — обращенный мартингал, то (Xn, ^n)i^n^oo также есть обращенный мартингал. 32. Можно ли в формулировке теоремы 13 отказаться от условия G1)? Теорема 15 (усиленный закон больших чисел; Колмогоров). Пусть ?ь ?г? • • • — независимые одинаково распределенные векторы со значениями в Ж171, для ко- которых Е ?i = a Е Мт . Тогда при п —У оо 1 п Хп:= -^бс -+а п.н. и в П k=i Доказательство. Без ограничения общности можно считать, что а = 0 G Ж В силу упражнения 27 обращенным (векторным) мартингалом является процесс где5п = ^i Н \-?тП = 1,..., А^. Итак,Хп = Е (Х\ \ ^n,iv) п.н. дляп = 1,... ,7V. Пользуясь теоремой Л еви (применяемой для каждой компоненты) и полагая N —у оо, оо получаем, что Xn = E (Xi | ^п) п.н., где ^n = a{Sk, к ^ п}. Пусть ^ = f| ^п. п=1 Согласно обращенной версии теоремы Л еви находим (см. G2)), что при п —У оо Е (Xl | <?„,) -У Е (Xi | ^оо) п.н. и в L1(Q, ^, Р). По закону 0 шш 1 Колмогорова сг-алгебра ^^ является вырожденной. Следовательно, Е (Xi | ^оо) = EXi = 0. Но Хп = E(Xi | ^п) и, значит, Хп ->> 0 п.н. ив L1^, &, Р). П Обобщение теоремы Колмогорова на зависимые слагаемые (в частности, попарно независимые), данное Этемади, можно прочитать, например, в [101; с. 66]. Следующее утверждение содержит в себе одновременно и теорему Леей и тео- теорему о предельном переходе под знаком условного математического ожидания (включая теорему Лебега о мажорируемой сходимости). Теорема 16 (Блэкуэлл и Дубине). Пусть последовательность действитель- действительных случайных величин {Xn}n^i на (П,^, Р) такова, что Хп —у Хоо п. н. при п —У оо и E(sup|Xn|j < оо. Пусть (&п)п^1 — семейство а-алгебр в 3*, ко- , оо торые либо возрастают, либо убывают I соответственно, 3*^ — \j 3n или П &п • п=1 У Тогда lim E(Xn\3k) = E(Xoo\3oo) п.н. и в Ь\п,3,Р). G4) п,к—>-оо
Гл. IV. Мартингалы. Дискретное и непрерывное время 141 Доказательство. Обозначим Um= sup Е(Хп\&к), Уш= inf Е(Хп\&к), m^l. к^ Тогда существуют (п.н.) U = lim иш и V = lim Уш (в силу монотонности по- т—юо т—юо следовательностей {С/т} и {Ут}). Пусть Ут = sup Хп, т ^ 1. Заметим, что Е \Yn\ ^ Е (sup |Xn| j < оо, т ^ 1. Следовательно, при каждом т ^ 1 либо по теореме 8, либо по обращенной версии теоремы Леви ^E(ym|^oo) п.н. и в L^O^P) при fe ^ оо. G5) Поскольку Xn ^ Ym при n ^ m, то Е (Xn | i^) ^ Е (Ут | &;^) п.н. для п ^ т,к ^ 1. Тогда в силу G5) получаем С/ ^ lim sup E(Fm | ^) ^ HmsupE(Fm | J^oo) п.н. Из того, что Ym I Xoo, имеем E (Ym \ 3<oo) I E (X^ \ 9^^) п.н. при т ->• оо. Та- Таким образом, С/ ^ Е (Хоо | <^оо). Аналогично показывается, что V ^ Е (Х^ \ 3*оо)- Поэтому U = V и.я. Доказательство L1 -сходимости в G4) предлагается в качестве упражнения. ? Существуют разнообразные обобщения теоремы Дуба (теорема 6) о сходимости субмартингалов и мартингалов. Так, если М = (Mn, ^n)n^i ~ квадратично интег- интегрируемый мартингал с квадратической характеристикой (М) = ((M)n, &n-i)n^i (см. B1)), то {(М>оо < оо} С {М ^}, т.е. на множестве {(М)^ < оо}, где (М)^ = lim (M)n, последовательность п—юо {Mn}n^i сходится (п.н.) к конечному пределу. Кроме того, Мп = о(/((М)п)) при п —>¦ оо для всякой функции /: М_|_ —>- М+ такой, что г Jo (в частности, для /(?) = t1/2(log+ t)a, где а > 1/2); подробнее см. [101; с. 53]. 33. Пусть (^t)t^o — неубывающий поток сг-алгебр (необязательно полных) в полном вероятностном пространстве (П,^, Р). Тогда &t+ — ^t+5 T«e. операции расширения классом Р-нулевых множеств и замыкания фильт- фильтрации по непрерывности справа перестановочны (см. упражнение 2). 34. Можно ли утверждать, что для винеровского процесса и его естественной фильтрации (&t)t>o выполнено соотношение ^ — ^t+ ПРИ всех t ^ О?
142 А. В. Булинский, А. Н. Ширяев Определение 10. Мартингал (субмартингал) X = (Xt,^t)t^o называется удовлетворяющим обычным условиям, если фильтрация (^t)t^o расширена клас- классом Р-нулевых множеств и непрерывна справа. 35. Пусть (Xt, &t)t^o — субмартингал (мартингал) имеющий п.н. непрерывные справа траектории. Пусть для каждого t Е М+ существует 8 = 8(t) > О такое, что Е( sup \XS\] < оо. G6) vse[tt+<5] J Покажите, что тогда X = (Xt, &t+) — субмартингал (мартингал) (указа- (указание: используйте теорему 16). Убедитесь, что условие G6) выполнено для винеровского и пуассоновского процессов (напомним, что траектории винеровского процесса п.н. непрерыв- непрерывны, а траектории пуассоновского процесса п. н. непрерывны справа). В связи с двумя последними упражнениями приведем следующий фундаменталь- фундаментальный результат о существовании у субмаргингалов модификации с "хорошими" тра- екторными свойствами. Теорема 17 (см. [148; с. 16]). Пусть X = (Xt,^t)t^o — субмартингал и фильтрация (^t)t^o удовлетворяет обычным условиям. a) Тогда процесс (Xt)t^o имеет модификацию (Xt)t^o c траекториями, не- непрерывными справа, тогда и только тогда, когда функция t \->EXt из М+ в Ж непрерывна справа. b) Если эта непрерывная справа модификация существует, то она может быть выбрана так, что будет иметь в каждой точке t Е @, оо) предел слева и будет согласована с фильтрацией (^t)t^o- При этом X = (Xt,^t)t^o есть субмартингал. Замечание 3. В общей теории случайных процессов обычно сразу предпола- предполагается, что фильтрация (^t)t^o удовлетворяет обычным условиям, а мартингалы (субмартингалы, супермартингалы) имеют траектории из пространства Скорохода D = D([0, оо), т. е. являются непрерывными справа в каждой точке t ^ 0 и имеющи- имеющими пределы слева в точках t > 0. Имеется много различных обобщений понятия мартингала. Пожалуй, наиболее важным является понятие локального мартингала (субмартингала, супермартин- супермартингала) . Определение 11. Процесс М = (Mt,^t)t^o c траекториями из пространства D = D([0,oo)) есть локальный мартингал (локальный субмартингал, локальный супермартингал), если найдется неубывающая последовательность марковских мо- моментов {тп}п^1, тп *\ оо (п.н.) такая, что при каждом п остановленные процессы Мп = (MtArn, &t)t^o являются (обычными) мартингалами (субмартингалами, су- супермартингалами) . С помощью понятия локальных мартингалов вводится понятие семимартингала, являющегося основным процессом, с которым связано развитие стохастического ис- исчисления (см., например, [46]).
Гл. IV. Мартингалы. Дискретное и непрерывное время 143 Определение 12. Процесс X = (Xt,&t)t^o c траекториями из D называется семимартингалом, если этот процесс допускает (вообще говоря, неединственное) представление X = Хо + М + А, где М = (Mt, &t)t^o — локальный мартингал с Mq = 0, а А = (At, &t)t^o —про- —процесс с траекториями ограниченной вариации (на каждом промежутке @, ?], ? > 0) с Ао = 0. Упомянем еще такие родственные мартингалам процессы, как демимартингалы (см. [168]), миксингалы ([162]), а также мартингалы с многопараметрическим индек- индексом ([119, 195]). Для дальнейшего чтения о мартингалах и их применениях можно обратиться, например, к книгам [26, 46, 95, 138,146,148,182,184,197]. Стохастические модели, основанные на понятии семимартингала, и стохастичес- стохастическое исчисление для них интенсивно используются в финансовой математике — одном из бурно развивающихся разделов современной теории случайных процессов. В качестве краткого введения в некоторые разделы финансовой математики рассмот- рассмотрим одну простую модель финансового рынка для случая дискретного времени. Пусть на финансовом рынке эволюция безрискового актива В (банковские счета) и рискового актива S (акции) описываются системой разностных уравнений (п ^ 1) АВп=гпВп-и G7) ASn=pnSn-1, G8) где, как обычно, полагаем АХп = Хп — Хп-\. Кроме того, предположим, что все случайные величины Вп, Sn, тп и рп заданы на фильтрованном вероятностном про- пространстве (П, ^, (^vOi^n^iV, Р), где &n = &•> N < оо. Положим для п = 1,..., N k=0 k=0 где {rn} и {pn} — стохастические последовательности, т.е. 3*п \ Зё(Ж)-измеримые при каждом (рассматриваемом) п. Будем предполагать также, что ^о = {0, П} и &п = сг{5о,..., Sn} при п ^ 1. Определение 13. Описанная выше модель называется (В, S)-рынком. 36. Докажите, что решения уравнений G7) и G8) записываются в виде (дискрет- (дискретных) стохастических экспонент, т.е. Bn = Bogn(U), Sn = S0Sn{V), где к = 1
144 А. В. Булинский, А. Н. Ширяев Определение 14. Инвестиционной стратегией, или портфелем, называется двумерная стохастическая последовательность тг = (/3n57n)o^n^iv такая, что для всех п Р&\ЗД 7п е &n- Капиталом портфеля тг называется стохастическая последовательность Х* = {Х%}, где XZ = /3nBn+lnSn, O^n^N. Величины /Зп и 7п интерпретируются соответственно как количества активов В и S в момент времени п. Определение 15. Класс портфелей тг, обладающих при всех п = 1,..., N свой- свойством называется самофинансируемым и обозначается SF. 37. Проверьте, что капитал самофинансируемого портфеля можно представить в виде XI = XS + к=0 где ДБ0 = Д50 = 0. Определение 16. В классе SF выделим подкласс арбитражных портфелей &rh (он может оказаться пустым), который состоит из таких тг, что Х^ = 0, Х^^О Р-п.н. при n^N X^j > 0 с положительной вероятностью. Таким образом, экономический смысл данного определения состоит в том, что можно получить прибыль, не рискуя. С последним определением тесно связано сле- следующее. Определение 17. Вероятностная мера Р*, эквивалентная Р (т.е. Р* < Р и Р ^С Р*), называется мартингалъной, если на введенном фильтрованном простран- пространстве {Sn/Bn}i^n^N — мартингал относительно Р*. Обозначим Р* класс всех мартингальных мер Р*. Теорема 18. Пусть в модели [В,8)-рынка случайные величины {rn}i^n^jv являются предсказуемыми (т. е. rn G &п-1 I &(Щ) и гп > —1. Тогда Р* G Р* эквивалентно тому, что {Vn — ?/n}i^n^jv — мартингал относительно Р*, где Un и Vn определены согласно G9). Теорема 19 (основная теорема финансовой математики). Пусть в моде- модели (В,Б)-рынка {rn}i^n^jv — детерминированный набор чисел такой, что гп > — 1 при 1 ^ п ^ N. Тогда
Гл. IV. Мартингалы. Дискретное и непрерывное время 145 Важной характеристикой рынка, позволяющей получать точные формулы в раз- различных финансовых расчетах, является его полнота. Определение 18. Говорят, что (В, S)-рынок полный, если для любой функции / е 3* | Зё(Ж) найдется стратегия тг е SF такая, что Х^(ш) = f(u),u ей. Свойство полноты рынка с математической точки зрения обеспечивает доступ- доступность всех фигурирующих на рынке активов и отсутствие ограничений для инвести- инвестирования в эти активы. Теорема 20 (о полноте (В, S)-рынка). Пусть Р* ф 0. Тогда следующие ут- утверждения эквивалентны: 1) (В,S) -рынок является полным, 2) Р* содержит единственный элемент Р*, 3) любой мартингал (Мп, &п, P*)o^n^iv допускает представление п Мп = М0 + ^ k=l Замечание 4. Поскольку Anik = Sk-\Bk (pk — г^), для величин {Мп} спра- справедливо эквивалентное представление п -rk), (80) Нам понадобится еще несколько понятий. Определение 19. Платежным обязательством с датой погашения N назы- называют пару (/, JV), где неотрицательная случайная величина / G &n \ ?%(Щ. Участ- Участник рынка, который должен погасить данное обязательство, строит свою инвести- инвестиционную политику так, чтобы XJj ^ /. Процедура построения такого портфеля тг называется хеджированием, а сам портфель — хеджирующим портфелем. Одна из важнейших задач, связанных с хеджированием платежных обязательств, возникает в связи с расчетами опционов. Представим на финансовом рынке эмитен- эмитента, выпустившего производную ценную бумагу (на покупку, продажу и т. д.) некото- некоторого актива. Чтобы стать держателем такой бумаги, требуется заплатить эмитенту некоторую премию С. Тем самым приобретается право предъявить данную бумагу к исполнению в момент времени N и получить выплату в размере /. Такая произ- производная ценная бумага называется опционом европейского типа (на покупку, про- продажу и т. д. актива), а сама сделка— контрактом с опционом. Имеются и опцио- опционы американского типа, когда держатель производных ценных бумаг имеет право предъявить их в любой момент п G {0,..., N} и получить выплату в размере /п. Продавец такой ценной бумаги, получивший при ее продаже премию С, должен ор- организовать свой портфель тг так, чтобы X^{uj) ^ /п(^) для всех uj G П и п ^ N. Обратимся к опционам европейского типа.
146 А. В. Булинский, А. Н. Ширяев Определение 20. Пусть на (В, S)-рынке задано начальное значение капитала х > 0 и платежное обязательство (/, N). Самофинансируемый портфель тг называ- называется (ж, /, N)-xedotceM, если для любого и; Е П XS = х и Х% 2 /. Обозначим П(ж, /, N) множество (ж, /, А^)-хеджей. Инвестиционной стоимостью платежного обязательства (/, N) называется величина C(N) = М{х > 0: П(ж, /, N) ф 0}. (81) Формула (81) отвечает идее одновременно удовлетворить и продавца (который на данном рынке может обеспечить обязательство (/, N)) и покупателя (который пла- платит в определенном смысле минимальную премию продавцу). Поэтому C(N) назы- называют также справедливой ценой опциона. Широкую известность имеют опцион колл (опцион на продажу) и опцион пут (опцион на покупку). Первый из них — это опцион с платежным обязательством f = (g/v — if)+, согласно которому его держатель может приобрести некоторые акции по фиксированной цене К в момент N (в то время, как рыночная цена акций равна Sn). Разумеется, такой опцион предъявляется к исполнению, если Sn > К, и не предъявляется, когда Sn ^ К. Опцион пут — это опцион, по которому держатель имеет право продать акции по фиксированной цене К в момент N, и в этом случае платежное обязательство / = (К — Теорема 21. Пусть (B,S)-pbiHOK не допускает арбитражных возможнос- возможностей и полон. Пусть детерминированная последовательность тп такова, что тп > —1 при п = 0,1,..., N. Тогда для опциона европейского типа с платеж- платежным обязательством (f,N) выполнены следующие утверждения: 1) справедливая цена определяется формулой где Е* обозначает усреднение по единственной (в силу теоремы 20) мартингалъной мере; 2) существует минимальный (C(N), /, N)-xedopc тг* = (/^7п)п^аг, т.е. такой, что Х^ = /', и при этом здесь {jn}n^N — предсказуемая последовательность из разложения (80) для мартингала {Е*E^1/ | ^rn)}n=o,...,iv- О приведенных выше результатах и о многих других интересных моделях финансо- финансового рынка как с дискретным, так и непрерывным временем, можно прочитать в ра- работе [86], где имеется обширная библиография.
Глава V Слабая сходимость мер. Принцип инвариантности Слабая сходимость мер в метрических пространствах. Сходимость случайных элементов по распределению. Критерии слабой сходимости. Сохранение слабой сходимости под действием непрерывных отображений. Слабая сходимость мер в пространстве С(Т, S). Относительная слабая компактность и плотность семейства мер. Теорема Прохорова. Принцип инвариантности Донскера-Прохорова. Мно- Многомерная центральная предельная теорема Линдеберга, лемма о максимуме сумм независимых случайных величин. Схема доказательства критерия согласия Кол- Колмогорова. Броуновский мост как условный винеровский процесс. Метод одного вероятностного пространства, теорема Скорохода. Метризация слабой сходимости. Метрика Леви-Прохорова. § 1. В этой главе мы затронем обширный круг вопросов, связанных с аппрокси- аппроксимацией распределений вероятностей одних процессов другими. Основной резуль- результат этой главы — принцип инвариантности Донскера-Прохорова (§6). При первом чтении для понимания этого результата требуется ознакомиться с понятиями сла- слабой сходимости мер и сходимости по распределению случайных элементов со значе- значениями в метрическом пространстве (§ 1 и § 2). Существенную роль при этом играет теория слабой сходимости мер в пространстве непрерывных функций С[0,1] (изло- (изложенная в § 5), опирающаяся на идею установления слабой сходимости конечномер- конечномерных распределений и относительной компактности мер. Последнее свойство удается эффективно проверять благодаря фундаментальной теореме Прохорова, доказатель- доказательство которой можно прочитать в приложении 2. Очень полезной является теорема Скорохода (§ 10), дающая метод переопределения случайных элементов на новое ве- вероятностное пространство с получением сходимости почти наверное. К остальным результатам главы V желательно вернуться при повторном чтении. Начнем с понятия слабой сходимости (вероятностных) мер. Определение 1. Пусть (S, р) — метрическое пространство с борелевской сг-ал- геброй ?&{S) и Q, Qn (n ^ 1) — меры на (S,^(S)). Говорят, что меры Qn слабо сходятся к мере Q (обозначение: Qn => Q), если для любой функции / Е Cb(S, Ж), т. е. непрерывной ограниченной функции /: S —> М, / f(x) Qn(dx) -> / f(x) Q(dx), n -> oo. A) s is
148 А. В. Булинский, А. Н. Ширяев Равносильное определение получается, если в A) брать комплекснозначные функции / Е Cb(S,C), т.е. непрерывные ограниченные функции /: S —> С. Вместо последовательностей мер можно рассматривать более общие структуры, на- например, сети, т.е. семейства мер, индексированные направленной переменной, например, {Qaj a > 0}. Интеграл функции / по мере Q (когда он определен) будем обозначать также </,Q>- Если слабый предел существует, то он единствен, как показывает Лемма 1. Пусть (/, Q) = (/, Q) для всех f е Cb(S,R). Тогда Q = Q. Доказательство. Пользуясь рассуждениями, приведенными после формулы (III.22), видим, что Q(F) = Q(F) для всех замкнутых множеств F. В силу леммы 4 главы I получаем, что Q = Q на S$(S). ? Заметим, что в этой лемме достаточно использовать лишь / Е Cb(S, М+). Напомним, что граница дВ множества В С S есть совокупность тех точек (из S), в любой окрестности которых имеются точки из В и S \ В. Множество дВ замкнуто для любого В С S, поэтому входит в ?&{S). Следующая теорема дает удобные для проверок критерии слабой сходимости. Теорема 1 (А. Д. Александров). Пусть Q, Qn (n ^ 1) — меры на мет- метрическом пространстве (S,p). Тогда слабая сходимость Qn => Q (n —> оо) равносильна каждому из следующих утверждений: 1°. limsup Qn(F) ^ Q(F) для любого замкнутого множества F G п—юо 2°. liminf Qn(G) ^ Q(G) для любого открытого множества G G п>оо 3°. lim Qn(B) = Q(??) дл^ любого множества В G 5S(S) такого, что 71—>>ОО О(ав) = о. Доказательство. Прежде всего заметим, что исходное определение A) равно- равносильно тому, что для любой функции / е C&(S, Ж) справедливо соотношение limsup(/,QnK</,Q>. B) п—юо Действительно, если B) имеет место для / G Cb(S, M), то заменяя / на —/, полу- получим liminf (/, Qn) ^ (/, Q) и, следовательно, A) выполнено. Обратное очевидно. п—^оо а) Пусть Qn => Q. Докажем утверждение 1°. Для замкнутого множества F и произвольного е > 0 определим функцию ff(x) = ip(p(x,F)/e) G Cb(S,M), где <р дается формулой (III.23). Тогда Qn(F) = AF, Qn) ^ (/f, Qn), поскольку 1F ^ /f. В силу B) HmsupQn(F) ^ limsup(/f ,Qn) ^ (/f, Q) 71—ЮО и остается воспользоваться тем, что по теореме Лебега (ff, Q) —> (If, Q) = при s —> 0.
Гл. V. Слабая сходимость мер. Принцип инвариантности 149 Ь) Пусть имеет место 1°. Докажем, что Qn => Q при п —У оо. Неравенство B) достаточно проверить при дополнительном условии 0 < f(x) < 1, выполнения кото- которого можно добиться линейным преобразованием af(x) + 6, где а > О, Ъ Е Ш. Возь- Возьмем произвольное/с Е N и рассмотрим замкнутые множества Fi = {х: f(x) ^ i/k}, г = 0,..., к. Положимd = Fi-i \F% = {(* — !)/& ^ /(ж) < */^}? * = 1? •••?&. Тогда [ № Q(dx) s к . к . к l | i +1Е г=1 г=1 г=1 Теперь заметим, что lQ^ = Е | г=1 Отсюда, вместе с аналогичным преобразованием суммы в левой части C), имеем к к у Т Q(Fi) 4: [ fix) Q(dx) < 7 + 7 Подобные неравенства верны и с заменой Q на Qn. Тем самым, к к lim sup(/, Qn> ^ - + lim sup - V Qn(Fi) 4 - + - V Q(F») ^ - + (/, Q). 1=1 1=1 Учитывая произвольность к, приходим к неравенству B), и, значит, Qn => Q. c) Эквивалентность условий 2° и 1° очевидна. (Достаточно от соответствующих множеств перейти к их дополнениям.) d) Покажем, что условие 1° влечет условие 3°. Обозначим В0 внутренностъ множества В, а через [В] — его замыкание. Тогда в силу 1° и 2° Q(B°) ^ liminf Qn(#°) ^ liminf Qn(B) ^ n^oo n^oo lim sup Qn(B) 4 lim sup Qn([B]) ^ Q([B]). E) n—^oo Ho Q([5]) = QE) = Q(?°), поскольку Q(95) = О ([Б] \ВсдВпВ\В°С дВ), поэтому из E) следует условие 3°. е) Пусть выполнено 3°, покажем, что тогда верно 1°. Возьмем замкнутое множес- множество F и рассмотрим множества F? = {x G S: p(x,F) < г}, где г > 0. Заметим, что dF? С {х: p(x,F) = г}, поэтому dF? П dFs = 0 при г ф S. Следовательно, Q(dF?) > 0 не более чем для счетного множества чисел г. Выберем последователь- последовательность Sk I 0 такую, что Q(dF?k) = 0, k ^ 1. Тогда HmsupQn(F) ^ lim Q для любого &. Остается учесть, что Q(F?k) -у Q(F) при к -у оо. П § 2. Пусть даны вероятностные пространства (П, ^, Р), (Пп, ^п, Рп), метричес- метрическое пространство S и случайные элементы X: П —>¦ S, Хп: Пп —у S (т. е. 3* \ 3e(S) и )-измеримые отображения соответственно), п ^ 1.
150 А. В. Булинский, А. Н. Ширяев Определение 2. Случайные элементы Хп называются сходящимися по рас- распределению к X (пишут Хп —у X или Хп —™ X), если Рхп => Рх при п —У оо, где Рхп и Рх — распределения вероятностей элементов Хп и X (см. A.6)). Пользуясь A.23), видим, что Хп —у X тогда и только тогда, когда Enf(Xn) -у Ef(X) F) для любой функции / Е Cb(S, М), где Е, Еп означают усреднения по мерам Р, Рп. Теорема 2. Пусть (S,^(S)); (V,^(V)) — метрические пространства uh — непрерывное отображение S в V. Если Хп —у X (Хп, X принимают значения в S), то h(Xn)%h(X), n^oo. G) Если Qn ^ Q на (S,SS(S)), то Qn/i~1 ^> Q/i wa (V,^(V)) npn n ^ оо. (8) Доказательство . Возьмем непрерывную и ограниченную функцию д: V —>¦ М. Тогда функция ^o/i принадлежит пространству Сь (S, М) как суперпозиция непрерыв- непрерывной и ограниченной непрерывной функций. Следовательно, Eng(h(Xn)) —> Eg(h(X)), п —у оо, что доказывает G). Теперь заметим, что если X: П —>¦ S есть 3* \ ^-измеримое отображение, а отобра- отображение h:S—y\/ является ё$ \ ^-измеримым ((S, Ж) и (V, 8$) —некоторые измеримые пространства), то Pxh-1 = PHX), (9) поскольку Pxh-^B) = Pxih'^B)) = Р(Х е Л^)) = Р(Л(^) е В) = РцХ)(В) для любого 5gS. Пусть Qn => Q. По лемме 8 главы I определены случайные элементы Хп, X та- кие, что Qn = Рхп, п ^ 1, и Q = Рх- Таким образом, Хп —у X при п —у оо. В силу (9) имеем Qnh~1 = Рцхп)? Qh-1 = Рцх)- Поскольку слабая сходимость Рцхп) к Ph(x) Уже доказана, то отсюда получаем требуемое утверждение (8). ? § 3. Как и для числовых последовательностей, при изучении слабой сходимости семейства мер важную роль играет рассмотрение сходимости мер по подпоследова- подпоследовательностям. Лемма 2. Сходимость Qn => Q, п —у оо, имеет место тогда и только тог- тогда, когда из каждой последовательности {rik} С N можно извлечь подпосле- подпоследовательность {nfk} такую, что Qn/ => Q при к —У оо. Доказательство. Необходимость очевидна. Докажем достаточность. Допус- Допустим, что выполнено утверждение о подпоследовательностях, но Qn ф> Q при п —у оо. Тогда существуют функция / G Cb(S, Ж) и последовательность {rrik}k^i такие, что для некоторого г > О К/, Qmk) ~ (/, Q)| > е при всех к ^ 1. Отсюда видим, что из последовательности {rrik}k^i нельзя извлечь подпоследова- подпоследовательность {га^}/^!, для которой Qm/ => Q при к —у оо. Полученное противоречие доказывает утверждение о достаточности. ?
Гл. V. Слабая сходимость мер. Принцип инвариантности 151 Определение 3. Семейство мер {Qa5 & ? Л} на (S, 5§(S)) называется слабо относительно компактным, если из любой последовательности {Qan } можно из- извлечь слабо сходящуюся подпоследовательность {Qan, } (сходящуюся, вообще гово- говоря, необязательно к элементу этого семейства). Любая слабо сходящаяся последовательность является, очевидно, слабо относи- относительно компактной. Легко построить слабо относительно компактную последова- последовательность, не имеющую слабого предела. (Достаточно взять две последовательнос- последовательности, слабо сходящиеся к разным пределам, и составить из них одну.) Полную связь между двумя введенными понятиями слабой сходимости и слабой относительной компактности раскрывает Теорема 3. Для того чтобы заданная на борелевской а-алгебре S$(S) метри- метрического пространства S последовательность мер {Qn}n^i имела слабый пре- предел, необходимо и достаточно одновременное выполнение следующих условий: 1) последовательность {Qn}n^i слабо относительно компактна; 2) имеется такой класс функций Ж С Cb(S,M), что 2а) существует lim (ft, Qn) при каждом ft G Ж, п—>-оо 2Ь) для любых мер Q и Q на (S,^(S)) соотношение (ft, Q) = (ft, Q), верное при всех ft е Ж, A0) влечет на ?&{S) равенство Q = Q. Доказательство . Необходимость очевидна — надо взять Ж — Cb(S, Ж) и вос- воспользоваться леммой 1. Достаточность. Согласно 1) из произвольной последовательности {п^}к^1 мож- можно выбрать подпоследовательность {п'к }k^i такую, что Qn/ => Q при к —У оо. Тогда (/? Qn' ) "^ (/? Q) Для всех / ? Cb(S, M). Поэтому условие 2а) влечет соотношение lim (/i, Qn) = (h,Q) при каждом ft, G J^f. A1) n—>-oo Допустим, что Qn 7^ Q ПРИ n —> оо. Тогда в силу условия 1) и леммы 2 найдется последовательность {тп^}к^1 такая, что Qmfc => Q при fc —у оо и Q ф Q. Из A1) вытекает, что (ft, Q) = (ft, Q) для всех ft G Ж, и согласно условию 2Ь) имеем Q = Q, а это противоречит предположению Q ф Q. ? §4. Рассмотрим процессы X = {Xtj t G Г} и X^n) = {Xt(n), ? G Г}, где п ^ 1, заданные соответственно на некоторых вероятностных пространствах (П, ^, Р) и (пп, ^п, Рп)и принимающие при каждом t G Т значения в измеримом пространстве (St,^t)- Учитывая теорему 3 главы I, естественно было бы определить слабую сходимость процессов Х^ к X как слабую сходимость их распределений на цилиндрической сг-алгебре S&t- Но слабая сходимость была введена для мер на бо- борелевской сг-алгебре метрического (или топологического) пространства, и в общем случае нельзя утверждать, что пространство St допускает метризацию, при которой S&t = S$(St)- Однако в некоторых случаях эту трудность можно преодо- преодолеть, учитывая, что траектории изучаемых процессов лежат (с вероятностью 1) в некотором собственном (метризуемом) подмножестве пространства St-
152 А. В. Булинский, А. Н. Ширяев В качестве такого подмножества рассмотрим пространство непрерывных функ- функций С{Т, S), заданных на некотором компакте Т (метрического пространства) и принимающих при каждом t G Г значения в польском пространстве S, снабженном равномерной метрикой рс (см. § 18 главы I). Напомним, что ё$(С(Т, S)) обозначает борелевскую сг-алгебру в пространстве С(Т, S), а ё$т(С(Т, S)) — цилиндрическую сг-алгебру в этом пространстве, т.е. сг-алгебру, порожденную "непрерывными ци- цилиндрами" вида Kt~1]...,tk(B)> где В ? S$(Sk), t\,... ,tk G T, к ^ 1, и отображение 7Tti,...,tfc : С(^\ S) —>¦ Sfc задается формулой *t1,...,tkx = (rr(ti),..., rr(tfc)), x G C(T, S). A2) В силу леммы 12 главы I имеем Зёт(С(Т, S)) = $(С(Т, S)). Теорема 4. Пусть С{Т, S) — введенное выше пространство. Тогда спра- справедливы следующие утверждения: a) еслг/ г/ .мер Q, Q, заданных на ё%{С{Т, S)), совпадают все конечномерные распределения, т. е. Q^!...,tfc = Q^!...,tfc ua* ti,...,^ gt, &gn, A3) moQ = Q wa^(C(T,S)); b) последовательность мер {Qn}n^i Wft ?$(C(T, S)) имеет слабый предел при п —у оо тогда и только тогда, когда она слабо относительно компактна и существуют слабые пределы у всех конечномерных распределений, т. е. при каждых t\,...,tk G Т и к ^ 1 последовательности {G^tt^ tfc}n^i имеют слабые пределы. c) Qn => Q wa S$(C(T, S)) npn n —>- оо тогда и только тогда, когда после- последовательность {Qnjn^i слабо относительно компактна и все конечно- конечномерные распределения Qn слабо сходятся к соответствующим конечно- конечномерным распределениям Q. Доказательство, а) Если меры Q и Q совпадают на алгебре ("непрерывных цилиндров" в С(Т, S)), то по теореме Каратеодори они совпадают и на порожденной ими сг-алгебре, т. е. на ?$т(С(Т, S)). Значит, они совпадают на борелевской сг-алгебре b) Необходимость вытекает из того, что слабо сходящаяся последовательность мер является слабо относительно компактной, а сходимость конечномерных распре- распределений обеспечивает теорема 2 в силу непрерывности отображений A2). Для до- доказательства достаточности воспользуемся теоремой 3. Предположение слабой от- относительной компактности последовательности {Qn}n^i означает справедливость условия 1) упомянутой теоремы. Рассмотрим класс функций Ж={Н = дко 7rtl,...,tfc, где gk G Cb{Sk, Ж), h,..., tk G T, k > l}. Согласно формуле A.23) для h = gk ° 7Tt1,...,tfc G Ж имеем </i,Qn> = <5fc,Qn^...,tfc>- A4)
Гл. V. Слабая сходимость мер. Принцип инвариантности 153 Слабая сходимость последовательности мер {Qn^^j^n^i влечет в силу A4) вы- выполнение условия 2а) теоремы 3. Та же формула A4), записанная для произвольных мер Q, Q вместо Qn, и утверждение а) показывают, что выполнено условие 2Ь) тео- теоремы 3. Таким образом, последовательность {Qn}n^i имеет слабый предел. с) Если Qn => Q при п —У оо, то требуемое утверждение получается аналогич- аналогично доказательству необходимости в пункте Ь). Пусть теперь {Qn}n^i слабо отно- относительно компактна и конечномерные распределения мер Qn слабо сходятся к ко- конечномерным распределениям Q (п —у оо). Тогда из любой последовательности Qnk можно выделить подпоследовательность Qn/ , слабо сходящуюся к некоторой мере Q. Пользуясь единственностью слабого предела (лемма 1), видим, что все ко- конечномерные распределения мер Q и Q совпадают. Следовательно, по доказанному выше пункту а) меры Q и Q совпадают на 8ё(С(Т, S)). Утверждение о слабой сходи- сходимости Qn к Q вытекает теперь из леммы 2. ? Покажем, что условие слабой относительной компактности мер Qn (по-прежнему рассматриваемых на 8ё(С(Т, S))) и условие сходимости их конечномерных распреде- распределений "независимы" в следующем смысле. Легко построить пример слабо относительно компактной последовательности Qn, для которой не имеет место сходимость всех конечномерных распределений. Доста- Достаточно взять две различные меры Q и Q' и затем составить из них чередующуюся последовательность Q,Q',Q,Q',.... Теперь приведем пример последовательности {Qn}5 Для которой все конечномер- конечномерные распределения слабо сходятся к конечномерным распределениям некоторой ме- меры Q, но при этом последовательность {Qn}n^i не является слабо относительно компактной (для этого в силу теоремы 4 достаточно убедиться, что не существует слабого предела Qn при п —> оо). Пример 1. Пусть мера Qn сосредоточена на функции хп(-) Е С[0,1], изобра- изображенной на рис. 14, т.е. Qn = 5Хп, п ^ 1 (как обычно, "дельта-мера" SX(A) = 1а(х) есть мера, "сидящая" в точке ж, и С [0,1] = С ([0,1], Ж)). xn(t) 1/Bга) 1/п 1 t Рис. 14 Легко видеть, что для любого В е 88(Жк), к ^ 1 и ti,..., tk G [0,1] SXnirri]...ttk(B)=Sxn{xeC[O,l]:(x(t1)^ При всех достаточно больших п имеем хп(и) = 0, г = 1,..., к. Поэтому при этих п lB(xn(ti),...,xn(tk)) = 1В(О,...,О) = SX07T^l^tk(B),raex0(t) = 0, t G [0,1]. Следовательно, конечномерные распределения мер Qn слабо сходятся к соответст- соответствующим конечномерным распределениям меры SXQ при п —у оо.
154 А. В. Булинский, А. Н. Ширяев Допустим, что существует мера Q такая, что 8Хп => Q при п —У оо на ?$(С[0,1]). Тогда по теореме 2 (или пункту с) теоремы 4) для всех к ^ 1 и t\,..., ?& Е Т В силу пункта а) теоремы 4 получаем, что Q = SXQ. Возьмем в С[0,1] замкнутое множество F = <ж(-): sup x(t) ^ 1 f. Очевидно, SXQ(F) = ОийЖп(^) = 1 для 1 t<E[O,l] J n ^ 1. Согласно теореме 1 (пункт 1°) соотношение 8Хп => 5Хо при п —у оо невозмож- невозможно. Полученное противоречие доказывает, что построенная последовательность мер не является слабо относительно компактной. Функции хп из примера 1 не имеют предела в пространстве С [0,1], поэтому другой способ проверить, что у мер 8Хп нет слабого предела, дает упражнение 1. § 5. Со слабой относительной компактностью семейства мер связано следующее важное понятие. Определение 4. Семейство мер {Qa}aeA на борелевской сг-алгебре метричес- метрического пространства (S, р) называется плотным, если для любого г > 0 можно найти компакт К?, такой что Qa(K?) > 1 — ? при всех a G А. Введенное понятие плотности позволяет сформулировать следующий фундамен- фундаментальный результат теории слабой сходимости, доказательство которого дается в приложении 2. Теорема 5 (Прохоров). Если на метрическом пространстве (S,p) семейст- семейство мер плотно, то оно слабо относительно компактно. Если к тому же пространство (S,p) является польским, то слабо относительно компактное семейство мер плотно. Утверждения теорем 4 и 5 дают весьма эффективный метод доказательства сла- слабой сходимости последовательности мер {Qn}n^i K мере Q на введенном выше мет- метрическом пространстве С(Т, S), основанный на идеях конечномерной сходимости и плотности. Для иллюстрации этого метода обратимся к польскому пространству С[0,1]. Напомним Определение 5. Модулем непрерывности функции / G С[0,1] называется функция A(f,S)=sup{\f(t)-f(s)\:s,te[O,l], |s-t|<<*}, S > 0. Заметим, что функция А( •, S) непрерывна на С[0,1] и, следовательно, Зё(С[0,1])- измерима. Следующий результат из теории функций (см., например, [32; с. 302]) описывает компакты в С[0,1].
Гл. V. Слабая сходимость мер. Принцип инвариантности 155 Теорема 6 (Арцела-Асколи). Замыкание множества функций Ж С С[0,1] является компактом в этом пространстве тогда и только тогда, когда sup |/@)| < оо и lim sup A(f,8) = 0. (Указанные условия равносильны тому, что семейство функций Ж равномерно ограничено и равностепенно непрерывно.) Отсюда легко выводится следующая теорема. Теорема 7. Семейство мер {Qa}aeA плотно на Й§(С[0,1]) тогда и только тогда, когда одновременно выполнены следующие два условия: 1°. Для любого г > 0 существует М = М(е) > 0 такое, что Qa(f- 1/@I > М) < г при всех а е А. 2°. Для любых е, v > 0 найдется 8 = 8{е, v) > 0 такое, что Qa(/: A(f,S) > v) < г при всех a Е А. Доказательство. Пусть семейство {Qa, а е Л} плотно. Для данного г > О возьмем компакт К? С С[0,1] такой, что Qa(K?) > 1 — г при всех a G Л. По теореме Арцела-А сколи для любых г, v > 0 существуют М = М(е) > Он S = S(e,v) > О такие, что Поэтому необходимость условий 1° и 2° очевидна. Пусть теперь выполнены условия 1° и 2°. Для каждого г > 0 найдем М = = М(е) > 0 такое, что Qa(#o) < е/2 при всех а е Л, где Бо = {/: |/@)| > М}. Выберем <5/с > 0 таким образом, чтобы Qa(Bk) < ?2~(/c+1) при всех а е Л и & ^ 1, г оо -| где Вк = {/: Д(/А) > 1/&}. Положим К? = f| Б/, , где Б/, = С[0,1] \ 5fc, L/c=o J [ • ] — замыкание множества в С[0,1]. По теореме Арцела-Асколи ife — компакт. оо Кроме того, Р(К?) ^ Yl e2~(k+1") = г. Следовательно, семейство мер {Qa, a e А} fc=o плотно.П Замечание 1. Если А = N, то в теореме 7 условие 2° можно заменить на следую- следующее ему равносильное условие: 2'. Для любых е, v > 0 найдутся S = й(г, v) > 0 и по = по(е, г/, J) такие, что Qn(/: A(f,5) > v) < г при всех п > п0. Действительно, любое конечное семейство мер в польском пространстве являет- является плотным в силу леммы Улама (см. в дополнении к этой главе лемму 6). Следо- Следовательно, по доказанной теореме 7 для любых г, v > 0 существует 8' = 8'(s, v) > О такое, что Qn(f:A(f,Sf)>i/)<? при всех п = 1,...,п0. Теперь надо лишь взять минимум из 8' и 8, фигурирующего в 2'.
156 А. В. Булинский, А. Н. Ширяев Вместо 2' часто бывает удобнее проверять обеспечивающее его условие: 2". Для любых г, v > 0 существует N = N(e, v) G N такое, что N г=1 при всех п > no (s, г/, TV). В самом деле, для любых N ^ 1 и г/ > О iV {/: А(/, 1/ДО < Зг/} э f| { sup |/(s) - /((г - 1)/N)\ < Л. Поэтому условие 2" влечет условие 2'. § 6. Перейдем к рассмотрению, пожалуй, одного из самых известных и интересных применений общей теории слабой сходимости мер в функциональном пространстве непрерывных функций — к так называемому принципу инвариантности, являюще- являющемуся функциональным аналогом (и обобщением) центральной предельной теоремы. Начнем с необходимых определений. Пусть (П, ^, Р) — вероятностное простран- пространство и Хп^ i = 1,..., mn, n ^ 1, — серии независимых (при каждом п) действи- действительных случайных величин, заданных на этом пространстве, таких, что Е Хп^ = О, тп г = 1,.. .,?тгп, и Yl an i = !j где ап i = Е^п г Если Fn^ (г = 1,.. .,шп) —неза- г=1 висимые величины с конечными дисперсиями и хотя бы одна из этих величин невы- невырожденная, то к указанной схеме можно перейти нормировкой, положив тп Xn,i = (Ynii - EYn,i)/Bn, B2n = Y,DYn,i- г=1 3 3 Обозначим tn,о = 0, tnj = Е^^г (тогда tn,rnn = 1), 5П,О = 0, Snj = Y^xn,i, j — 1,..., ттгп. Зададим теперь при каждом ио G П функцию 5n(t, w),tG [0,1], как непрерывную ломаную с узлами (tn?j, Snj), j = 0,... ,mn, т.е. пусть Sn(t,(j) = 5n,i-l + " _^г~1 ^п,г ДЛЯ t G [*те,г-1, *п,г] A5) (см. рис. 15). Здесь и далее будем считать, что все оп^ > 0, иначе говоря, мы ис- исключаем из рассмотрения величины Хп^ = 0 п.н. Формула A5) показывает, что Sn(t, uj) при каждом t G [0,1] является случайной величиной, а так как траектории Sn( •, и) непрерывны при всех ио G П, то Sn = {5n(t), t G [0,1]} является случайным элементом в пространстве С[0,1] по теореме 7 главы I. Пусть Рп — распределение вероятностей Sn на Зё(С[0,1]). Теорема 8 (Прохоров). Пусть для описанных выше величин Хп^, где п ^ 1 г/ г = 1,... ,mn, выполнено условие Линдеберга: ЕХ^1{|Хгг .|>е} ->• 0 ^л^ любого г>0 при п ->• оо. A6) г=1 Тогда Тп => W (п —>- оо), здесь W — .мера Винера. Другими словами, Sn —> W при п —У оо, где VF = {W(t), ? G [0,1]} — винеровский процесс на отрезке [0,1].
Гл. V. Слабая сходимость мер. Принцип инвариантности 157 Рис. 15 Утверждение о слабой сходимости распределений случайных ломаных к мере Винера часто называют принципом инвариантности Донскера-Прохорова. М. Донскер исследовал случай независимых одинаково распределенных величин Х\, Х2,... с нулевым средним и единичной дисперсией. Полагая Sn = Х\ + • • • + Хп, п ^ 1, So = 0, он рассматривал случайные ломаные с узлами (г/п, Si/л/п), i = 0,...,п. Теорема 8 (с условием Линдеберга), охватывающая и случай, рассмотренный М. Донскером, доказана Ю. В. Прохоровым. Полезно подчеркнуть, что в приведенной формулировке теорема 8 включает в се- себя обычную центральную предельную теорему для серий независимых действи- действительных случайных величин в условиях Линдеберга. Действительно, функционал h(x(-)) = хA) — это непрерывное отображение С[0,1] в 1. Поэтому согласно тео- теореме 2 имеем при п —У оо h{Sn{-))%h{W{-)), т.е. Sn(l) = Sn>mn4w4l)~N@,l). Функциональный характер теоремы 8 объясняет, почему ее называют также функ- функциональной центральной предельной теоремой. Понятно также и происхожде- происхождение термина принцип инвариантности — инвариантность понимается в том смыс- смысле, что предельное распределение h(Sn( •)) будет таким же, как у h(W( •)), какие бы серии независимых случайных величин, удовлетворяющих условию Линдеберга, мы ни брали (при этом h может быть любым непрерывным отображением, заданным наС[0,1]). Таким образом при больших п распределение (непрерывных ограниченных) функ- функционалов h{Sn{ •)) "близко" к распределению функционала h(W( •)) от винеровско- го процесса. Это разъяснение полезно и потому, что можно двигаться в обратном на- направлении — для нахождения распределения вероятностей интересующего нас функ- функционала h(W( •)) от винеровского процесса иногда удается построить такое случай- случайное блуждание S = {5п,п ^ 1}, что исследование распределения h(Sn( •)), п ^ 1, становится простой задачей (см. упражнения 11 и 13). Доказательство теоремы 8 будет опираться на идеи "слабой сходимости конечно- конечномерных распределений и плотности", изложенные выше в теоремах 4 с), 5 и 7. § 7. Нам понадобятся два вспомогательных утверждения, представляющих и са- самостоятельный интерес. Первое утверждение — это многомерная центральная пре- предельная теорема, которая будет играть ключевую роль при доказательстве слабой сходимости конечномерных распределений процессов Sn (•), п ^ 1.
158 А. В. Булинский, А. Н. Ширяев Для формулировки этой теоремы (ее доказательство дано в приложении 3) рас- рассмотрим в Жк случайные векторы ^п^,г = 1,..., mn, независимые для любого фик- фиксированного п ^ 1 (в каждой серии) такие, что Е?п^ = 0 G Rk и Е ||?n^||2 < оо для всех п, г, где || • || — евклидова норма в Ж . Обозначим В2^ i = D?n^ дисперсионные (ковариационные) матрицы, т.е. мат- матрицы, состоящие из ковариаций компонент векторов ?п^, г=1 г=1 Теорема 9 (Линдеберг). Пусть для описанного массива случайных векторов 1 == 1 777 77 "^ 1 г ??2 —У В2 поэлементно при п —У оо, A7) Е ||?n^||2l{||?n ^||>е} ""> 0 для любого г>0 при п —У оо. A8) г=1 Тогда 5n^N@,52), n^oo. A9) Второе вспомогательное утверждение — максимальное неравенство, требуе- требуемое при оценке модуля непрерывности процессов Sn( •), п ^ 1, т. е. при доказатель- доказательстве плотности семейства их распределений, дается в следующем предложении. Лемма 3 (неравенство Колмогорова). Пусть (д,..., ?т — независимые дей- действительные случайные величины с EQ = 0, а2 = DQ < оо, г = 1,... ,7Т7. Поло- Положим Si = S}=i Cjj 1 ^ i ^ m, d2^ = D5m. Тогда для любого A G 1 справедливо неравенство max |5»| ^A^m) ^ 2P(|5m| ^(A-v^)dm). B0) Доказательство. При А ^ у/2 утверждение леммы, очевидно, справедливо. Поэтому предположим, что А > у/2. Пусть Aj = <тах|5г| < Adm, \Sj\ ^ Аб?т \. Тогда событие т А = { max |5г| ^ Adm 1 = I I At, 1 причем Ai П Aj = 0 для i ф j vi вероятность P(A) = P{A П {|5m| ^ (A - л/2)йт}) +РDП {|5т| < (A - ^ P(|5m| ^ (A - >/2)dm) + J] P(^ П {\Sm\ < (A - Заметим, что АтП{|5т | < (X — y/2)drn} = 0. Для1 ^ j < m имеем A j П {15m | < < (A — \/2)drn} С Aj П {|5m — Sj\ > \f2dm}, поскольку из того, что \Sj\ ^ A(im,
Гл. V. Слабая сходимость мер. Принцип инвариантности 159 \Sm\ < (А - V2)dm, следует, что |5m - Sj\ ^ \Sj\ - \Sm\ > л/2<^т. События Aj и {|Sm — Sj\ > л/2 dm} зависят от разных групп независимых случайных величин и поэтому независимы. Используя неравенство Чебышёва, получаем (учитывая, что Р(А)= ) ^ Р(А) ^ P(|Sm| ^ (Л - л/2 )dm) ^ Р(|5т| ^ (Л - y/2)dm) + ^ откуда и следует требуемое неравенство B0). ? § 8. Доказательство теоремы Прохорова (теорема 8) начнем с установления сла- слабой сходимости конечномерных распределений: т.е. имея в виду (9), покажем, что для любых ti,..., ?& G [0,1], к ^ 1 (достаточно рассматривать лишь 0 ^ t\ < • • • < tk ^ 1) (Sn(h),...,Sn(tk))%(W(h),...,W(tk)), 7W<x). B1) С этой целью для каждой точки tj (j = 1,..., к) и заданного п найдем ближай- ближайшую к ней слева точку tj среди введенных в начале §6 точек tn^i — 1,..., тп. Иначе говоря, пусть t^ = max {tn^ ^ tj}. Из условия Линдеберга вытекает (см. приложение 3), что max а2п г ->• 0, п ->• оо, B2) и поэтому max (tn^ — tn^-i) —>- 0 при п —У оо. Следовательно, t п —>- tj при п —> оо для каждого j = 1,..., к. Если tj = tn?/, / = lj(n), to В силу B2) имеем сходимость по вероятности Sn(tj) — Sn(t n ) —У 0 при п —У оо для - Sn(tnii)\ ^ |ХП)/+1 дмеем сходимость по вероятности *! i = i,...,fc. Нетрудно проверить, воспользовавшись утверждением 1° теоремы 1, что если Zn, Yn, Z — случайные векторы со значениями в Ж такие, что Zn —У Z, a Yn —у О (т. е. все компоненты Yn сходятся по вероятности к нулю), то Zn + Yn —У Z. Таким образом, для доказательства B1) достаточно убедиться в том, что Zn=(Sn(t[n)),...,Sn(t[n))) % Z=(W(h),...,W(tk)), n^oo. B3)
160 А. В. Булинский, А. Н. Ширяев В рассматриваемом нами случае при п —У оо (г, j = 1,..., к). Поэтому для Zn и Z, фигурирующих в B3), получаем, что DZn —> DZ, т. е. дисперсионные матрицы сходятся поэлементно. Далее, если t zz. ~F l ~F '^z. ~F i "p 7TP / • zz. / • i т) i ^ 41 tt) V тп ^ ^^ /- -i —I— • • • —I— /- 7 где^П5г, г = 1,.. .,i независимые случайные векторы со значениями в + Xn'h+1 +¦¦¦+ с (Лп) Пользуясь тем, что у каждого вектора ?n^ все компоненты (кроме части из них, воз- возможно, равных нулю) совпадают с Хп^, получим в силу A6), что 1к(п) тп г=1 г=1 Тем самым, все условия теоремы Линдеберга выполнены и, следовательно, имеет место сходимость конечномерных распределений B1). Проверим теперь плотность семейства распределений {1Рп}п^ъ воспользовав- воспользовавшись теоремой 7 и леммой 3. Условие 1° теоремы 7 выполнено, так как Sn@) = 0, п ^ 1. Проверим условие 2" замечания 1. Зафиксируем N ^ 1. Пусть tn \,tn \ — со- соответственно самая правая из точек tnj ^ (г — l)/iV и самая левая из tnj ^ i/N, i = 1,..., N. Из B2) следует, что при всех достаточно больших п * г = 1,..., N. B4) Обозначим VUti = sup ()/ гг(*) ~ Sn((i — ) тогда sup Sn(t) - Sn(*$) + 5nD^) - Sn((i - 2 sup tB) Sn(t) - Sn(t«) I = 2 max 5n(tn,,) - Sn(t«) I и для любого v > 0, всех г = 1,..., N, N ^ 1, Pn,i = Р(УП,» > v) max 5n(tn5J) - Sn(t^\)\ > v jl\. B5)
Гл. V. Слабая сходимость мер. Принцип инвариантности 161 С учетом леммы 3 и равенства D (Sn (tn^) - Sn (tn^)J = tn^ - tn^ находим n <2P(k (t{2))-S (t{1))\ > (\-\/2)xt{2) где Л = v I \2ytn \ — tn\ j, г и п — те же, что и выше. Пусть tn^ = tn,ii, ^n]i — tn,ri, ji = ji(n), ri = ri(n). Рассмотрим в n-й серии величины с номерами I из множества «7г- = {I- ji < I ^ r^}, i = 1,..., N. Обозначим Sn = ^2 Xn,i = Sn(tn \) — Sn(tn {). Тогда в силу B4) и A6) для любого г/ > 0 г у- Е /" хп,г при п —у оо. Следовательно, по одномерной (т.е. при к = 1; см. начало § 7) теореме Линдеберга для i = 1,..., N и любого г/ > 0 имеем при всех п > по(г, г/, N) л/2)| < e/B7V), B6) | > Л - где ? ~ N @,1). Здесь мы воспользовались тем, что в центральной предельной теоре- теореме сходимость функций распределения равномерна на всей оси; см. упражнение 12. Согласно B4) при всех достаточно больших п для i = 1,... ,7V получаем Л ^ ^ vy/N /{2л/2). Поэтому, если N = N(s, v) достаточно велико, то применив A1.29), придем к оценке Р(|?| >Л-л/2) <e/27V. N Из последнего неравенства, B5) и B6) заключаем, что ^2 Pn,i < ?• В силу теоремы 7 г=1 и замечания 1 плотность семейства мер {Pn}n^i установлена. Вместе с соотноше- соотношением B1) это завершает доказательство теоремы 8. ? § 9. Наметим схему, по которой с помощью изложенной теории слабой сходимости можно доказать один из центральных результатов математической статистики — критерий согласия Колмогорова. Пусть ?i, ?2 ? • • • — независимые одинаково распределенные действительные вели- величины, имеющие функцию распределения F = F(x). Определим согласно A.34) эм- эмпирические меры Рп. Взяв В = (—оо, ж], получим эмпирические функции распре- распределения п Fn{x,u) = п-1^1(_ОО5Ж](^Н), n^i, хеш. г=1 Теорема 10 (Колмогоров). Если функция распределения F = F(x) непрерыв- непрерывна, то для всех z > 0 при п —у оо ( sup ^lft(^)-FW)K^l-2^(-l)fc+1e-2tV=]fD B7) / k=1
162 А. В. Булинский, А. Н. Ширяев Доказательство . Воспользовавшись заменой Q = Fmv(^), i ^ 1, где Finv(t) = inf{x: F(x) ^ ?}, te [0,1], — обобщенная обратная функция к функции F = F(z), можно (учитывая непрерыв- непрерывность функции F) свести все рассмотрения к тому случаю, когда величины ?i, ?2 ? • • • имеют равномерное распределение на [0,1]. Таким образом, надо доказать, что для таких величин при всех z ^ 0 Р( sup \Yn(t)\^z)^K(z), n^oo, 4 J B8) где Yn(t) = фг (Fn(t) -t),te [0,1]. Определим функционал h(x( •)) = sup )|. Естественно напрашивающийся путь доказательства B8) состоит в том, чтобы найти слабый предел распределений Yn (•) и затем воспользоваться теоремой 2. Сложность заключается в том, что про- процессы Yn имеют разрывные траектории (они принадлежат пространству Скорохо- Скорохода D[0,1]). Однако доказательство утверждения B8) можно дать и в рамках изло- изложенной выше теории. Пусть ?(!),..., ?(п) — вариационный ряд, построенный по ?i,... ,?п, т.е. при каж- каждом ио величины ?i(a;),..., ?п(^) перенумерованы в порядке возрастания (п.н. все точки ?i,..., ?п различны). Пусть при каждом ио функция Gn(t, w),t G [0,1], есть непрерывная случайная ломаная с узлами (i/(n + 1), ?^)(о;)), г = 0,..., п + 1, где ?(о)(^0 — 05 ?(n+i)(Cc;) = 1- Так определенные функции Gn( •, о;) (см. рис. 16) явля- являются случайными элементами в пространстве С[0,1] по теореме 7 главы I. Заметим, что sup \Fn(t)-Gn(t)\ 0<t<l п.н., ПОСКОЛЬКУ Fn(t,u) = г/пдля? G ),i = O,...,n. i 2 n+1 1 n+1 ?@) = 0 ' Si1* / n у /^ Fn{t) >- m 1 *• Рис. 16
Гл. V. Слабая сходимость мер. Принцип инвариантности 163 Обозначая Zn (t) = л/п (Gn (?)—?),? G [0,1], находим, что с вероятностью единица sup \Yn(t)-Zn(t)\^n-^2. Следовательно, предельное распределение случайной величины h(Yn( •)) будет сов- совпадать с предельным распределением случайной величины h(Zn(-)). Доказательство теоремы Колмогорова сводится, тем самым, к проверке следую- следующих двух утверждений: 1) Zn —> W° в С[0,1] при п —> оо, здесь W0 — броуновский мост, определяе- определяемый как процесс W°(t) = {W(t)-tW(l),t е [0,1]}, где W = {W(t),t e [0,1]} — винеровский процесс на отрезке [0,1]; 2) р( sup ) 4 Первое из этих утверждений доказывается по той же схеме, что и теорема 8 ("схо- ("сходимость конечномерных распределений и плотность"). Соответствующее доказа- доказательство см., например, в [2; гл. 2, § 13]. Остановимся на доказательстве второго утверждения. Определим семейство (условных) вероятностных мер на (С[0,1], Зё(С[0,1])) Qe(C) = P{W(.)eC\ W(l) е [-е,е]), е > 0. B9) Лемма 4. Если г I 0, то Q? => W°, где W° — распределение броуновского моста в С[0,1]. Доказательство. В силу теоремы 1 (которая, очевидно, верна для мер, индек- сированых параметром г > 0) достаточно показать, что limsupQ?(F) ^ P(W° G F), C0) где F — произвольное замкнутое множество из Й§(С[0,1]). Для любых 0 ^ h < • • • < tk ^ 1, k e N, вектор (H/0(ti),..., W°(tk)) не зависит otWA). Действительно, (W°(ti),..., H/0(t/c), W(l)) имеетгауссовскоераспределе- имеетгауссовскоераспределение как линейное преобразование гауссовского вектора. Кроме того, EW°(t)W(l) = = EW(t)W(l) - tEW(lJ = min{t, 1} - t = 0, t G [0,1], что и влечет указанную независимость. Таким образом, для любого цилиндрического множества С G Зё(С[0,1]) и любого множества В G P(w°( -)eC, W(i) ев) = P(w° e С) P(W(i) e в). C1) Поскольку цилиндры образуют алгебру, порождающую Зё(С[0,1]), то по лемме 3 главы I равенство C1) справедливо для любого множества В G Зё(Ж) и всех мно- множеств С G Й§(С[0,1]). Следовательно, P{w°(-)eC\w(i) e [-г,s}) = P(w°(-)eC) при каждом е > 0.
164 А. В. Булинский, А. Н. Ширяев Заметим, что Pc{W('),W°(.))= sup \W(t)-(W(t)-tW(l))\ = \W(l)\, C2) где рс — равномерная метрика в С[О,1]. Возьмем теперь замкнутое множество F е 38(С[0,1]). Изтого,что|И^A)| <SnW(-) G F, в силу C2) имеем W° eFs = {xeC[Q,l]:p(x,F) <S}. Тем самым для любого е < S получаем \V G F | \WA)\ ^ г) ^ P(V° G F* | \WA)\ ^ е) = P(W° G Fs) и, значит, HmsupQ^) ^ ^(F5). Переходя в этом неравенстве к пределу при S 4- 0 и учитывая, что f] F = i^1, при- <5>о ходим к соотношению C0), из которого следует утверждение леммы. ? Замечание 2. Лемма 4 показывает, что распределение броуновского моста естественно интерпретировать как распределение условного винеровского процесса (при условии W(l) = 0). Согласно (9) имеем W0/* (В) = P(h(W°) e В). Кроме того, Qsh-\B) = P(W( •) G h-\B) | W(l) e [-e,e]) = P{h(W) e В I W(l) e [-e,e]), где В G 3&(R), e > 0, h(x(-)) = sup \x(t)\ для x( •) G С[0,1]. Поэтому лем- [] ма 4 и теорема 2 дают, что для тех значений z ^ 0, где функция распределения Р ( sup | W° (t) | ^ z) непрерывна (т. е. за исключением не более, чем счетного мно- жества значений z), Pf sup |W°(t)| sC Pf sup \W(t)\^z, \W(l)\^e) \ir[n 11 / Pf sup |) , inf W{t) > -z, W(l) e [-e,e]) Совместное распределение величин inf te[o,i] W(t), suptG[0,i] W(t),W(l) известно (см. в дополнении к этой главе теорему 15). Тем самым можно найти вероятность, стоящую в числителе правой части формулы C3). Поделив полученное выражение на значение вероятности P(WA) G [—?, е]) и со- совершив предельный переход при е \. 0, получим в пределе в точности функцию K(z), определяемую в B7). Эта функция К = K{z) непрерывна для всех z > 0. Поэтому
Гл. V. Слабая сходимость мер. Принцип инвариантности 165 требуемое утверждение 2) установлено. Тем самым в C3) сходимость будет при всех z > О, что завершает доказательство теоремы Колмогорова. ? § 10. В этом параграфе мы обратимся к вопросу о взаимоотношении сходимости почти наверное и слабой сходимости случайных элементов. Будем предполагать, что S — метрическое пространство. Пусть X,Хп: П —У S и X, Хп G 3* | 88(S) для п ^ 1. Если Хп —у X п.н. при п —у оо, то в силу F), очевидно, Law(Xn) => Law(X), n —У оо. Обратное утверждение неверно даже для действительных случайных величин, заданных на одном вероятностном пространст- пространстве (приведите пример). Однако в должном смысле удается обратить приведенный результат, как показывает следующая теорема. Теорема 11 (Скороход). Пусть (S,p) — польское пространство и Qn => Qoo при п —У оо, где Qn (n G NU {°°}) — вероятностные меры на 8&(Ъ). Тогда существуют вероятностное пространство (П, ^, Р) и такие случайные эле- элементы Хп: П —у S, Xn G 3 | ^(S), n G N U {оо}, что Law(Xn) = Qn для всех п G N U {оо} и Хп -у Хоо п. н. при п -у оо. Доказательство. Сначала построим определенную систему измельчающихся разбиений S. Для каждого k G N в силу сепарабельности S имеются открытые ша- шары Gk,mj т ? N, радиуса 2~(fc+2) такие, что (J Gk,m — S. Меняя радиус каждого т шара в пределах от 2~(fc+2) до 2~(к+1\ возможно получить покрытие S открыты- открытыми шарами Вк^ш такими, что Qn(dB^^rn) = 0 при всех /c,m G 1Чип G NU {оо}. гп-1 Положим D/сд = Б^д и для т ^ 2 пусть Dk:Tn = В^^ш \ (J В^^г. При каж- г=1 дом к G N множества D/c,m, m G N, образуют разбиение S, причем diamD^^ ^ ^2~fc,?7iGl4, и Qn(9D/C5?7l) = 0 для всех рассматриваемых п, /с, ттг. Напомним, что diamD = sup{^(x,у): х,у G D}, D С S. /с Введем теперь множества 5ib...,ifc = П -^зЛз (все индексы из N). Они образуют измельчающиеся с ростом к вложенные разбиения S такие, что: 1) Sib...,ifc ^Sju...jk = 0при(гь...,г/с) 7^ (ji, • • •, Зк)] 2) U^j = S^ {Jsh,...,ikJ = sh,—>ik Для всех ?,21,...,^ G N; j j 3) diamSib...,ifc ^ 2~fc для всех fc,zi,... ,г^ G N; 4) Qn(95ib...,ifc) = 0 для всех Mi,...,^ GN,nG NU{oo}. Начнем строить искомую последовательность случайных величин на (П, ^, Р), где п = [0,1),^ = ^([0,1)), Р есть мера Лебега (как всегда, считаем сг-алгебру 3 пополненной). Для краткости пишем | • | вместо mes( •). Для каждого nGNUoon&GN рассмотрим разбиение [0,1) полуинтервалами = QnEib...,ifc), а именно, =
166 А. В. Булинский, А. Н. Ширяев Аналогичным образом каждый из полуинтервалов Д^ разбивается на полуинтер- полуинтервалы А^п- (последовательным прикладыванием их, начиная с левого конца Д^ ) и т.д. Интервалы Д^ ^ получаются упорядоченными в лексикографическом смысле. Для к Е N в каждом непустом множестве 5^,...,^ выберем точку ж^,...,^ и для каждого n G N U {оо} определим на П = [0,1) функции XZ(Lj)=xil,...,ik при .€Д«.Л (если Qn(Sii,...,ifc) = 0, то Д^ i = 0 — промежуток вида [а, а), и определять функцию на таком промежутке не требуется). Очевидно, Х% Е ^|^(S) при всех к, п. Заметим, что р(Хк(ии),Хк+ш(ии)) <С 2~к для любого со е [0,1) и всех к,п,т C4) по свойствам 2) и 3) построенных разбиений. В силу полноты S существует Хп(и>) = lim Х%(и>) для и е [0,1), п е NU {оо}, C5) к—юо причем по лемме 5 главы I имеем Хп G ^ | ^(S) для всех п G N U {оо}. Пользуясь свойством 4) и пунктом 3° теоремы 1, получаем, что при всех /с, ii,..., г& G N Поэтому, если Qoo(Si1:...:ik) > 0 (и, значит Д^ i ф 0), то для каждой точки ^ G {ai™likA™lik) найдется номер п^Н такой, что cj G (a-^.i^^^J при п ^ ^ nk(oj). Тогда для рассматриваемого ио получим X^(uj) = X^(cj) и, учитывая C4), имеем при п ^ Пк(оо). Очевидно, Хп(и) —у Х^^) при п —У оо для uj Е [0,1) за исклю- исключением, быть может, счетного множества концов промежутков вида Д^ i . Тогда Хоо Е 3 | 8e(S) в силу леммы б главы I. Осталось показать, что Law(Хп) = Опдляп G NU{oo}. По построению при всех n,?7i,ii,... ,im и /с ^ ттг Любое открытое множество G в S можно представить как объединение конечно- конечного или счетного числа множеств 5^,...,^ ПРИ разных & и ii,..., г&. Действительно, (счетная) совокупность множеств Si1^_^k (к, ii,..., г& G N) образует базу тополо- топологии польского пространства (S, р). Это сразу вытекает из [35; теорема 3, § 5 гл. 2], поскольку в любом шаре с центром в произвольной точке х G S найдется множество упомянутой базы, содержащее х и целиком содержащееся в данном шаре.
Гл. V. Слабая сходимость мер. Принцип инвариантности 167 Возьмем множества Gn t G при N —У оо, где Gn состоит из конечного числа ттгдг непересекающихся множеств указанной базы. Тогда Р(Х% Е G) ^ Р(Х% G Gjv) для каждого 7V и, учитывая C6), имеем liminf P(X% e G) ^ liminf P(X^ G Gjv) = Qn(GN). к—юо /с—юо Таким образом, liminf P(X^ Е G) ^ Qn(G). Согласно пункту 2° теоремы 1 получа- к —юо ем, что Law(X^) => Qn при к —У оо для n Е N U {оо}. Но по доказанному Х^ —у Хп п.н. при к —У оо для всех п, поэтому Law(X^) => Law(Xn) при /с —у оо (n G N U оо). Лемма 1 дает, что Law(Xn) = Qn, n ? N U оо. ? §11. Для рассмотрения вопроса о метризации слабой сходимости введем ^с — класс равностепенно непрерывных функций /: S —У М, имеющих Лемма 5. Если (S,p) — польское пространство и Qn => Q па 5§(S), то для любой константы С > О sup{K/,Qn)-</,Q>|:/e0c}-X), n^<x). C7) Доказательство. На некотором вероятностном пространстве (П, ^, Р) выбе- выберем случайные элементы X, Хп, п G N, фигурирующие в теореме Скорохода (вместо Qoo и Хоо пишем соответственно Q и X). В соответствии с формулой A.23) требуется проверить, что sup{\Ef(Xn)-Ef(X)\:fe<Sc}^0, n -»¦ оо. В силу равностепенной непрерывности для любого г > 0 найдется S = S(e) > О такое, что \f(x) — f(y)\ ^ s для всех / G ^с и х, У таких, что р(х,у) ^ S. Тогда, очевидно, \Ef(Xn) - Е/рО| < E\f(Xn) - f(X)\l{p(Xn,X) + Е \f(Xn) - f(X)\l{p(Xn,X) >6}^ <:е + 2СР(р(Хп,Х)>ё). Теперь заметим, что р есть непрерывная функция, а р(Хп,Х) в силу сепарабель- сепарабельности S есть действительная случайная величина. Поскольку Хп —у X п.н., то р(Хп,Х) —У 0 п.н., а значит, сходится и по вероятности. Следовательно, P(cj: р(Хп(ш),Х(ш)) > S) —У 0 для любого S > 0 при п —>¦ оо, что завершает доказательство леммы. П Пусть всюду далее (S, р) — польское пространство. Для В С S и е > 0 пусть, как обычно, В? = {ж G S: /)(ж, 5) < г}, где ^(ж, 5) = inf{p(x,y): у G 5}.
168 А. В. Булинский, А. Н. Ширяев Определение 7. Метрикой Леви-Прохорова называется величина tt(P,Q) = Ы{г>0:Р(В) <С Q(??)+s, Q(B) ^ Р(В?)+едляВ е 3&(S)}. C8) В качестве упражнения нетрудно проверить, что тг( •, •) обладает всеми свойст- свойствами метрики. Теорема 12. Сходимость Qn => Q равносильна тому, что 7r(Qn, Q) —у 0 при п —у оо. Доказательство. Пусть 7r(Qn, Q) -у 0 (п -у оо). Тогда в силу C8) для любого е > 0 и каждого замкнутого F С S имеем Qn(F) ^ Q(Fe) + е при всех п ^ п(г). Следовательно, lim sup Qn(F) ^ Q(Fe) + г для любого е > 0. Остается устремить п—юо г к нулю и воспользоваться пунктом 1° теоремы 1. Пусть теперь Qn => Q. Совокупность функций /^( •), введенных в доказательстве теоремы 1 (где г > 0 и F — замкнутое множество в S), обладает тем свойством, что О ^ ff(') ^ 1и \ff(x) - ff(y)\ ^ s~1p(x,y) для всех ж, 2/ G S. Следовательно, при каждом фиксированном г > 0 класс ^е = {//"(•): множества .F замкнуты в S} С С ^i, где классы ^с фигурируют в лемме 5. Из C7) вытекает, что для любого е > О A^=sup{|(/,Qn)-(/,Q)|:/G^?}^0, n ->• оо. C9) Если множество F замкнуто иг > 0, то учитывая, что 1f( •) ^ ff(') ^ 1f? (*), и л (е) принимая во внимание определение An , получаем Q(F?) ^ </f, Q) ^ </f, Qn> - A(f) ^ <1F, Qn> - A(f) = Qn(F) - Д^. D0) Заметим, что Ane ^ в при n ^ no(e). Обозначая, как и ранее, [5] замыкание мно- множества В из 5§(S), имеем в силу D0), что при всех достаточно больших п Аналогичные оценки справедливы, если поменять местами Q и Qn. Поскольку [В]? С В2е для любого е > 0, приходим к неравенству 7r(Qn,Q) ^ 2е при всех достаточно больших п. Таким образом, 7r(Qn,Q) —> 0 (п —>- оо). П Дополнения и упражнения 1. Докажите, что меры 5Хп, заданные на борелевской сг-алгебре метрического пространства (S,p), имеют слабый предел тогда и только тогда, когда су- существует такой элемент х G S, что хп —у х при п —> оо (тогда 5Хп => 5Х при п —у оо). 2. Докажите, что Qn => Qnpnn —>- оо (меры заданы на 3e(S), где (S,p)—метри- (S,p)—метрическое пространство) тогда и только тогда, когда свойство A) выполнено для всех действительных ограниченных липшицевых функций, т.е. таких функ- функций /: S —У М, что Ц/Цоо = sup \f(x)\ < оо и L(/) = sup{|/(a;) - f(y)\/p(x, у)} < оо. D1)
Гл. V. Слабая сходимость мер. Принцип инвариантности 169 Из теоремы Скорохода легко вытекает (с учетом известных фактов о сходимости п.н.), что ЕХп —у ЕХ, если S = М, Хп ч!и если семейство {Хп, n ^ 1} равно- равномерно интегрируемо. Этот результат полезен для следующего упражнения. 3. Пусть функция /: S —У V, где S, V — метрические пространства (измеримость / не предполагается). Покажите, что множество Df = {х: функция / раз- разрывна в точке х} Е S$(S). Докажите, что Qn => Q тогда и только тогда, ког- когда свойство A) выполнено для любой ограниченной функции / Е ?$(S) | ё$(Ж) такой, что Q(Df) = 0. 4. Пусть S есть пространство Ж171 с евклидовой метрикой. Будет ли верно, что Qn => Q5 если свойство A) выполняется для функций / Е Со°(Жгп), т.е. для всех бесконечно дифференцируемых функций с компактным носителем? Лемма 6 (Улам). Пусть (S,p) — польское пространство и Q — мера на ?$(S). Тогда для каждого В Е ?$(S) и любого е > 0 существует компакт К? С В такой, что Q(B \ К?) < г. оо Доказательство. Для п Е N в силу сепарабельности S = |J ??i/nB/n,m), где т=1 В?(у) = {х Е S: р(х,у) < г}, а точки уп,т, тп = 1, 2,..., образуют l/n-сеть (т.е. для любого у G S можно указать точку уп,т-> такую что р(у,уп,гп) < 1/п)- При > 1 — е2 2~п~ / Зп \ фиксированному > 0 выберем jn так, чтобы Q ( |J B1/n(yn^rn) 1 оо jn Положим R? = P| U B1/n(yn^rn). Тогда для замыкания [R?] множества R? име- п=1 т=1 ем Q(S\[i?e])^Q(S\i?e) = Q# Множество [R?] — компакт, так как в полном пространстве оно замкнуто и вполне ограничено (т. е. для любого г > 0 существует конечная г-сеть); например, 2/п-сетью могут служить точки уП:ГП,т = 1,... ,jn. По лемме 4 главы I существует замкнутое множество F? С В такое, что Q(B \ F?) < г/2. Тогда К? = F? П [R?] есть компакт как пересечение замкнутого множества с компактом и Q(B \ К?) < г. ? Заметим, что для S = Жд вместо [R?] в доказательстве леммы б можно взять за- замкнутый шар достаточно большого радиуса, однако в бесконечномерном нормиро- нормированном пространстве замкнутый шар не является компактом. Определение слабой сходимости A) применяется и для любых конечных (необя- (необязательно вероятностных) мер. В этой связи предлагается упражнение 5. Пусть Q, Qn (n ^ 1) —конечные меры на (S, S$(S)). Докажите, что теорема 1 останется верной, если к ее условиям добавить требование Qn (S) —У Q (S) при п -У оо. Определение 8. В измеримом пространстве (S, si) класс множеств ^ С si на- называется классом, определяющим меру, если равенство любых двух мер на ^ вле- влечет их равенство на si. Пусть S — метрическое пространство. Класс множеств Ж С Se(S) называется классом, определяющим (слабую) сходимость, если для
170 А. В. Булинский, А. Н. Ширяев любой последовательности мер Q, Qi, Q2, ... такой, что Qn(B) —у Q(B) при всех В е Л с Q(dB) = 0, имеем Qn => Q. 6. Покажите, что класс множеств, определяющий (слабую) сходимость, явля- является классом, определяющим меру. Объясните, почему обратное утвержде- утверждение не обязано выполняться. 7 (сравните с упражнением б и примером 1). Докажите, что в польском про- пространстве М°°, снабженном метрикой вида борелевская сг-алгебра совпадает с цилиндрической, а сходимость Qn => Q при п —У оо имеет место тогда и только тогда, когда слабо сходятся все конеч- конечномерные распределения. Иначе говоря, класс цилиндров в Ж°° одновремен- одновременно является классом, определяющим и меру, и сходимость. Пример 2 (доказательство формулы (IV.19)). По центральной предельной тео- теореме Sn/л/п -у ? ~ N@,1) при п -У оо и, следовательно, \Sn\/y/n ~> 1?1 согласно теореме 2. Кроме того, последовательность {\Sn\/y/n }n^i равномерно интегрируе- интегрируема, поскольку для нее при 7 = 2 выполнено условие (III.53). Поэтому Е \Sn\/y/n —У -У Е |?| = д/2/тг, п ->• оо. Для случайного вектора ? в Мт пусть i^ и у?^ обозначают соответственно функ- функцию распределения и характеристическую функцию (см. § 14 главы I). В курсе тео- теории вероятностей доказывается Теорема 13 (см., например, [85; т. 1, с. 414]). Соотношение ?п —у ? для случай- случайных векторов в Ж171 равносильно любому из следующих двух условий: 1. F^n(x) —у Fg(x) при п —У оо в каждой точке х G Mm, в которой непре- непрерывна функция F^; 2. <^? (А) —У <?t(A) при п —У оо для каждого Л G Мт. Кроме того, если для случайных векторов ?п в Ж171 имеет место сходимость (fgn (А) —у ip(\) при всех Л G Мт, где функция if непрерывна в точке О G Мт, то <р( •) = <^t( •) для некоторого случайного вектора ? и ?п —У ? при п —У оо. 8 (метод Крамера-Уолда). Докажите, что ?п —у ? в (Мт, Й§ (Мт)) тогда и толь- ко тогда, когда (а, ?п) —>- (а, ^) при п —>- оо для каждого неслучайного вектора a G Mm, где (•, •) - скалярное произведение в Ж171. Это упражнение показывает, как можно сводить изучение слабой сходимости случайных векторов в Ж171 к изучению слабой сходимости случайных величин. В связи с теоремой 13 заметим, что понятие характеристической функции для ме- меры на (Мт, 88 (Ж171)) допускает естественное обобщение на более общие пространства. Так, пусть S — действительное сепарабельное банахово пространство, S* — сопря- сопряженное пространство (т. е. пространство непрерывных линейных функционалов на S) и 3^(S) — пространство вероятностных мер на 8e(S).
Гл. V. Слабая сходимость мер. Принцип инвариантности 171 Определение 9. Характеристическим функционалом меры Q Е 3^(S) (или случайного элементаХ: п —У S, X Е 3* | ^(S), такого, что Law(X) = Q) называется отображение ^q : S* —У С, задаваемое формулой <pQ(x*) = Е ехр{г(Х,ж*)}, ж* G S*, где (?/, ж*) обозначает действие линейного функционала ж* на элемент у Е S. Пусть ^ — совокупность цилиндрических множеств в S, т. е. множеств вида где Б G3S(Mn), *?,... ,2* Е S*. 9 (сравните с теоремой 13). Покажите, что Ч? порождает S$(S). Докажите, что если Qn => Q в 3^(S), то cpQn —у cpQ поточечно. Обратно, пусть cpQn —у ср поточечно, где ср: S* —У Си семейство {Qn, n ^ 1} плотно. Тогда ср = (^q для некоторого Q G 3^(S) и Qn ^> Q. 10. С помощью теоремы 13 главы III докажите принцип инвариантности (теоре- (теорему 8) для схемы серий, порожденной последовательностью независимых оди- одинаково распределенных величин с нулевым средним и единичной дисперсией. Результаты о слабой сходимости сумм независимых случайных величин в схеме серий приводят к следующему утверждению. Теорема 14 (Дуб). Пусть процесс X = {Xt, t ^ 0} принимает при каждом t значения в Ж171 и имеет независимые приращения. Если траектории процесса непрерывны п. п., то процесс Y = {Yt = Xt — Xq, t ^ 0} является гауссовским. При этом для 0 ^ s ^ t < оо E(Xt-Xs) = Mt-Ms, D(Xt-Xs)=Gt-Gs, D2) 2 где М: М+ —У Ж171 и G: M+ —У Ж171 - непрерывные функции. Если в условиях этой теоремы величина Xq имеет вырожденное распределение, то и сам процесс {Xt, t ^ 0} будет гауссовским. Если процесс X удовлетворяет условию теоремы 14 на отрезке [а, Ь], то ее утверждение справедливо для процесса {Yt = Xt — Xaj t G [a, b]}. (Достаточно ввести процесс Xt = Xt+a для t G [0, b — a] и Xt = Xfr при t ^ 6.) Технически довольно сложное доказательство теоремы 14 и ряда вспомогательных утверждений, представляющих самостоятельный интерес, отнесено в приложение 3. Покажем, как, применяя принцип инвариантности (теорему 8), можно найти рас- распределение функционала sup W(t). Согласно формуле (III.34) это распределение *G[0,l] совпадает с распределением случайной величины |WA)|. Для иного доказательст- доказательства этого свойства построим случайные ломаные Sn( •) по сериям величин Хп^ = , г = 1,..., п, где Xi, X2,... независимы, одинаково распределены и Р{Хг = -1) = Р(Хг = 1) = 1/2.
172 А. В. Булинский, А. Н. Ширяев 11 (сравните со следствием 1 главы III). Докажите, что для любого целого не- неотрицательного числа j р( max Sk > j) = 2P(Sn > j) + P(Sn = j), D3) где So = 0, Sk = X\ + • • • + Xfc, к ^ 1, a Xi, X2,... — введенные выше величины. Теперь заметим, что для каждого z > 0 Р( sup Sn(t) ^ z) =P( max Sk/y/n ^ z) =р( max Sk ^ jn\ где jn — наименьшее целое число, большее или равное Zy/n, другими словами, jn = = — [—zyfn\, где [ • ] — целая часть числа. Пользуясь центральной предельной тео- теоремой и учитывая равномерную сходимость функций распределения нормированных сумм к функции распределения стандартного нормального закона, получаем, что для любого z > 0 P(Sn > jn) = P(Sn/V^ > jn/yfc) -> P(f > z), D4) где f ~N@,l). Полезно отметить, что использованное свойство равномерной сходимости функ- функций распределения в общем случае формулируется следующим образом. 12. Пусть ?п —>- ?, где ^, ^п, п ^ 1, действительные случайные величины, причем функция F%(x) непрерывна на R. Тогда sup \F^n(x) - F^(x)\ ->• 0, п -»> оо. Для получения из соотношений D3) и D4) искомого результата (III.34), описы- описывающего распределение максимума броуновского движения на отрезке [0,1], остает- остается выполнить простое упражнение: 13. Докажите, что для введенных перед упражнением 11 двузначных величин Xi, i G N, и их сумм Sk = X\ + • • • + Х^, к ^ 1, имеем max P(Sn = j) —У 0 при п —у оо. j Теперь обратимся к более общему результату, нежели (III.34), описывающему совместное распределение величин т = inf W(t),M= sup W(t)wW(l). *e[oi] Пусть, по-прежнему, Xi, X2,... — независимые одинаково распределенные слу- случайные величины, P(Xi = -1) = P(Xi = 1) = 1/2 и 50 = 0, Sk = Х\ Л V Хк, к ^ 1. Обозначимтп = min Sk, Mn = max 5/c. Введем непрерывное отображение h: С[0,1] —>¦ /l(x(-))=f inf x(t), sup v^[oi] В силу теорем 2 и 8 имеем h(Sn( ¦)) = (mn/v^,Mn/v^,5п/лА) 4 (m, M, W(l)), D5) где 5n(t), 0 ^ t ^ 1, — случайная ломаная с узлами (fc/n, Sk/y/n), к = 0,...,п. Более сложные рассуждения, чем приведенные выше для нахождения распределе- распределения М, позволяют с помощью предельного перехода D5) установить следующий ре- результат (см., например, [2; с. 113-115], [7; с. 18-21]).
Гл. V. Слабая сходимость мер. Принцип инвариантности 173 Теорема 15. Для а < 0 < Ъ, а < г < s < Ъ и ? ~ N@,1) имеем Р(а < т ^ М < Ь, г < W(l) < s) = оо a) <?<s + 2k(b-a))- к = — оо оо Приведем еще несколько результатов о слабой сходимости и компактности рас- распределений процессов с непрерывными траекториями. Теорема 16 (Прохоров; см., например, [146; с. 259]). Пусть X, Х^ (п ^ 1) — случайные элементы со значениями в пространстве С(Т, S), где Т — компакт, a S — польское пространство. Соотношение Х^ —у X справедливо тогда и только тогда, когда конечномерные распределения процессов Х^ слабо схо- сходятся к конечномерным распределениям процесса X (п —> оо) и lim lim sup E (A(X^, S) Л 1) = 0, 810 А(Х(п\5) — модуль непрерывности случайной функции Х^п\ Рассмотрим теперь пространство С(Т, S), где S — метрическое пространство, а Т — локально компактное хаусдорфово пространствосо второй аксиомой счетности (последнее означает, что в пространстве имеется счетная база). Снабдим С(Т, S) топологией равномерной сходимости на компакт ах,введя метрику оо SUp p(x(t),y(t)) где х = (ж(?), teT),y = (y(t), t e T) е С (Г, S), а Г представимо в виде оо Г = У Кп, где Кп — компакты, Кп С Кп+1, п е N. D7) 71=1 14. Докажите, что Х^ —у X во введенном выше пространстве С(Т, S) тогда и только тогда, когда ХУ^ —У Х*к в С (К, S) для любого компакта К С Т; здесь, как и прежде, Y\K означает сужение функции Y = {Yt,t G Т}до функ- функции Y\K = {Yt, t е К}. Этот результат, упражнение 8 и теорема 8 приводят к следующему утверждению. 15. Пусть ?i, ?2 ? • • • — независимые одинаково распределенные случайные векто- векторы в Мт, для которых E?i = 0, E||?i||2 < 00, где || • || — евклидова норма. Тогда процессы х(п) = n-i/2 f^Sk + int- H)?[nt]+i) , t> 0, п G N, где [ • ] — целая часть числа, сходятся по распределению при п —У оо в про- пространстве С ([0, оо), Ж171) к стандартному m-мерному броуновскому движе- движению, домноженному на константу.
174 А. В. Булинский, А. Н. Ширяев 16. Пусть Х^п\ п ^ 1, — непрерывные процессы на Rd со значениями при каж- каждом t Е Md в метрическом пространстве S. Докажите, что распределения этих процессов плотны в С (Ж , S), если для некоторых констант а, /3 > О s,teRd, где || • || — некоторая норма в Rd. По изложенной выше схеме изучения слабой сходимости в пространстве С(Т, S) может быть также проведено исследование слабой сходимости в пространствах Скорохода D([0, l]q) и D([0,oo)g), q ^ 1. Эти пространства состоят из функций, определенных соответственно на [0, l]q и [0,оо)д, принимающих действительные значения и таких, что они непрерывны сверху в каждой точке и в каждой точке t ф 0 имеют предел снизу (предел функции / сверху в g-мерной точке t = [t\,..., tq) означает, что берется предел f(s) при s —У t для s ф t таких, что s^ ^ th, к = 1,..., q; аналогично определяется предел снизу). Введенные пространства с помощью специальным образом заданных метрик могут быть превращены в польские, в них также вводится аналог модуля непрерывности. По сравнению с пространствами непрерывных функций пространства Скорохода обладают следующими преимуществами. В этих пространствах близкими оказы- оказываются не только функции, получающиеся "малой деформацией" в области их зна- значений, но и функции, получающиеся "малой деформацией" временного аргумента t (например, функции fx(t) = l[X:OO^(t)nfy(t) = l[2/)OO)(t) не сближаются в простран- пространстве С при х —> у, но сближаются в пространстве D). С данным кругом вопросов можно ознакомиться в [2] и [26]. В связи с вышесказанным рассмотрим следующую модель случайно блуждаю- блуждающей частицы. Пусть Y^\ F2 ° ? • • • — независимые одинаково распределенные ве- величины и P(Y}a) = -а) = P(Y}a) =а) = 1/2, а > 0. тт /-(А) ДА) Пусть ^ , ^ ? • • • — независимые величины, экспоненциально распределенные с па- параметром Л > 0, причем последовательности {Yn } и {?т } независимы при всех а и Л. Предположим, что состояния частицы изменяются на а или —а в моменты к тт. = ^2 ^- , к = 1, 2,... (в момент t = 0 частица находится в нуле). Точнее го- (Л) А Л,(а) воря, пусть в момент т^, координата частицы равна 2^ ^7 •> эт0 состояние сохра- г (А) (А) ч г , няется на промежутке [т^ , т^+1) и, таким образом, положение частицы в момент t описывает процесс Х\а' ' = ^2 Y^a\ t ^ 0, где N\{t) = max{&: r^ ^ t} (счи- таем TVa(O) = 0, Xq ' — 0). Процесс TV = {TVa(^), ? ^ 0} является пуассоновским согласно теореме 2 главы II). 17. Исследуйте сходимость конечномерных распределеннийпроцессов Х^а> ' при условии, что а —> 0 и Л —> оо таким образом, что а2А = сг2 > 0 (т.е. вели- величина смещения уменьшается и соударения учащаются). Будут ли процессы Х(а,\) слао~о сходиться в пространстве D[0,oo)?
Гл. V. Слабая сходимость мер. Принцип инвариантности 175 Замечание 3. Изложенный в данной главе метод доказательства предельных теорем, основанный на идеях плотности и конечномерной сходимости, "хорошо работает" тогда, когда предельный процесс является относительно простым (напри- (например, процессом с независимыми приращениями). В тех же случаях, когда предель- предельный процесс является более сложным (например, процессом диффузионного типа, марковским процессом, мартингалом, семимартингалом), полезно применение мар- тингальных методов (см., например, [26]) или методов, опирающихся на аппарат марковских полугрупп (см., например, [91]). Наряду с принципом инвариантности Донскера-Прохорова, относящимся к сла- слабой сходимости распределений процессов, в теории случайных процессов важную роль играют сильные принципы инвариантности, в которых рассматривается не близость распределений процессов, а близость их траекторий. Пусть задана последовательность независимых (или должным образом "слабо зависимых") величин ?i, ?2 ? • • • со значениями в Ж (или в Ж171, или даже в некотором банаховом пространстве). Тогда можно перейти к новой последовательности величин 77i, 772, • • •, заданной, вообще говоря, на другом вероятностном простран- пространстве вместе с некоторым броуновским движением W = {W(t), t ^ 0} так, что (при справедливости ряда условий) будут выполнены следующие два свойства: Law(?b ?>,..•) = Lawful, 772,...) и п.н. Vk - W(t) = O(h(t)) при t -+ 00, D8) где h — некоторая неслучайная положительная функция. Запись D8) означает су- существование для n.B.wGl) такого С (и) > 0, что при t > to(u,C(uj)). k^t Вместо D8) рассматриваются также соотношения вида ^Vk- W(t) = o(g(t)) п.н. при t -+ 00, D9) k^t j]k — W(t)\ g(t) —> 0 при t —У оо для п.в. uj G П, где g — не- означающие, что которая неслучайная функция. Приведем один из результатов, относящихся к этому кругу вопросов. Теорема 17 (Штрассен, [188]). Пусть ?ь?2?--- — независимые одинаково распределенные случайные величины такие, что E^i = 0 и Е^ = 1. Тогда справедливо соотношение D9) с g(t) = (tinhitI/2, t > е. Данное в [188] доказательство этой теоремы основано на представлении Скорохо- Скорохода (см. (III. 78)). Другой мощный метод получения оценок типа D8), так называемый ^венгерский метод", был предложен И. Комлошем, П. Майороми Г. Тушнади [152]. Заметим также, что наряду с аппроксимациями сумм J^ 77/0 винеровским процессом (как в D8), D9)) используются также приближения с помощью специальным обра- образом построенных последовательностей гауссовских величин [122].
176 А. В. Булинский, А. Н. Ширяев Из курса теории вероятностей известно, что даже для действительных случайных величин при исследовании асимптотических задач важен правильный выбор ти- типа сходимости. Так, согласно центральной предельной теореме для независимых одинаково распределенных величин Хк, к ^ 1, со средним 0 и дисперсией 1, имеем n-i/2 ^ Хк —У X при п —У оо, где X ~ N@,1). В то же время, как в этом нетрудно убедиться, не существует случайной величины Y такой, что п/2 ^ Хк —У Y к = 1 при п —У оо (а значит, нельзя говорить о пределе рассматриваемых нормированных сумм ни почти наверное, ни в среднем). В этой связи обсудим некоторые типы сходи- сходимости вероятностных мер на измеримых пространствах (S, si). Основная идея здесь заключается в том, чтобы обеспечить определенную бли- близость интегральных функционалов (/, Qn) к (/, Q) для функций / из тех или иных классов функций. Пусть S — польское пространство. Обозначим B(S, Ж) пространство, состоящее из ограниченных si \ 8ё(Ж) — измеримых функций /: S —У М, снабженное нормой || • ||оо. Пусть SP — 3^(S) —пространство вероятностных мер на (S,si). Определим расстояние по вариации || • || между мерами Р, Q Е SP формулой ||Р - Q||= sup{|(/, P) - (/, Q)|: / е B(S, Ж), Ц/IU «С 1}. 18. Проверьте, что z/(P,Q) = ||Р — Q|| является расстоянием на 3^, которое мет- ризует топологию равномерной сходимости на 3^, задаваемую системой окрестностей U(Q, S) = {Р G 3^: г/(Р, Q) < S}, где Q G &, S > 0 (о топологи- топологических пространствах см. [35; гл. 2, § 5]). 19. Докажите, что для расстояния по вариации справедлива следующая фор- мула: ||Р - Q|| = 2 sup \P(A) - Q(A)\. 20. Пусть меры Р, Q G S* и пусть Р и Q абсолютно непрерывны относительно не- некоторой меры A G 3й (такая "доминирующая" мера существует, достаточно взять Л = (Р + Q)/2). Обозначим р = dP/d\, q = dQ/dX производные Радо- на-Никодима мер Р и Q по мере Л. Докажите, что ||Р — Q|| = \\р — q\\b1(x)^ где 1/1(Л) = L1(S, d, Л). Покажите, что г/(Р, Q) ^ 2, причем равенство до- достигается только если PJ_Q (меры сингулярны, т. е. имеется A G si такое, что P(A) = 1hQ(A)=0). 21. Докажите, что C^(S), v) — полное метрическое пространство и что это про- пространство несепарабелъно, если S — несчетно. Что можно сказать о сепа- сепарабельности C^(S), г/), если S — конечно или счетно? Используя расстояние по вариации, можно дать следующий вариант теоремы Пуассона об аппроксимации биномиальных вероятностей. п Теорема 18. Пусть Sn = J^ X&, где Xi,... ,Хп - независимые величины, к = 1 Р(Хк = 1) = рк, Р(Хк = 0) = 1-рь 0 < рк < 1, к = 1,...,п. Пусть У — пуассоновская случайная величина с параметром Л = р\ + • • • + рп. Тогда k=l для любого Bel.
Гл. V. Слабая сходимость мер. Принцип инвариантности 177 Доказательство этого результата можно найти, например, в [192; с. 39]; в [85; т. 1, с. 87, 479] указаны также уточнения приведенной оценки. В приложениях часто полезна следующая теорема. Теорема 19 (Шеффе; см., например, [2; с. 306]). Пусть Qn, n G NUoo, — такие меры на измеримом пространстве (S,?/), что Qn <С Л для некоторой а-ко- нечной меры А на (S,?/). Пусть dQn/d\ —у dQoo/dX п. н. по мере X. Тогда Qn сходится к Qoo no вариации. С расстоянием по вариации тесно связано расстояние Какутани-Хеллингера, определяемое, с учетом обозначений в упражнении 20, формулой /1 Г \1/2 d(P,Q)= [^J2 22. Проверьте, что d(P, Q) — метрика на 3^(S), значения которой не зависят от выбора доминирующей меры Л. Заметим, что где для a Е @,1) интеграл Хеллингера определяется формулой H(a;P,Q) = (paq1-a,Q). По поводу разнообразных применений, использующих метрику <i(P, Q) и интегра- интегралы Хеллингера Н(а; Р, Q), см., например, [85; т. 1, гл. III, § 9, 10]. Теорема 20 (Штрассен). Пусть Р, Q G 3^(S), где S — польское пространст- пространство. Тогда для метрики Леви-Прохорова верно следующее равенство: tt(P,Q) = Ы{е > 0: P(F) ^ Q(F?) +e для всех замкнутых F С S}. E0) 23. Рассмотрим банахово пространство BL ограниченных действительных лип- шицевых функций на польском пространстве S, снабженное нормой ||/||вь = = ||/||оо + L(f), см. D1). Докажите, что функция ||Р - Q\\*BL = sup{|(/, P) - </,Q)|: / е BL, \\f\\BL ^ 1} задает метрику на 3^(S), сходимость в которой также эквивалентна слабой сходимости. Более того, для любых Р, Q G 3^(S) имеют место неравенства ||Р - Q\\*BL <: 2тг(Р, Q), Л(тг(Р, Q)) ^ ||Р - Q\\*BL, где h(t) = 2t2/(t + 2),t^0. Теорема 21 (Штрассен, см., например, [27; с. 59]). Пусть (S,p) — польское пространство. Метрика Леви-Прохорова тг является минимальной метрикой для метрики Ки Фан к (метризующей сходимость по вероятности), т. е. ir(P,Q)=mf{x(X,Y)}, E1) где нижняя грань берется по всем парам случайных величин (X, Y) (каж- (каждая пара задана на своем вероятностном пространстве (П,^, Р)) так, что Law(X) = P, Law(F) = Q и k(X,Y) = Ы{г > 0: P(p(X,Y) > г) < г}.
178 А. В. Булинский, А. Н. Ширяев Соотношение E1) иллюстрирует идею каплинг-метода (иначе — метода "соеди- "соединения") для оценки расстояния тг(Р, Q), состоящего в подборе пар случайных эле- элементов (X, Y) с заданными маргинальными распределениями, таких, что при этом X и Y близки по вероятности в метрике р. С общей теорией каплинг-метода и его разнообразными приложениями можно ознакомиться по монографиям [159, 190]. Использованию различных метрик в предельных теоремах для сумм независимых случайных величин, а также для описания устойчивости стохастических моделей, посвящены монографии [27, 175]. Отметим также, что метрический подход является весьма полезным в теории слу- случайных процессов. Например, при широких условиях этот подход позволяет полу- получить оптимальную оценку скорости сходимости в принципе инвариантности Донске- ра-Прохорова. Предположим, что Хп^ (г = l,...,mn; п ^ 1) — последовательность се- серий невырожденных случайных величин, независимых в каждой серии, для которых EXni = 0, E|Xn^|s < оо при некотором s > 2. Будем считать Хп^ нормированными так, что ^2 ап % ~ 1> где ап % ~ DXn^- Положим г=1 тп , Ln s = ^2 E|Xn^|s- Эта величина называется дробью Ляпунова (при отсут- г=1 ' V ствии условия нормировки соответствующая величина определялась бы как дробь тп / /тп \s/2\ Ln,s = ^2 Е \Xnji — EXU:i\s / f ^2 an i) )• Легко видеть, что условие Ляпунова Ln,s ~^ 0 при п —> оо влечет условие Линдеберга A8). Полезно отметить, что условие Линдеберга явля- является не только достаточным, но и необходимым для выполнения центральной пре- предельной теоремы (см. приложение 3), если слагаемые Хп^ удовлетворяют B2). Ис- Используется (см. [50, с. 102]) и "условие бесконечной малости": max P(|Xn^| ^ s) —> 0 при п —У оо для каждого е > 0. E2) Необходимые и достаточные условия выполнения центральной предельной теоремы без предположения E2) можно найти в [27, 85]. Теорема 22 (Боровков). Пусть для серий независимых центрированных слу- случайных величин Хп^, г = 1,..., mn, п ^ 1, при некотором s G B, 3] E\Xn,i\s < оо, г = 1,.. .,шп, п ^ 1, тп а также ^2 DXn^ = 1, ^ ^ 1- Тогда для случайных ломаных 5П(-), фигуриру- г=1 ющих в теореме 8, справедлива следующая оценка: 7r(Pn,W) ^cLl/}ss+1\ E3) где Рп — распределение случайной ломаной Sn( •), W — мера Винера в С[0,1], и с — некоторый множитель, не зависящий от п.
Гл. V. Слабая сходимость мер. Принцип инвариантности 179 Подробное доказательство этого результата вместе с библиографическими ссыл- ссылками и необходимым вспомогательным материалом можно найти в [7] (там же разо- разобран пример Т. Арака, показывающий, что оценка E3) оптимальна с точностью до выбора постоянного множителя с). Для s = 3 доказательство приведено в [4]. В заключение этого раздела затронем вопрос, относящийся к случайным мерам. Пусть S — локально компактное хаусдорфово пространство со второй аксиомой счетностии M(S) —пространство локально конечных мер \i = /i( •) на борелевской сг-алгебре ?&{S) (т.е. пространство мер, являющихся конечными на ограниченных множествах пространства S), снабженное топологией, в которой непрерывны отобра- отображения = для функций / из класса СJ непрерывных неотрицательных функций с компактным носителем. 24. Докажите, что M(S) — польское пространство. Для этого заметьте, что если /ъ /25 • • • — множество, плотное в Cj, то функция /i) - (Д,|/)| Л 1), »,ие M(S), метризует введенную топологию. При этом 5§(M(S)) порождается как отображениями тг/, / Е Cj (т.е. является наименьшей сг-алгеброй, отно- относительно которой измеримы все эти отображения), так и отображениями 7гв: 1л ^ 1л{В) для В G SM,rae/iG M(S),SM = {В е JI: ц{дВ) =0},а^ — совокупность ограниченных подмножеств пространства S. Определение 10. Пусть заданы вероятностное пространство (П, ^, Р) и введен- введенное выше пространство локально конечных мер М (S). Случайной мерой называется 3* | Й§(M(S))-измеримое отображение \i\ п —У M(S). Точечный случайный процесс определяется как случайная мера, принимающая значения в (замкнутом относитель- относительно введенной топологии) подмножестве N(S) С M(S), состоящем из целочисленно- значных случайных мер. 25. Пусть /i, /ii, /i2,... — случайные меры на S — локально компактном хаусдор- фовом пространстве со второй аксиомой счетности. Докажите, что следую- следующие три условия эквивалентны: 1. fin —У \i при п —У оо, 2. (/,/in) 4 (/,//> Для/ G C+ (п -> оо), 3. (/xn(Bi),...,/xn(Bfc)L(/x(Bi),...,/x(Bfc))OTflBi,...,BfcGSAi,fcGN, raeSM = {Б G М\ц(дВ) = 0п.н.}. Дальнейшие сведения о слабой сходимости мер и о различных аппроксимациях случайных процессов можно получить, обратившись, например, к следующим кни- книгам и статьям: [2, 65, 71, 97, 98, 105,109,117,168,172,193].
Глава VI Марковские процессы. Дискретное и непрерывное время Эквивалентные определения марковского процесса. Марковость процессов с не- независимыми приращениями со значениями в R . Примеры. Цепи Маркова, их построение по переходным вероятностям и начальному распределению. Пуассо- новский процесс как марковская цепь. Переходная функция марковского процесса. Нахождение переходной функции d-мерного броуновского движения. Конечномерные распределения марковского процесса, их выражение через начальное распределение и переходную функцию. Однородные марковские процессы. Эргодическая теорема для однородных цепей Маркова. Следствия. Инвариантная мера. Инфинитезимальная матрица Q стохастической по лугруппы (P(t))^>o- Обратная и прямая системы диффе- дифференциальных уравнений Колмогорова. Стационарное распределение как собственный вектор матрицы Q*. Формулы Эрланга. Модель системы массового обслуживания, приводящая к этим формулам. § 1. Замечательная идея А. А. Маркова, лежащая в основе всей развиваемой те- теории "марковских процессов", состоит в том, что выделяется класс процессов, для которых эволюция во времени может быть описана следующим образом: поведе- поведение процесса после момента t определяется не всей его предысторией, а лишь значением, которое процесс принял в момент t. Здесь уместна аналогия с клас- классическим описанием движения "частицы", поведение которой после момента t опре- определяется лишь ее положением (координатами) и скоростью в момент t. Если вре- время дискретно (t = 0,1,...), то сказанное особенно наглядно: изменение состояний изучаемого объекта представляется последовательностью шагов, где каждый шаг определяется предыдущим. Отсюда и возникло понятие цепи Маркова, послужив- послужившее основой разнообразных дальнейших обобщений. Формализуем сказанное и рассмотрим ряд эквивалентных определений. Некото- Некоторые обобщения отнесены в дополнение. Всюду в этой главе X = {Х^, t Е Т} — случайный процесс, заданный на некотором (полном) фильтрованном вероятностном пространстве (П, &, (^t)teTi Р) с фильтрацией, расширенной классом Р-нулевых событий, и при каждом t Е Т С Ж принимающий значения в измеримом пространстве (Sj, $&t). Определение 1. Процесс X, согласованный с фильтрацией (&t)teT, называется марковским (по отношению к этой фильтрации и мере Р), если для каждого s Е Т
Гл. VI. Марковские процессы. Дискретное и непрерывное время 181 и любого события С е &^s = cr{Xu, u^ s,u еТ} S) п.н. A) Напомним, что Р(А \ sd) = Е A^ | sd), P(A \ rj) = Е A^ | сг{^}), где А е &, сг-ал- гебра sd С 3*, т\ — случайный элемент, заданный на вероятностном пространстве (П,^,Р). Если соотношение A) справедливо для некоторой фильтрации, с которой согла- согласован процесс X, то оно, очевидно, выполняется и для естественной фильтрации Wx = (&?)seT, где &х = <г{Хи, и^ s,ueT},s еТ. Множества из сг-алгебр &* и 3^s интерпретируются соответственно как события из "прошлого" и "будущего" процесса X (относительно фиксированного "настоящего" момента s). Когда гово- говорят, что иХ — марковский процесс", то, как правило, подразумевают естественную фильтрацию. Перед чтением этой главы было бы полезно ознакомиться с главой VIII в [85], где рассматриваются марковские цепи с дискретным временем (Т = Z+) и дискретным пространством состояний. Для таких цепей там проводится классификация состоя- состояний, поэтому в данной книге эти результаты не излагаются. Упомянутое ознаком- ознакомление целесообразно, но для понимания приводимого ниже материала не является необходимым. При первом чтении главы VI следует усвоить формулировки различных определе- определений марковского процесса (без доказательства их эквивалентности), содержащиеся в § 2-5. Полезно обратиться к примерам из § б, а далее сосредоточиться лишь на ма- материале, относящемся к марковским цепям с дискретным и непрерывным временем (§ 7-9). Важной для приложений являются эргодическая теорема в § 13 (для пони- понимания которой требуется определение однородной марковской цепи, данное в § 12), а также ее простые следствия (§ 14). Желательно изучить прямые и обратные сис- системы уравнений Колмогорова (§ 17), хотя бы для случая конечного пространства со- состояний (см. § 18). § 2. Заметим прежде всего, что всякий процесс X, состоящий из независимых дей- действительных случайных величин Х±, где t G Г, будет марковским (это сразу выте- вытекает из теоремы 2, доказываемой в § 5). Далее, условие A) напоминает определение мартингала и чтобы лучше понять, в чем сходство и отличие упомянутых определе- определений, а также для построения примеров нам понадобятся следующие вспомогатель- вспомогательные утверждения. Лемма 1. Случайный процесс X является марковским тогда и только тог- тогда, когда для каждого s Е Т и любой ограниченной 3*^s \ &(Щ-измеримой функ- функции F: п -> R S) п.н. B) Доказательство. Очевидно, A) есть частный случай B), когда F = 1с, С G &^s- Выведем B) из A). Пользуясь линейностью условного математического ожидания*) получаем, что B) будет справедливо для "простых" функций вида F = Е OLi\Cl, где щ G Ж, С{ G &>8, г = 1,..., q. г=1 *) Е (af + PC | sd) = aE (f | sd) + CE (C | sd) п.н. для a, /3 G R и сг-алгебры sd С &, если | <oo, E|C| <oo.
182 А. В. Булинский, А. Н. Ширяев Из соотношения A.4) следует, что любая ^/-измеримая ограниченная функция ft: ft —У Ж ( sup | ft (о;) | ^ Н для некоторой константы Н > 0) есть равномерный предел "простых" ^/-измеримых функций hn = ^2 fn,k^-Dn к^ гДе \rn,k\ ^ ^\ Dn,k G «с/, fc = —2n,...,2n — 1, а множества Dn^ образуют разбиение п при каждом п Е N. Имеем sup |ftn(cj)| ^ H и sup |/in(W) - ЛН| «С 2"и, C) откуда для любой а-алгебры Ж С 3* |E(ftn|^)-E(ft|^)| ^ E(|ftn-ft| | Ж) ^2"n, nGN. Полагая sd — 3*^s-> $? — &s-> а затем J^ = cr{Xs}, видим, что A) => B). Заметим, что если (п —у ( п.н., (п —у ( п.н. и при этом ?п = ^п п.н. (n G N), то, учитывая полноту вероятностного пространства, имеем ( = ( п. н. ? Для введенного выше процесса X = {Xt, t G Т} и множества U С Т определим совокупность ^(U) случайных величин, являющихся конечными линейными комби- m нациями функций вида Yl /s^(-^sj5 где /Sfc : SSfc —>¦ М — ограниченные измеримые г=1 функции (варьируются ттг, точки Sk G С/ и функции /Sfc),& = l,...,m. Обозначим Ун (U), где константа i? > 0 и множество С/ С Т, совокупность вели- величин f G ^(?7), для которых |^| ^ Я п.н. Положим <тх(Е/) = cr{^t, teU},U СТ. Лемма 2. Пусть действительная случайная величина h является <Jx{U)- измеримой (U С Т), причем \h\ ^ Н п. н. для некоторой константы Н > 0. Тогда h может быть получена как предел п. н. и предел в L1(Q, ^, P) последо- последовательности случайных величин из Доказательство. Положим si = cr{Xtj t G U} и выберем, как при доказатель- 2п-1 стве леммы 1, функции hn = J^ fn^Dn k •> для которых верно C). Заметим, что /с = -2™ si — а{^?], где алгебра 5? состоит из множеств вида L = {ио\ (XSl,..., XSfn) e С}, si,... ,sm G С/, Cij G ^s^ причем объединяемые "прямоугольники" не пересека- пересекаются, г = 1,..., m, j = 1,..., q, m,q G N. Для каждого г > 0 и любого D G «с/ по лемме 3 главы I можно выбрать L? G «S? так, что P(D Л Le) < е. Поэтому для каждого п ^ 1, всех /с = —2П,..., 2П — 1
Гл. VI. Марковские процессы. Дискретное и непрерывное время 183 и любого еп Е @,1) найдутся множества Lnk Е «S? такие, что P(Dn^k A Ln^k) < ~ 1п-\ < гп2-п-1. Пусть hn= J2 rn,klLn^k. Тогда (все \rn,k\ ^ Я) E) k = -2n Перейдем к непересекающимся множествам k-l и (J LnJ при /c =-2 и положим /in = ^2 rn,k^-Z ' n ^ ^* Тогда |/in| ^ Я при всех n G N. Кроме k = -2n ' n'fc того, E \hn — hn\ ^ 4i??n2n, n G N. Следовательно, при каждом п E\h-hn\^E\h- hn\ + E \hn - hn\ + E \hn - hn\ ^ 2~n + Hen(l + 2n+2). Выберем еп так, чтобы гп2п —у 0 при п —у оо. Тогда /in —>¦ /i в пространстве Ь1(П,с?,Р)прип ->> оо. Если последовательность сходится в среднем, то она сходится и по вероятности, а значит, имеется подпоследовательность (скажем, {hnk}), сходящаяся п.н. Теперь заметим, что для введенных выше множеств L G «S?, где С задается соотношением D), q 1ьИ = X 1су (Х81 (и)) • • • lCmj (XSmИ). F) Таким образом, величины /infc обеспечивают искомую аппроксимацию (по некото- некоторой подпоследовательности {пк}). ? §3. Ранее предполагалось, что процесс X = {Xt, t G Т} принимает при каждом ? G Т значения в произвольном измеримом пространстве (Sj, 5§t)« Дадим теперь критерий марковости процесса в предположении, что его значения лежат в борелевских пространствах. Теорема 1. Пусть (St, ?$t)teT — семейство борелевских пространств. Слу- Случайный процесс X является марковским тогда и только тогда, когда для каж- каждых s ^ ? (s,t G Т) и любой ограниченной Sfet-измеримой функции /: St —У Ж \X8). G) Последнее соотношение равносильно тому, что для всех В € Set B\xs). (8)
184 А. В. Булинский, А. Н. Ширяев Доказательство. Очевидно, B) влечет G). Пусть теперь выполнено G). По- Покажем вначале, что B) верно для функций F вида п F=Y[fi(Xti), (9) г=1 где берутся произвольные п ^ 1, s ^ t\ < • • • < tn {s и все ti из Т) и любые огра- ограниченные Зёц-измеримые функции fi'.Sti —> Ж (г = 1,...,п). Для этого применим индукцию. Пусть п ^ 2 (формула G) отвечает случаю п = 1). Свойства условного математического ожидания и соотношение G) дают = Е (h(Xtl) ¦ ¦ • /„-ipf^JE (/„№„ = E (/!(Xtl) • • • U-iiXt^E (fn(Xtn здесь/n_i(Xtn_1) = fn-i(Xtn_1)E(fn(Xtn) | Х4п_1)имывоспользовалисьтем,что если ? — действительная случайная величина с Е |?| < оо, а случайный элемент ту принимает значения в борелевском пространстве (S, 5S), то где функция if. S —>¦ Ш. является ^-измеримой (это утверждение нетрудно получить, рассмотрев доказательство в [85; т. 1, с. 219], проведенное там для действительной случайной величины 77). Кроме того, мы учли, что |Е (? | rj)\ ^ Н, если |^| ^ Н. Напомним также, что равенства и неравенства для случайных величин считаются выполненными с вероятностью единица. Аналогично, поскольку a{Xs} С &tn-i •> т0 E(F\Xs) = E(f1(Xtl)--Jn_1(Xtn_1)\Xs). Отсюда соотношение B) следует для функций F вида (9), а значит, и для их линей- линейных комбинаций. Взяв в лемме 2 множество U = [s, 00) П Т, т.е. ax(U) = ^^s, и ограниченную функцию h = F G i^s | Й§(М), получим E\E(F\X8)-E(hn\X8)\^E\F-hn\, где /in G Ун(U), n G N и |.F| ^ i?. Поэтому существует подпоследовательность {п/с}, для которой Е (/infc | J^s) -»> Е (F \ &'s) п.н. и Е (/infc | Xs) -)> Е (F | Xs) п.н. при & —>¦ оо. По доказанному (для функций F вида (9)) Е (/in \&s) — Е (/in | Xs) п.н. при всех n G N, что приводит к выполнению B) в общем случае. Равносильность G) и (8) проверяется совершено аналогично доказательству лем- леммы 1. ?
Гл. VI. Марковские процессы. Дискретное и непрерывное время 185 Замечание 1. Если параметрическое множество Т = Ъ+ (= {0,1,...}), то в ус- условиях G) и (8) теоремы 1 достаточно брать лишь ? = s + l,sEZ+. § 4. Доказанная теорема 1 позволяет следующим образом сопоставить опреде- определения мартингала и марковского процесса. Далее параметрическое множество ТсМ, вообще говоря, не предполагается дискретным. Следствие 1. Действительный марковский (относительно некоторой фильтрации (&t)teT) процесс X = {Xt, t G Т} является мартингалом (Xt,&t)teT в том и только том случае, если E|Xt|<oo и E(Xt\Xs) =XS для s^t (s,teT). A1) Доказательство. Возьмем для п ^ 1 функцию f(x) = ж-1{|ж|^п}И, подставив ее в соотношение G), совершим предельный переход (см. [85; т. 2, с. 299]) при п —> оо. В итоге приходим к искомому утверждению. ? § 5. Дадим еще одну удобную форму проверки марковости процесса (со значени- значениями в борелевских пространствах) относительно естественной фильтрации. Теорема 2. Марковость процесса X = {Xt, t G Т}, принимающего при каж- каждом t Е Т значения в борелевском пространстве (St,?$t), равносильна следую- следующему свойству, для любого т ^ 1 и всех s\ < • • • < sm < s ^ t (s, t и все si изТ), а также для произвольной ограниченной Sfet-измеримой функции f Е (f(Xt) \Xsl,...,XSm,Xs) = E (f(Xt) | Xs). A2) Последнее соотношение эквивалентно тому, что для любого множества в eS&t ?(xt ев\х31,...,х3т,х3) = P(xt g в | х8). A3) Доказательство. Пусть выполнено свойство G). Тогда = E(E(/(Xt) \XS)\XS1,..., XSn, Xs) = E(f(Xt) | Xs), т. е. имеет место и свойство A2). Пусть теперь справедливо A2). Случайная величина E(f(Xt) \XS) является a{Xs}-измеримой и, следовательно, 3*^--измеримой. Поэтому для доказательст- доказательства G) надо убедиться в том, что при любом A G 3*^ E\AEC(Xt)\Xs) = E\Af(Xt). A4) Согласно A2) последнее равенство верно для событий А, входящих в алгебру, по- порождающую 3^, т. е. для множеств вида А = {(XSl,..., XSfn, Xs) G С}, где С е @S1 0 • • • 0 ^Srn ®&s, si < ''' < Sm < s, m ^ 1.
186 А. В. Булинский, А. Н. Ширяев Алгебра является тг-системой, а совокупность множеств А, удовлетворяющих соот- соотношению A4), очевидно, образует А-систему. Поэтому требуемое утверждение вы- вытекает из теоремы о монотонных классах (теорема 1 главы I). Равносильность утверждений A2) и A3) устанавливается аналогично доказа- доказательству леммы 1. ? § 6. Рассмотрим некоторые примеры. В классе марковских процессов со значе- значениями в пространстве Rd особую роль (ввиду их относительной простоты) играют процессы с независимыми приращениями. Свойство марковости этих процессов обеспечивает Теорема 3. Пусть X = {Xt, t G T} — процесс с независимыми прираще- приращениями, принимающий значения в Ж , d ^ 1. Тогда X — марковский процесс {относительно естественной фильтрации). Доказательство. Имеем a{XSl,... ,XSrn,Xt) = cr{XSl,XS2 — XSl,... ,Xt — XSfn}, поскольку векторы в фигурных скобках связаны невырожденным линейным преоб- преобразованием. Нетрудно показать (например, используя лемму 2), что если ? и ? — независимые случайные векторы соответственно со значениями в!^ иЕг,а функция д:Жк х!1 —у Ж есть ограниченная борелевская функция (т. е. ?$(Жк+1) | ^(М)-изме- римое отображение), то Е(Ж,С)|С = 2/) = Е5(?,2/), yeR1 A5) (ср(у) = Е (у | ? = у) означает, что в качестве Е [у \ Q можно взять <р(С)? см. A0)). Теперь заметим, что для 0 ^ s\ < • • • < sm ^ t ^ и (точки из Т) и ограниченной $(Rd) | ^(М)-измеримой функции / , , m+l E(f(xu)\xsl,...,xSm,xt) = e(/U+ Y1 С» где Ci = XS1, C2 = XS2 - XS1J ..., Сш = XSrn - XSrn_1J (rn+i = Xt - XSrnJ ? = Xu — Xt. Следовательно, для всех ?д,..., 2/m+i G Md m+l e fU+y,ti г=1 Cl=2/l,...,Cm+l=2/m+l) = m+l г=1 где ^ — (ограниченная) борелевская функция. Это легко устанавливается для "прос- "простых" функций, фигурирующих в лемме 1, которые равномерно по и G П аппроксими- аппроксимируют функцию /(?(о;) + х) при каждом фиксированном xGlrf. (Для этого с помо- помощью теоремы о монотонных классах проверяется, что Р(? + х G В) есть борелевская функция по х при каждом 5 G S$(Kd), а затем применяется лемма 5 главы I.) Таким образом, Р-п.н. m+l
Гл. VI. Марковские процессы. Дискретное и непрерывное время 187 Осталось теперь лишь заметить, что m+l m+l г=1 m+l г=1 m+l Sm+1 m+l E г=1 Сг = г=1 Следствие 2. Процесс частных сумм Sn = ?о + * * * + 6ю п ^ О (где ?о? ?ъ • • • — независимые векторы в Жл), пуассоновский процесс и броуновское движение в Ж<* являются марковскими процессами. Естественное обобщение процесса частных сумм содержится в следующем при- примере. Пример 1. Пусть на вероятностном пространстве (П, ^, Р) заданы независимые случайные векторы ?i, ?2 ? • • • со значениями в ld и не зависящий от них случайный вектор Хо: П —у Ж171. Положим гдеборелевскиефункции/in: Mm хЖA —У Ж171. ТогдаX = {Xn, n ^ 0} — марковский процесс. Для проверки этого утверждения воспользуемся замечанием 1. При любом п ^ 0, произвольной ограниченной борелевской функции /: Ж171 —у Ж и всех жо,..., хп G Ж171 имеем в силу A5), что Е (f(X n+i) \Xo=xo,...,Xn = хп) = = жо,...,Хп = хп) = Аналогично, Е (f(Xn+1) | Хп = жп) = д(хп), откуда и следует искомый результат. Отметим, что этот пример, в определенном смысле, носит общий характер (см. упражнение 19). Обширный класс марковских процессов дают решения стохастических дифферен- дифференциальных уравнений (см. далее § 18 главы VIII). § 7. Обратимся к наиболее важному случаю, когда при каждом t G Т С Ж мар- марковский процесс принимает значения в одном и том же множестве S, называемом фазовым пространством или пространством состояний (точнее говоря, будем предполагать, что (St, $&t) — (S, Й§) при всех t G Т, где (S, Й§) — некоторое измери- измеримое пространство).
188 А. В. Булинский, А. Н. Ширяев Пусть вначале фазовое пространство S изучаемого марковского процесса диск- дискретно, т. е. состоит из конечного или счетного числа (занумерованных) точек. Точ- Точку Xi G S отождествим с ее номером г, при этом будем считать, что г пробегает либо конечное множество {0,1,...,г}, либо счетное 14 U {0}. Пусть ё% состоит из всех под- подмножеств S. Пространство (S, ё$) является польским, если ввести метрику, положив р(х, у) = 1 при х/р р(х, х) = 0, где ж, у G S. Заметим также, что в рассматрива- рассматриваемом "дискретном" случае любая функция g: S —У Ж является ^-измеримой. Определение 2. Марковский процесс с дискретным пространством состоя- состояний называется цепью Маркова (и в случае дискретного, и в случае непрерывного времени). Проанализируем подробнее в рассматриваемом случае марковское свойство. Пусть si обозначает сг-алгебру, порожденную разбиением П на множества Aj, где j G J, J — конечное или счетное множество ( |J Aj = П, Ai П Aj = 0 при г ф ])• KjeJ J Если ? — случайная величина с Е |?| < оо, то условное математическое ожидание Е (? | si) = Е (? | si)(и) на множествах разбиения Aj с P(Aj) > 0 может быть задано формулой Если P(Aj) = 0, то Е (? | si) (и) для со G Aj можно считать произвольным числом aj. Взяв в A7) ? = 1в, получим P{B\si){uj) = P{BC\Aj)/P{Aj) при ueAj и P{Aj) > 0. Для изучаемого процесса X = {Xt, t G Т} и любого набора точек ti,..., tn G T сг-алгебра cr{Xt1,..., Х^те } порождается разбиением пространства П на события ви- вида {Xtl = ji, ...Д4п = Jn}, где ji,...,jn G S. С учетом сказанного из A3) выводим, что процесс X = {Xt, t G Т} является цепью Маркова тогда и только тогда, когда для любого т ^ 1, всеж точек s\ < ••• < sm < s ^ t (являющихся точками из Т) и любых i,j,ii,... ,im G S справедливо равенство P(Xt = j I XS1 = гь ... ,XSm = im, Xs=i)= P(Xt =j\Xs=i) A8) (во всех тех случаях, когда P(XSl = ii,..., XSfn = im, Xs = г) Ф 0). Положим Ss = {г G S: P(XS =i)/0},sG T. Определение З. Переходными вероятностями марковской цепи называются функции Pij(s,t) = P(Xt=j\X8=i), где s^t(s,tGT), г G Ss, j G St. A9) Из этого определения легко получить, что функции p^j (s, ^) удовлетворяют усло- условиям: 1) pij(s,t) ^ 0 для любых г G Ss, j G St, и s ^ t, s, t G T;
Гл. VI. Марковские процессы. Дискретное и непрерывное время 189 2) ^2 Pij(sJ t) = 1 для любых г G Ss и s ^ ?, s, t G T; S 3) Pij(s, s) = E^j для любых г G Ss, j G St и s G T, где ^j — символ Кронекера; 4) для любых i G Ss, j G St и всех s ^ и ^ t (s,u,t ? T) Pij(s,t) = J^ Pik(s,u)pkj(u,t). B0) Действительно, условия 1)-3) очевидны. Проверка соотношения 4), иногда назы- называемого уравнением Маркова, проводится (с учетом A8)) следующим образом: Pij(s,t) = P(Xt=j\Xs=i) = = ? P(XS = i) P(Xt= j, Xu = k,Xs=i)_ k - i I x - k x - л p{Xu = k>x°=j) -j\xu-k,xs-t) I, . D / V /ЛЛЛ Л-iH \<S / = j | Xw = k)P(Xu = k\X8=i) = Pik(s,u)pkj(u,t). B1) Замечание 2. Отправляясь от функций Pij(s,i), заданных для s ^ t (s,? G T), i G Ss, j G St, можно оперировать с их продолжениями, определенными для s ^ t (s,t G Т) и всех г, j G S. Для этого достаточно положить Pij(s,t) = 0 при i G Ss, j ? St и pij(s,t) = Pioj(s,t) при г ? Ss, j G S, где г0 = го(в) G Ss выбрано произвольно (Ss т^ 0). При этом легко убедиться, что свойства 1)-4) переходных вероятностей будут верны и для их продолжений. Именно поэтому, как правило, предполагается, что такое продолжение уже осуществлено и St = S для всех t G Т. Определение 4. Начальным распределением марковской цепи X = {Xt, t G T}, где 0 G Т = [0, оо), называют меру \i — La Ясно, что Р(Х0 ? В) = Y1 Рг@) для любого В С S, где г), г G S. B2) § 8. Обратимся к рассмотрению свойств конечномерных распределений марков- марковских цепей. Теорема 4. Конечномерные распределения марковской цепи X = {Xt, t G T}, где 0 G Т С [0, оо), однозначно определяются переходными вероятностями Pij(s,t) и начальным распределением /i, т.е. набором Pi@) (s,t G T, s ^ t,
190 А. В. Булинский, А. Н. Ширяев Доказательство. Для 0 ^ t\ < • • • < tn (?& G Г, к = 1,..., n), n ^ 2, и любых Л, • • •, jn G S, обозначив А = {Xtl = jb ..., Xtn_1 = jn-i}, имеем P(Xtl = л, ..., Xtn = jn) = P(Xtn = jn I A)P(A) = = P(Xtn=jn\Xtn_1=jn_1)P(A) = = Jb -.., ^tn_i = jn-l) •Pjn_1^jn{tn-1,tn). Этот вывод, основанный на определении условной вероятности и формуле A8), пред- предполагает, что P{Xtx = л,... ,Xtn_1 = jn-i) Ф 0- Однако полученное равенство справедливо и при Р[ХЧ = ji,... ,Xin_i = jn-i) = 0 (функцияр^-ь^^п-ь^п) всегда определена, согласно замечанию 2). Таким образом, приходим к формуле P(Xtl = л, ...,Xtn =in) = P(Xtl =ii)-Pjltj2(t1,t2)---pjn_ltjn(tn-1,tn), B3) имеющей вполне наглядный смысл. А именно, будем считать, что в момент t\ некоторая "система" начинает свое движение из состояния j\ •> переходя за время от ti до ^2 в состояние J2,..., за время от tn _ i до tn система совершает переход из jn _ i в jn. Тогда формула B3) означает, что для подсчета вероятности "маршрута" {Xt1 = л, ..., X-tn = jn} надо поступить следующим (весьма естественным образом, если принимать во внимание, что рассматриваемая система функционирует по принципу "беспоследействия"): взять вероятность начального состояния Р(Х^ = л) и умножить ее на (условные) вероятности переходов "системы" (jk-i —> jk) за соответствующее время от tk-i до ?&, где к = 2,..., п. По формуле полной вероятности p(xtl =ii) = что позволяет, в силу B3), для любого В С Sn найти искомое выражение конечно- конечномерных распределений: p((xtl,...,xtn)eB)= J2 YlPi(o)Pijl{o,ti)---pjn_1,jn{tn-utn). a (ju---Jn)eB г B4) Справедливо и обратное утверждение. Теорема 5. Пусть 0 G Т С [0, оо), а некоторые непустые множества Ss С S, s GT, где S — конечно или счетно. Пусть задан набор Pi@) > 0, г G So, wa- кой, что Y1 Рг@) = 1, и заданы функции pij(s,t) при s ^t (s,t G Г), i G Ss, is j G St, (?л^ которых выполнены приведенные в §7 условия 1)-4). Тогда на неко- некотором вероятностном пространстве (П, ^, Р) существует марковская цепь X = {Xt, < G Г} с пространством состояний St при каждом t G Т (т. е. Xt(u;) G St при любом t е Т для всех и; G П) такая, что справедливы соотно- соотношения B2) г/ A9).
Гл. VI. Марковские процессы. Дискретное и непрерывное время 191 Доказательство. Если бы марковский процесс X с данными характеристика- характеристиками B2) и A9) существовал, то его конечномерные распределения должны были бы определяться формулой B4). Поэтому ничего другого не остается, как ввести меры Ptb...,tn на ё%п (для 0 ^ t\ < • • • < tn, n Е N) с помощью правой части равенст- равенства B4) и проверить их согласованность. То, что при этом на ё$п возникают веро- вероятностные меры, обеспечивается требованиями, налагаемыми в условиях теоремы на pi@) и Pij(s, t) (счетная аддитивность имеет место, поскольку ряды из неотри- неотрицательных чисел можно суммировать в любом порядке). Для согласованности мер Pti,... ,tn в силу замечания 3 главы I достаточно убедиться лишь в выполнении усло- условия 3° § 11 главы I. Оно верно в силу условий 2), 3) и 4), которым удовлетворяют функции Pij(s, ?); условие 3) обеспечивает согласованность, когда рассматриваются и не только несовпадающие друг с другом точки ti,..., tn. Следовательно, по тео- теореме Колмогорова существует процесс X = {Xt, t Е Т} с данными конечномерными распределениями Ptb...,tn (О ^ ti < ••• < tn, n ? Щ- Этот процесс будет мар- марковским, поскольку B4) влечет B3), откуда немедленно вытекает A8), а также A9). Кроме того, полагая в B4) п = 1 и t\ = 0, получаем B2). ? § 9. Важнейшим представителем марковских цепей с непрерывным временем яв- является пуассоновский процесс, определенный в § 2 главы П. Теорема 6. Процесс N = {Nt, t ^ 0} является пуассоновским (с ведущей локально-конечной мерой т на Й§([0, оо), т([0, оо) = оо) тогда и только тогда, когда N — цепь Маркова с пространством состояний S = {0,1,2,...}, для которой Pi@) = Sio и переходные вероятности (j-i)\ ' J^' B5) 0, j < h где 0 ^ s < t, i, j G S upij(s,s) = Sij для s ^ 0, i, j G S @° считается равным 1). Доказательство. Необходимость. Пусть N — пуассоновский процесс: No = 0 п.н., процесс имеет независимые приращения и Nf — Ns ~ 7rm((s,t]) ПРИ^ > s ^ 0,т. е. приращение Nt — Ns распределено по закону Пуассона с параметром m((s,t]). В силу теоремы 3 процесс N является марковским, при этом п.н. Nt = Nt — Щ ~ тгт((о,*])? t > 0, и P(Nt G S) = 1, где S = {0,1,... }. Очевидно также, что pi@) = P(NO = г) = = 5io, i G S. Для 0 ^ s < t имеем P(Nt -Ns=j-i,Ns= P(NS = i) Pij(s,t) = P(Nt=j\Ns=i) = V " ZZ_:: = P(Nt-Ns=j-i), что в силу указанного свойства распределений Nt — Ns дает формулу B5). При s ^ 0 для всех г, j G S находим, что Pij(s, s) = P(NS = j \ Ns = г) = Sij. Достаточность. Пусть N = {Nt, t ^ 0} — цепь Маркова с переходными веро- вероятностями B5) и начальным распределением, сосредоточенным в нуле. Очевидно,
192 А. В. Булинский, А. Н. Ширяев Щ = 0 п.н., поскольку pi@) = 8ю, г G S. Для s < t и к ^ 0 имеем, пользуясь B3) и B5), оо оо P(N* — N — к) — \^ Р(N* — N — к N — Л — \^ Р(N* — к 4- / /V — Л — oo oo 1=0 i ^(m(@,s]))< _m((o,sl ^ /! e (=0 (m((e,t]))\_mae,tl) iMM))\- c Итак, Nt — Ns ~ 7rm((s,t])j 0 ^ s ^ t. Для доказательства независимости прира- приращений воспользуемся формулами B3) и B6). С их учетом находим, что для п ^ 2, 0 = to ^ ti < • • • < tn и fci,..., кп G N U {0} вероятность P(JVtl = кг, Nt2 -Ntl=k2,..., Ntn - Ntn_x = kn) = = P(Ntl =k1,Nt2=k1 + k2,...,Ntn=k1 + --- + kn) = Ы = n Отсюда следует независимость соответствующих приращений. П § 10. Введем понятие переходной функции, играющей ключевую роль при по- построении марковских процессов. Пусть (St, S$t)teT — семейство измеримых пространств, множество Tel. Определение 5. Функция P(s,x,t,B), заданная для s ^ t (s, t G Г, ж G S, В G 5§tK называется переходной функцией (или марковской переходной функци- функцией), если: 1) при фиксированных s, ж, ? функция P(s, x,t, •) является мерой на (St, $&t)\ 2) при фиксированных s,t, В функция P(s,-,t,B) является , 3) P(s,x,s,B) = 5Х(В) при любых s е Т, х е Ss, В е ?%s; 4) при всех s < и < t (s,u,t G Т), ж G Ss, В ? ?$t выполняется уравнение Колмогорова-Чепмена: P(s,х,t,B)= f P(s,x,и,dy)P(u,у,t,В). B8) J S7/
Гл. VI. Марковские процессы. Дискретное и непрерывное время 193 Заметим, что в силу требования 3) соотношение B8) верно для переходной функции и при s ^ и ^ t. Определение 6. Говорят, что марковский процесс X = {Xt, t G Т} (со значе- значениями в (St, Set) ПРИ каждом t G Т) обладает переходной функцией P(s,x,t, В), если при всех s ^ t (s,t G Г), В G Set P(XteB\X8)=P(s,X8,t,B) п.н. B9) или, что то же самое, P(s, ж, ?, Б) = P(Xt е В \XS = х) п.п. по мере Pxs • Понятен смысл данного определения: у марковского процесса с переходной функ- функцией существует "хороший" (в смысле сформулированных выше свойств 1)-4)) ва- вариант условного распределения, стоящего в левой части B9). Пользуясь свойствами условного математического ожидания, в случае борелев- ских пространств (St, ?%t)teT легко пояснить происхождение условия B8). В силу B9), (IV.2)hA3) дляв ^ и ^ t (s,u, t G Г) и В G SSt имеем (п.н.) Иначе говоря, для Pxs -почти всех х P(s,х,t,B) = E (Р(щXu,t,B)\X8=x). C0) Теперь заметим (см. [85; т. 1, с. 281]), что если имеется регулярная условная ве- вероятность Г(С) = P(XU G С | Xs = х) (где Г зависит от s,u,x), то для любой ограниченной функции g G S%u \ ?$(Щ получаем Е (g(Xu) \Xa=x)= ( д(у) Y(dy) (Рх,-п.н.). C1) Напомним, что регулярная условная вероятность Р(у G B\Q заведомо существует, если случайный элемент v принимает значения в борелевском пространстве. Итак, из C0) и C1) вытекает, что для произвольных марковских процессов имеет место лишь ослабленный вариант равенства B8), справедливый не для всех х G S, а лишь для почти всех х G S (по мере Pxs)- Отметим, что для марковских цепей, рассмотренных в § 5, переходная функция су- существует и дается формулой P(s,i,t,B)= зев где s ^ t (s, t G T), г G S (S конечно или счетно), В — любое подмножество S. В общем же случае вопрос о существовании у марковского процесса переходной функции не является тривиальным. Для процессов с дискретным временем, принимающих значения в произвольных измеримых пространствах, наличие пере- переходной функции — факт известный (см., например, [17; т. 1, гл. II, §4]). Существо- Существование же переходной функции для марковских процессов с непрерывным временем, принимающих значения в универсально измеримых пространствах, доказано лишь сравнительно недавно С. Е. Кузнецовым.
194 А. В. Булинский, А. Н. Ширяев Пример 2. Пусть W = {W(t),t ^ 0} — m-мерное броуновское движение. Тог- Тогда, применяя A5), получаем (обозначив || • || евклидову норму в Мт), что при s < t P(W(t) е В | W(s) =x) = E {l{W(t)-w(s)+w(s)eB} I W(s) = x) = = E l{W(t)-w(s)+xeB} = P(W(t) - W(s) e В - x) = I2 2(t-s) Отсюда видно, что для s < t переходная функция здесь дается формулой 1 Г -\\y-x\\2 P(s,X,t,B)={Mt_e))m/2JBe*-* dy, C3) поскольку при s < t все требования 1)— 4), очевидно, выполнены (для проверки B8) надо вспомнить, что свертка нормальных распределений дает нормальное распреде- распределение) . Если же t = s, то A5), независимость W(s) и 0 G Mm влекут p(w(s) е в | w(s) = х) = p(w(s) + о е в \ w(s) = х) = р(х + о е в) = 6Х(В). Следовательно, P(s, ж, s, В) = 8Х(В). §11. Пусть марковский процесс X = {Х^, t G Т} принимает значения в измери- измеримых пространствах (St, S%t)teT и обладает переходной функцией P(s, ж, t, 5). Ана- Аналогично рассмотрению в § 8 марковских цепей можно доказать, что при любом п ^ 1, всех so ^ t\ < • • • < tn (точки из Т) и произвольных В^ G ^tfc ? fc = 1,..., п, конеч- конечномерные распределения процесса X задаются формулой Ptl9...,tn(C)= [ Qso(dx) f ^sso Jstl / Jst C4) где С = Bi x • • • x Bn, QSo = Law(XSo), а интегрирование ведется последовательно справа налево (нетрудно проверить, что каждый раз получаются должным образом измеримые ограниченные функции). Заметим также, что для любых v^.s(v,s?T)vLBESes Q8(B)= f P(v,x,s,B)Qv(dx). C5) = f Действительно, QS(B) = P(XS eB) = E (E(lXseB\Xv)) = ,Xv,s,B) = / P(v,x,s,B)Qv(dx).
Гл. VI. Марковские процессы. Дискретное и непрерывное время 195 В частности, если Т = [О, оо), то мера Qs для любого s > 0 определяется по мере Qo и переходной функции. Здесь уместна аналогия с гауссовскими процессами, конеч- конечномерные распределения которых задавались также двумя объектами — функцией среднего и ковариационной функцией. Возникает естественный вопрос, можно ли по переходной функции P(s, ж, ?, В) и мере QSo (на ^So, где so Е Т, Т С [so, оо)) построить марковский процесс X = {Xt, t G Т} с данной переходной функцией и Law(XSo) = QSo. Оказывается, что для борелевских пространств (Sj, S$t)teT ответ утвердительный, что устанавливается с помощью теоремы Колмогорова. Следующее замечание полезно с точки зрения терминологии, используемой в об- общей теории марковских процессов. Пусть X = {Xt,t Е [0,оо)} — марковский процесс с начальным распределением \i [р, — PXq на Зёо). Чтобы подчеркнуть это обстоятельство, распределение процес- процесса X, т. е. меру на цилиндрической сг-алгебре Й§[о,оо)? часто обозначают Р^. В част- частности, запись Рж, ж Е So, означает, что \± — Sx, иначе говоря, процесс в момент t = О "выпускается" из точки х (при этом считается, что ё$о содержит все одноточечные множества {ж}, х G So). § 12. Теперь мы рассмотрим один из наиболее важных подклассов марковских процессов. Определение 7. Марковский процесс X = {Xt,t G Т}, принимающий при каж- каждом t G Г С 1 значения в измеримом пространстве (S, Й§) и имеющий переходную функцию P(s, ж, ?, В), называется однородным, если P(s, ж, ?, В) при любых х G S, БЕЙ зависит лишь от разности ? — s. Для таких процессов вместо переходных функций P(s,x,s + u,B) достаточно опе- оперировать лишь с функциями Р(х, и, В) = Р@, ж, и, В). По своему смыслу Р(х, и, В) — это условная вероятность того, что, выходя из точки ж, через время и "система" окажется во множестве В. Чтобы сдвиги точки s G Т на величину и G Т не выводи- выводили из множества Т, обычно рассматриваются множества Т = [0, оо) или Т = {&А, к ^ 0},где А > 0. Согласно C3) m-мерное броуновское движение — это однородный марковский процесс. Пуассоновский процесс однороден тогда и только тогда, когда ведущая мера m(s,t] = X(t — s), A > 0, 0 ^ s < t < 00, т.е. процесс имеет постоянную интенсивность Л. Для однородного марковского процесса X = {Х^, ? ^ 0} условия 1)-4) на пере- переходную функцию Р перепишутся для s,? ^ 0,ж G S,?? G йв следующем виде: 1°) при фиксированных ж, t функция Р(ж, t, •) есть мера на Й§; 2°) при фиксированных ?, Б функция Р(-,t,B) e 3&\3&(Ж); 3°) Р(ж,0,Б) = ?ж(Б)прилюбыхж, Б; 4°) при всех х, s,twB справедливо уравнение Колмогорова-Чепмена: Р(х, s + t,B)= [ P(x, s, dy)P(y, t, В). C6)
196 А. В. Булинский, А. Н. Ширяев Отметим, что выполнение соотношения C6) при всех рассматриваемых значе- значениях аргументов лежит в основе применения в теории марковских процессов мощных методов теории полугрупп (см., например, [23]). В силу C2) однородность марковской цепи X = {Xt, t G Т} означает, что пе- переходные вероятности pij (s, t) при каждых г, j Е S зависят лишь от разности t — s. В этом случае вместо pij (s,s-\-u) пишут pij (и) (как уже отмечалось выше, рассмат- рассматривается Г = [0, оо) или Г = {/сД, к ^ 0}, А > 0). Пусть P(t) — матрица, состоящая из элементов Pij(t), i,j G S. Тогда уравнение B0) приобретает следующий вид: Pij(s + t) = ^2pik(s)Pkj(t), где i,j е S, C7) к то есть P(s + t) = P(s)P(t), s,teT. C8) Таким образом, с каждой однородной марковской цепью можно связать стохасти- стохастическую полугруппу матриц P(t) = (Pij(?))i,jes такую, что при всех t G Т Pij(t) > 0, ^Pijit) = 1, Pij@) = Si:j C9) j (последнее условие в C9) означает, что Р@) — I — единичная матрица). Если время дискретно (Т = Z+), то в однородном случае переходные вероятности за п шагов, т. е. р^ (п), находятся как элементы матрицы Рп, где Рп есть п-я степень матрицы Р = РA). С другой стороны, располагая стохастической полугруппой матриц (Pij(t))t^o, по теореме 5 всегда можно построить марковскую цепь с переходными вероятностя- вероятностями pij (t). Заметим, что изучение неоднородных марковских процессов может быть, в прин- принципе, сведено к исследованию однородных за счет расширения исходного фазового пространства (см., например, упражнение 18). § 13. В этом параграфе будут рассматриваться важные и интересные вопросы асимптотического поведения марковских цепей в предположении, что Т = [0, оо); переформулировки на случай дискретного множества Т не вызывают затруднений. Теорема 7 (эргодическая теорема). Пусть X = {Xt, t ^ 0} — однородная марковская цепь (с произвольным начальным распределением) такая, что для некоторого jo G S и некоторых h>0u0<5^1 Pijo(h) ^ ^ пРи всех i ? S. D0) Тогда для любого j G S существует и не зависит от начального состояния г G S предел причем для всех t > 0 |ру(*)-р,-|^A-^*//11, D2) где [•] — целая часть числа.
Гл. VI. Марковские процессы. Дискретное и непрерывное время 197 Наглядный смысл этого "эргодического" результата состоит в том, что за боль- большое время система, описываемая марковской цепью, как бы "забывает", из какого начального состояния она "стартовала" Доказательство. Обозначим для каждого t е T rrij (t) = inf pij (t), Mj (t) = sup pi j (t). 1 i Очевидно, ттг j(t) ^ Pij(t) ^ Mj(t) при всех i,j G Sut G Т. Для доказатель- доказательства свойства D1) достаточно показать, что rrijit)/* и Mj(t)\f при t —> оо и что Mj(t) — rrij(t) —> 0 при t —> оо. Оценка D2) будет вытекать из устанавливаемой далее оценки D3). Для s, t G Т имеем, учитывая C7), что TTlj (s + t) = inf ^2 Pik (s)Pkj (t) ^ mj (t) inf ^ Pik (s) = mj (*) j 1 к Ъ к Mj (S + t) = SUp ^ pi/c (s)pkj (t) ^ Mj (t) SUp ^ Pik (s) = Mj (*) • Тем самым, rrij (t)/* и Mj (t)\ при t —> oo. Далее, для t, /i, t — h G T, снова воспользовавшись свойством C7), находим, что Mj(t) - mj(t) = SUppaj(t) a b (j(t) -Pbt(t)) = sup^(pa/c(/i) -Pbk(h))pkj(t -h) = a,b a,b ?, ^ (Pak(h) -Pbk(h))pkj(t ~h) + ^2 (Pak(h) - Pbk(h))pkj(t - h) \ к к > M^t - h) ^2+(Pak(h) -Pbk(h)) + mj(t - h) ^~(Pak(h) -Pbk(h))\, к к где J^+ обозначает сумму по тем к, для которыхрак (h) —рьк (h) ^ 05 а ^~ — сумму /с /с по & таким, что pak(h) — Pbk(h) < 0- (Рассматриваемые суммы J^+ и J^~ зависят, ^ к к конечно, от значений ft, a и Ь. 1 Поскольку ^2pak(h) — ^Pbk(h) = 1, то ^ t. t. = 0. Поэтому u (PafcW -Pbk(h)). a,b
198 А. В. Булинский, А. Н. Ширяев Теперь заметим, что если "специфическое" состояние jo (см. D0)) не входит в J^+, к то в силу D0) J]+(Pafc(ft) -Pbk(h)) ^ J]+Pafc(ft) ^ l-PajQ(h) ^1-6; к к если же jo входит в J^+> т0> снова учитывая D0), получаем к к к Следовательно, Mj(t) - mj(t) ^ A - S)(Mj(t - ft) - rrij(t - ft)) ^ ^ A - SJ(Mj(t - 2ft) - mj(t - 2ft)) ^ где и — t — [^] ft. Но, очевидно, Mj(u) — rrij(u) ^ 1. Тем самым, для всякого t > 0 где 0 < й ^ 1. Отсюда, поскольку TTij(t)/1, a Mj(t)\ при t —>¦ оо, выводим, что существуют и равны (скажем, значению рЛ пределы lim Mj (t) и lim rrij (t), и при ЭТОМ mj(t) - Mj(t) ^ m^t) -pj ^ рф) -pj ^ Mj(t) -pj ^ Mj(t) - mj(t). Значит, IPij(t) -Pi| ^ Mj(t) - mj(t) ^ A - J)[*], что доказывает не только свойство D1), но и оценку D2), устанавливающую ско- скорость сходимости (при t —у оо) переходных вероятностей pij (t) к предельному значе- значению pj. ? § 14. Остановимся на некоторых следствиях доказанной эргодической теоремы. Следствие 3. Пусть для любых i^j G S справедливо соотношение D1). Тог- Тогда для всех j G S существует lim pj(t) =pj, D4) t—ь-оо где Pj(t) = P(Xt = j), причем если выполнено условие D0), mo
Гл. VI. Марковские процессы. Дискретное и непрерывное время 199 Доказательство. По формуле полной вероятности pj (t) = ^2pi(O)pij(t). Поэ- i тому в силу теоремы Лебега о мажорируемой сходимости для каждого j Е S (t) -Pj) ->U при i В предположении D0) что и доказывает искомое утверждение. ? Следствие 4. Пусть для любых i,j Е S справедливо соотношение D1). <5а прг/ каждом t ? Т и всех j Е S Pij(*). D5) Иначе говоря, вектор-столбец р = (po,Pi, • • • )* есть собственный вектор мат- матрицы P*(t), где * обозначает транспонирование. Доказательство. В силу D4) для любых j E S и N Е N рj = lim pj(s + t) = lim ^рДф^(г) ^ lim J^ Pi(s)pij(t) = s—юо s—>>oo что дает неравенство pj ^ ^2PiPij(t)- Допустим, что здесь для некоторых j E S и г ? ^ 0 имеет место строгое неравенство г Согласно D1) для любого N и каждого г имеем ^2 Pj = nm S Pij(t) ^ 1? поэ~ тому J ^ -^ J ^N j Тем самым, если D6) выполнено, то j j г i j i Полученное противоречие показывает, что D6) невозможно. ? Следствие 5. Пусть для любых i,j G S справедливо соотношение D1). Тог- Тогда либо ^2pj = 1, т. е. величины pj, j G S, задают распределение вероят- з _ ностей, называемое стационарным распределением, либо ^2pj = 0, т. е. все Pj = О- j
200 А. В. Булинский, А. Н. Ширяев Доказательство. Очевидно, что имеют место лишь две возможности: либо ^2pj = 0, либо ^2pj ф 0. Если ^2pj ф 0 (ряд сходится согласно D7)), то возьмем з з з Pi@) = pi /^2pjj г G S, и рассмотрим цепь с таким начальным распределением з и заданными pij(t), что возможно в силу теоремы 5. Тогда по формуле полной вероятности и следствию 4 для всех t ^ 0 и j G S T,PiPij(t) ^ ^ P j Z^ P j 3 3 Из D4) имеем pj = Pj(O). Следовательно, если ^2pj > 0, то J^pj = 1. П з з § 15. Приведенный выше термин "стационарное распределение" связан со сле- следующим понятием. Определение 8. Процесс X = {Xt, t G Т}, где Г С I, называется стацио- стационарным (строго стационарным или. стационарным в узком смысле), если для всех п ^ 1, ti,..., tn G Т и всех ft G 1 таких, что ti + h,..., tn + /i G T выполнено следующее свойство: Law(Xh+tl,..., Xh+tn) = Law(Xtl,..., Xt J. Иначе говоря, все конечномерные распределения такого процесса должны не меняться при сдвигах (не выводящих из параметрического множества Т; это заме- замечание существенно, если, например, Т = Z). Теорема 8. Пусть однородная марковская цепь X = {Xt, t ^ 0} имеет ста- стационарное распределение /i, определяемое набором {pj, j G S}. Пусть Y = {Yt, t ^ 0} — (однородная) марковская цепь, имеющая те же переходные вероят- вероятности, что X, и начальное распределение \i. Тогда процесс Y будет строго стационарным процессом. Доказательство. Прежде всего заметим, что существование процесса Y с дан- данными переходными вероятностями и начальным распределением \i следует из тео- теоремы 5. Из B4), записав Y вместо X, получим для 0 ^ t\ < ••• < tn, B^ С S, к = 1,... ,nnn ^ 1, что P(YtleB1,...,YtneBn) = = Yl где Pj(t) = P(Yt = j) = pj при всех j G S, t ^ 0 в силу D8). Требуемое в теореме утверждение о стационарности следует непосредственно из того, что Pij (s,t) = pij(t- s) = pij (s + h,t + h) для любых i,jGS,O^s^t,/i^ —s. ?
Гл. VI. Марковские процессы. Дискретное и непрерывное время 201 Подчеркнем, что при доказательстве теоремы 8 мы использовали не само соотно- соотношение D1), а лишь D8), т. е. наличие такого начального распределения \i = Law(Xo), которое не меняется во времени (иначе говоря, Law(Xt) = \i при всех t Е Т). Такое распределение \i называется инвариантной мерой процесса X = {Xt, t G Т}. § 16. Цель дальнейших рассмотрений — получение дифференциальных уравне- уравнений, которым (при некоторых условиях) удовлетворяют переходные вероятности од- однородной марковской цепи, заданной на [0, оо), и обсуждение приложений этих и ра- ранее установленных результатов. Для этого нам потребуются новые понятия. Определение 9. Однородная цепь Маркова X = {Xt, t ^ 0} (или порожденная ею полугруппа P(t) = (pij(t))) называется стандартной, если P(t) —у I при t —у 0+, т. е. для всех г, j Е S y(*) = <*«> D9) где, как обычно, Sij — символ Кронекера. Лемма 3. Для переходных вероятностей однородной марковской цепи X = = {Xt, t ^ 0} при всех i,jeSu любых ?, h ^ 0 справедливо неравенство \pij (t + h)- pij (t) | ^ 1 - pa (h). В частности, если цепь стандартна, то для каждых i,j G S функция Pij(t) равномерно непрерывна на [0,оо). Доказательство. С одной стороны, Pij(t + h) - Pi j{t) = ^2pik(h)pkj(t) -Pi jit) = k = Pij(t)(pu(h) - кфг = l-pu(h). кфг С другой стороны, pi jit + h) - pij(t) ^ Pij(t)(pu(h) - 1) ^ pu(h) - 1. Оба эти неравенства дают требуемое в утверждении леммы соотношение. ? Теорема 9. Если (P(t))t^o — стандартная стохастическая полугруппа, то существует правая производная = Q, E0) t=o т. e. существуют правые производные в нуле у всех переходных вероятностей d+Pij(t) При этом 0 ^ Qij < °° для г ф j и qi = —qu G [0, оо]. E2)
202 А. В. Булинский, А. Н. Ширяев Доказательство. Зафиксируем некоторое состояние i Е S и установим спра- справедливость E1) сначала в случае j = г, т.е. для переходных вероятностей рц{Ь). В силу условия D9) найдется S > 0 такое, что рц{К) > 0 при ft Е [0, S]. Для t > 0, применяя C7), получим, что pu(t) ^ (pu(t/n))n, п ^ 1. Взяв п = n(?, J) так, чтобы t/n < 5, видим, что pu(t) > 0 для всех t ^ 0. Поэтому на [0, оо) определена функция H(t) = -\npu(t). Из C7) имеемpu(s +1) ^ Pu(s)pu(t) дляз,? ^ 0, откуда H(s + t) ^H(s)+H(t) для s,te[0,oo). E3) Положим q = swpH(t)/t. E4) t>o Очевидно, g' G [0, оо]. Рассмотрим отдельно два случая: q < оо и g = оо. 1) Пусть q < оо. Тогда для любого е > 0 найдется to = to (г) > 0, такое, что H(to)/to ^ q — г. Для /i G @, to) представим to в следующем виде: to = nh + А, где n = [to/h], 0 ^ А < /i, a [•] —целая часть числа. Пользуясь E3), получаем q-e^ H(to)/to «S (пЯ(Л) + Я(Д))/*0 = ^ • ^ + ^^ • E5) a to to В силу D9) Я(А) -»> 0 при А -»> 0 (А -»> 0, если ft -»> 0+). Поэтому ^-г^Нтт?Я(Д)/Д. E6) Из E4) вытекает, что limsup H(h)/h ^ g'. Следовательно, существует и, значит, д= Шп /0 = lim = nm ft h->0+ ft /i^0+ ft что и доказывает (для г G S) соотношение E1) в случае q < оо. 2) Пусть теперь q = оо. Тогда для любого М > 0 найдется to = to(M) > 0 такое, что Н(to)/to ^ М. Действуя точно так же, как при доказательстве E6), получаем, что M. Таким образом, существует lim H(h)/h = оо. Учитывая E7), получаем соотно- шение E1) в случае q = оо. Снова фиксируя некоторое состояние г G S, установим теперь справедливость E1) для вероятностей pij (t) в случае j ф i. Обозначим /г-• ^(ft) условную вероятность (при условии Хо = г) перехода из состояния г в то же самое состояние г ровно за к последовательных шагов длины ft (и не за меньшее число шагов), но так, чтобы
Гл. VI. Марковские процессы. Дискретное и непрерывное время 203 при этом ни разу не побывать в состоянии j в моменты вида m/i, где т = 1,..., к. Покажем, что тогда для п ^ 2 справедливо неравенство п-1 h) > Yl к = 1 Действительно, учитывая замечание 2, получаем для переходных вероятностей Pi j (nh) следующую оценку снизу Pi j (nh) = ^2 P{Xnh = j, X{n_1)h = jn_i, ..., Xh = ji | Xo = г) ^ Jb---,Jn-l?S n-1 ^ X/ X/ X/ P(^^ = h X(n-i)h = jn-i, • • •, X(k+1)h = j/c+i | X/,^ = г) x k=l Jk = «j ^"(fc-i)Ai = J/c-i, • • •, Xh = ji \Xo = г), где Jk = {(ji, • • • jjk-i) '• jrn ? {hj}j m — 1? • • • •> к — 1} и мы воспользовались тем, что в силу A) P 1 Xn/l = j, Х^.!)^ = jn-i, • • •, Х/^ = г, ..., Xh = ii, Xo = г) = P(X0 = г) x P{Xnh = J, Х(п_1)/г = jn_b ..., X(k+1)h = J/c+i I Xkh = г, ..., Xo = г) х x Р(Х^=г,...,Х0 = г) = = P{Xnh =j, -.., X(jfe+1)^ = j/c+i \Xkh =i)P{Xkh = г, -.., Xh = ji |X0 =г). Для получения требуемого неравенства E8) надо теперь лишь учесть, что ,^ = г) = ( Обозначим Pij (h) условную вероятность перехода из состояния г в состояние j ровно за к шагов длины h (и не за меньшее число шагов). Аналогично E8) получаем для к ^ 1 fe0 = 0) к-1 J2 {™\{{k - m)h). E9) k-1 , , Очевидно, Yl Pij1 (h) ^ 1, поэтому m=l Щ (h) ^ Pu(kh) - max Pji((k - m)h) J l<m<k — l
204 А. В. Булинский, А. Н. Ширяев (максимум по пустому множеству считаем равным нулю). В силу D9) для любого г > 0 находим to = to(e, i,j) > 0 такое, что Pji(t) ^ г, Pii(t) ^1-e, Pjj(t) ^1-е при t e [0,t0]. F0) Тем самым, f\f{h)^l-2e при nh^to и /с = 1,... ,п - 1. F1) Замечая, чтоPij((n - k)h) ^ Pij(h)pjj((n - к - l)h) при к = 1,... ,п - 1 ип ^ 2, из E8)—F1) выводим, что п-1 Pij(nh) > A - 2e)Pij(h) J2 Рзз((п ~к~ !)Л) > (! " 2^)(! " ?){п - 2)рф). к=1 Отсюда, выбрав п = [to/h] и учитывая непрерывность Pij(t) (лемма 3), получим Устремив to к нулю, видим, что Поскольку г > 0 выбиралось любым, из этого неравенства заключаем, что при j ф г существует конечный предел qij = lim pij(t)/t. Теорема 9 доказана. П § 17. Пусть выполнено условие стандартности D9). Для любого N ^ 1 и t > 0, учитывая, что ^2Pij(t) = 1, получаем, что j Поскольку сумма в левой части этого неравенства содержит лишь конечное число членов, то полагая t 4- 0 и пользуясь существованием производных (см. E1)), нахо- находим, что ^2 Qij ^ Qi- Следовательно, эф* ^Q^Q F2) (При qi = оо утверждение F2) тривиально.) Определение 10. Цепь называется консервативной, если все элементы qij ин- финитезималъной матрицы Q конечны и при этом для любого i j = (n- (б3)
Гл. VI. Марковские процессы. Дискретное и непрерывное время 205 Примером консервативной цепи может служить процесс Пуассона с интенсивнос- интенсивностью Л > 0. Действительно, согласно B5) матрица -Л Л 0 ... = | 0 -Л Л и свойство F3), очевидно, справедливо. Теорема 10 (обратная система уравнений Колмогорова). Пусть однородная цепь Маркова консервативна. Тогда для всех t > 0 F4) т. е. при любых i,jeSut>0 существуют производные p'^t) и справедливы следующие (обратные) уравнения: F5) (В силу E1) и поскольку Pij(O) = Sij, уравнение F5) будет верно и при t = 0, если все производные понимать как правые производные.) Доказательство. Для ? ^ 0, /г > 0 и г, j G S положим 1 „ кфг Тогда Pij(t + h) - 7 h h Для всякого N ^ 1 из E1) заключаем, что F6) HmsupLij(M) ^ limsup J^ -^—Pkjif) = J^ QikPkj(t). k^N k^N Итак, имеет место следующая оценка снизу: lim sup Lij (h,t)^^2 VikPkj (t). F7) Учитывая, что Pkj(t) ^ 1 и ^2pik(h) — 1 Для всех hj ^ S и t ^ 0, h > 0, видим, /с что Lij(h, t) при N > i может быть оценено сверху величиной E Pik(h) T 2_^Pik(h) 2^ Г k>N кфг ^ /c^iV кфг
206 А. В. Булинский, А. Н. Ширяев Откуда, при h —> 0+ находим, что lim sup Lij(h,t) ^ V] qikPkj(t) - qu - У2 qik' Л">°+ кфг k^N k^N кфг Следовательно, в силу условия F3) lim sup Lij (h,t)^^2 QikPkj 0) + Qi ~ ^ qik = ^2 ^kPkj (*) • F8) h~>°+ кфг кфг кфг Обратимся теперь к формуле F6). Из нее и из E1), F7) и F8) очевидным образом вытекает справедливость уравнения F5) для правой производной. Аналогичным об- образом рассматривается вопрос о существовании левых производных функций pij (t) при t > 0 и выполнении для них уравнения F5). Тем самым теорема доказана. ? Теорема 11 (прямая система уравнений Колмогорова). Пусть для полугруппы (P(t))t^o однородной марковской цепи существует инфинитезималъная мат- матрица Q с конечными элементами qij, причем для всех г ф j Pij (h) = qij h + otij (h), F9) где равномерно по i aij(h)/h -^ 0 при h -> 0 + . G0) Тогда для всякого t > 0 существует P'{t) {т. е. существуют все производ- производные p'ijit), г, j e S) и P\t) = P(t)Q. G1) Иначе говоря, при t > 0 справедливы следующие (прямые) уравнения: Pij(t) = ^2Pik(t)qkj для всех iJeS. G2) к Доказательство. Пусть t ^ 0, h > 0 и г, j e S. Тогда G3) Для любого г > 0 при фиксированном j в силу F9) находим ho(e,j) > 0 такое, что \akj(h)\/h < г при всех к и 0 < h < ho(e,j), поэтому '? > Pik(t) ^e. G4) :—. 11 " . Из F9), G4) и того, что -rY\Pik{t)pkj(h) 2^Pik(t)Pkj(h) ^ ^^ , кфз
Гл. VI. Марковские процессы. Дискретное и непрерывное время 207 заключаем, что для каждого t ^ 0 и г Е S сходится ряд в правой части G2). Со- Совершив в G3) предельный переход при h —у 0+, получим, что G1) справедливо для правой производной. Аналогичным образом доказывается справедливость G1) и для левых производ- производных. Тем самым теорема доказана. ? § 18. Рассмотрим некоторые важные частные случаи. Замечание 3. Если пространство состояний S конечно и матрица P(t) стан- стандартна, то справедливы обе системы уравнений Колмогорова. Действительно, для любых i,jGSnt>0 и ее ли число состояний конечно, то теорема 9 обеспечивает консервативность. При этом выполнено условие F9), а условие G0), очевидно, имеет место, так как i при- принадлежит конечному множеству S. Названия обратная и прямая системы уравнений Колмогорова связаны с поло- положением матрицы Q в F4) и G1) ("перед" или "после" P(t)). Следствие 6. Пусть справедливы условия теоремы 11, и пусть для всех г, j Е S существуют пределы tl}*%oPij(t)=Pj G5) (например, условия теоремы 7 обеспечивают выполнение G5)). Тогда для лю- любого j ^2pkQkj=0, G6) k т. е. вектор-столбец р есть собственный вектор матрицы Q* (Q* обозначает транспонированную матрицу Q), отвечающий нулевому собственному значе- значению. Иначе говоря, Q*p = 0. Доказательство. Повторяя рассуждения, проведенные при доказательстве те- оремы 11, видим, чторяд^р/с(^)^/с^-,гдер/с(^) = P(Xt = k), сходится для каждого j, к причем при любом начальном распределении существует предел Если ^2jPj = 0, т.е. все pj = 0, то G6), очевидно, верно. Если ^2pj ф 0, то возьмем начальное распределение Pi@) = pi,i G S (см. следствие 5). Тогда соглас- согласно D8) Pj(t) =Pj- Следовательно, Pj(t) = 0, и требуемое соотношение G6) вытекает из G7). ?
208 А. В. Булинский, А. Н. Ширяев § 19. Рассмотрим теперь модель (ее происхождение объясним позднее), в которой S = {0,1,..., п}, а вероятности переходов Pij(h) при h —У 0+ имеют вид, изобра- изображенный на следующей "диаграмме переходов": n\ih + o(h) n- Xh + o(h) rib • - Xh + o(h) kfih + o(h) • Лп JU 1 / Д. v_ Рис АЛ- + o(/i) J . 17 Из этого рисунка видно, что возможны (с ненулевой интенсивностью) переходы только па одип шаг вправо или влево, а также разрешается оставаться па мес- месте] в "концевых точках" 0 и п допускаются переходы соответственно из 0 в 1 и из п в п — 1, при этом также можно оставаться на месте (Аи/i — положительные пара- параметры). Точнее говоря, пусть все остальные Pij(h) = o(h) при h —У 0+. Отметим, что существование марковской цепи с соответствующей инфинитези- мальной матрицей Q обеспечивает результат упражнения 36. Покажем, что выполнены условия эргодической теоремы 7. Возьмем jo = п. Для любого г = 0,..., п Pin(nt) Найдем to > 0 такое, что Pk-i,k(t) = At + o(t) > 0, к = 1,..., п (о( •) зависит от к) W-Pnn(t) > 1/2 при 0 < t ^ to- Выбрав h = to/n, t = to, получаем, что справедливо условие D0), и значит, существуют pj = lim Pij(t). t—too В силу замечания 3 выполнены обе системы Колмогорова, при этом, очевидно, ^2pj = 1, так как S конечно, a ^2pij(t) = 1. з з Найдем теперь стационарное распределение р. Согласно следствию б, для этого надо решить векторное уравнение Q*p = 0, т. е. систему /-А fJL A -(A + \ 0 А -(А + Аух) (k-\-l)fjL — (А + (n — А 0 \ —п\±) = 0.
Гл. VI. Марковские процессы. Дискретное и непрерывное время 209 С этой целью ко второй строке матрицы Q* прибавим первую, ..., к к-и. — все предыдущие строки B ^ к ^ п — 1). Тогда /-Л \ 0 -Л 2/i 0 \ —Л n\i А —тцл/ и значит, покоординатно, -Хрк + fi(k + l)pk+i = 0, к = 0,..., п - 1. Следовательно, p/c+i = рРк/(к + 1), где /? = A//i. Отсюда имеем р^ = ттРо, /с. и поскольку для решения выполняется условие ^ р& = 1,торо = ( S Р /к\) к=0 4=0 ^ Тем самым, справедливы следующие формулы: Рк = рк/к\ G8) называемые в "теории массового обслуживания" формулами Эрланга. § 20. Объясним, как естественно в прикладных вопросах возникает рассмот- рассмотренная выше модель с переходными вероятностями, указанными на рис. 17. Пусть имеется некоторая "система массового обслуживания" (СМО), состоя- состоящая из п одинаковых приборов. Пусть в эту систему поступает поток заявок на об- обслуживание. Будем предполагать, что заявки образуют пуассоновский поток, т.е. требования на обслуживание появляются в моменты ?i, ?i + ?2 ? • • • ? где {?& } порож- порождает пуассоновский процесс (см. теорему 2 главы II), считая, что в каждый из таких моментов поступает одно требование. Рассмотрим "систему с отказами", т.е. такую, что если все приборы заняты об- обслуживанием поступивших ранее заявок, то требование покидает систему (происхо- (происходит "отказ"). Так бывает, например, когда телефонная линия занята. Если имеется свободный прибор, то поступившее требование мгновенно начинает обслуживаться (можно рассмотреть различные дисциплины обслуживания, мы этого не делаем для простоты и при этом считаем несущественным, на какой из свободных одинаковых приборов попадет на обслуживание заявка; каждый прибор обслуживает только од- одну заявку). Естественно считать, что состояние системы в момент t — это число Xt занятых приборов. Уточним также характер обслуживания. Предположим, что время обслужива- обслуживания каждого требования представляет собой экспоненциально распределенную ве- величину г\ с параметром \i > 0. Кроме того, пусть каждое требование обслуживается независимо от других, и пусть случайные величины, описывающие длительности об- обслуживания, не зависят от последовательности {?/с}5 задающей поток требований.
210 А. В. Булинский, А. Н. Ширяев Обычно системы массового обслуживания обозначают а|Ь|с, где символ а указы- указывает распределение величин^ (по-прежнему, времена прихода заявок т^ =?]_ + •• •+ + ?д., где ?д. — независимые одинаково распределенные случайные величины, к ^ 1), Ъ обозначает распределение длительности обслуживания (все длительности — независимые одинаково распределенные величины, не зависящие от {?/с}M с — ко- количество приборов. При этом для экспоненциального распределения традиционно используется буква М. Таким образом, рассматриваемая нами система — это сис- система М\М\п. Запись G|G|c указывала бы, что не делается специальных предполо- предположений об используемых распределениях ("G" — general). Возможны также даль- дальнейшие обобщения, например, рассмотрение групповых заявок, когда в момент т^ приходит не одно требование и т. д. Если N = {Nt, t ^ 0} — пуассоновский процесс, построенный по последова- последовательности {?/с}5 и событие Am(t, t + ft) состоит в том, что на промежутке (?, t + ft] поступило ровно m требований, то P(Am(t, t + h)) = P(Nt+h -Nt=m) = -^-e~Xh , m = 0,1, Поэтому P(A0(t,t + h)) =e~Xh = l-\h + o(h) при h -+ 0+; P(Ai(?,? + /i)) =\he~Xh = \h + o(h) при h -+ 0+; P f |J Am(t, t + h) j = 1 - e~Xh - Xhe~Xh = o(h) при h -+ 0 + . Для времени обслуживания г\ плотность е~^х, х ^ 0, {о, \<о, (80) и, значит, для х,у ^ 0 Pn(z)dz е-К*+у) ' ~Хх P( > x). (81) Jy Pv(Z)dZ e У Тем самым вероятность того, что работающий прибор проработает еще время ж, не зависит от того, сколько этот прибор уже проработал (в классе распределений, об- обладающих плотностью, экспоненциальное распределение является единственным, удовлетворяющим условию (81)). Заметим, что ph = Р(?7 < ft) = 1 - e~^h = fih + o(h) при ft -»> 0 + . Каждый прибор работает независимо от других. Поэтому вероятность того, что из к работающих в момент t приборов ровно I закончат эту работу за время ft, есть
Гл. VI. Марковские процессы. Дискретное и непрерывное время 211 Clkplh(l - Ph)k~l ("схема Бернулли"). Следовательно, обозначив Bk^i(t,t + ft) со- событие "из к работающих приборов I закончат работу па промежутке времени (?, t + ft]", получим, что м(?, t + ft)) = fc(l - е-М)е o(ft))(l - (k - l)iih + o(h)) = o(h), ft^0+; (82) = 1 - A - Ayxft) -kfih + o(h) = o(ft), ft -^ 0 + . Событие Cij (?, ? + ft), состоящее в том, что система осуществила переход из состо- состояния г в состояние j на промежутке (?, t + ft], можно представить в следующем виде (опуская для простоты записи аргументы t и t + ft у рассматриваемых множеств). Если l^i^n — 1, то d,i+r = ArBifl\jAr+1Bi^\J---\jAr+iBij для r = l,...,n-i, Ci^i-r = Aoi^,r U AiBi^r+i U • • • U Ai-rBi^ для г = 1,..., г. Аналогичные выражения можно написать и для Со,к и Для Сп,/с5 & = 0,1,...,п. Пользуясь теперь независимостью событий {^4^} и {i^^} и применяя G9) и (82), приходим к формулам для pij (ft) = Р (Ci j (t, t+ft)), содержащимся на представленной выше "диаграмме переходов" Подчеркнем, что мы лишь описали наводящие соображения в пользу модели, при- приводящей к формулам Эрланга. Здесь же заметим, что мы столкнулись с важной и сложной задачей: можно ли вводить марковский процесс с помощью инфинитези- мальных характеристик (задавая матрицу Q и, например, решая системы Колмого- Колмогорова). Оказывается, ответ на этот вопрос совершенно нетривиален (см. дополнение к этой главе). § 21. В заключение этого раздела еще раз обратимся к формулам G8). Для к = п величинарп может интерпретироваться как вероятность потери требования в ста- стационарном режиме,т.е. через большое время вероятность того, что все п приборов будут заняты, окажется близкой к рп (следствие 3 обеспечивает экспоненциально быструю скорость сходимостиpj (t) Kpj). Следующая таблица (см. [19; с. 48, 49]) п = 2, п = 4, п = 6, Р = 0,3, Р = 0,6, Р = 0,9, Р2 Р4 Рб ~0 -0 -0 ,0335 ,0030 ,0003 показывает, что при увеличении коэффициента нагрузки р = Л/\± пропорциональ- пропорциональное увеличение количества обслуживающих приборов может существенно умень- уменьшить вероятность потери требования (в стационарном режиме). Название коэффи- коэффициент нагрузки для р связано с тем, что экспоненциально распределенная величина г\ (см. (80)) имеет Е r\ = 1/fi. Поэтому р есть отношение среднего времени обслужива- обслуживания одного требования к среднему времени между приходом заявок.
212 А. В. Булинский, А. Н. Ширяев Заметим также, что среднее число занятых приборов (в стационарном режиме) п равно J2 крк = рA -Рп)- k=i Отметим, что формулы Эрланга будут действовать и в более общей ситуации, ког- когда среднее время обслуживания одного требования по-прежнему равно l//i, но это время необязательно имеет экспоненциальное распределение (см. [66]). Дополнения и упражнения 1. Докажите, что определение A) равносильно тому, что для каждого s G Ти любых iGfs,5G <^>s выполнено равенство P(AC\Xt) = P(A\Xt)P(C\Xt). (83) Заметим, что, если (^s)seT — естественная фильтрация марковского процесса X = {Xt, t G Т}, то определение марковости в виде выполнимости свойства (83) имеет симметричный (по отношению к "прошлому" и "будущему") характер и означает условную независимость "прошлого" и "будущего" процесса X при фиксированном "настоящем" 2. Пусть X = {Xt, t G T} — марковский процесс, Т С 1. Объясните, поче- почему {Xt, t G С/}, где U С Т, также марковский процесс. В частности, если {Xt, t ^ 0} — марковский процесс, то для любого А > 0 процесс ХА = — {XkA, к ^ 0} будет марковским. Верно ли обратное утверждение? 3. Пусть X = {Xt, t ^ 0} — марковский процесс со значениями при каждом t в измеримом пространстве (S, 88). Пусть (V, ё) — измеримое пространство и ht: S —У V, ht G 8& | ё, t ^ 0. Докажите, что если ht для каждого t ^ 0 яв- является взаимно-однозначным отображением, то Y = {Y^ = ht(Xt), t ^ 0} также есть марковский процесс. Приведите пример, показывающий, что ес- если отказаться от условия взаимной однозначности отображения ht, то это утверждение может не выполняться. 4. Пусть X = {Xt, t ^ 0} — марковский процесс со значениями bS С I По- Положим Yt = [Xt], где [ • ] — целая часть числа. Можно ли утверждать, что процесс Y = {Yt,t ^ 0} является марковским? 5 (сравните с упражнением 3). Пусть W = {W(t) = (VFi (t),..., \УШ (?)), t ^ 0} {/ т ч 1/2 Хт (?) = I ^2 Wy, (t)) , 4=i у ? ^ 0 !> — процесс Бесселя, т.е. радиальная часть процесса W. Будет ли процесс Хш марковским? Отметим, что X\(t) = |VFi(t)| при t ^ 0, т = 1. 6. Пусть X = {Xt, t ^ 0} и 7 = {It5 ? ^ 0} — действительные марковские процессы. Будут ли процессы X + Y = {Xt + Yt, t ^ 0} и ХУ = {XtYt, t ^ 0} марковскими? Что можно сказать о частном случае, когда У^ = с(?), где c(t) — детерминированная функция? 7. Пусть {Х&, & = 0,1,... } — действительный марковский процесс. Положим для? G [fc, fc+1), fc = 0,1,..., т.е. рассмотрим
Гл. VI. Марковские процессы. Дискретное и непрерывное время 213 на [0, оо) непрерывную ломаную с узлами (к, Х&). Будет ли процесс X = {Х^, t ^ 0} марковским? Будет ли марковским процесс Y = {Yt = Х[ф t ^ 0}, где [ • ] — целая часть числа? 8. Пусть ?i, ^2? • • • — независимые одинаково распределенные случайные вели- величины, принимающие значения 1 и —1 с вероятностью 1/2. Положим So = О, Sn = ?Н Ь^п,^ ^ 1,и1п = max S&. Докажите, что процесс X = {Хп, п ^ 0} не является марковским. 9. Приведите пример мартингала, не являющегося марковским процессом. Постройте пример действительного марковского процесса X = {Xt, t G Г} с Е \Xf | < оо при всех t G Т, который не является мартингалом. 10. Пусть ?7, ?i, ^2, • • • — независимые величины, причем ?i, ?г? • • • равномерно распределены на [0,1], a rj — действительная случайная величина с функцией распределения Fr](x). Положим Sn = 1, если ?п ^ 77, и 5П = —1, если ?п > г] (п = 1,2,...). Будет ли {5n, n ^ 1} однородным марковским процессом? 11. Верно ли, что действительный гауссовский процесс X = {Xt,t ^ 0} является марковским, если условие A2) выполнено лишь для функции /(ж) = ж, х G Ш 12. Докажите, что действительный гауссовский процесс X = {Xt, t G Т}, ТсМ, будет марковским тогда и только тогда, когда для любых t\ < ?2 < ?3 (tijt2jts G Т) справедливо равенство где r(s, t) = cov(Xs, Xt), s, ? G Т. Будет ли марковским процессом фракталь- фрактальное броуновское движение? 13. Является ли процесс Орнштейна-Уленбека (определение 13 главы III) од- однородным марковским процессом? 14. Пусть Р(Хо = 1) = Р(Хо = —1) = 1/2, причем Хо не зависит от пуассо- новского процесса N = {Nt, t ^ 0} интенсивности Л > 0. Пусть процесс X = {Х^, t ^ 0} меняет свое значение на значение противоположного знака в точках скачков пуассоновского процесса. Будет ли процесс X, называемый телеграфным сигналом, марковским? Нарисуйте график траектории этого процесса. 15. Для х ^ 0 пусть тх = inf{? ^ 0: W(t) = ж}, где W = {W(t), t ^ 0} — винеровский процесс. Докажите, что процесс {тж, х ^ 0} есть процесс с неза- независимыми приращениями (следовательно, марковский по теореме 3). 16. Пусть X = {Х^, ? ^ 0} — марковский процесс. Для s > 0 определим процесс У = {It = Xs-tj t G [0, s]}. Будет ли У марковским процессом? Обязан ли процесс Y быть однородным процессом, если таковым является X? 17. Докажите, что броуновский мост (см. определение в §9 главы V) — это не- неоднородный марковский процесс.
214 А. В. Булинский, А. Н. Ширяев 18. Пусть X = {Xt,t ^ 0} — случайный процесс на [0, оо), и Докажите, что процессы X и Y могут быть марковскими только одновремен- одновременно. Докажите, что если Y — марковский процесс, то этот процесс обязатель- обязательно является однородным. Как связаны переходные функции марковских про- процессов 1иУ? Интересно отметить, что к одним и тем же конечномерным распределениям мар- марковского процесса могут приводить разные переходные функции, как показывает Пример 3. Пусть S = М, 88 = 88(Ж), Т = [0, оо). Возьмем начальное распреде- распределение Q = So (мера Дирака, сосредоточенная в точке 0). Для ж Gt,t^0,S введем функции P(x,t,B) = 1в(х), P(x,t,B) = где sgnx определяется согласно (IV. 16). Легко видеть, что Р и Р обладают свойствами 1°)-4°), приведенными в § 12, и, следовательно, являются переходными функциями. Пользуясь формулой C5), полу- получаем, что РиР приводят к конечномерным распределениям процесса, тождественно равного нулю. 19. Пусть X = {Xt, t ^ 0} — однородный марковский процесс со значениями (при каждом t) в некотором польском пространстве S. Докажите, что су- существуют измеримые функции hs : S х [0,1] ->• S, s ^ 0 (т.е. hs G 88(S) (g) Й§([0,1]) | 5§(S), s ^ 0) и равномерно распределенные величины #t,s5 незави- независящие от {Хи, 0 ^ и ^ ?} приt, s ^ 0, такие, что Xt+S — hs(Xtj0t:S) П-Н- для всех ?, s ^ 0. 20. Пусть однородный марковский процесс принимает значения в Ж171 и его пере- переходная функция такова, что Р(х, ?, В) = Р(х + y,t,B + у) для всех ж, у G Ж171, t ) 0, 5 G ^(Mm). Докажите, что тогда этот процесс имеет независимые приращения. 21. Докажите, что для стандартного пуассоновского процесса интенсивности Л > 0 имеет место "усиленный закон больших чисел": N(t)/t —у Л п.н. при t -У оо. 22 ("парадокс времени ожидания"). Пусть пуассоновский процесс N = {Nt, t ^ 0} строится согласно теореме 2 главы II по последовательности {?j } неза- независимых экспоненциальных (с параметром Л) случайных величин. Найдите Nt + 1 при фиксированном t > 0 распределение случайной величины Ct = ^2 ?>j —t, j=i т. е. величины "перескока" за уровень t накапливающихся частных сумм Sk = к = Е О''" ^ 1- С этой задачей связан следующий занятный вопрос. Пусть ?j — это случайные интервалы между временами прихода автобусов на дан- данную остановку. Спрашивается, сколько времени Вы будете ждать появления ближайшего автобуса, если оказались на остановке в момент ??
Гл. VI. Марковские процессы. Дискретное и непрерывное время 215 23 (продолжение упражнения 22). Докажите, что для каждого фиксированно- фиксированного t > 0 введенные выше величины Ct ? ?iv*+2j ?л^+з?... образуют последо- последовательность независимых экспоненциально распределенных случайных вели- величин с параметром Л. В связи с пуассоновским процессом обсудим более детально элементарную кон- конструкцию точечного случайного процесса без кратных точек. Пусть на некотором вероятностном пространстве (П, 3*, Р) задана последователь- последовательность случайных величин {т^}к^1 таких, что для и Е По, где Р(По) = 1? О < т\ < Т2 < ... и тп —У оо при п —У оо. (84) Определение 11. Точечная случайная мера /а: Й§(М_|_) х П —у Ъ+ — Z+U {оо} задается формулой оо = ХХ(а;)(В), 5G«(I+), 00 СП, (85) к=1 где последовательность {тк}к^1 описана выше, 8Х — мера Дирака. При каждом фиксированном и Е По, очевидно, /i( •, uS) есть целочисленная мера на Й§(М_|_). Условие (84) влечет, что /i( •, о;) для каждого w G По является мерой Радона, т.е. мерой, конечной на компактных подмножествах М+, а, следовательно, /i( •, о;) есть сг-конечная мера на ^(М+) для этих о;. Заметим, что приведенная конструкция дает случайную точечную меру без крат- кратных точек, т. е. меру, для которой /a({t},uj) ^ 1 при всех t G M+ и и; G По- Определение 12. Точечный случайный процесс Y = {Y(t), t ^ 0}, где У @) = 0, вводится для t > 0 следующим образом: оо У(*,и) =/х(@,М = ^l@,t](^H), (86) где точечная случайная мера \i дается формулой (85). Очевидно, {Y(t) ^ п} = {rn ^ t} для t G M+ и n G Z+ = {0,1,...}, поэтому Y(t) G 3*\8i для каждого t G M+, где «с/ — сг-алгебра, состоящая из всевозможных конечных или счетных подмножеств Z+. 24. Определим т(В) = Е fi(B, •) для всех В G Й§(М+), где /iE, •) фигурирует в (85). Будет ли функция т{ •) мерой на ^(М+)? Можно ли представить в ви- виде (86) пуассоновский процесс с локально конечной мерой интенсивности т? 25 (сравните с теоремой 14 главы V). Докажите, что точечный случайный про- процесс (86) является пуассоновским, если Y имеет независимые приращения и Y стохастически непрерывен на М+ (см. A.56)). 26. По теореме 3 величины т& = ?i + ••• + ?&, fc E N, образуют цепь Маркова, если слагаемые — независимые действительные случайные величины. Будет ли марковским процесс (86), если {т^, k E N} —произвольная цепь Маркова, удовлетворяющая условию (84)?
216 А. В. Булинский, А. Н. Ширяев Определение 13. Пусть на некотором вероятностном пространстве (П, ^, Р) за- заданы пуассоновский процесс N = {7V(t),t ^ 0} интенсивности Л = 1 и независящий от него процесс Л = {Л(?), t ^ 0} с п.н. неубывающими, конечными и непрерывными справа траекториями, выходящими из нуля. Процессом Кокса называется процесс Z = {Z(t),t ^ 0}, где Z(t,uj) =N(k(t,u)),u)),t ^ 0, ш е п. 27. Является ли процесс Кокса марковским? Определение 14. Будем говорить, что случайный процесс X = {X(t), t ^ 0} является "дробовым шумом" если (87) где ограниченная борелевская функция ф: Ж —у Ж финитна, случайные величины Т& (k G N) удовлетворяют условию (84) (обеспечивающему, что ряд (87) сходится при каждом t ^ 0 на множестве По вероятности единица). Заметим, что в литературе встречаются и другие варианты процесса "дробового шума" Например, в сумму (87) вводятся случайные амплитуды и масштабные мно- множители. Рассматриваются также случайные поля дробового шума, заданные на Жс1. 28. Найдите ковариационную функцию процесса (87), если последовательность {тк }к ^ 1 задает точечный пуассоновский процесс интенсивности Л. Достаточ- Достаточно ли для сходимости ряда (87) с вероятностью единица при каждом t ^ 0 потребовать, чтобы ограниченная функция ф была интегрируема по Лебегу? Дадим определение маркированного точечного процесса, являющегося естест- естественным обобщением введенного выше понятия "точечного случайного процесса" Пусть кроме последовательности {rk}k^i случайных величин, удовлетворяющих условию (84), на том же вероятностном пространстве (П, ^, Р) имеется некоторая последовательность {r]k}k^i действительных случайных величин, "привязанная" к моментам т&, fc ^ 1. Например, можно себе вообразить, что в случайный момент времени Т& страховая компания выплачивает некоторую случайную сумму rj^- Интерпретируя величины rj^, k ^ 1, как "марки", "метки", дадим следующее Определение 15. Маркированным точечным случайным процессом называется процесс (индексированный множествами С) следующего вида к = 1 Функция Y(t,D;u)=ji(@,i\xD;u;), t ^ 0, De называется считающей функцией (сравните с (86)). Согласно (85) имеем j) = Ji(B x Ж;и).
Гл. VI. Марковские процессы. Дискретное и непрерывное время 217 Кроме того, {О () Y(t) ? lD(Vk) к=1 где процесс Y задается формулой (86). 29. Нарисуйте график траектории Y(t,D),t ^ 0, при фиксированном D Е 30. Пусть N = {N(t), t ^ 0} — пуассоновский процесс с ведущей мерой т = = га(-), определяемый последовательностью {т^}к^ъ задающей точки скачков N. Пусть {f]k}k^i — последовательность независимых одинаково распределенных действительных случайных величин с Law^/c) = Р. Возь- Возьмем!) Е Й§(М) такое, что P(-D) > 0. Докажите, что "прореженный процесс" Y(t,D), t ^ 0, определенный формулой (88), является пуассоновским с ве- ведущей мерой тп( • )P(D). О точечных случайных процессах и полях см., например, [111, 178]; специально о маркированных процессах см. [156]. Дадим наглядное представление о связи инфинитезимальной матрицы Q (стан- (стандартной) марковской цепи X = {X(t), t ^ 0} с поведением самой цепи. Пусть qi := — qu < оо для всех i G S. (89) Процесс X является стохастически непрерывным на всей полупрямой [0, оо) и, согласно теореме 13 главы I, он имеет сепарабелъную модификацию (с любым счетным множеством сепарабельности R С [0, оо)), которую мы и будем рассмат- рассматривать. Пусть в момент s ^ 0 процесс находится в состоянии i с вероятностью P(X(s) = г) ф 0. Тогда для любого t > 0 (учитывая сепарабельность процесса X) имеем Р(Х(и) =i, s ^u^s + t\ X(s) =i) = \ T- lim P(X(u)=i, = I) П-5-OO V = n/v,\T lim P(X(S) = I) П-5-OO = lim (Pii(t2-n)Jn = exp{-git}, (90) n—>-oo поскольку из определения qi получаем Pu(h) = 1 — q%h + o(h) при h —> 0 + . Если t = 0, то левая часть формулы (90) равна 1, поэтому данное соотношение спра- справедливо для всех t ^ 0. Левая часть (90) представляет собой вероятность пребывания процесса X пос- после момента s в состоянии i в течение времени, не меньшего ?, при условии, что в момент s процесс находился в состоянии i. Таким образом, результат не зависит от s, как и следовало ожидать для однородного процесса. Поэтому можно говорить о длительности пребывания процесса X в состоянии г, начиная с произ- произвольного момента времени.
218 А. В. Булинский, А. Н. Ширяев Итак, доказана (сравните с теоремой 2 главы II) Теорема 12. Пусть однородная марковская цепь X = {X(t), t ^ 0} удовле- удовлетворяет условию (89). Тогда условная длительность пребывания этого про- процесса в любом состоянии г имеет показательное распределение с парамет- параметром qi. Для марковских цепей, как уже упоминалось в § 1, важным представляется вопрос 0 классификации состояний (возвратные, периодические состояния и т. д.). Для це- цепей Маркова с дискретным временем об этом можно прочитать, например, в книгах [85,101,78]. Определение 16. Состояние г, для которого 0 ^ qi < оо, называется устойчи- устойчивым. Оно называется поглощающим, когда qi = 0. Состояние i называется мгно- мгновенным, если qi = оо. Если процесс попадает в поглощающее состояние, то он остается там "навсегда" (это следует из (90) при qi — 0). Происхождение названия "мгновенное состояние" объясняет упражнение 31. Пусть в состоянии г значение qi = оо. Тогда вероятность пребывания про- процесса X в этом состоянии положительное время равна нулю, т. е. попав в это состояние, процесс мгновенно его покидает. Теория цепей Маркова, имеющих мгновенные состояния, чрезвычайно сложна. Интересно отметить, что существуют примеры однородных цепей, все состояния ко- которых мгновенны (см. [21]). Пусть теперь процесс X консервативен (см. F3)). Тогда при qi ф 0 величины Qij/Qii 3 Ф h можно интерпретировать как интенсивности вероятностей перехо- переходов из состояния i в состояние j. Точнее говоря, пусть для j ф г, t > О Fij(t) := P{X(s + t)=j\ X(s) = i, X(s + 1)фг). Легко видеть, что Fij(t) = Pij(t)/A - pu(t)) ^ qij/qi при t->0 + . (91) ЕслиО < qi < oon ^2 qij < ^,то1— J^ qij/qiMOжпoпoшшдLTь как интенсивность вероятности "ухода процесса на бесконечность" Данная выше интерпретация элементов матрицы Q делает естественным следую- следующий способ построения марковских цепей, предложенный Дж. Дубом. Пусть S — конечное или счетное множество, и пусть X = {X(t), t ^ 0} — кон- консервативная марковская цепь с инфинитезимальной матрицей Q, причем qi G @, оо), 1 G S. Построим рекуррентным образом последовательности случайных величин {Vn}n^i и {rn}n^i со значениями в S и М+ = [0, оо) соответственно и так, что 771 имеет произвольное распределение, P(r1>t\r]1=i)=e~qi\ ?>0, г е S, (92) иприп ^ 1, i,j,ib...,in e S, жь...,жп е М+ пусть P( j \ri=x1,.. .,тп=хп,гц=п,... ,rjn-i=in-i,rin=i) = qij/qi, г Ф j, i -rn>t \т1=х1,.. .,тп=хп,гц=п,... ,rjn=in,rin+i=j) = e~q^. (93)
Гл. VI. Марковские процессы. Дискретное и непрерывное время 219 32. Докажите, что на подходящем вероятностном пространстве (П, ^, Р) сущест- существуют описанные выше последовательности случайных величин {r]n}n^i и {Tn}n^ij удовлетворяющие условиям (92) и (93). При этом 0 = то < т\(и) < < T2(uj) < • • • п.н. Можно ли утверждать, что всегда lim тп = оо п.н.? п—>>оо Докажите, что последнее заведомо справедливо, если sup qi < оо. г 33. Пусть tq(uj) = 0 п.н. Используя случайные величины rjn, тп (п ^ 1), фи- фигурирующие в упражнении 32, и считая выполненным условие (84), введем процесс F = {F(t),t^0}, положив Y(t,Lj) =7}n(u) При Tn_i(cj) ^ t < Tn(u), П^1 (для uj таких, что rn-\(uS) = rn{uj) при некотором п ^ 1, пусть Y(t,u) = О, t ^ 0). Докажите, что Y — марковский процесс, имеющий те же конечно- конечномерные распределения, что и процесс X, который определял матрицу Q. Как модифицировать приведенное построение, если некоторые величины qi равны нулю? В связи с последними двумя упражнениями заслуживает внимания следующее уп- упражнение. 34. Докажите, что sup^ < оо тогда и только тогда, когда pu(t) —у 1 при г t —у 0+ равномерно по г G S (или, что эквивалентно, Pij(t) —У Sij при t —у 0+ равномерно по г, j G S). 35. Приведите пример переходных вероятностей, для которых не выполнено усло- условие стандартности D9). Приведите пример, когда выполнено свойство D9), а (89) не имеет места. В упражнении 33 указано, как, располагая консервативной инфинитезимальной матрицей Q, построить марковский процесс, имеющий полугруппу переходных ве- вероятностей с инфинитезимальной матрицей Q. Другая постановка этой задачи — восстановление полугруппы (P(t))t^o п0 ее генератору Q (см., например, [23]). Если нам удастся построить стохастическую полугруппу (P(t))t^o? имеющую ин- инфинитезимальной матрицей заданную матрицу Q, то дальше теорема 5 обеспечит построение самого процесса с переходной матрицей P(t) (даже семейства марков- марковских процессов, поскольку можно варьировать начальное распределение). В этой связи предлагается упражнение 36. Пусть пространство состояний S состоит из конечного числа N элементов, а переходная матрица P{t) стандартна. Докажите, что тогда эта матрица допускает следующее представление: P(i) = exp{tQ}, t 2 0, (94) где N х TV-матрица Q удовлетворяет условию qij ^ 0 для гф j и 2_^ Qi j = 0 ПРИ каждом г. (95) з Обратно, если для матрицы Q выполнены условия (95), то формула (94) опре- определяет стандартную матрицу переходных вероятностей.
220 А. В. Булинский, А. Н. Ширяев Рассмотрим теперь более общую матрицу Q = (qij)ijes c действительными эле- элементами, где S счетно, такую, что qij ^ 0 при г ф j, qi = -qu ^ 0, и ^qij ^ 0 для г G S. (96) з Поставим следующий вопрос: существует ли стохастическая полугруппа имеющая своей инфинитезимальной матрицей матрицу Q? Оказалось, что искать решение этой задачи удобно сначала в классе полустохас- полустохастических матриц (P(t))t^Oj Для которых, по-прежнему, выполнено уравнение C7), а в C9) вместо условия ^2pij(t) = 1, ? ^ 0, г Е S, налагается менее ограничитель- з ное требование ^2pij(t) ^ 1, ? ^ 0, г Е S. Решение такой задачи часто называют з Q-решением, подразумевая следующее. Если окажется, что J2pij(t) = 1 для всех з t ^ 0, г Е S, то говорят о собственном решении (или процессе), а если выполняется лишь неравенство ^2 pij (t) ^ l,t ^ 0, г Е S,to — о несобственном решении. Впер- з вом случае действительно существует марковская цепь (разумеется, можно менять начальное распределение) с переходными вероятностями р^(t), t ^ 0, г, j G S. Во втором же случае можно построить соответствующий марковский процесс X, рас- расширив пространство состояний S еще дополнительной точкой, скажем, оо и опреде- определяя для него переходные вероятностиp..(t) следующим образом: Pij(t) = pij(t) при iJeS и Poo,j(t) = <$oo,j, PiiOO(t) = l В связи с вышеизложенным рассмотрим проблему существования и единствен- единственности решений систем (прямых и обратных) уравнений Колмогорова, исследовав- исследовавшуюся многими авторами с помощью разнообразной техники. В. Феллер использо- использовал преобразование Лапласа. Г. Ройтер и В. Ледерман применяли метод, основан- основанный на приближениях матрицы Q "усеченными матрицами" Т. Като опирался на теорию возмущений (подробнее об этом см., например, [1]). Интересно отметить, что сами уравнения F5) и G2) можно понимать либо в узком смысле (т. е. что все р1^ (?) непрерывны при t ^ 0 и изучаемые уравнения справедли- справедливы при всех t ^ 0), либо в широком смысле, т.е. что Pij(t) абсолютно непрерывны и рассматриваемые уравнения верны для почти всех (по мере Лебега) t ^ 0. 37. Докажите, что для систем уравнений Колмогорова обе данные выше интер- интерпретации (в узком и широком смыслах) эквивалентны. Для прямых уравне- уравнений это упражнение надо отнести к числу весьма трудных. Теорема 13. Пусть выполнено условие (96). Тогда существует Q-решение, удовлетворяющее как обратной, так и прямой системам Колмогорова. Кро- Кроме того, обратное уравнение F4) имеет в классе полустохастических матриц единственное собственное Q-решение тогда и только тогда, когда то же са- самое верно для прямого уравнения G1). Подробное доказательство можно прочитать, например, в [1].
Гл. VI. Марковские процессы. Дискретное и непрерывное время 221 Теорема 14 ([180]). Пусть матрица Q, удовлетворяющая условию (96), кон- консервативна. Тогда любое из следующих двух условий необходимо и достаточно для существования единственного Q-решения. 1. Для некоторого Л > 0 единственным ограниченным решением уравнения (Q - \1)х = 0 является х = @,0...)*, а ограниченность решения означает, что sup \xi\ < оо. г 2. Для некоторого Л > 0 единственным неотрицательным ограниченным решением х = (xi,?2,... )*, где все xi ^ 0, указанного уравнения является х = @,0, •••)"¦ 38 (см. также упражнение 34). Докажите, что если sup qi < оо, то обе системы г уравнений Колмогорова имеют единственное решение. Интересно отметить, что, как показывает нижеследующая теорема, проблема су- существования и единственности решения уравнений Колмогорова тесно связана со свойствами траекторий марковской цепи. Теорема 15 (см. [82; с. 341]). Пусть все qi < оо. Тогда для того, чтобы были справедливы и система прямых, и система обратных уравнений Колмогорова, необходимо и достаточно, чтобы почти все траектории обладали следующим свойством: если X(s,lo) —> оо при s —> t с одной стороны (слева или справа), то X(s,uj) ->• оо при s ->• t с обеих сторон. Единственность и неединственность решений уравнений Колмогорова удобно рас- рассмотреть на примерах процессов рождения и гибели. Определение 17. Однородный марковский процесс X = {X(t), t ^ 0} с прост- пространством состояний S = {0,1,... } называется процессом рождения и гибели, если его инфинитезимальная матрица Q имеет следующий вид Яг,г-1 = Дг, Qii = ~(Аг + Hi), Qi,i+1 = A», qij = 0 при \l - j\ > 1, где /io = 0, \±i > 0, i ^ 1 и Xi > 0, i ^ 0. Если \±i — 0 для всех i ^ 0, то говорят о процессе рождения, а если А^ = 0, i ^ 0, то — о процессе гибели. 39. Пусть процесс рождения таков, что ^2 Х~ < оо. Докажите, что тогда систе- г ма прямых уравнений Колмогорова имеет единственное решение, а систе- система обратных — бесконечно много. Докажите, что если для процесса гибели ^211^ < оо, то система обратных уравнений имеет единственное решение, г а система прямых — бесконечно много, среди которых ровно одно собствен- собственное. Чтобы сформулировать общий результат о процессах рождения и гибели, введем некоторые обозначения. Положим тго = 1, тгп = (A0Ai... An_i)/(/ii/i2 • • • A^n), n e N, ОО П ОО 71 — 1 ОО п=1 г=1 п=2 г=0 n=0
222 А. В. Булинский, А. Н. Ширяев Теорема 16 (Ледерман-Ройтер). Пусть матрица Q консервативна и удов- удовлетворяет условию (96). Тогда 1. Если R = оо, то существует ровно одно Q-решение, оно собственное и удовлетворяет системе прямых уравнений. 2. Если R < оо и S = оо, то имеется бесконечно много Q-решений. Толь- Только одно из них удовлетворяет системе прямых уравнений, но это решение несобственное. 3. Если R < оо и S < оо, что эквивалентно условию Т < оо, то существует бесконечно много Q-решений, удовлетворяющих системе прямых уравнений. Ровно одно из этих решений является собственным. Выше прямые и обратные уравнения Колмогорова рассматривались лишь для марковских цепей, для которых (по определению) пространство состояний S дис- дискретно. В этой связи отметим, что в классической работе 1931 года А. Н. Колмого- Колмогорова "Об аналитических методах в теории вероятностей" (см. [33]) прямые и обрат- обратные уравнения даются и для более общего пространства состояний, в частности, для S = M,d. При этом соответствующие прямые и обратные уравнения будут уже урав- уравнениями в частных производных. (Некоторый класс таких прямых уравнений был ранее рассмотрен в физике Фоккером и Планком.) Об этом направлении исследова- исследований, приведшему к моделям диффузионных процессов, можно прочитать, например, в [12, 192, 76]. В главе VIII мы рассмотрим конструкцию диффузионных процессов как решений стохастических дифференциальных уравнений. Наряду с понятием марковского процесса, в общей теории важным является по- понятие марковского семейства случайных процессов. Пусть (St, S$t)teT — борелевские пространства, Тс1и P(s, x, t, В) — переход- переходная функция. Возьмем произвольное s G Гив качестве начального распределения выберем меру Qs( •) = Sx(-), где х Е Ss. На некотором вероятностном пространстве существует марковский процесс XS>X = {X°'X, teTs:=[s,oo)nT}, имеющий заданную переходную функцию и такой, что Хрх = х п.н. Этот про- процесс XsIх с помощью теоремы Колмогорова может быть непосредственно задан (см. A.42)) на пространстве (St3,$t3), снабженном вероятностью Qs,x = Law(Xs'x). Переопределим процесс Xs'х на п = St формулой Г/'» := X°>x(nT}Tstu), tET8, o>gSt, (97) где тгт Т обозначает отображение сужения функции с множества Т на множество Ts. Введем сг-алгебру 3*^3 '-— ^т т ^Ts и зададим на ней меру Рs,x — Qs,x^t t • Нетрудно видеть, что Ys^ = {Yts'x, t е Ts }- случайный процесс на (п, ^s, Ps,x)- Теорема 17. Для каждых s G Г и х G Ss процесс Ys^x = {Y^x{u), t G Ts, и G Щ будет марковским на (п, &^s, Ps,x) с переходной функцией P{s,x,t, В). Кроме того, У/'ж = х (п. н.) по мере Ps,x-
Гл. VI. Марковские процессы. Дискретное и непрерывное время 223 Доказательство. Равенство У"/'х = х очевидно, поскольку Qs = Sx. Прове- Проверим, что для любых точек и и t из Т [и ^ t) и В G S&t Pa,x(Yta>x G B\Y?*) = P{u,Y*>x,t,B) (Р^-п.н.). Для этого достаточно убедиться в том, что для любого D G ё%и Ps,x(YtS'X € B,Y°'X e D) = f P{u,Y^x,t,B)dPs,x. (98) Пользуясь марковостью процесса Xs'x на пространстве (St3 , ?$т3, Qs,x), имеем [ Р(и, Y°'x, t, В) dPS:X = f P(u, Xsu>x, t, B) dQs,x = = Qs,x(x?'x e в, xsu'x €D) = Ps,x(Yt'x e в, Y*>x e D), что и доказывает (98). Марковость процесса Ys'x, т.е. свойство A2), проверяется аналогично. ? Отметим, что довольно сложное обозначение (97) для процесса Ys'x было введено для удобства применения формулы A.23) при переходе к интегрированию с одного пространства на другое. Однако очевидно, что Yts'x(uj) = u(t) для teTs, и; G П. По-другому, можно сказать, что у нас имеется один непосредственно заданный про- процесс Y = {Y(t), t G Г}, т.е. Y(t,u) = u(t), t G Г, и; G П = St, который может рассматриваться для разных s из Т при t ^ s (t G Т). Этот процесс, как процесс на Ts и (П, ^^s, Ps,ccM будет марковским с заданной переходной функцией, причем в момент s он начинается из точки х G Ss (Р5)Ж-п.н.). Заметим, что обозначение &^s полностью согласуется с использованным ранее, поскольку сг-алгебра &^s порождается семейством координатных отображений тгт,г пространства St при t G Ts. Иначе говоря, &^s = o-{Yt, t G Ts}. Определение 18. Введенные на вероятностном пространстве (St,^t,Ps,cc) марковские процессы Ys>x называются марковским семейством. Поскольку 3*^,8 С ?%т для каждого s G Т, то меру Ps,x можем продолжить на 3* — ?%т нулем для тех множеств из S&Ti которые не входят в 3^s. Итак, появляется возможность на одном и том же измеримом пространстве (П, 3), задавая разные ме- меры Ps,x Для s G Т, х G Ss, "выпускать" марковские процессы в момент s из точки х (Р5)Ж-п.н.), причем у всех процессов этого семейства Ys>x = {У/'ж, t G Ts} будет одна и та же заданная переходная функция. Легко прояснить смысл переходной функции для марковского семейства: Ps,x(Yt'X ^B) =P(s,x,t,B) Р,,ж-п.н, где s,teT (s^t), x G Ss, В G 0Bt. Мы воспользовались здесь тем, что Р(^ G В \ rf) = P(? G В) п.н., если г] = с п.н., где с = const. Определение марковости A) и другие рассмотренные нами определения можно обобщать в различных направлениях. Например, в монографии [23] дается опреде- определение марковского процесса, время жизни которого случайно, т. е. траектории могут обрываться в случайный момент времени (наглядно можно представить частицу, которая "сгорела"). Чтобы наметить путь дальнейших обобщений понятия марко- марковости на случайные поля нам понадобится следующее определение.
224 А. В. Булинский, А. Н. Ширяев Определение 19. Пусть дано вероятностное пространство (П, ^, Р) и сг-алгеб- ры 8, ?/]_, ?^2 С 3*. Говорят, что сг-алгебра g 'расщепляет, 4и^2, если P{AXA2 | g) = P(Ai | g)P(A2 | ^) для любых Ak e dk, k = 1,2. (99) Согласно этому определению, соотношение (83) означает, что сг-алгебра cr{Xt} расщепляет сг-алгебры 3t и 3*^t ПРИ каждом t Е Т. 40 (сравните с A)). Покажите, что если g расщепляет si\ и &?2 •> то g расщепляет d\ V g и ^2 V (? (по-прежнему, d У ё — o{d, g}). Проверьте, что условие (99) эквивалентно тому, что P(A\si1\/g) = P(A\g) для всех А е d2. Пусть сг-алгебры ga С &4\ и пусть при каждом a G / сг-алгебра (?а рас- расщепляет д?\ и ^2- Убедитесь, что тогда f] ga расщепляет 4 и 5/2, а если выполнено свойство (99), то si\ П si2 С g. В частности, в (83) фигурирует минимальная расщепляющая алгебра, поскольку сг{Х^} С 3*t П 3*^t- Заметим, что независимость сг-алгебр si\ и ^2 равносильна тому, что их расщеп- расщепляет тривиальная сг-алгебра g — {0, Q}. Вкратце остановимся на том случае, когда параметрическое множество Т не обя- обязательно есть подмножество числовой прямой Ж. Случайная функция X = {Xt, t G Т} порождает семейство сг-алгебр &4(IJ) = = a{Xt, t G С/}, С/ С Т. В обшем случае у нас нет естественных понятий "прошлого" и "будущего", в терминах которых удобно определять понятие марковости. Поэтому для введения марковских случайных функций могут представляться разные возмож- возможности, позволяющие определять, какие сг-алгебры s4{U{) и ?/(?/2) для C/i, С/2 С Т и какой сг-алгеброй должны расщепляться (см., например, [62; гл. 2], где изучается се- семейство сг-алгебр, индексированных некоторой системой А открытых множеств в ло- локально-компактном метрическом пространстве Т). Мы не затрагиваем здесь также концепции марковости обобщенных случайных полей. О гиббсовских и марковских полях, заданных на решетке Zd, см., например, [54]. Для дальнейшего чтения о марковских процессах и полях можно обратиться к [23, 24, 47, 56, 59, 62, 76, 82,107,113,150,184].
Глава VII Стационарные процессы. Дискретное и непрерывное время Ортогональные случайные меры и их сг-конечные структурные меры. Построение ортогональной случайной меры, отвечающей данной структурной мере. Интеграл по ортогональной случайной мере, его свойства. Теорема Карунена о факторизации ко- ковариационной функции и представлении процесса в виде интеграла по ортогональной случайной мере. Стационарные в широком смысле процессы и их ковариационные функции. Теорема Герглотца. Теорема Бохнера-Хинчина. Спектральное представ- представление стационарных процессов с непрерывным и дискретным временем. Эргодичность в L (Q). Процессы скользящего среднего. Статистическое оценивание ковариаци- ковариационной функции и спектральной плотности. Задача линейного прогноза. Регулярные и сингулярные процессы. Разложение Вольда. Регулярные процессы как физически осуществимые фильтры. Критерий Колмогорова регулярности процесса. Теорема Колмогорова-Сегё. § 1. Основным объектом изучения в этой главе будут стационарные в широком смысле случайные процессы. Акцент делается на их канонических представлениях с помощью "просто" устроенных объектов. При первом чтении необходимо усвоить теорему Карунена (§8), дающую общий подход к построению канонических пред- представлений случайных процессов и полей. Для ее понимания необходимо разобрать- разобраться с интегрированием детерминированных функций по ортогональным случайным мерам, детально изложенным в § 1-7. Далее в § 9 можно ограничиться рассмотре- рассмотрением процессов с дискретным временем, их спектральным представлением, а также применением этого представления к установлению варианта закона больших чисел (§ 10). Важный класс стационарных процессов дают процессы скользящего средне- среднего, вводимые в § 13. Желательно также ознакомиться с проблемой статистического оценивания спектральной плотности (§ 14). К остальным вопросам можно вернуться при повторном чтении. Для интересующих нас так называемых канонических представлений случай- случайных функций надо довольно подробно рассмотреть вопросы построения ортогональ- ортогональных случайных мер по заданной стуктурной функции и дать конструкцию с тохасти- ческого интеграла по таким мерам. Пусть Ж — полукольцо подмножеств некоторого множества Л, т. е. АП В Е Ж для любых А, В Е Ж, а если А С В, то В \ А можно представить в виде конечно- конечного, зависящего от А и В, объединения множеств из Ж. Предположим, что каждому
226 А. В. Булинский, А. Н. Ширяев множеству В Е Ж сопоставлена комплекснозначная величина Z(B) = Z(B,u) со значениями из L2(u) = L2(Q, J^, P). Определение 1. Семейство Z = {Z(B), В Е Ж} в L2(Q), индексированное множествами полукольца Ж, называется ортогональной случайной мерой, если 1) Z(B) ±Z(C) для В, С Е Ж,ВПС = 0,T.e.(Z(B),Z(C)) = 0, где (•,-) — скалярное произведение в L2(Q); оо 2) для каждого В = (J В^,тт В, Bi,B2,... е ЖиВпП Вш = 0 {п ф тп), к=1 п.н., A) к=1 а ряд сходится в среднем квадратическом. Подчеркнем, что множество тех ш Е П, для которых выполнено свойство A), хотя и имеет вероятность 1, но зависит от рассматриваемых множеств В\,Въ,... Е <Ж (всюду далее равенство случайных величин понимается как равенство п.н.). Отме- Отметим, что свойства 1) и 2) обеспечивают также конечную аддитивность Z на Ж. Введем структурную функцию (или структурную меру) меры Z, положив /i(B) = E|Z(?)|2, в еж. Пользуясь непрерывностью скалярного произведения, легко убедиться, что функция множеств \i = /i( •) обладает свойством счетной аддитивности. Если множество A G Ж, то \i является конечной мерой на Ж. Мы будем рассматривать более общую ситуацию, когда \± есть а-конечная мера, т. е. множество А может быть представлено оо в виде А = U Ап, где Ап Е Ж, /л(Лп) < оо и Ап П Ат = 0 при всех п ф т. 71=1 Нетрудно проверить, что если на полукольце Ж задана сг-конечная мера /i, то эквивалентное определение ортогональной случайной меры Z (на Ж), имеющей структурную меру /i, таково: Z(B) G L2(u) для всех В G Ж и (Z(B),Z(C))= /i(BnC) для любых В,СеЖ, B) напомним, что (?, Т]) = E?rj для ?, 77 G L2(Q, ^, P). Ортогональную случайную меру \± называют центрированной, если Е Z(B) = О для любого В ? Ж. Пример 1. Пусть А = [0,оо), Ж = {[а, 6), 0 ^ а ^ 6 < оо} ([а, а) = 0). Положим Z([a, 6)) = W(b) — W(a), где W — винеровский процесс. Учитывая не- независимость приращений винеровского процесса, видим, что Z есть ортогональная случайная мера на Ж, имеющая структурной функцией \i меру Лебега. § 2. Покажем как построить (даже центрированную) ортогональную случайную меру Z, имеющую своей структурной функцией данную сг-конечную меру \i на полу- полукольце Ж. Мы увидим, что такое построение неединственно.
Гл. VII. Стационарные процессы. Дискретное и непрерывное время 227 Теорема 1. Пусть \± есть а-конечная мера на полукольце Ж подмножеств Л (можно сразу продолжить ее на si = а{Ж}). Тогда существуют вероят- вероятностное пространство (П, 3, Р) и центрированная ортогональная случайная мера Z = Z(-), заданная на П и системе множеств <S = {В Е si: /л(В) < оо}, имеющая своей структурной функцией меру \i. Доказательство. Требуемое вероятностное пространство (П, ^, Р) построим следующим образом. Возьмем П = Л, 3* — si. Если0 < /i(A) < оо (случай/i(Л) = Отривиален),тоположимР(??) = /л(В)//i(A). Введем ортогональную случайную меру Z, но пока еще без свойства центрирован- центрированности, формулой Z(В, и) = у//л(АIв(ш), В Е si. Очевидно, Z(B,u) Е Ь2(п) и для любых множеств В\, B<i G si имеем (опуская аргумент и G Q) т. е. выполнено свойство B). Если же /i(A) = оо и Ai, Л2,... — разбиение Л, причем /i(An) < 00, П ^ 1, ТО ДЛЯ A G «С/ ПОЛОЖИМ [} Ясно, что Р — вероятностная мера на si ( Р(Л) = J^ 2~п = 1 ^ п=1 В рассматриваемом случае (/i(A) = оо) определим для Бе п=1 где 5П = В П An, n ^ 1. Ряд сходится в L2(Q) (суммируются величины с непересе- непересекающимися носителями), при этом оо оо 2>(ЛП)Е 1Вп = ^ 2>(ЛП)^-^ = М(В) < оо. 71=1 П=1 П=1 /^lAnJ^ Теперь заметим, что в силу непрерывности скалярного произведения (Z(B),Z(C)) = n=l n=l где 5n = В П Лп, Сп = С П Лп, п ^ 1, т.е. снова выполнено свойство B). Построенная мера Z, как в случае /i(A) < оо, так и в случае /i(A) = оо, еще не является центрированной. Для получения центрированной ортогональной слу- случайной меры Z с заданной структурной функцией /i, поступим следующим образом. (Отметим, что взятие Z(B) — Е Z(B), В G ^, не приводит к желаемому результату.) Рассмотрим на некотором вероятностном пространстве (П7, 3*\ Р') действительную случайную величину 77 такую, что Е'ту = О, Е'(?72) = 1, где Е' обозначает усредне- усреднение по мере Р'. Пусть Z — построенная выше на (П, ^, Р) ортогональная случайная
228 А. В. Булинский, А. Н. Ширяев мера. Возьмем (П, &, Р) = (П, ^, Р) ® (П', ^', Р') и определим на этом пространстве Z(B,u) := Z(B,uo)t](u;/) для Б G Й,й= (w,w;) G Ох (У. Легко видеть, что так построенная мера Z есть уже искомая центрированная ортогональная случайная мера. ? § 3. Ортогональная случайная мера Z была задана изначально на полукольце Ж. В то же самое время, структурная мера \i однозначно продолжается на сг-алгебру si = а{Ж}. Естественно поэтому заняться вопросом о возможности продолжения случайной меры Z на множества из si. Как будет видно из дальнейшего, такое про- продолжение возможно на множества совокупности ^ = {В Е si: /л(В) < оо}. Для этого понадобится ввести интегрирование по мере Z. Поскольку \i есть сг-конечная мера на Ж, то существуют множества An Е Ж, оо п ^ 1, такие, что А = (J Ап, причем Ап П Ат = 0 для п ф т и /л(Ап) < оо, 71=1 п ^ 1 (если \i — конечная мера, то А = Ai). Обозначим Жп = Ж П Ап, т.е. пусть Жп = {5 ? Ж: В С Лп}, и рассмотрим ?/п = а{Жп} — сг-алгебру с единицей Лп. Тогда оо A G d = сг{Х} <^ i= |Jin, где Ап е sin, C) 71=1 при этом п(АпК °о, D) 71=1 где /in — лебегово продолжение меры \± | ^ с полукольца JTn на sin. Нетрудно пока- показать, что si не зависит от выбора разбиения Л на An G Ж, п ^ 1, и при этом правая часть формулы D) будет давать одно и то же значение для разных разбиений. Предположим сейчас, а также в последующих §4 и §5, что /i(A) < оо (в качест- качестве Л может выступать любое из введенных выше множеств Лп, п ^ 1, для которых /i(An) < оо). Алгебру, состоящую из конечных объединений непересекающихся множеств, вхо- входящих в Ж (Л G Ж), обозначим 8. т Определение 2. Для В = (J Б^, где Bi G X, г = 1,..., m, Bi П ^ = 0 (г ^ j), г=1 введем ZE) формулой: E) г=1 Данное определение корректно. Действительно, пусть множество В имеет также г представление В — (J Dj, Dj G J^T, j = 1,... , r, Dj П D/ = 0 (j 7^ Z). Тогда получим m m r r r m У^ z(b ¦) = У^У^ г=1 i=lj=l
Гл. VII. Стационарные процессы. Дискретное и непрерывное время 229 (равенства случайных величин всюду выполняются п. н.). Отсюда г=1 j = l Займемся теперь конструкцией стохастических интегралов по ортогональным слу- случайным мерам. Назовем функцию /: Л —> С простой, если т / = $>1в4, (в) где Ci Е С, Bi Е ё, г = 1,..., т, причем (J 5^ = Л и ^ П ??j = 0 для г ф j, т.е. г=1 множества 5^ образуют разбиение Л. Определение 3. Интеграл (обозначаемый J(f)) no ортогональной случай- случайной мере для простой функции / вида F) задается формулой . G) г=1 Докажем, что это определение корректно. Пусть наряду с F) функцию / мож- г но представить также в виде / = ^2 djlo , где dj G С, Dj G 8, j = 1,... ,r; r U Dj = A, DiH Dj = 0(i/ j). Но тогда, если fi(Bi П ?>j) 7^ 0, то с» = dj. Следовательно, учитывая, что Z@) = 0, имеем г г т т г=1 j: откуда видно, что значение J(f) не зависит от способа представления (простой) функции /. ? § 4. Чтобы распространить данное определение интеграла J(/) на более широкий класс функций /, рассмотрим свойства построенного интеграла для простых функ- функций. Лемма 1. Пусть fug — простые функции. Тогда (J(f),J(g)) = (f,g), где (•, •) — скалярное произведение в L2(Q), а (•, •) — скалярное произведение в пространстве L2(A) = L2(A,?/, /jl).
230 А. В. Булинский, А. Н. Ширяев г Доказательство. Пусть функция / задается формулой F); д = ^2 djlDj •> где Di,..., Dr образуют разбиение А. Тогда / т г \ т г (J(/), J(g)) = (TciZiBi), Y, djZ{Dj) ) = E E ddjKBi П Dj) = \=1 j = l ' i=l 3 = 1 m r „ = ^212с^з / 1BinDj i=lj=l JA г=1з=1 г г = / E^1^E^1^^A)= / jAi=l j = l JA Следствие 1. Отображение f \-> J(f) есть линейное отображение линей- линейного многообразия простых функций в пространство Ь2(П). Доказательство . Если / и д — простые функции, то, очевидно, функция af + (Зд,а,C G С, также есть простая функция. В силу "полуторалинейности" скалярно- скалярного произведения и леммы 1 (J(af + fig) - a.J(f) - (iJ{g), J(af + fig) - aJ(f) - fJJ(g)) = = (af + fig, af + fig) - a(f, af + fig) - f3(g, af + f3g)- - a(af + f3g, f) + aa{f, f) + fia{g, /)- - 0(af + fig, g) + a0(f, g) + 00{g, g) = 0. Таким образом, сохраняющее скалярные произведения отображение линейно: J(af + fig) = aJf + fiJg. ? (8) Лемма 2. Простые функции вида F) плотны в пространстве L2(A). Доказательство. Пусть / е L2(A). Для любого г > 0 выберем Н = Н(е) > 0 так, чтобы Используя A.4) и лемму 3 главы I, легко аппроксимировать /(АI{|^(д)|^д-} после- последовательностью ступенчатых функций вида F), что и доказывает требуемую плот- плотность в 1/2(А). ? §5. Распространим теперь введенное понятие интеграла J(f) с простых функ- функций / на функции / из L2 (А).
Гл. VII. Стационарные процессы. Дискретное и непрерывное время 231 Определение 4. Для функций / Е L2(A) положим J(/)=l.i.m.J(/n) (l.i.m.—предел в L2(Q)), (9) п—юо где {/п}п^1 — последовательность простых функций таких, что fn —у / при п —У оо. Докажем, что определение 4 корректно. По лемме 2 существует последователь- последовательность простых функций fn —у / при п —У оо. Пользуясь следствием 1 и леммой 1, имеем, обозначив || • || норму в L2(Q), \\J(fn)-J(frn)\\ = \\J(fn-frn)\\ = \fn-frn\^O При П,Ш^00, где | • | — норма в L2(A), поскольку сходящаяся последовательность {/п} является фундаментальной. В силу того, что пространство Ь2(П) полно, получаем, что су- существует предел J(fn) в Ь2(П). Покажем, что этот предел не зависит от выбора х « тт ?2(Л) ? аппроксимирующей последовательности простых функции. Пусть дп —у j, где дп — простые функции. Тогда \дп - fn\ ^ \дп - /| + |/п - /| -^ 0 при п -)> оо. Следовательно, Но если J(jn) —> <iw-J{gn) —у ^ то J(jn) — J(gn) —у ? — (\ 1аким образом, \\J(fn) - J(gn)\\ -»• 11^ - СИ = 0, т.е. ? = <п.н. ? Следствие 2. Дл^ любых функций f,g€ L2(A) J(/) есть линейное отображение L2(A) в Ь2(п). Пусть hn G L2(A) (необяза- (необязательно простые) и hn —у /. Тогда J(hn) ^ J(f), п -> оо. A1) Доказательство. Для проверки свойства A0) достаточно взять простые функ- функции fn,gn,n ^ 1, такие что fn —у /, дп —у д (см. лемму 2), воспользоваться леммой 1 и учесть непрерывность скалярного произведения. Линейность J = J(f) устанавливается так же, как при доказательстве следствия 1. Пользуясь линейнос- линейностью J и соотношением A0), имеем \\J(f) - J(hn)\\ = \\J(f ~ Mil = I/ - M "> 0, n -> оо. ? § 6. Приведенная выше конструкция интеграла J(/) для / G Ь2(Л) = 1/2(Л, ?/, /i) предполагала, что /i(A) < оо. Рассмотрим теперь построение интеграла J(f) для функций / G Ь2(Л, «с/, /i) в том случае, когда \i есть сг-конечная мера.
232 А. В. Булинский, А. Н. Ширяев Рассмотрим разбиение множества А на множества An Е Ж с /i(An) < оо, п ^ 1. Тогда п ^ 1, 2 A2) 71=1 -*АП где ?/п = сг{Жп}, \in — лебегово продолжение меры /i\^ с Жп = Ж П Лп на ?/п. При этом значение интеграла / |/(А)|2 /i(dX) не зависит от выбора разбиения Л на Лп е Ж, п е N. Определение 5. Для функции / Е L2(A) введем интеграл J(f) по ортогональ- ортогональной случайной мере Z формулой оо ~~ A3) 71=1 где ряд сходится в L2(Q), функции /п фигурируют в A2), отображения Jn опреде- определены на L2(An) описанным выше способом (индекс п подчеркивает связь с Ап и /лп). Проверим корректность определения A3). Если / G L2(A), то, согласно A2), имеем fn G L2(An) для всех п ^ 1. Теперь заметим, что Jn(fn) _L Jm(fm) ПРИ п ф т. Действительно, если fn и /т — простые функции, то это — очевидно, так как (Z(B), Z(D)) = 0 для В G Sп и D G Sш (&п — алгебра, порожденная Жп в Ап, п G N). Для функций fn и /т, не являющихся простыми, надо взять их аппроксима- аппроксимации простыми функциями. Следовательно, MVN MVN n=MAN n М N V J if )-Vj (f / j Jn\Jn) 2_^ Jn\J? n=l n=l при TV, M —у оо в силу изометричности Jn и сходимости ряда в A2). оо Покажем, что величина J(f) не зависит от вида разбиения Л. Пусть Л = (J Гп, 71=1 Гп G Ж, Гп П Гт = 0 (п / ш), МГП) < оо, n G N. Обозначим НП:ГП = /|Л пГ •> оо ??г,п е N. Тогда /п = /|л = Z) /|л пГ , n G N. Теперь заметим, что П 771=1 П m S ^n,m —^ fn ПРИ TV —>¦ оо и, значит, в силу A1) 7П=1 оо тп=1 где ряд сходится в Ь2(п). По построению Jn,m для h G L2(An П Гт) С L2(An) имеем Jn,m(h) = Jn(h), п,т ^ 1. Учитывая, что счетное объединение множеств меры нуль имеет меру нуль, получаем оо оо оо Jnyjn) — / J / J ^n,7n(^n,7nj- П=1 7П=1
Гл. VII. Стационарные процессы. Дискретное и непрерывное время 233 Аналогично, обозначив дш = /I и Jm — отображение, которое строится на L2(Fm) так же, как Jn на L2(An), m, п ^ 1, имеем оо оо 9m) — / J / J Е- m=l ?ti=1 n=l Остается учесть, что Jn,m(hn,m) -L Jk,i(h>k,i)i если (n, m) т^ (fc, Z), и то, что данные ряды из ортогональных величин можно суммировать в любом порядке, поскольку оо оо оо оо « % Л % Л . . , \ I I 2 \. Л % Л .. ||Q / I /» / А \ I 2 /7Л\ 71=1 771=1 71=1 771=1 Для действия J (см. A3)) на функции / G Ь2(Л, ?/, /л) обычно используется ин- интегральная запись J(f)= [ /(A)Z(dA). A4) ./л Интеграл в правой части A4) принято называть стохастическим интегралом по ортогональной случайной мере Z (от детерминированной функции / G Ь2(Л, «с/, /i); сравните данное определение с определением в главе VIII стохастического интеграла Ито от случайных функций / = /(А; о;)). §7. Основное свойство введенного стохастического интеграла J = «/(/) содержит следующая теорема. Теорема 2. Стохастический интеграл A4) является изометрическим отображением L2(A,?/, /i) (где /i есть а-конечная мера) на некоторое подпространство L2Z С L2(Q,^, P). Дрг/ этом Z продолжается до орто- ортогональной меры (со структурной функцией /i) на систему множеств Ш = = {В G «с/: 1л(В) < оо} по формуле Z{B) = JAB). A5) Доказательство. Случай, когда /л(А) < оо, был подробно разобран выше. На- оо до рассмотреть лишь случай /i(A) = оо. Пусть Л = (J Лп, где Ai, Л2,... — разби- п=1 оо оо ение Л и /л(Ап) < оо, п ^ 1. Возьмем /, ^ G L2(A). Тогда f = ^2 /п? # — где /п = /1 л 5 #п = ^ I л 5 причем ряды сходятся в L2 (А). Отсюда 71=1 71 771 7 Х71=1 771=1 N N = lim У^ (Jnfm JnQn)= lim iV—юо ^-^ iV—^oo 71=1 71=1 iV iV iV^oo\ П=1 771=1
234 А. В. Булинский, А. Н. Ширяев где учтено то, что Jn(fn) -L Jm(gm) при n/тито, что Итак, J — это изометрия. Очевидно, образ L2(A) при изометрии J есть подпро- оо странство в 1/2(П), обозначенное L2Z. Для В G Ж имеем \i(В) = Y1 М^п) < ею, 71=1 оо где Вп = В П An, n G N. Далее, 1в G Ь2(А, ^,//)и1б= ^ 1-Вп > где ряд сходится 71=1 также в L2(A). Следовательно, из A3) JAb) = 71=1 ОО В силу G) имеем Jn(li?J = ^(Бп), а ? ZEn) = ZE) (ряд сходится в 71=1 Таким образом, для любого В е Ж получаем JAb) = Z(B), т.е. формула A5) определяет продолжение Z с Ж на Я!. Кроме того, для В, С G ^ по свойству изо- метричности J (Z(B),Z(C)) = (JAB), JAc)) = <1в, 1с> = KB П С). П Замечание 1. В силу A1) легко видеть, что если Z — центрированная ортого- ортогональная случайная мера (на Ж), то Е («/(/))= 0 при всех /GL2(A). В частности, Е Z(B) = 0 для В е <S. §8. Пусть X = {X(t), t G T} - к о мплек спознанный L2-nponecc, заданный на некотором вероятностном пространстве (П,^, Р), т.е. Е|Х(?)|2 < оо при каждом t G Т. Напомним, что для комплекснозначного L2-nponecca ковариационная функция определяется формулой = E(X(s)-EX(s))(X(t)-EX(t)), s,teT. A7) Будем в дальнейшем считать, что EX(t) = 0, t G Т. Это предположение не огра- ограничивает общности, и к этому случаю все можно свести, переходя от величин X(t) к центрированным величинам X(t) = X(t) — EX(t). Один из основных результатов относительно возможности представления случай- случайного процесса с помощью "просто" устроенных объектов (в рассматриваемом случае под этим понимаются ортогональные случайные меры) дает следующая теорема.
Гл. VII. Стационарные процессы. Дискретное и непрерывное время 235 Теорема 3 (Карунен). Пусть ковариационная функция центрированного комплекснозначного L2-процесса X = {X(t), t G Т}, заданного на вероят- вероятностном пространстве (П,^, Р), допускает факторизацию, т. е. представима в виде r(s,t) = [ /(s, АOМ)М<*А), s,teT, A8) Ja где /(?, •) G L2(K) = L2(A, «с/,/i) для каждого t G T', a \i — некоторая а-конечная мера. Тогда существует центрированная ортогональная случайная мера Z, заданная на системе множеств <$ = {A G si: /л(А) < оо} и на, вообще говоря, расширении исходного вероятностного пространства, имеющая структурную меру \i, такая, что X(t)= / f(t,X)Z(d\) для каждого teT. A9) Если система функций {f(t, •), t G Т} полна в пространстве L2(A,si,ц), т. е. Ф f f(t,XLf(X)fi(d\) =0 \fteT =Ф Ф = 0^п.н, B0) то требуемую меру Z можно построить и на исходном вероятностном про- пространстве. Замечание 2. Если процесс X = {X(t), t G Т} допускает представление A9) (с функциями /(?, •) из 1/2(Л, si, \i), где \i есть сг-конечная структурная мера цент- центрированной ортогональной случайной меры Z), то свойство A8) справедливо в силу теоремы 2. При этом, согласно замечанию 1, L2-nponecc X будет центрированным. Это замечание показывает, что достаточные условия интегрального представ- представления A9) являются и необходимыми. Доказательство теоремы. Пусть выполнено условие B0). Для t G Т рас- рассмотрим отображение G, определяемое формулой G:f(t,-)^X(t,-), B1) и продолжим его по линейности: B2) гдеС/с G С, tfc eT, k = 1,...,п. Покажем корректность этого определения, т. е. покажем, что если 2_^ckf(tk, •) = k=l 1=1
236 А. В. Булинский, А. Н. Ширяев где di Е С, si Е Г, / = 1,..., т, то к=1 1=1 Для этого, учитывая A8), заметим, что Z=l fc=l Z=l (п.н.). / f(tk, Л Значит, к = 1 1 = 1 / П - (s> 4 = 1 Е< k=l 1=1 z=i Z=l Z=l откуда и следует требуемое утверждение о корректности определения B2). Как изометрия отображение G продолжается на подпространство L2[f], полу- п ченное замыканием в L2(A) функций вида J2 ckf(tk, •)? где функция / — это к=1 функция из представления A8). При этом в силу B0) имеем L2[f] = I/2(A, ?/, /i) и G(L2[f]) = L2[X], где Ь2[Х] — замыкание линейной оболочки процесса X(t), т.е. в L2(Q, ^, Р) берется замыкание линейных комбинаций вида c\X(t\) + • • • + cnX(tn) Для Тогда , ,,) , очевидно, 1# G L2(A), и мы можем определить (Z(B),Z(C)) = B3) Следовательно, Z = Z(-) — ортогональная случайная мера на ^ со структурной мерой \i. Мера Z центрирована, поскольку Z{B) G Ь2[Х] для каждого В G ^, а Е ^ = 0 для каждого ( G L2[X], так как процесс X имеет нулевое среднее. Для любой функции h G L2 (А) можно ввести стохастический интеграл j(h) = / h(\)Z(d\). По теореме 2 J есть изометрическое отображение Ь2(Л) на L2Z. Итак, имеются две изометрии G: L2(A) -> L2[X] С Ь2{п) и J: L2(A) -> L| С
Гл. VII. Стационарные процессы. Дискретное и непрерывное время 237 при этом в силу B3) и A5) справедливы равенства GAb) = JAb) Для В G ^. Но конечные линейные комбинации таких индикаторов плотны в L2(A), что обеспечи- обеспечивается леммой 2. Следовательно, G = <7на1/2(Л) и, крометого, L2[X] = L2Z. Из B1) вытекает, что J(/(t, •)) = X(t). Таким образом, получаем представление A9) (без расширения исходного вероятностного пространства). Пусть теперь условие B0) не выполнено. Тогда L2[f] С L2(A) и L2[f] ф L2(A). Рассмотрим L2 (Л) 0 L2 [/], т. е. ортогональное дополнение к L2 [/] в L2 (Л), и возьмем в нем какой-либо базис из функций д(и, •) G Ь2(Л), и G Г', где Тг П Г = 0 (см. [60; т. 1, с. 59]). Введем функцию p(s,t) = [ Очевидно, для cj~ G C, tk G T', к = 1,..., n и n ^ 1, 2 /x(dA) ^ 0. В силу теоремы 4 главы II существует комплекснозначный центрированный гауссов- ский процесс {Y(t), t G Т'}, заданный на некотором вероятностном пространстве (п',^',?'), такой, что cov(y(s), Y(t)) = E fY(s)Y(t) = p(s, ?), s,t e T'. Возьмем (ft, &, P) = (ft, &, P) x (ft7, &\ P'). Тогда для каждых t G Г, s G T' ве- величины X(t) и Y(s) будут независимы на этом пространстве (для и = (о;, о;') G ft имеем X(t) = X(t, и) = X(t, о;), l^(s) = У(s, 5) = l^(s, cj')). Введем на (Г U Г;) х ft центрированную случайную функцию V D- , Л 4- CL Т1 А ^6, UJ) , 6 t 1 , I [t,OJ J, t kz 1 . Для нее о, s g r, t ef или seTf, teT. Иначе говоря, где и учтено, что /(t, •) _L g(s, •) в L2(A) для t eTus eTf.
238 А. В. Булинский, А. Н. Ширяев Кроме того, L2[h] = L2(A), так как $ Ф е L2(A): Ф _L h(t, •) для всех t е Г U Г'. Итак, по доказанному существует центрированная ортогональная случайная мера Z 2Z у х(] такая, что Ь2[?] = L2Z w = / h(t, A) Z(dA), teTUT'. Ja Но при t G T имеем ?(?, 5) = X(t, 5). Следовательно, для всех t G T X(t,^) = X(t,cj)= [ h(t,\)Z(d\)= f f(t,\)Z(d\). ? ./л Ул § 9. Как следует из названия главы, наш основной интерес связан с рассмотрением стационарных процессов. Оказывается, что приведенная выше теорема Карунена, примененная к таким процессам, позволяет получить для них стохастические пред- представления, допускающие весьма прозрачную "спектральную" интерпретацию. Напомним, что суть понятия стационарности случайного процесса состоит в том, что статистические характеристики такого процесса инвариантны относительно сдвигов t н-» t + и. Сказанное нуждается в пояснении. Прежде всего, чтобы иметь возможность производить сдвиги аргумента t исследуемого процесса X = {X(t), t G Т}, предполагается, что Т — некоторая группа (всюду — по сложению). В качестве временного множества Т, как правило, будут рассматриваться мно- множества Ъ — {0,=Ы,...} или Ж = (—оо, оо), что соответствует изучению процессов с дискретным или непрерывным временем. Кроме того, значения X(t) при каждом t G Т считаются принадлежащими одному и тому же пространству S (обычно S = С илиБ = Ж). Определение 8 главы VI стационарного в узком смысле процесса сохраняется для случая, когда Т является группой. Во многих исследованиях представляют интерес свойства случайных процессов, зависящие лишь от смешанных моментов определенных порядков, т. е. от функций вида EX(ti)fcl '"X(tn)kn, гдеп G N, ?ь ..., tn е Г, къ ..., кп е Z+. Для класса L2-процессов это делает естественным Определение 6. Комплекснозначный (в частности, действительный) L2-npo- цесс X = {X(t), t G Т}, где Т — некоторая группа, называется стационарным в широком смысле (или стационарным второго порядка), если EX(t)=a при любых t G Г, B4) r(s,t)=cov(X(s),X(t))=r(s-t,Q)(:=R(s-t)) при всех s,teT. B5) Легко видеть, что всякий стационарный в узком смысле L2-nponecc будет ста- стационарным в широком смысле. Для гауссовских процессов эти понятия совпадают. Последовательность независимых одинаково распределенных величин, не имеющих математического ожидания, дает пример стационарного в узком смысле процесса, для которого бессмысленно говорить о стационарности в широком смысле. Далее мы сосредоточимся на процессах, стационарных в широком смысле. Теория таких процессов тесно связана с теорией кривых в гильбертовом пространстве и со свойством неотрицательной определенности детерминированных функций.
Гл. VII. Стационарные процессы. Дискретное и непрерывное время 239 Определение 7. Комплекснозначная функция R = R(t), t G Т, заданная на не- некоторой группе Т, называется неотрицательно определенной, если неотрицатель- неотрицательно определена функция r(s, t) = i?(s — ?), s, t G T, т. е. выполнено условие A1.13). Из теоремы 4 главы II вытекает, что класс неотрицательно определенных функций R = R(t), t G Т, где Т — группа, совпадает с классом ковариационных функций стационарных гауссовских процессов X = {Х(?), t G Г}. Описание неотрицательно определенных функций на группе Т = Z = {О, =Ы,... } дает следующая Теорема 4 (Герглотц). Функция R = R(n), n G Z, является неотрицательно определенной тогда и только тогда, когда справедливо "спектральное пред- представление" R(n) = Г einA Q(dA), nGZ, B6) У — тг где Q — конечная мера ("спектральная мера") на ?$([—тг,тг]). В формуле B6) и далее интеграл от —тг до тг понимается как интеграл по отрезку [-тг,тг]. Доказательство . Достаточность. Очевидно, функция B6) является неотрица- неотрицательно определенной, поскольку для всех t\,..., tn G Z, z\,..., zn G Си любых n G N n ,itk\ k,q=l zkzqR(tk-tq)= Г J~7T Q(dX) ^ 0. B7) Необходимость. Для N ^ 1иА G [—тг, тг] введем непрерывные и неотрицательные (в силу неотрицательной определенности R = R(n)) функции 9N(X) = f- ^ ^ R(k - q)e-^e^ = ^ k=lq=l — y^ П _ m\/N)R(m)e~irnX, B8) ^ |m|<iV где переход от двойной суммы к однократной справедлив, поскольку имеется N —\тп\ пар (k, q), для которых к — q = m (здесь k,q G {1,..., TV}, |m| < TV). Определим на Зё([—тг, тг]) меру Qjv с плотностью ^дг по мере Лебега, т. е. положим QN{B) = f gN(X)dX, В G й8([-тг,7г]). Тогда, согласно формуле A.25), для N ^ 1 имеем У-7Г J-7T { 0, |П| ^ N. Заметим, что Eдг([—тг,тг]) = Д@) < оо для всех 7V (в силу B9) при п = 0). Поэтому по теореме Прохорова для конечных мер (см. приложение 2), взяв компакт
240 А. В. Булинский, А. Н. Ширяев К = [—тг, тг], находим подпоследовательность {N^} С N такую, что Qnu => Q-> где Q — некоторая конечная неотрицательная мера на [—тг, тг]. Тогда, учитывая B9), получим для каждого п Е Z соотношения / einAQ(dA)= lim / einX QNk(dX) = R(n). ? J — ТГ J — 7Г Доказанная теорема позволяет легко получить "спектральное представление" стационарных (в широком смысле) процессов. Теорема 5. Пусть X = {Xt, t Е Z} — центрированный стационарный в ши- широком смысле процесс, определенный на некотором вероятностном простран- пространстве (П,^, Р). Тогда на том же вероятностном пространстве существует ортогональная случайная мера Z, заданная на ?$([—тг,тг]), такая, что (п. н.) имеет место стохастическое "спектральное представление" Xt= Г eitxZ(dX), teZ. C0) J Доказательство. По теореме Герглотца для s,tGZ r(s,t) = cov(Xs,Xt) = Г e^-^QidX) = Г eisX^Q(dX), C1) J —тг J —ТГ где Q — конечная (неотрицательная) мера на ?%([—тг, тг]). Значит, выполнено усло- условие A8) теоремы Карунена (теорема 3), с /(t,A) = ег*л, Л G [—тт,тт], t G Z. При этом также выполнено и условие B0), поскольку любую функцию из пространства L2 = L2([—тг, тг], Зё([—тг, тг]), Q) можно аппроксимировать в L2 непрерывной функци- функцией, принимающей одинаковые значения в точках — тг и тг, а такая функция равномерно приближается суммами Фейера (см., например, [35; гл. VIII, § 2, п. 1]). Тем самым, требуемое представление C0) вытекает непосредственно из теоремы Карунена. ? Напомним, что в силу теоремы Карунена спектральная мера Q, фигурирующая в формуле C1), является ни чем иным как структурной мерой (см. B)) для орто- ортогональной случайной меры Z, по которой ведется интегрирование в C0). Меру Q, использующуюся в B6), можно переопределить (не меняя обозначений), перенеся "массу" Q({—тг}) из точки — тг в точку тг, где возникнет "масса" Q({—тг}) + + Q({tt}) . При этом значение интеграла в правой части формулы B6) не изменится, поскольку е~гП7Г = егП7Г при всех п G Z. Указанное переопределение делается лишь для того, чтобы представить интеграл по промежутку (—тг, тг] как интеграл по единичной окружности. Если такое перенесение массы произведено, то Z({—тг}) = 0 п.н. в силу теоремы 2, поэтому и в C0) интегрирование фактически ведется по (—тг,тг]. Разумеется, аналогичным образом интегрированние в C0) можно свести к интегрированию по полуинтервалу [—тг, тг). Завершая рассмотрение вопроса о спектральном представлении C0), отметим, что данная ("спектральная") терминология навеяна тем, что (согласно C0)) значе- значения Xt как бы "складываются" из спектральных гармоник е с соответствующими "весами" Z(dX). § 10. В качестве приложения стохастического спектрального представления C0) рассмотрим следующий вариант закона больших чисел.
Гл. VII. Стационарные процессы. Дискретное и непрерывное время 241 Теорема 6. Пусть выполнены условия теоремы 5. Тогда N-l 2 Xk —У Z({0}) при N —у оо. к=0 j- C2) Доказательство. Воспользуемся представлением C0). В силу линейности сто- стохастического интеграла по ортогональной случайной мере лг-1 лг-1 1 лг-1 лг-1 мТ,х»= ыТ,eik Флг(А) = I TV A = 0. Следовательно, в силу теоремы 2 ЛГ-1 = \XN(\)\2Q(d\), (зз) J-тг где х^у (А) = Фдг(А) — 1 jo} (A), Q( •) — структурная мера, отвечающая ортогональ- ной случайной мере Z(-). Учитывая, что XN @) = 0 и для A G [-7Г, тг] \ {0} — У^ е i\k 1, А е М, а также то, что N по теореме Лебега о мажорируемой сходимости из C3) получаем требуемое свойст- свойство C2). П Замечание 3. Если X = {Xt, t G Z} — стационарный в широком смысле про- процесс и Е Xt = a, t G Z, то Xt - а = Г eitA Z(dA) J—тг и- . Следовательно, k=0 а при TV ^ oo C4) тогда и только тогда, когдаЕ |Z({0})|2 = 0; это равносильно условию Q({0}) = О, означающему, что у спектральной меры нет атома в нуле. Отметим, что о свойстве C4) часто говорят как о свойстве эргодичности процес- процесса X в1?(п). §11. Имея спектральное представление C0) стационарных процессов с дискрет- дискретным временем, естественно теперь задаться вопросом о соответствующем представ- представлении для процессов с непрерывнем временем. С этой целью нам будет полезно следующее определение.
242 А. В. Булинский, А. Н. Ширяев Определение 8. Пусть X = {X(t), t G 1} есть комплекснозначный!/2-процесс, заданный на некотором вероятностном пространстве (П, ^, Р). Процесс X непреры- непрерывен в среднем квадратическом в точке t G 1, если \\X(s)-X(t)\\^0 при s^t, C5) где ||?|| = (Е l^l2I/2, ? G L2(n,^, P). Непрерывность на некотором множестве озна- означает непрерывность в каждой точке этого множества. Теорема 7. Стационарный в широком смысле процесс X = {X(t), t G 1} непрерывен в среднем квадратическом на Ж тогда и только тогда, когда ко- ковариационная функция R = R(t), определенная в B5), непрерывна в нуле. Доказательство. Необходимость. Процессы X(t) и X(t) — а, где а = const, очевидно, одновременно являются непрерывными (в среднем квадратическом). По- Поэтому без потери общности можем считать процесс X имеющим нулевое среднее. Его ковариационная функция r(s,t) = EX(s)X(t) = R(s — t) для s,t G ffiL Ясно, что \R(t)-R@)\ = |r(t,O) -r@,0)| ^ \\X(t)-X@)\\ \\X@)\\. Следовательно, непрерывность процесса X в среднем квадратическом на Ж (даже лишь в точке 0) влечет то, что R(t) —У Д@), когда t —> 0. Достаточность следует из того, что \\X(s) - X(t)\\2 = -(R(t -а)- Д@)) - (R(s -t)- Д@)) -»• 0 при s —У t, поскольку ковариационная функция R = R(t) непрерывна в 0. ? Следствие 3. Неотрицательно определения на Ж функция R = R(t) непре- непрерывна на М, если она непрерывна в нуле. Доказательство. Пусть функция R неотрицательно определена на Ж и непре- непрерывна в нуле. По теореме 4 главы II построим центрированный стационарный в ши- широком смысле (гауссовский) процесс X = {X(t), t G Ж} с ковариационной функцией r(s, t) = EX(s)X(t) = R(s -t),s,tGl. В силу теоремы 7 этот процесс непрерывен в среднем квадратическом на Ж. Тогда \R(s) - R(t)\ = \ЕХ(8)Х~Щ- ЕХ(*Щб)| ^ ||ВД - X(t)\\ \\Х@)\\ -»¦ 0 при s —У t. Таким образом, функция R = R(t) непрерывна в каждой точке t G ffiL ? § 12. Описание ковариационных функций стационарных (в широком смысле) про- процессов, непрерывных в среднем квадратическом на Т = М, дает следующая Теорема 8 (Бохнер-Хинчин). Пусть R = R(t), t G 1, — непрерывная в нуле неотрицательно определенная функция. Тогда для любого t G 1 R(t)= f°° eitxG{d\), C6) J — oo где G = G(dX) — некоторая неотрицательная конечная ("спектральная") ме- мера наЩЖ). Очевидно, что функция, задаваемая правой частью C6), является непрерывной на всей прямой. Совершенно аналогично B7) убеждаемся, что она также и неотрица- неотрицательно определена. Таким образом, условия теоремы 8 необходимы. Доказательство достаточности этих условий отнесено в приложение 4.
Гл. VII. Стационарные процессы. Дискретное и непрерывное время 243 Определение 9. Если спектральная мера в представлениях B6) или C6) для ко- ковариационной функции стационарного процесса X = {X(t), t G Л} (где Л = [—тг, тг] или Л = Ж) имеет плотность д = д(Х) по мере Лебега, то функция д называется спектральной плотностью процесса X. При наличии спектральной плотности д у меры G видим из C6) и A.25), что -L R(t) = / eUAg(X) dX. Ja Из доказательства теоремы 8, проведенного в приложении 4, вытекает, что если ковариационная функция R Е L1(M), то у меры G существует непрерывная и огра- ограниченная на всей прямой спектральная плотность д = д(Х), причем эта плотность может быть найдена по следующей формуле: д(Х) = — \ e~itxR(t) dt. C7) Теорема 9 (Крамер). Пусть на некотором вероятностном пространстве (П, &, Р) задан центрированный стационарный в широком смысле непрерыв- непрерывный в среднем квадратическом случайный процесс X = {X(t), t G 1} (дос- (достаточно непрерывности только в нуле). Тогда на том же вероятностном пространстве существует ортогональная случайная мера Z = Z(-), заданная на Й§(М), такая, что имеет место стохастическое "спектральное представ- представление^ X(t) = / eitxZ(dX), t е Ж. C8) J — oo Доказательство . Функция R(t) = EX(t)X@) в силу теоремы 7 и следствия 3 непрерывна на М и согласно теореме 4 главы II неотрицательно определена. По тео- теореме Бохнера-Хинчина имеет место представление C6). Следовательно, r(s,t) =cov(X(s),X(t)) = = f° J — где G(-) — конечная (неотрицательная) мера на сг-алгебре ё$(Ж). Таким образом, получена факторизация ковариационной функции в виде A8) с f(t,X) = eltx (t,Ae Ж). В рассматриваемом случае справедливо также условие B0). Действительно, лю- любую функцию / = /(А) из L2 = L2(M, 3e(R),G) можно аппроксимировать "срез- "срезкой" /(АI{|д|^а}, которую затем приближаем в 1? непрерывной функцией, имею- имеющей одинаковые значения в точках а и —а, после чего берем конечные линейные ком- комбинации функций ехр{шАтг/а}, п G Z. Тем самым требуемое представление C8) вытекает из теоремы 3 (Карунен). От- Отметим, что в силу этой же теоремы 3 спектральная мера G совпадает со структур- структурной мерой ортогональной случайной меры Z, по которой ведется интегрирование вC8).П § 13. Обратимся к изучению спектральных плотностей стационарных процессов, которые (в соответствии с уже данной в конце § 10 интерпретацией) характеризуют "мощность", с которыми входят гармоники elXt в спектральное представление. Для простоты ограничимся рассмотрением процессов с дискретным временем.
244 А. В. Булинский, А. Н. Ширяев Определение 10. Центрированный и ортонормированный (в Ь2(п)) процесс г = {еП5 п Е Z} называется белым шумом. Легко видеть, что такие процессы являются стационарными в широком смысле со спектральной плотностью д(А) = 1/Bтг), A Е [—тг,тг]. Иначе говоря, подобно "бе- "белому свету" процесс г можно рассматривать в виде смеси (в смысле спектрального представления) гармонических колебаний одинаковой интенсивности. Общий результат о структуре случайных процессов с дискретным временем, у ко- которых существует спектральная плотность, дается в следующей теореме. Теорема 10. Стационарный в широком смысле центрированный процесс X = {Xt, t G Z}, определенный на вероятностном пространстве (П,^, Р), имеет спектральную плотность тогда и только тогда, когда найдутся чис- числовая последовательность {ck}kez ? I2 и белый шум е = {еп, n Е Ъ\, задан- заданный, быть может, на расширении исходного вероятностного пространства, для которых (п. н. относительно меры этого "расширенного пространства" Xt= ^ cket-k 1= ^ ct_kek), teZ, C9) k = — oo k = — oo где ряд сходится в среднем квадратическом. Напомним, что {ck}kez ? I •> если ^2 \ск\ < °°- к = — оо Прежде, чем переходить к доказательству, отметим, что процесс X, допускаю- допускающий представление C9), называется процессом скользящего среднего. Говорят так- также, что процесс X получается с помощью фильтра, на вход которого подается белый шум е. Фильтр называется физически осуществимым, если с& = 0 для к < 0. Это соответствует тому, что значения процесса X в момент t определяются величина- величинами Ek с к ^ t (т. е. в формировании величин Х± на "выходе" фильтра не участвуют "будущие" значения "входящего" процесса е). Доказательство. Пусть справедливо соотношение C9), где {с^} и {вк} опи- саны в условиях теоремы. Легко видеть, что поскольку Е ^2 сг-к^к — О Для — оо < М ^ N < оо, и ряд в C9) сходится в Ь2(п, §<, Р), имеем EXt = 0 при всех t G Ъ, где Е — усреднение по мере Р. По равенству Парсеваля для любых s,tG Z r(s, t) = Е ^ к = ^2 cs-kct-k = ^2 cjCj-(s-t) = R(s - t). D0) к j Следовательно, X — стационарный в широком смысле центрированный процесс. Введем функцию Ф(А) = -j= ? с-*™. D1) к — — оо
Гл. VII. Стационарные процессы. Дискретное и непрерывное время 245 Поскольку {ck} G I2 и О для s ф t (s, t G Z), то ряд D1) сходится в L2 [-тг, тг] = L2 ([-тг, тг], Й§[-тг, тг], mes), где mes — мера Лебега (Ф в D1) является представителем класса эквивалентных функций в L2[—тг, тг]). В том же самом пространстве L2[—тг, тг] для функции Ф(А)е* справедливо пред- представление D3) л/ 2тг , — л/ 2тг . к = — оо j = По равенству Парсеваля из D3) и D2) имеем с учетом D0), что Г е^-*)л|Ф(А)|2йА= Г Ф(\)eisXЩj^d\ = ^2cs-jct-j=r(s,t). D4) J — тг •/ — тг Функция Ф( •) G L2[—тг,тг], то есть, |Ф( • )|2 G L1[—тг,тг]. Поэтому D4) показывает, что если имеет место C9), то функция д(Х) = |Ф(А)|2, A G [—тг, тг], есть спектральная плотность процесса X = {Xt, t G Z}. Для доказательства обратного утверждения предположим, что д = д(А) — спектральная плотность процесса X = {Xt, t G Z}. Тогда д( •) G L1[—тг,тг] и g(A) ^ 0 п.в. по мере Лебега (для спектральной меры G процесса X имеем G(B) = J g(X)d\, В е Й8([-7г,тг])У Обозначим Ф(А) = л/дЩ (на множестве меры нуль, где д(Х) < 0, полагаем Ф(А) = 0). Тогда Ф(А) G Ь2[-тг, тг], а полнота и ортонормированность в этом пространстве системы функций {Bтг)~1/2ег/сЛ, A G М}, & G Z^aiOT оо /с = —оо где {с/е} G I2 и ряд D5) сходится в L2[—тг, тг]. Снова пользуясь равенством Парсева- Парсеваля, имеем , - ?) = Л ei(s"t)A^(A) ^A = J — тг = Г е^АФ(Л)е**ЛФ(Л)ЙЛ = ^cs_fcct_fe. D6) ^ — тг и Возьмем Л = Z, пусть \i — считающая мера на сг-алгебре Й§ всех подмножеств Z, т.е. сг-конечная мера, сосредоточенная на множестве Z, для которой [х({к}) = 1, к е Z. Пусть f(t,a) = с*-а,где*,а G Z, {ск} G L2(A, ^,/i), т.е. {с^} G /2. Тогда
246 А. В. Булинский, А. Н. Ширяев формула D6) может быть переписана в виде A8) и, следовательно, по теореме 3 су- существует (возможно, на расширении (П, ^, Р) исходного вероятностного простран- пространстве (П, ^, Р)) центрированная ортогональная случайная мера Z( •) такая, что Xt= f(t,a)Z(da) = JA D7) k = — oo = Z({k}), а ряд в D7) сходится в L2(Q, ^ P). При этом центрированные вели- величины г /с, к Е Z, будут не только ортогональны, но и ортонормированы, поскольку Замечание 4. Из данного доказательства видно, что представление C9) рав- равносильно тому, что стационарный в широком смысле центрированный процесс X = {Х±, t G Z} обладает следующей спектральной плотностью: 1 Л G [-7Г,7Г]. § 14. Остановимся на важном для приложений вопросе статистического оцени- оценивания спектральной плотности,, показывающей, как уже отмечалось выше, с ка- какими "весами" в спектральное представление стационарного процесса входят те или иные спектральные частоты. Пусть X = {Xt, t G Z} — центрированный стационарный в широком смысле процесс с ковариационной функцией R(п) = EXn+/cX/c, &,n G Z. Формула B9) показывает, что функция ^дг(Л), введенная в B8), может рассматриваться как ап- аппроксимация спектральной плотности g(Л) (когда последняя существует). Отсюда следует, что если в качестве оценки ковариационной функции R(m) по наблюдениям Xq , Х\,..., Xjsf—1 взять величину RN(m) = N-rn-1 N -m RN(-m) 0 Хш+кХк для 0 ^ m ^ N - 1, для - (N - 1) ^ m < 0, для остальных m G Z, D8) то естественной оценкой для спектральной плотности g(Л) будет (ср. с B8)) функция 1 2^ -iAm \т\ Простые преобразования показывают, что функция дд/- (А) может быть представлена в виде 2tt7V iV-l E- /c=0 n — i\k D9)
Гл. VII. Стационарные процессы. Дискретное и непрерывное время 247 называемом периодограмм ой. Легко видеть, что поскольку ER iv(m) = R(m) длят таких, что т N — 1, то Заметим, что = gN(\). E0) к=0 1=0 N-1N-1 Г l -I' J —7Г где функция (ядро Фей ер а) к=о 1=0 iV-liV-1 2ttN k=o 1=0 N-l 2 i(v-\)k k=0 -\)g{v)dv, g(y) dv = 2ttN N-l k=0 ei\k /sinGVA/2)\ = ) 2ttN V sin(A/2) N E1) A = 2тг?7г, ттг G Z. Таким образом, формулы E0) и E1) показывают, что периодограмма является асимптотически несмещенной оценкой спектральной плотности, если для п. в. A G [-7Г,7Г] /* Фдг(г/ - \)g(v) dv ->• TV ->• оо. E2) Из свойств ядра Фейера вытекает (см., например, [35; гл. VIII, §2]), что так будет для всякой функции g Е С[—тг,тг]. Однако среднеквадратическая ошибка Е |^iv(A) — #(А)|2 с ростом N, как правило, не стремится к нулю. Поэтому на практике вместо оценок ^дп(Х) используются сглаженные оценки ^]^, т.е. оценки вида ~w Г N J^ N N где так называемые "спектральные окна" Wn( •), N G N, определяющие эти оцен- оценки, выбираются так, чтобы: а) функция Wn (А) имела резкий максимум в окрестности точки 0; б) Г WN(X)d\ = 1; в) Е\д%(\) - д(Х)\2 -^OupuN ^ оодляА G [-тг,тг].
248 А. В. Булинский, А. Н. Ширяев Например, оценки Бартлета строятся с помощью окон где л 2 а последовательность адг /* оо, адг/N ->• О (N ->• оо). По поводу условий а), б), в) и других видов спектральных окон см., например, [85; гл. VI, §4], а также [6, 80]. § 15. Один из самых интересных (и с теоретической точки зрения, и с точки зре- зрения приложений) разделов теории стационарных процессов — это раздел, связанный с прогнозом (экстраполяцией) "будущих" значений этих процессов по их "прошлым" значениям. Начнем с простейших постановок задачи прогноза. Пусть дан L2-nponecc X = = {X(t), t G T}, Tel. Требуется найти наилучшее приближение случайной ве- величины X(t) (по норме || • || пространства L2(Q)), основываясь на предыстории про- процесса X до некоторого момента s < t. Если считать, что мы располагаем всеми вели- величинами у, измеримыми относительно сг-алгебры 3<s — а{Х(и), и ^ s,u G Т} и квад- квадрат модуля которых интегрируем, т. е. рассматриваемые величины у G L2 (П, 3*s, Р), то дело сводится к отысканию величины и того значения у, на котором этот inf достигается (такой элемент существует). Согласно теории гильбертовых пространств Н всякий элемент х G Н может быть представлен (для заданного подпространства L С Н), и единственным обра- образом, в следующем виде: х = у + z, где у G L и z G lA (ортогональное дополнение к L в Н). Величина у обычно обозначается Рг^ ж, где оператор Рг^ называется ор- топроектором в Н на L. Тем самым, беря Н = L2(Q, ^, Р) и L = L2(Q, §<Sl P), находим, что В упражнении 1 главы IV отмечалось, что если сг-алгебра si С 3* и случайная величина У G ?2(П,«?, Р), то (Р-п.н.) Таким образом, оптимальной в среднем квадратическом смысле оценкой для X(t) по величинам пространства L2(Q, 3S, P) является условное математическое ожидание E(X(t)\&a). Нахождение условного математического ожидания относительно сг-алгебры 3's является, как правило, довольно сложной задачей. Однако если ограничиваться только линейным прогнозом, основанным на линейных комбинациях величин Хи cu^s,u€T(w пределах таких величин в пространстве L2(Q)), то задача сущест- существенно упрощается. Для ее решения весьма полезны методы теории гильбертовых пространств, позволяющие также получить и ряд интересных результатов о струк- структуре некоторых важных классов стационарных процессов.
Гл. VII. Стационарные процессы. Дискретное и непрерывное время 249 Имея в виду изучение линейного прогноза, введем для sGT пространство Hs (X) как замыкание в среднем квадратическом линейной оболочки величин Х(и),и ^ s, и Е Т, и положим Я_оо(Х) = р| HS(X), Н(Х) = L2[X], зет где Ь2[Х] — замыкание в L2(Q) линейной оболочки всех величин Х(и), и Е Т. Яс- Ясно, что Н(Х) — наименьшее подпространство Ь2(П), содержащее все пространства Hs(X),s Е Т. Рассматриваемая нами задача линейного прогноза величины X(t) по "прошлому" до момента времени s ставится теперь как задача наилучшего прибли- приближения X(t) (по норме пространства Ь2(п)) элементами пространства Н3(Х). При этом ошибкой линейного прогноза естественно называть величину A(s,t)=mi{\\X(t)-y\\:yeHs(X)}. E3) Решением поставленной выше задачи линейного прогноза является величина PsX(t), где для краткости Ps обозначает ортопроектор Pi#s (х) (вН(Х)нз>Н3(Х)). Введенная ошибка линейного прогноза A(s,t) удовлетворяет следующему свой- свойству: A(s, t) = A(s + и, t + и) при всех s,t,u?T. E4) (Достаточно заметить, что если у = c\X{s\) + ••• + cnX(sn) G HS(X) и z = = c1X(s1 +u)-\ \-cnX(sn +u) e Н3+и(Х),гдеск G C, sk G T, sfc ^ s, то в силу стационарности процесса X имеем \\X(t) - у\\2 = \\X(t + и) - z\\2.) Из E4) следует, что величина A(s, s + u) не зависит от s и, следовательно, можно положить 5(и) := A(s,s + u), и G Г; при этом <5(г/) = 0 при и ^ 0 (u G Т) и ?(г>) ^ <5(г/) для v ^u (v,u G Т). E5) Смысл свойства E5) вполне ясен: ошибка прогноза величины X (?) по "прошлому" Hs (X), где s ^ t, растет с убыванием s. §16. Вопрос поведения ошибок прогноза S(t), t ^ 0, оказывается тесно связанным со следующими понятиями. Определение 11. Пусть Г = R или Г = Z и X = {X(t), t G Г} - центри- центрированный стационарный случайный процесс. Процесс X называется сингулярным, или. детерминированным, если Н-^Х) = Н(Х), т.е. HS(X) = Щ(Х) при всех s,t G Т. Процесс X называется регулярным или чисто недетерминированным, еслиЯ_оо(Х) = 0. Теорема 11. Центрированный стационарный процесс X является 1) сингулярным тогда и только тогда, когда S(to) = 0 для некоторого t0 > 0, t0 G Г (тогда S(t) = 0 npn всеж ? G Г); 2) регулярным тогда и только тогда, когда S2(t)^R(O) при t-юо, E6) где R@) — значение в нуле ковариационной функции R(t) процесса X (в нашем случае R(t) = ЕХ(?)Х@)).
250 А. В. Булинский, А. Н. Ширяев Доказательство. 1) Если процесс X сингулярен, то X(t) G Ht(X) = HS(X) и, следовательно, A(s, t) = 0 при любых s, t G Т. Пусть теперь S(to) = 0 для некоторого to > 0 (to G Т). Тогда X(s + to) G HS(X) для любого s G Т. В силу E5) имеем X(t) G HS(X) для t ^ s + to- Отсюда сле- следует, что Hf(X) = HS(X) при s ^ t ^ s + to- Ввиду произвольности s видим, что процесс X сингулярен. 2) Пусть выполнено соотношение E6). Для s,t ? Т имеем Д@) = ||Х(*)||2 = \\PsX(t)\\2 + \\X(t) - PsX(t)\\2 = \\PsX(t)\\2 + S2(t - s). E7) Поэтому ||PsX(t)|| —> 0 при s —У —оо для каждого t G Т. Теперь заметим, что если Li, Z/2 — подпространства гильбертова пространства Н и L\ С 1/2, то II-Pli^H ^ ||-Pl2^II Для любого х е Н. Таким образом, ||P_ooX(t)|| ^ ||PsX(t)|| для каждых s,t € Т. Итак, P_ooX(t) = 0, т.е. X(t)J-H-oo(X) для t € Т, откуда Н(Х)±Н-ОО(Х). Поскольку Я_оо(Х) С Н(X), получаем, что Я_оо(Х) = 0. С другой стороны, пусть Н-ОО(Х) = 0. Учитывая E7), видим, что требуемое соотношение E6) непосредственно вытекает из следующего утверждения. Лемма 3. Пусть Hi, t G T, — убывающее семейство подпространств гиль- гильбертова пространства Н, т. е. Hs С Ht при s ^ t, s,t G Т. .Еслг/ Q Hs = О, то Psx —У 0 npn s —>¦ — оо <5л^ каждого х G Я, г<?е Ps - ортопроектор (в Доказательство. Поскольку ||Psx|| ^ ll^t^ll для каждого ж G Я при s ^ t, то достаточно показать, что ||^^Ж11 ~^ 0 при к ->• оо, где t^+i < t^, /с G N и lim t^ = /с—>>оо = -оо. Обозначим Lk := Я*й 0 Я^+1, т.е. пусть Щк+1 Ф Lk = Щк,к еК Тогда для любого х G Я и ттг > к имеем т — 1 т — 1 где Qj—ортопроектор в Я на Lj. Отсюда ||Ptfcx||2 = ll^tm^||2 + Yl \\Qjx\\2- j=k оо довательно, J^ IIQj^ll2 ^ ll^ti^H2 ^ IM|2- Таким образом, при т > к 3 = 1 \\ptkx-Ptmx\\'z = ^\\Qjx\\'z ^0, если fe,i (очевидным образом можно рассмотреть и случай к ^ т). В силу полноты прост- пространства Я существует предельный элемент у = lim Ptkx. Поскольку Ptkx G Htrn оо при к ^ т, то ^/ G Я^т для любого тгг G N. Получаем, что у G П Я^т. В силу т=1 оо монотонности семейства Ht выполнено свойство f] Htrn = П Ht. Так как по 771=1 предположению Q Ht = {0},r т — 1 \2 = ^2 \\Qjx\\2 Тем самым, лемма 3 и теорема 11 доказаны. ?
Гл. VII. Стационарные процессы. Дискретное и непрерывное время 251 § 17. Материал, излагаемый в этом параграфе, носит технический характер. Читателю рекомендуется сначала обратиться к следующему § 18 и затем вернуться к чтению этого параграфа. Введем в Н(Х) операторы сдвига Su,u G Т (Т = Ж или Т = Z), положив Su(ciX(*i) + • • • + cnX(tn)) = ciX(ti + и) + • • • + cnX(tn + и), где Ck G С, tk G T, к = 1,..., n, n G N. Данное определение корректно. Действи- п m тельно, если Yl ckX(tk) = Yl djX(sj), где dj G С, Sj G Г, j = 1,..., m, m G N, то fc=i j=i в силу стационарности X имеем для и G Т 0 = п и) 2 Аналогично устанавливается, что при каждом и G Т (Sux,Suy) = (х,у), х,уе Lin{X(t), t G Г}. Иначе говоря, Su — изометрический оператор на линейной оболочке Lin{X(t), t G Г} величин X(t), t G Г, причем \\Sux\\ = ||ж|| для х G Lin{X(t), t G Г}, и ?Т. Следовательно, 5W однозначно продолжается до изометрического оператора наЯ(Х). Нетрудно показать (проверив вначале на элементах из Lin{X(t), t G Т}), что Ew)wgt образует группу операторов в Н(Х), т.е. 5W+V = 5W5V для u,v ?T, где So = I — тождественное отображение. Итак, построенные операторы сдвига Su,u G Г, образуют группу изометричес- изометрических (значит линейных) операторов на Н(Х). Лемма 4. Дл^ любых u,v G Т E8) где Pt — ортопроектор в Н(Х) па Щ(Х), t G T U { —оо}. Доказательство. Длям,г> G Г, очевидно, что 5w(Lin{X(?), t ^ v, t e Г}) С С HU+V(X). Поэтому SUHV(X) С HU+V(X). Учитывая, что SUS-U = /, имеем ^w+v(^) = SUS-UHU^.V(X) С SUHV(X). Первое равенство в E8) доказано. Еслиж G i^-oo^), тож G Щ(Х) для всех ? G Т. По доказанному 5W x G Щ+и(Х) для любого ? G Г. Следовательно, 5^ж G Я_оо(Х), т.е. SUH-OO(X) С Н-^Х). Таким образом, () 5и5_иЯ_оо(Х) С SuH- что завершает доказательство второго равенства в E8).
252 А. В. Булинский, А. Н. Ширяев В силу линейности операторов Su, Pv, u,v G T, для доказательства равенств в E9) достаточно проверить совпадение результатов действия на X(t), t G Т, соот- соответствующих операторов, фигурирующих в этих равенствах. Имеем Pu+vSuX(t) = Pu+vX(t + и) для u,v,t G Т. С другой стороны, X(t) = = yv(t) + zv(t), где Vv(t) = Pvx{t) e hv(x), Zv(t)±Hv(x). Далее, X(t+u) = SuX(t) = Suyv{t)-\-SuZv{t)^TjxeSuyv{t) ? HU+V(X), а для любого h G HU+V(X) в силу первого равенства E8) получаем h = Sug, где g G HV(X). Отсюда (/i,S^(?)) = (?,*«(*)) =0. Если гильбертово пространство Я = L0L, то любой элемент ж G Я единственным образом записывается в виде х = у + z, где у € L, z € lA. Поэтому SuPvX(t) = — Suyv{t) = Pw+-yX(t + u). Тем самым первое равенство в E9) доказано. Аналогично, P_oo5wX(t) = Р_ооХ(?+?/), Х(?) = 2/_(Х5(?)+г_(»(?),где2/_оо(?) G G Я_оо(Х)и^_оо(^)_1Я_оо(Х). ТогдаХ(^+^) = SuX(t) = S^-ooW + S^-ooW- Поскольку Suy-oo(t) G Н-ОО(Х) и 5lt2:_oo(t)_LJH"_oo(X) (согласно второму равен- равенству в E8)), то SuP-ooX(t) = Suy-oo(t) = P-ooX(t + и). П § 18. Введенные выше понятия сингулярности и регулярности процессов оказы- оказываются полезными при описании общей структуры стационарных процессов. Теорема 12 (Вольд). Центрированный стационарный в широком смысле процесс X = {X(t), t G Г}, где Т = R или Т = Z, допускает (для каждого uj G П) разложение вида x(t) = M(t) + N(t), teT, F0) где М = {М(?), t G Г} — сингулярный, а N = {N(t), t G T} — регулярный про- процессы, причем M_L7V, т.е. M(s)J_iV(?) <?л^ s,t ? Т. Кроме того, процессы М и N стационарны в широком смысле и центрированы. Указанное разложение процесса X единственно, если потребовать следующее условие подчиненнос- подчиненности: M(t) G Ht(X) (тогда и N(t) G Щ(Х)) при каждом t G Г. Доказательство. Пусть M(t) = Р-^Х^) и N(t) = Х(?) - М(?) для t G Г. Ясно, что так определенные процессы М = {М(?), ? G Г} и TV = {iV(?), ? G Г} приводят к представлению F0). По определению процессов М и N имеем M(t) G Н-ОО(Х) и 7V(t)_Li?_oo(X) для t еТ. Следовательно, M±N. Из F0) вытекает,чтоX(t) G Ht(M)®Ht(N), поэтому С Я*(М) 0 Ht(N). С другой стороны, M(t) G Я-ооРО С Я*(Х). Отсюда С Щ(Х), и тогда в силу F0) имеем Я*(^) С Щ(Х) при t G Г. Таким образом, Ht(M) 0 Ht(N) С Ht(X) и получаем fft(X) = ^(М) ®Ht(N), te Т. F1) Теперь заметим, что Ht(N)±H-oo(X) при t Е Т, откуда H-OO(N)±H-OO(X). В то же время Я*(iV) С Щ(Х), поэтому H-OO(N) С Я_оо(Х). Итак, H-OO(N) =
Гл. VII. Стационарные процессы. Дискретное и непрерывное время 253 О, т.е. N — регулярный процесс. Мы знаем, что Щ(М) С Н-ОО(Х), t Е Т. Из F1) следует, что Н-ОО(Х) С Щ(М) 0 Ht(N). Учитывая, что Ht(N)±H-oo(X)J получаем Н-ОО(Х) С Ht(M), t е Т. Следовательно, Ht(M) = Н-ОО(Х), t е Г, и М — сингулярный процесс. Проверим, что М, 7V — стационарные (в широком смысле) процессы. Поскольку M(t) е Н-ОО(Х) С Н(Х), то ЕМ(?) = 0, и, следовательно, EiV(t) = 0, t Е Г. Таким образом, М w N — центрированные процессы. Для t,u,v G Т, пользуясь леммой 4 и изометричностью оператора Su, имеем (МО + u),M(t + и)) = (P-ooX(v + w), P-ooX(t + и)) = = (SuP-ooX(v),SuP-ooX(t)) = = (M(v),M(t)). Аналогично, u),N(t + u)) = (X(v + и) - M(v + u),X(t + и) - M(t + и)) = = (X(v),X(t)) - (P-ooSuX^SuXit))- - (SuX{v),P-ooSuX(t)) + (M(v),M(t)) = (M(v),M(t)) = = (X(v) - M(v),X(t) - M(t)) = Тем самым, процессы М и TV, обеспечивающие представление F0), являются про- процессами, стационарными в широком смысле. Докажем теперь единственность разложения F0). Допустим, что наряду с F0) имеет место другое разложение X(t) = U(t) + V(t), где U = {U(t),t G T},V = = {V(t), t G T} — соответственно сингулярный и регулярный процессы, причем U±V и U(t),V(t) eHt(X),te Т. Тогда щ(х) = Ht(u) + Ht(v) = н(и) e Ht(v), t e r. Следовательно, = f) (н(и) е я*(т/)) = н(и) е f| я*(У) = щи) + о = В итоге находим, что M(t) = P-ooX{t) = PvH{u) X(t) = = PrH(t7) Uit) + PvH{u) V(t) = PvH{u) Uit) + 0 = Uit), и, очевидно, N(t) = V{t) для всех t G T. Все утверждения теоремы 12 доказаны. ?
254 А. В. Булинский, А. Н. Ширяев Замечание 5. Разложение F0) было впервые получено Вольдом для стационар- стационарного процесса с дискретным временем. Г. Крамер доказал, что любой центрирован- центрированный L2-nponecc X (необязательно стационарный), заданный на множестве Г С 1, допускает разложение на ортогональные компоненты, одна из которых сингуляр- сингулярна, а другая — регулярна. Условие подчиненности (см. теорему 12) обеспечивает единственность упомянутого разложения. Доказательство теоремы 12 проведено таким образом, что нами установлен и результат Крамера. Полезно подчеркнуть, что в то время как стохастические представления C0) и C8) действуют в спектральной (частотной) области, разложение F0) носит иной ха- характер — оно действует во временной области. Отметим также, что лемма 4 позволяет немедленно получить соотношение E4). В самом деле, A(s + u,t + u) = \\X(t + и) - Ps+uX(t + и)\\ = \\SuX(t) - Ps+uSuX(t)\\ = = \\SuX(t) - SuPsX(t)\\ = \\X(t) - P8X(t)\\ = A(s,t). § 19. Рассмотрим более детально структуру регулярных процессов. Определение 12. Белый шум г = {еп, п Е Z} называется обновляющим про- цессом для центрированного L2-nponecca X = {Хп, n Е Z}, если Нп(Х) = Нп{е) при всех п Е Z. С помощью этого понятия описание всех регулярных процессов с дискретным вре- временем, дается в следующем предложении. Теорема 13. Для того, чтобы невырожденный центрированный стацио- стационарный в широком смысле процесс X = {Хп, п G Ъ\ был регулярным, необхо- необходимо и достаточно, чтобы нашлись обновляющий процесс г = {гп, п G Z} и последовательность комплексных чисел {сп}^=0 G I2 такие, что п. н. оо Хп = ^2 ck?n-k, пеЪ, F2) k=o где ряд сходится в среднем квадратическом. Доказательство . Пусть X — регулярный процесс. Легко видеть, что для каж- каждого т G Ъ пространство НШ(Х) состоит из замыкания в L2(Q) элементов вида hm-i + сХ(т), где /im_i G НШ-1(Х), с е С. Поскольку Х(т) = gm-i + ^ш, где gm-i G НШ-\(Х) и zrnJ-Hrn-i(X), то НШ(Х) представляет собой замыкание в 1/2(П) элементов вида /im_i + сгш, а значит, в точности состоит из всех элементов такого вида. Заметим, что гш ф 0 @ — класс функций в L2 (Q), равных 0 п. н.). Дей- Действительно, если гш = 0, то тогда НШ-\(Х) = НШ(Х) и в силу E8) получили бы, что Hn-i(X) = Нп(Х) для любого п G Z,4to противоречит регулярности. Выберем ?n = 2n/||;zn|| и воспользуемся рассуждениями, проведенными при доказательстве леммы 3. А именно, для фиксированного п G Z возьмем последова- последовательность tk = п — к, к G Z+. В рассматриваемом случае подпространство L& := := Htk 0 Щк+1 порождается вектором ?n_fc (к G Z+). Поскольку ||Р^А,^(^)|| —> 05
Гл. VII. Стационарные процессы. Дискретное и непрерывное время 255 к —у оо, то приходим к равенству типа F2), в котором к=0 Следовательно, F2) доказано для произвольного фиксированного п Е Z, поэтому коэффициенты ск, k Е Z, вообще говоря, зависят от п. Другими словами, пока лишь установлено, что k=0 где ряд сходится в Ь2(п) и с^. = (Хп,?п-к), п,к Е Z. Для получения F2) проще всего теперь перейти к другому базису в Н(Х). А имен- именно, положим ек = Sk^o, к Е Z (по-прежнему, S& — оператор сдвига в Н(Х)). Учи- Учитывая E8), видим, что г/с Е Нк(Х) и ?д._1_Я&_1(Х). Кроме того, ||е^|| = ||?о|| = 1, fc E Z. Поскольку одномерное подпространство Нк(Х) 0 Нк-\(Х) порождается вектором ^, то, следовательно, ек = ак?,к, ак Е С, fc E Z и {?&} — обновляющий процесс для процесса X. В итоге оо оо к=0 к=0 В силу непрерывности и линейности операторов Sn (n G Z), образующих группу, получаем оо оо оо оо Хп = SnX0 = ^ ckSne-k = ^ ckSnS-k(;o = ^ ckSn-k(;o = ^ скгп-к. к=0 к=0 к=0 к=0 Докажем теперь достаточность условия регулярности. Если г = {гп, n G Z} — произвольный белый шум (т.е. необязательно обновляющий процесс для X), то F2) показывает, что X — центрированный стационарный процесс, причем Нп(Х)с CHn(s),n G Z. Отсюда Я_оо(Х) С Нп(е) для любого п G Z. Но sn+i-LHn(s), п G Z, поэтому Sn-LH-oo (s) для всех п G Z. В то же время г = {гп, n G Z} — базис в Н(Х). Следовательно, Н-ОО(Х) = 0, т.е. процесс X является регулярным. ? Следствие 4. Центрированный стационарный процесс X = {Xn, n G Z} является регулярным тогда и только тогда, когда он задается физически осу- осуществимым фильтром, на вход которого подается белый шум. Доказательство. Если процесс X регулярен, то согласно теореме 13 этот про- процесс записывается в виде F2) с белым шумом е = {sn, n G Ъ\ (являющимся обнов- обновляющим процессом). Пусть теперь X задается физически осуществимым фильтром, т.е. формулой F2) с некоторым белым шумом г = {гп, п G Z} (не обязательно обновляющим процессом). Из доказательства достаточности теоремы 13 вытекает регуляр- регулярность процесса X, значит, найдется и обновляющий процесс е, обеспечивающий разложение F2). ?
256 А. В. Булинский, А. Н. Ширяев § 20. Полученные результаты позволяют легко найти величину ошибки линейно- линейного прогноза на п шагов. Заметим, что для произвольного центрированного стационарного процесса X = {Хп, п Е Z} разложение Вольда приобретает вид оо Хп = Мп + Nn = Мп + ^ скгп-к, п е Z, F3) к=0 где Мп — сингулярная составляющая, г = {гп, п Е Ъ } — обновляющий процесс для регулярной компоненты Nn (n Е Z), а числовая последовательность {ск}^0 G I2. Теорема 14. Для процесса X = {Хп, п G Z}, задаваемого формулой F3), ошибка прогноза 8(п) на п шагов вперед дается формулой п-1 пе k=0 Доказательство. Учитывая,чтоРг#т(х) Мп+т = Мп+тиЯтA) = Нт(е) для тгг, n G Z, получаем с учетом представления F2), что = Е \Хп+ш - РгЯт(х 2 = Е | s n+m = Е п—1 2 ra — 1 . ? F4) Формула F4) еще раз показывает (сравните с E6)), что для регулярного центри- центрированного стационарного процесса X = {Xn, n G N} справедливо (при п —у оо) следующее свойство: k=0 где R = R(n) — ковариационная функция X. Из замечания 4 и следствия 4 вытекает Следствие 5. Центрированный стационарный процесс X = {Хп, п Е Z} является регулярным тогда и только тогда, когда он обладает спектральной плотностью g = g{X), задаваемой формулой оо 2 ^C/ce"i/cA , Ае[-тг,тг], {ск}е12. к=0 Исчерпывающее описание спектральных плотностей, допускающих такое пред- представление, дает Теорема 15 (Колмогоров). Стационарный процесс X = {Хп, п Е Z} с ну- нулевым средним является регулярным тогда и только тогда, когда его спек- спектральная плотность g = g(X) удовлетворяет условию Г. lng(A) d\ > —оо.
Гл. VII. Стационарные процессы. Дискретное и непрерывное время 257 Доказательство этого результата, основанное на описании граничных свойств аналитических в единичном круге функций класса Харди Ж^-, см., например, в [85; т. 2, с. 621]. § 21. Согласно теореме 14 величина ошибки прогноза 6A) на один шаг равна ве- величине |со|, где со определяется разложением Вольда F3). Оказывается, для про- процессов, имеющих спектральную плотность, величина 62A) = |со|2 допускает иное выражение в терминах спектральной плотности, определяемое ниже формулой Кол- могорова-Сегё (см. теорему 16). Обозначим L1 = L1([—тг,тг], ^([—7r,7r]),mes), где mes — мера Лебега. Для неот- неотрицательной функции v арифметическое среднее A(v) и геометрическое среднее G(v) определяются формулами A(v) = -!- Г v(X) d\ G(v) = exp{A(\nv)} 27Г J-7T в предположении, что г? и In г? интегрируемы на отрезке [—тг, тг] (формально полагаем 1пО = -оо). Пусть f(X)=2ng(X), Ае[-7г,тг]. F5) Теорема 16 (Колмогоров-Сегё). Для центрированного стационарного про- процесса X со спектральной плотностью g = g(X) либо In/ G L1 и тогда 62A) = G(f) (= expj— / ln/(A)dA либо In/ ^ L1 и тогда 6A) = 0. Доказательство этой теоремы, иллюстрирующее связь вероятностных задачи задач классической теории функций, отнесено в приложение 5. Дополнения и упражнения 1. Покажите, что пуассоновская случайная мера (см. §9 главы I), заданная на полукольце ограниченных борелевских множеств в Жп, является ортогональ- ортогональной (нецентрированной) случайной мерой. 2. Пусть {Z\, X G М} —комплекснозначный L2-nponecc такой, что 1) Е \Z\ — Zv\2 —У 0 для любого i/Gl при А ^ г/, 2) приращения ортогональны, т.е. для всех Ai < A2 < A3 E(ZA2-ZAl)(ZA3-ZA2)=0. На полукольце Ж = {(а, 6], —оо < а ^ Ъ < оо}, где (а,а\ = 0, введем семейство случайных величин Z((a, b]) := Z(b) — Z(a). Докажите, что Z — ортогональная случайная мера на Ж.
258 А. В. Булинский, А. Н. Ширяев Если мера порождается указанным выше процессом Z\, A Е М, то вместо интег- интеграла / /(A) Z(dX) используют также запись / /(A) dZ\. 3. Пусть {Z\, A Е М} — центрированный гауссовский процесс с ортогональ- ортогональными приращениями, непрерывный справа в среднем квадратическом (выяс- (выясните, обязан ли тогда процесс быть непрерывным слева в среднем квадрати- квадратическом). Пусть при каждом t Е Т функция g(t, -):Ж —У С удовлетворяет условию L <oo, где \i — структурная мера, отвечающая ортогональной случайной мере Z, порожденной процессом {Z\, A Е М} (см. упражнение 2). Докажите, что тогда процесс Y = lr(t) = J g(t,X)dZx, t является центрированным гауссовским процессом. 4 (замена меры в интеграле). Пусть Z — ортогональная случайная мера, за- заданная на П х ^ и имеющая сг-конечную структурную функцию (меру) \± на из- измеримом пространстве (Л, d) (система множеств ^ = {А Е d: fi(A) < оо}). Пусть h: Л -»> С, h E I/2(A, d, /i). Введем M>=<Bed: / \h(X)\2 fi(dX) < оо } и положим V{B) = f 1в(А)Л(А) Z(dA), БеЖ F6) •/Л Объясните, почему V является ортогональной случайной мерой (центриро- (центрированной, если центрирована мера Z) с сг-конечной структурной мерой и(В) = / \h(X)\2 /i(dX) < оо, В еЖ. JB Докажите, что еслид: А —у С, д Е Ь2(А, cr{J^}, г/), то / ^(А) V(dX) = / ^(А)Л(А) Z(dA). F7) Ja J a В связи с теоремой Карунена (см. также [38, 58, 134]) вновь обратимся к вопросу о канонических представлениях случайных процессов. Рассмотрим пространство Ь2[а,Ь], состоящее из комплекснозначных функций, квадрат модуля которых интегрируем по Лебегу на [а, Ь]. Скалярное произведение в этом гильбертовом пространстве определим, как обычно, формулой (/,<?)= / WWJdt, f,geL2[a,b]. J a
Гл. VII. Стационарные процессы. Дискретное и непрерывное время 259 Следующий пример является хорошей иллюстрацией теоремы 3. Пример 2. Для винеровского процесса W на отрезке [0, 2тг] справедливо пред- представление (Карунена) где Zk — ортонормированные центрированные случайные величины, ряд сходится при каждом t Е [0, 2тг] в среднем квадратическом (при к = 0 считаем A — elkt)/ik = = -*)• Действительно, для s, и Е [0, 2тг] имеем оо 53 cfc(8)eifctt. F9) 1 оо 5 к = — оо Функции A/л/2тг)ег/с^, к Е Z, образуют полную ортонормированную систему в пространстве 1/2[0, 2тт]; при каждом s G [0, 2тг] ряд F9) сходится в этом простран- пространстве и коэффициенты Фурье о 3](и)е~гки du = —т=A - e~lks)lik, гтт Jo л/2тг (co(s) = s/y/2ir). Равенство Парсеваля показывает, что при s, t G [0, 2тг] г , . . 1 ^ (l-e-iks)(l-eikt) mm{s,t\ = (l[o,s],l[o,t]) = тг- >. г^ • Z7T /е /с = —оо Представление F8) следует из теоремы 3, в которой надо положить Л = Z, и выбрать меру \± считающей мерой на Л, т. е. такой, что /i({fc}) = 1, fc G Z. П 5. Докажите, что представление F8) справедливо (п.н.) с независимыми слу- случайными величинами z^ ~ N@,1), k G Z. Точнее говоря, докажите, что при таком выборе величин z^ (к G Z) правая часть F8) имеет непрерывную модификацию, являющуюся центрированным гауссовским процессом с кова- ковариационной функцией min{s, ?}, s,tG [0, 2тг]. Приведем некоторые необходимые нам в дальнейшем утверждения, относящиеся к теории операторов в гильбертовых пространствах. С непрерывной на [а, Ь] х [а, Ь] ковариационной функцией г = r(s,t) свяжем огра- ограниченный линейный оператор Фредголъма А, который действует в L2[a, b] и опре- определяется следующим образом: гЪ Af(s)= r(s,t)f(t)dt, feL2[a,b]. G0) J a Функция г = r(s,t) называется ядром интегрального оператора А.
260 А. В. Булинский, А. Н. Ширяев 6. Пусть X = {X(t),t Е [а, Ь]} — комплекснозначный!/2-процесс, непрерывный в среднем квадратическом на отрезке [а, Ь]. Докажите, что это свойство рав- равносильно тому, что на [а, Ь] непрерывна функция m(t) = Е X(t) и на [а, Ъ] х [а, Ъ] непрерывна ковариационная функция г(s,t) = co\/(X(s),X(t)). 7. Докажите, что оператор А, заданный формулой G0), компактен, т. е. всякое ограниченное по норме множество оператор А переводит в предкомпактное (т. е. множество, замыкание которого есть компакт). Введенный оператор А — оператор Гильберта-Шмидта (см. [35; гл. IX, § 2, п. 1]), поскольку ГЪ r(s,t)\2 dsdt < oo. G1) rb rb J a Ja Заметим также, что оператор А является самосопряженным, поскольку (см. [35; гл. IX, § 2, п. 2]) его сопряженный оператор А* задается эрмитово сопряженным ядром r(t, s), а для ковариационной функции r(s,t) справедливо равенство r(s,t) = r(t, s) при всех s, t. Напомним (см. [35; гл. 4, §6]) классический результат. Теорема 17 (Гильберт-Шмидт). Пусть А — компактный самосопряженный оператор в (комплексном) гильбертовом пространстве Н. Тогда существует ортонор мир о ванная система {фп, п G J} собственных векторов оператора А, отвечающих ненулевым собственным значениям, такая, что любой элемент h G Н записывается единственным образом в виде пфп + и, G2) где сп G С, п G J, и G КегА (т.е. Аи = 0). Множество J всегда конечно или счетно (в последнем случае ряд G2) сходится по норме \\-\\ = (-,-I/2). Более того, в силу компактности оператора А число собственных значений Хп во внешности любого круга с центром в 0 конечно и поэтому их можно перенумеровать в порядке невозрастания модулей \\\\ ^ |А2| ^ •••• Если J счетно, то lim An = 0. п—юо Отметим, что собственные значения самосопряженного оператора А в Н действи- действительны, а собственные векторы, отвечающие разным собственным значениям, орто- ортогональны, в частности, (фп,и) = 0, п G J. Для оператора А из G0) в силу сепарабельности Н = L2[a, b] в подпространстве Кег А можно выбрать ортонормированный базис {фк}кем, где М — конечно или счетно (считаем множества М и J непересекающимися; если Кег А = 0, то М = 0). Дополнив систему {фп,п G J} до ортонормированного базиса {фп,п G JU М}в Н, перепишем G2) в виде разложения по собственному базису оператора А: h= Y^ скФк, ск = (Н,фк), keJUM. G3) keJuM Если семейство индексов JUM счетно, то ряд Фурье в G3) сходится по норме L2 [а, Ь].
Гл. VII. Стационарные процессы. Дискретное и непрерывное время 261 В силу упражнений б и 7 для ковариационной функции г непрерывного в среднем квадратическом процесса X = {X(t), t G [a, b]} имеем согласно G3), что при каж- каждом t Е [а, Ъ] где Тем самым, fb = / Ja Если J — счетно, то ряды G3), G4) сходятся в L2 [а, Ь] при любом t Е [а, Ь]. Отметим, что здесь все А& действительны и А& = 0, к G М. 8. Докажите, что (в силу неотрицательной определенности функции г) А& > О при к G J. Следующая теорема показывает, что соотношение G4) допускает усиление (см., например, [60; с. 263]). Теорема 18 (Мерсер). Пусть г = r(s,t) — непрерывная на [a,b] x [a, b] кова- ковариационная функция. Тогда для всех s,t G [а, Ь] r(s,t) = ^А^(*Шг), G5) причем, если J — счетно, то на [a,b] x [a,b] ряд G5) сходится абсолютно и равномерно. (По-прежнему, А& и ф\~ обозначают положительные собствен- собственные значения и соответствующие собственные функции оператора А из G0).) В качестве следствия отсюда вытекает Теорема 19 (Карунен-Лоэв). Пусть X = {X(t), t G [а,6]} — непрерывный в среднем квадратическом на [а,Ъ] центрированный L2-процесс, имеющий ко- ковариационную функцию г = r(s,t). Тогда при каждом t Е [а, Ъ] t)zk n. и, G6) где Xkj фк — те же, что и в G5), а {г^, к G J} — центрированные ортонор- мированные случайные величины. Для счетного J ряд G6) сходится в среднем квадратическом. Доказательство. Положим Т = [а,Ь], А = Jwf(t,k) = л/ХкФк^) для t G Т, НА. Тогда свойство G5) превращается в соотношение A8), где \i — считающая мера на Л, т.е. \±({к\) — 1 для к G Л и требуемый результат о справедливости G6) непосредственно вытекает из теоремы Карунена (теорема 3). ? 9. Оценив величину Е X(t) - ^_ к<п
262 А. В. Булинский, А. Н. Ширяев докажите, что если выполнены условия теоремы 19, то в разложении G6) в качестве величин Zk можно выбрать центрированные ортонормированные величины X(t)</>k(t)dt. G7) V^k Ja Интеграл здесь определяется (и существует) как предел в среднем квадра- тическом соответствующих интегральных сумм Римана. Объясните, почему для непрерывной на [а, Ь] х [а, Ь] ковариационной функции г = r(s,t) собствен- собственные функции фк Е С[а, b],k E J. Кроме того, докажите, что среднеквадрати- ческая сходимость в G6) с данными ^,^G J, будет равномерна по t Е [а, Ь]. Пример 3. Найдем, пользуясь теоремой 19, разложение Карунена-Лоэва для винеровского процесса W(t) на отрезке [0,1]. Для этого определим собственные значения и собственные функции интегрально- интегрального оператора, отвечающего непрерывной на [0,1] х [0,1] ковариационной функции r(s, t) = min{s, ?}, s, t E [0,1]. Согласно упражнению 8 величины A& > 0 для к Е J и, как показывает упражнение 9, фь Е С[0,1]. Рассмотрим относительно А>0и^еС[0, 1] уравнение 1 r(s, ?H(?) dt = A0(s), s E [0,1], о т.е. уравнение fS f1 I Ьф\Лл dt -\- s I Ф\к) dt = Хфув). v $) ^0 J s Поскольку левая часть G8) дифференцируема в каждой точке s E [0,1] (в концах от- отрезка надо рассматривать соответствующие односторонние производные), то функ- функция ф дифференцируема и I <t>(t)dt = \<t>'(s), se [0,1]. J s Снова дифференцируя по s обе части этого равенства, приходим к тому, что требуе- требуемая функция 0(s) должна удовлетворять следующему уравнению: Общее решение этого уравнения задается формулой ф(в) = Acos(s/y/\) +Bsin(s/y/\), где А, В — константы. Учитывая, что ф@) = 0, ф'(Х) = 0, находим, что А = 0, (B/V\)cos(l/V\)=0, откуда Afc = ((^ + 1/2)тг)-2, fceZ+ = {0,l,...}. G9)
Гл. VII. Стационарные процессы. Дискретное и непрерывное время 263 Условия нормировки / (/>! (s) ds = 1 и равенства G9) дают следующие представле- представления для функций 4>k(s): фф) = Л/2 sin((fe + l/2Ors), s G [0,1], ке%+. (80) Легко проверить, что все найденные величины А& и (/>&, к Е Z+, удовлетворяют исходному уравнению G8). Таким образом, согласно представлению G6) находим, что где {zk, к Е Z+} — последовательность ортонормированных и центрированных (в силу упражнения 9) случайных величин; ряд в (81) сходится в среднем квадра- тическом равномерно на [0,1]. Замечание 6. Упражнение 4 главы II показывает, что L2[W(t), t G [0,1]] — га- уссовская система. Согласно упражнению 9 величины {z/с, к Е Z+}, фигурирующие в G7), где X(t) = W(t),a = 0nb= 1, образуют гауссовский процесс. Поскольку для центрированных действительных гауссовских величин ортогональность равно- равносильна независимости, то, применив упражнение 11 главы IV, получаем, что ряд (81) при каждом t Е [0,1] сходится с вероятностью единица, где независимые величины Zk ~ N@,1), к Е Z+, определены на том же самом вероятностном пространстве (П, ^, Р), что и данный винеровский процесс. 10. Найдите на отрезке [0,с] разложение Карунена-Лоэва для процесса Орн- штейна-Уленбека, имеющего ковариационную функцию r(s,t) = e-a\s-t\^ где a >O,s,tGl. 11. Пусть X = {X(t), t G 1} —непрерывный в среднем квадратическом процесс с ковариационной функцией r(s,t) = R(s — t). Пусть X — периодический процесс с периодом а, т.е. X(t + а) = X(t) п.н. для каждого t G I. Найдите для X разложение Карунена-Лоэва. Применения различных канонических разложений случайных функций даны, на- например, в монографии [58]. Важное значение канонические представления имеют для приложений теории случайных процессов к решению задач обнаружения сиг- сигналов в присутствии помех. В частности, большой интерес представляет вопрос, в каком случае в той или иной задаче можно ограничиться конечным числом членов, участвующих в каноническом разложении случайной функции. Иллюстрирующие примеры такого рода можно найти в [58; § 174]. Между теоремой 3 главы VII и теоремой б главы II имеется тесная связь. 12. Докажите, что если ковариационная функция г = r(s,t) допускает представ- представление A8), то гильбертово пространство Н с воспроизводящим ядром г = = r(s,i) состоит из функций h(t) = f g(\)f(t,\)v(d\), 9 G L2[f], t G T, (82) Ja
264 А. В. Булинский, А. Н. Ширяев где L2[f] — замыкание в 1/2(Л, ?/, /i) линейной оболочки функций /(?, •), t G Т. Скалярное произведение в Н задается формулой где h}~ vi gk, к = 1,2, связаны соотношением (82). Пусть Z — ортогональ- ортогональная случайная мера на ^ х П, имеющая структурной функцией меру \± (см. теорему 2), и пусть L2[f] = L2(A,?/, /i). Объясните, почему для процесса X = {Х(?), t G Г} вида A9), процесс У, фигурирующий в теореме б гла- главы II, задается формулой Y(h)= [ g(\)Z(d\), (83) Ja где h определяется по д согласно (82). 13. Докажите, что (действительное) гильбертово пространство Н с воспроиз- воспроизводящим ядром г(s,i) = min{s,t}, s,? G M_|_, являющимся ковариационной функцией винеровского процесса, состоит из функций rt roo h(t) = g(\)d\, tGlf, для которых / g2 ^0 -'О g2{\)d\ < oo (84) (интегрируем по мере Лебега). Объясните, почему L2[VF], т.е. замкнутая в 1/2(П, ^, Р) линейная оболочка W(t), t G M+ , состоит из величин вида (83), где Z — ортогональная случайная мера на ^ х П, порожденная самим вине- ровским процессом (см. пример 1), а функция д удовлетворяет условию (84). Рассмотрим ряд конкретных примеров стационарных и нестационарных процес- процессов (подразумевая под стационарностью стационарность в широком смысле). 14. Пусть X = {X(t) = ?e*(T*t+I/), t G Щ, где действительные случайные ве- величины ?, rj, v таковы, что v равномерно распределена на [0, 2тг] и не зави- зависит от (?,77), а Е?2 < оо. Убедитесь, что X дает пример центрированно- центрированного стационарного процесса с ковариационной функцией вида C6), где мера G — E?2F, a F — распределение вероятностей величины 77. Положим те- теперь Y = {Y(t) = f;cos(r]t + v\ t G Щ. Проверьте, что Y — стационарный процесс. Если X = {X(t), t G 1} —стационарный комплекснозначный про- процесс, то можно ли утверждать, что {ReX(t), t G Щ и {ImX(t), t G 1} — стационарные процессы? 15 (телеграфный сигнал). Пусть N = {Nt, t ^ 0} — пуассоновский процесс, имеющий постоянную интенсивность Л > 0. Пусть случайная величина ( не зависит от процесса N и Р(? = —1) = Р(? = 1) = 1/2. Введем процесс, называемый телеграфным сигналомнлн телеграфной волной, положив х(г,и) = с(и)(-1)м*(ш), t^o, и ей. Найдите cov(X(s),X(t)), s, t ^ 0 (сопоставьте ответ с упражнением 27 гла- главы III).
Гл. VII. Стационарные процессы. Дискретное и непрерывное время 265 16 (обобщение упражнения 15). Пусть {?п, п ^ 1} — последовательность неза- независимых одинаково распределенных действительных случайных величин, не- независящая от пуассоновского процесса N = {Nt,t ^ 0}, имеющего с вероят- вероятностью 1 единичные скачки в моменты т\ < t<i < • • • (см. теорему 2 главы II). Положим X(t,uj) = Ос(^) ПРИ rk(u) ^ t < Tk+i(u), где к = 0,1,... (то = 0). Если в последовательности {т^ (со), к ^ 0} найдутся совпадающие точки (что возможно лишь с вероятностью 0), то пусть X(t,uo) = 0 для t ^ 0. Считая Е (J < оо, найдите ковариационную функцию процесса X = {X(t), t ^ 0}. 17. Для процесса дробового шума (см. определение 14 главы VI) с ковариацион- ковариационной функцией г = r(s, ?) докажите, что возникает асимптотическая стацио- стационарность в том смысле, что существует lim r(s, s + t). Разнообразные обобщения процесса дробового шума (в том числе поля дробово- дробового шума) играют важную роль в построении различных моделей более сложных процессов; см. по этому поводу, например, [13, 111]. 18. Найдите спектральную плотность процесса Орнштейна-Уленбека. 19. Приведите пример (в связи с теоремой 8) неотрицательно определенной функ- функции R(t),t G М, непрерывной всюду, кроме точки t = 0. 20. Пусть /ii(t),..., hn(t) — комплекснозначные функции, заданные на множес- тве Т. Докажите, что г(s,t) = J2 hj(s)hj(t), s,t G T, является неотрица- тельно определенной функцией. Дайте пример негауссовского процесса, име- имеющего эту ковариационную функцию. 21. Пусть {Хп, п G Z} — стационарный процесс со средним значением а и кова- ковариационной функцией R = R(n), n G Ъ. Докажите, что при N —> оо к=0 к=0 22. Пусть в условиях предыдущего упражнения cN-t (85) N N m) k = lrn=l для некоторых с, 7 > 0 и всех N G N. Докажите, что тогда N-1 д? ^2 Хк ~^ ° п-н-при N ~^ °°- к=0 Покажите, что условие (85) выполнено, если R(n) = O(n~"r) при п —> оо.
266 А. В. Булинский, А. Н. Ширяев Теорема 20 (Гапошкин; [14]). Пусть X = {Х(п), п Е Z} — стационарный процесс со средним а = 0 и ковариационной функцией R = R(n), n Е Z [пусть Д@) = 1). Тогда (86) имеет место в том и только том случае, когда G({0}) = 0 г/ lim / Z(dA)=O п.н., где Z — ортогональная случайная мера, фигурирующая в спектральном пред- представлении C5), a G — ее структурная функция (мера). Отметим, что доказательство этого результата основано на представлении сумм к к~г Е хз Для 2П ^ к < 2П+1 (п = 0,1,...) в виде J <2_nZ(d\) + фк, где lim фк = 0 п.н. ив Ь2(П). к—юо 23. Пусть X = {Хп, п Е N} — центрированная действительная гауссовская стационарная последовательность с ковариационной функцией R = R(n),n Е Е Z. Докажите, что условие TV ^ ~^ °°' необходимо и достаточно для того, чтобы оценка Rn (тп) , введенная в D8), была состоятельной в среднем квадратическом, т. е. для т G Z E|,Rjv(m) - i?(m)|2 -»> 0 при N -»> оо. 24. Пусть X = {Xn, n G Z} — процесс скользящего среднего вида C9). По- оо требуем, чтобы ^2 \ск\ < °° и &к-> к ^ Z, были независимыми одинаково к = — оо распределенными величинами, имеющими Ег| < оо. Пусть /jv(A) -перио- -периодограмма D9). Найдите lim cov(f/v(A), /nM) для частот А, г/ G [—тт, тг1. iV^oo 25. Пусть Z — ортогональная случайная мера, отвечающая процессу Орнштей- Орнштейна-Уленбека с параметрами а, C > 0 (см. упражнение 27 главы III). Докажи- Докажите, что процесс /оо UX _ 1 • д , /э — J^- Z(dA), t ^ 0, (87) (по непрерывности полагаем (е — 1)/гЛ = ? при А = 0) является винеров- ским процессом. (Существование непрерывой модификации этого процесса легко получается с помощью теоремы Колмогорова (теорема 18 главы II).) Интересно отметить, что представление (87) дает возможность построения ви- неровского процесса, отправляясь от процесса Орнштейна-Уленбека, а формула (III.88) показывает, как процесс Орнштейна-Уленбека выражается через винеровс- кий процесс. В главе VIII мы рассмотрим также стохастическое дифференциальное уравнение Ланжевена, связывающее эти два замечательных процесса.
Гл. VII. Стационарные процессы. Дискретное и непрерывное время 267 Пользуясь упражнением 4, представление (87) можно переписать в следующем виде (сравните с примером 2) it\ roo it\ _ 1 W(t)= ——V(d\), J-oc гХ где ортогональная случайная мера V на полукольце Ж, фигурирующем в упражне- упражнении 4, задается формулой F6) с функцией h(X) = (гА + /3)/а, A G 1. Замечание 7. Согласно теореме 1 главы III траектории винеровского процесса п.н. недифференцируемы ни в одной точке t G М+. Докажите, что в каждой точке t G М+ не существует производной W(t) даже в смысле сходимости по вероятности. Тем не менее, если формально продифференцировать по t равенство (88), получим так называемый "белый шум" W(t) = / eitxV(d\). (89) J — оо Это название связано с тем, что формально получена смесь гармонических колеба- колебаний eltx, каждое из которых входит в (89) с одинаковой интенсивностью (поскольку структурная для V мера есть мера Лебега). Точный смысл понятию "белый шум" придается в теории обобщенных случайных процессов (сравните с определением 10); введение в эту область содержится в [77]. Результаты, изложенные в настоящей главе, распространяются также и на про- процессы с векторными значениями, а также на случайные поля. Остановимся на некоторых из этих результатов. Матричная функция C(s,t) = (Cjk(s,t))'Jlk=1, s,t G T, с комплекснозначными элементами называется неотрицательно (или положительно) определенной, ес- если для всех п ^ 1, любых t\,..., tn G Т и вектор-столбцов z\,..., zn G Cm j,k=l где символ * обозначает транспонирование и комплексное сопряжение всех элемен- элементов. Векторный процесс X = {X(t), t G Т} со значениями в Ст называется L2 -про- -процессом, если Е ||Х(?)||2 < оо для всех t G Т. Здесь и далее норма || • || в комплексном (в частности, действительном) евклидовом пространстве определяется обычным об- образом: где (z,w) = ^^ZkWk для z,weCm. k=i Определим векторную функцию среднего и матричную ковариационную (кор- (корреляционную) функцию L2-nponecca X, положив a(t) = EX(t) G Cm, r(s,t) = E(X(s)-a(s))(X(t)-a(t))*. 26. Докажите, что матричная функция R(s, ?), s, t G T (с комплексными элемен- элементами) является ковариационной функцией некоторого векторного L2-процес- L2-процесса X = {X(t), t G Т} тогда и только тогда, когда она неотрицательно опре- определена.
268 А. В. Булинский, А. Н. Ширяев Пусть Т — линейное пространство. L2-nponecc X = {X(t), t G T} со значениями в Ст называется стационарным в широком смысле, если a(t)=aeCm, r(s,t)=R(s-t), s,teT. (90) Матричная функция C{t), t G Т, называется неотрицательно определенной, если неотрицательно определена функция r(s, t) := C(s — t), s, t, s — t G T. Как и прежде, рассматриваемые процессы (и поля) считаем центрированными. Говорят, что последовательность векторов sn,n Е Z, со значениями в Ст явля- является белым шумом, если в (90) а = 0, Д@) = /, R(n) = 0 при п ф 0, здесь / — единичная матрица ттг-го порядка. Заметим, что для процессов с непре- непрерывным временем этот термин, как отмечалось выше, имеет более сложный смысл. Векторный процесс скользящего среднего вводится формулой Х(п)= ^ Aken-k, neZ, (91) k = — оо где г = {гп, п G Z} — белый шум, А^, к G Z, — матрицы (операторы), отображаю- отображающие Ст в Ст. Ряд в (91) понимается как сходящийся в среднем квадратическом. 27. Докажите, что для сходимости в L2(n,^, P) ряда (91) достаточно, чтобы оо / т \ 1/2 Е \Ак\2 < оо, где |А| := (ТгАА*I/2 = ( Е \ajq\2) для А = / т = ( Е \ajq ) — (cijqIjlq=i • Найдите ковариационную функцию векторного процесса сколь- скользящего среднего. Если Т — метрическое пространство с метрикой р, то L2-nponecc X = {X(t), t G T} со значениями в Ст называется непрерывным в среднем квадратическом в точке t G Г, когда E||X(s)-X(?)||2 ^0 при p(s,t)^>0. (92) Непрерывность в среднем квадратическом на Т означает выполнение свойства (92) в каждой точке t e Т. Ознакомившись с приложением 4, докажите, что справедлива следующая Теорема 21. Функция R = R(t), t G Iе1, является ковариационной функци- функцией стационарного в широком смысле непрерывного в среднем квадратическом комплекснозначного случайного поляX = {X(t), t G ld} тогда и только тогда, когда R(t) = [ е^'л> G(d\), teRd, (93) где G — некоторая конечная неотрицательная мера на ?$(Rd). При этом мера G определяется однозначно. Аналогичным образом может быть переформулирована и теорема 4. Комплекснозначное поле X = {X(t), t G Md} называется изотропным, если его ковариационная функция r(s,t) зависит только от t и ||s — t\\. Если поле еще и однородно (иначе говоря, является стационарным в широком смысле), то r(s,t) = R(\\s — t\\) для s,t G Md. В этом случае для R(t) можно дать иное представление.
Гл. VII. Стационарные процессы. Дискретное и непрерывное время 269 Теорема 22 (см. [16; т. 1, с. 262]). Для того чтобы функция R = R(u), где и G M+, была ковариационной функцией однородного, изотропного и непрерыв- непрерывного в среднем квадратическом комплекснозначного поля X = {X{t), t G 1 необходимо и достаточно, чтобы здесь /^(ж) — бесселева функция первого рода, Г — гамма-функция, Q — не- неотрицательная конечная мера на Й§(М_|_), причем ф(М+) = G(Mm) = Д@). Упомянем также понятие оdwopодного изотропного векторного поляХ, опреде- определенного на Ж х(]и принимающего значения в Ж . Помимо условия (90) здесь тре- требуется инвариантность распределений относительно группы вращений, т.е. тре- требуется, чтобы (S~1X(Sti),..., S~1X(Stn)) = (X(ti),..., X(tn)) для любого вра- вращения SbE^h любых точек t\,..., tn G Жс1 (п G N). Матричную функцию множества G(B) = (Gjk(B)IJlk=1, где 5 G JT и JT — полукольцо множеств Л, называют неотрицательно определенной, если матрица G(??) неотрицательно определена при каждом В ? Ж. Для непрерывного в среднем квадратическом однородного случайного поля X = {X(t), t G ЖA} со значениями в Ст и вектора г G Cm введем скалярное поле Y = {Y(t) = (X{t),r), t G ЖA}. Можно показать, что это поле также является однородным и непрерывным в среднем квадратическом. Пользуясь этим свойством, докажите, что верна Теорема 23. Для того чтобы функция R = R(t), t G lrf, была матричной ковариационной функцией непрерывного в среднем квадратическом однород- однородного векторного поля X = {X(t), t G Md} со значениями в Cm, необходимо и достаточно выполнение представления (93), где R понимается как т х т матричная функция, a G — как тхт матричная счетно-аддитивная функция на Векторной (со значениями в Ст) ортогональной случайной мерой Z на некотором полукольце Ж подмножеств множества Л называется L2-nponecc {Z(B), В G Ж} такой, что y =G(BnC), где G — матричнозначная счетно аддитивная мера на Ж, называемая структурной (матричной) мерой. Конструкция интеграла по скалярной ортогональной случай- случайной мере легко обобщается на векторный случай. Из теорем 3 и 23 выводится (см., например, [16; т. 1, с. 297]) Теорема 24. ПолеХ, фигурирующее в теореме 23, допускает представление вида X(t)= [ е^'Л> Z(d\), ?GMd, где Z — векторная ортогональная случайная мера на ^(Жл) со структурной функцией {мерой) G, отвечающей спектральному представлению ковариаци- ковариационной функции поля. При этом между пространством L2[X], т. е. замкну- замкнутой в Ь2(п,^, Р) линейной оболочкой величин X(t), t G Жс1, и пространством
270 А. В. Булинский, А. Н. Ширяев L2(v) = L2(Kd, Й§(М), i/), где г/ = TrG, устанавливается изометрическое соот- соответствие, при котором 1) Х(?) ^е^'л\ 2) еслг/ Ffc ^ gk(\), где Yk G L2[X], gk G L2{y), к = 1,2, то Получите отсюда, что справедлива Теорема 25 (Котельников-Шеннон). Пусть однородное, непрерывное в сред- среднем квадратическом поле X = {X(t), t G Md} имеет ограниченный спектр, т. е. структурная мера сосредоточена на некотором параллелепипеде (—ui,ui) х • • • х (—UdiUd) с ненулевыми длинами ребер. Тогда X(t)= V ]Vin("fc*fc~™fcW—>¦-¦,—У 04) г^е прг/ каждом t = (ti,..., td) G Md p^ сходится в среднем квадратическом. Смысл представления (94) состоит в том, что оно дает возможность восстана- восстанавливать значения поля в произвольной точке t G Md по его значениям в узлах ре- решетки вида (тгп! ju\,..., 7rnd/ud). В связи с каноническими представлениями случайных процессов упомянем еще одно направление исследований, относящееся к обновляющим процессам. Рассмот- Рассмотрим L2-nponecc X = {X(t),t G Г}, Г С М, и подпространства Ht(X), t G Г, со- состоящие из замыкания в среднем квадратическом линейной оболочки величин X(s), s ^ t (s,t G Т). Спрашивается, можно ли найти обновляющий процесс Y = {F(t), ? G Т}, т. е. процесс с ортогональными приращениями, такой, что Ht(X) = Ht(Y) при всех ? G Г. Вообще говоря, ответ отрицательный. Однако как показано Г. Крамером, можно ви- видоизменить постановку задачи и получать (при каждом t G Т) представление i^t (-X") в виде суммы ортогональных друг другу пространств Ht(Yn), порожденных долж- должными процессами Уп — {Yn(t), t G T},n = 1,..., N (N ^ оо). Такого рода пред- представления играют важную роль при иссследовании вопросов об абсолютной непре- непрерывности и сингулярности распределений случайных процессов. Стохастическое спектральное представление C8) можно получить, опираясь на теорию операторов в гильбертовом пространстве Н. Для этого напомним, что изо- изометрический оператор U (т.е. такой, что (Ux, Uy) = (ж, у) при всех ж, у G Н), ото- отображающий Н на себя, называется унитарным. Справедлив следующий класси- классический результат.
Гл. VII. Стационарные процессы. Дискретное и непрерывное время 271 Теорема 26 (Стоун). Пусть {[/t, t G 1} — группа унитарных операторов в гильбертовом пространстве Н (UtUs = Us+t, s,t E Ж) и функции t н->- (?,Щг)) непрерывны при всех ?,г] G Н. Тогда для каждого t G 1 roo Ut= eitxE(d\), J — oo где E(-) — операторно-значная мера со значениями в ортогональных проек- проекторах. О том, как определяется этот интеграл можно прочитать, например, в [64]. Для стационарного в широком смысле процесса X = {Х(?), t G М}, применив теорему Стоуна к группе унитарных операторов сдвига (St)teR на гильбертовом пространстве Н = Н(Х) (см. доказательство теоремы 12), получаем, что X(t) = StX@) = ( f eitxE(d\))X@) = f eitxZ(d\) для t G M, \Jr J Jr где Z(B) = E(B)X@) — ортогональная случайная мера, заданная на борелевских множествах. Тем самым, получено иное (нежели в теореме Крамера (теорема 9)) доказательство спектрального представления стационарных процессов (сравните с C8)). Приведенный способ получения спектрального представления, опирающийся на теорему Стоуна, был предложен А.Н. Колмогоровым (см. [64]). Аналогичным образом изучаются стационарные процессы с дискретным временем, а также случайные поля на Z и Ж . Отметим, что использованный в главе VII подход к каноническому представлению случайных функций, основанный на теореме Карунена, действует для процессов X = {X(t), t G Т}, заданных на произвольном параметрическом множестве Т. Различным аспектам исследования векторных процессов и полей посвящены мо- монографии [6, 44, 64, 80, 87,134,186]. Рассмотрим кратко некоторые вопросы, связанные с преобразованиями (главным образом, стационарных) процессов. Пусть имеется устройство ("система") А, на вход которой подается процесс X = = {X(t), t G Т} и на выходе получается процесс Y = {Y(t), t G 5}, который образно можно было бы представить в виде Y = АХ. Если считать, что А — отображение одного функционального пространства в другое, то надо потребовать, чтобы (почти все) траектории процесса X лежали бы в области определения Da отображения А и чтобы действие этого отображения на эти траектории приводило также к случай- случайному процессу (в смысле выполнения должных требований измеримости). Будем предполагать, что действие преобразования (отображения) А на процесс X приво- приводит в каждый момент t G S к случайной величине Y(t). Введем операцию дифференцирования в среднем квадратическом. Производ- Производную X1 {t) (комплекснозначного) L2-nponecca X, заданного в окрестности точки t G 1, определим как производную функции X (t) со значениями в банаховом (гиль- (гильбертовом) пространстве Ь2(п) = L2(Q,^, P), т.е. как такой случайный элемент (обозначаемый Xf(t)), для которого E\X'(t) - (X(t + h) -X(t))/h\2 ^0 при ft-^0. (95)
272 А. В. Булинский, А. Н. Ширяев Данный подход позволяет создать содержательное исчисление (с аналогом классической формулы Ньютона-Лейбница). Отметим, что если вместо сходимости в среднем квадратическом (95) требовать лишь сходимость по вероятности, то соответствующее исчисление не приводит бы к плодотворной теории как показывает утверждение следующего упражнения. 28. Докажите, что для пуассоновского процесса N = {Nt, t ^ 0} постоянной интенсивности Л > 0 производная по вероятности в каждой точке t Е М+ равна нулю п. н. В дальнейшем мы рассматриваем лишь производные в среднем квадратическом, не оговаривая этого специально. При этом, как обычно, производная k-го порядка в точке t определяется (если она существует) по значениям процесса Х^к~^ (?) в ок- окрестности этой точки. 29. Пусть X = {X(t), t G [a, b]} — это L2-nponecc с ковариационной функцией r(s, t). Докажите, что X'(t) существует в точке t Е (а, Ъ) тогда и только тог- d2r(s,t) да, когда существует обобщенная вторая производная —-—^— в точке (?, t) os dt (с теорией обобщенных функций это никак не связано), где по определению d2r(s t) ' := Hm (r(s + h,t + u) - r(s + h,t) - r(s,t + u) + r(s,t))/hu. C/o Cy L ft jUj г U Аналогичное утверждение справедливо и для производной в концевых точ- точках, если иметь в виду односторонние производные. Для доказательства этого и подобных утверждений ключевую роль играет сле- следующая элементарная Лемма 5. Функция /(t), t G [а, Ь], принимающая значения в гильбертовом пространстве Н со скалярным произведением (•,•), имеет предел в точке to G [a,b] тогда и только тогда, когда существует предел Hm (f(s),f(t)). s,t—>to 30. Докажите, что у стационарного процесса X = {X(t), t G 1} производная Х^ (t) существует в каждой точке t тогда и только тогда, когда оо X2k G(d\) < оо, — оо где G — спектральная мера X. Будет ли процесс Х^ стационарен? Если будет, то каково его спектральное представление? Наряду с дифференцированием важный пример линейной операции А, т. е. такой, что А(аХ + CV) = аАХ + CAV дляа,{3 G С и X, V G D^, дает интегрирование в среднем квадратическом слу- случайных процессов. Будем для L2-nponecca X = {X(t), t G [a, b]} под интегралом / X(t) dt (или J[a,b] v rb I X(t) dt) понимать интеграл Римана (если он определен) по отрезку [а, Ь] от слу- чайной функции Xt, т.е. берется предел в пространстве L2(Q) интегральных сумм
Гл. VII. Стационарные процессы. Дискретное и непрерывное время 273 Римана. Подробнее об интегрировании и дифференцировании в среднем квадрати- ческом см., например, [12; гл. 2]. Несобственные интегралы по всей прямой Ш. вводятся стандартным образом как пределы (если таковые существуют) в соответствующем смысле интегралов по от- отрезкам [а, Ь], когда а ->• — оо и Ь ->• оо. Для L2-nponecca X = {X(t), t G Щ с ковариационной функцией г = r(s,t) и комплекснозначной функции /i, заданной на МхМ, положим гоо Y(t) = / h(t, s)X(s) ds, t G M. (96) J — oo Нетрудно убедиться, что процесс У (?) в (96) определен и конечен (п.н.), если конечен несобственный интеграл Римана гоо гоо / / h(t,s)r(s,u)h(t,u) dsdu. J — оо J — оо Функция h = h(s, t), фигурирующая в (96), называется в приложениях импульс- импульсной переходной функцией системы. Если формально подставить в интеграл (96) вместо X(s) функцию Дирака Ss, то получим Y(t) = h(t,s). Другими словами, h(t, s) есть отклик системы в момент t на S-импулъс, подаваемый на ее вход в мо- момент s. Частным случаем процессов вида (96) являются однородные по времени процессы вида гоо Y(t)= h(t-s)X(s)ds, teR. (97) J — оо Соответствующие системы ("фильтры"), на выходе которых возникает процесс Y = {Y(t), t G 1}, определяются импульсной переходной функцией h = h(s), sGM, или функцией лоо H(i\) = / h(s)e~iXsds, AgI, J —оо называемой частотной характеристикой или коэффициентом передачи. Если функция h = h(s) интегрируема по Лебегу, то функция Н = H(iX), очевидно, оп- определена. Частотная характеристика имеет следующий наглядный смысл. Если h G L1(M), то для каждого Л G Ш. неслучайная функция X(s) = егЛв, s G 1, будет собственной функцией преобразования (97), отвечающей собственному значению Н(гХ). Это делает понятным смысл следующего упражнения. 31. Пусть на вход фильтра (97) подается стационарный в широком смысле процесс X = {X(t), t G Щ, имеющий спектральное представление C8) с ортогональ- ортогональной случайной мерой Z, структурная мера которой есть G. Докажите, что если частотная характеристика фильтра H(iX) G L2(M, Зё(Ж), G), то кова- ковариационная функция процесса Y = {Y(t), t G 1} имеет вид лоо R(t)= eiXt\H(i\)\2G(d\), teK, J — оо а сам процесс Y допускает следующее спектральное представление: (t) = / J — eiXt oo Y(t) = / eiXtH(iX) Z(dX), t G
274 А. В. Булинский, А. Н. Ширяев Вспоминая энергетическую интерпретацию спектральной меры, видим, что функция \H(i\)\2 показывает, во сколько раз изменяется энергия простых гармо- гармонических составляющих процесса X при прохождении через фильтр с частотной характеристикой i? = H(iX). 32. Какая импульсная переходная функция отвечает полосовому фильтру, т.е. фильтру, пропускающему без изменения гармонические составляющие процессов с частотами в полосе [а, Ь] (это значит, что H(i\) = 1[а^ (А), Л Е М)? Интересно применение спектральных представлений процессов к решению диф- дифференциальных уравнений вида гдеРпB:) = aozn + агг71'1 -\ \-an,Qm(z) = boz171 + b1zrn~1 Н Ь &т — много- многочлены с постоянными коэффициентами, X — заданный стационарный процесс, а не- неизвестный процесс Y = {Y(t), t G 1} предполагается дифференцируемым в среднем квадратическом п раз в каждой точке t G 1. Равенство (98) считается выполненным п. н. при каждом t Е ffiL О том, как решение Y ищется с помощью некоторого фильтра см., например, [17; с. 282-284]. Аналог уравнения (98), рассмотренного выше для процессов с непрерывным вре- временем, приводит в случае процессов с дискретным временем (t Е Т С Z) к изучению так называемых процессов "авторегрессии — скользящего среднего", см. подроб- подробнее [85; т. 2, с. 586]. В заключение приведем еще один классический результат, относящийся к теории процессов, стационарных в узком смысле. Введем на пространстве Ж°° оператор сдвига Т, положив Тх = у, где х = (... ,ж_1,жо,Ж1,...), у = (... ,?/_i, ?/о, 2/ъ- • •)> причем уп = xn+i, n G Z. Назовем множество В G ^(М°°) инвариантным, если Т~1(В) = 5. Легко видеть, что совокупность инвариантных множеств образует в ^(М°°) сг-алгебру, которую обозначим J>. Пусть X = {Xt, t G Z} — стационарный в узком смысле процесс, заданный на вероятностном пространстве (П,^, Р). Положим J>x = Х~г(^), где X рас- рассматривается как измеримое отображение (П, ^) в (М°° , ^(М°°)). Тогда ^х будет сг-алгебройв ^, называемой сг- алгебр ой инвариантных событий процесса X. Теорема 27 (Биркгоф-Хинчин). Пусть X = {X(t), t G Z} — стационарный в узком смысле действительный процесс и Е|Хо| < оо. Тогда Г^М Е *(') = №№, (99) t=M+l где предел существует как почти наверное, так и в L1(Q). Говорят, что процесс X является эргодическим, если Р{А) равно 0 или 1 для лю- любого A G J>x- В этом случае (при выполнении условий теоремы 27) в правой части (99) условное математическое ожидание Е (Xq \ J>x) = EXq п.н.
Гл. VII. Стационарные процессы. Дискретное и непрерывное время 275 Теория стационарных в узком смысле процессов теснейшим образом связана с тео- теорией динамических систем (с изучением преобразований, сохраняющих меру). В ка- качестве введения в эту область исследований можно прочитать гл. V в [85], где, в част- частности, дано простое доказательство соотношения (99) (при М = 0), предложенное А. Гарсиа.Обобщение теоремы 27 на случайные поля содержится в работе [15]. Эр- годической теории и динамическим системам посвящены, например, книги [36, 70].
Глава VIII Стохастический интеграл. Стохастические дифференциальные уравнения Стохастический интеграл для простых случайных функций по винеровскому про- процессу. Конструкция Ито стохастического интеграла для неупреждающих случайных функций. Свойства стохастического интеграла. Формула замены переменных Ито. Уравнение Ланжевена. Процесс Орнштейна-Уленбека. Теорема существования и единственности сильного решения стохастического дифференциального уравнения. Марковость решения стохастического дифференциального уравнения. § 1. В классическом анализе существуют различные подходы к операциям интегрирования, приводящие к таким, вообще говоря, отличающимся понятиям, как, например, интегралы Римана, Лебега, Римана-Стилтьеса, Лебега-Стилтьеса, Данжуа, Бохнера, Петтиса, А-интеграл Колмогорова и др. В теории случайных процессов также рассматриваются разнообразные подходы к интегрированию случайных функций по случайным процессам, случайным мерам, и др., приводящие также к различным конструкциям "стохастических интегралов". С одним (и простейшим) типом таких интегралов мы уже имели дело в преды- предыдущей главе, где интегрирование детерминированных функций велось по ортого- ортогональным случайным мерам Z. Там же было показано, как с помощью таких интег- интегралов удается дать стохастическое спектральное представление для стационарных в широком смысле случайных процессов (теорема 9 главы VII). Если ортогональная случайная мера Z порождается винеровским процессом W = {Wt, t ^ 0} (см. пример 1 главы VII) и детерминированный процесс / = {ft, t ^ 0} принадлежит классу L2 = 1/2([0, оо), Й§([0, oo)),mes),raemes — мера Лебега, то соответствующий интеграл «/(/) (см. (VII. 14)) обычно записывают в виде ¦/(/)=/ ftdWt A) J[0,oo) roo или J(f) = / ft dWt, подчеркивая тем самым аналогию с классическими процеду- процедурами интегрирования. Заметим, что стохастический интеграл в A) вообще говоря нельзя понимать как потраекторный интеграл, т.е. как интеграл Лебега-Стилтьеса при каждом фик- фиксированном ио Е П (Wt = Wt(uj))j поскольку в силу теоремы 1 главы III траектории винеровского процесса (броуновского движения) имеют Р-п.н. неограниченные ва- вариации.
Гл. VIII. Стохастический интеграл. Стохастические дифференциальные уравнения 277 В формуле A) функция / = ft, t ^ 0, была детерминированной. Однако во мно- многих вопросах стохастического анализа возникает необходимость рассмотрения ин- интегралов типа A), в которых подынтегральная функция / является случайной: / = ЛН, t^o, toe a B) Так, например, при рассмотрении стохастических дифференциальных уравнений (см. далее § 12-18) C) понимаемых как условная запись стохастических интегральных уравнений [ b(s,Ys)ds+ [ a(s,Ys)dWs, t > 0, D) o Jo сама собой возникает необходимость в придании смысла "стохастическим ин- интегралам" / a(s, Ys) dWs(uj), являющимся интегралами типа A) от случайной функции /1(о,t]. При первом чтении желательно ознакомиться с конструкцией интеграла Ито на отрезке [О, Т] и свойствами этого интеграла (§ 2—6), а также с формулой замены пере- переменных (теорема 5 § 11), позволяющей легко решить классическое уравнение Ланже- вена (§ 13). Следует разобраться с формулировкой теоремы существования и единст- единственности (сильного) решения стохастического дифференциального уравнения (§17). Заслуживает внимания § 18, в котором доказывается марковость решения стохасти- стохастического дифференциального уравнения. К техническим деталям приведенных дока- доказательств можно вернуться позднее. § 2. На чем, собственно говоря, основана надежда на то, что можно определить "стохастические интегралы" типа / fs(uj) dWs (uj) от случайных функций / (инте- грандов) по винеровскому процессу W (играющему роль интегратора), в то вре- время как ранее при определении стохастических интегралов по 1/2-процессам с ортого- ортогональными приращениями предполагалось, что интегранды / суть детерминирован- детерминированные функции? Дело здесь обстоит следующим образом. Винеровский процесс, как и всякий квадратично интегрируемый мартингал, входит в подкласс L2-процессов с орто- ортогональными приращениями. Но понятно, что, сужая класс интеграторов, можно рассчитывать на то, что соответствующий класс интеграндов окажется шире рассматриваемого ранее множества детерминированных функций /. В случае винеровского процесса W конструкция стохастических интегралов от случайных функций /, удовлетворяющих приводимым ниже условиям измеримости и интегрируемости, была дана японским математиком К. Ито. Это объясняет, поче- почему такие интегралы далее обозначаются It (/) (/ — от Ito). Сама идея Ито построения стохастического интеграла в сущности та же, что и в случае процессов с ортогональными приращениями: A) сначала интегралы опреде- определяются (вполне естественным образом) для кусочно постоянных "неупреждающих" случайных функций, а затем B) для более общих функций / путем их аппроксимации
278 А. В. Булинский, А. Н. Ширяев простыми функциями /(п) и, основываясь на идеях изометрии, определяются стохас- стохастические интегралы /т(/) как пределы (в среднеквадратическом смысле) величин () Эта конструкция работает (с привлечением разложения Дуба-Мейера) не только в случае винеровского процесса, но также и для других классов процессов, включая класс квадратично интегрируемых мартингалов, к которому принадлежит винеров- ский процесс. §3. Пусть W = {Wt, t ^ 0} — винеровский процесс (броуновское движение), заданный на вероятностном пространстве (П, ^, Р). По-прежнему, 3^ обознача- обозначает сг-алгебру, порожденную величинами Ws, 0 ^ s ^ t. Положим 3W = a{Wt, t ^ 0}. Будем считать, что вероятностное пространство полно и сг-алгебры 3W, 3^ (t ^ 0) расширены классом J/ событий нулевой вероятности. Итак, рассмат- рассматривается естественная фильтрация (^^Ot^o? удовлетворяющая обычным условиям (см. определение 10 главы IV). Займемся сначала конструкцией стохастического интеграла ЫЛ= I fs(oo)dWs(u;) E) J@,T] для некоторого фиксированного Т > 0, а затем определим стохастические интегралы hU)=f fs(oo)dWs(u;) F) уже для всех t G [0, оо). Если / является простой детерминированной функцией вида /t(cj) = /(а^](?), где t G [0,Т], (а, Ъ] С @,Т], то под стохастическим интегралом /т(/) понимают случайную величину IT(f)=Wb(L0)-Wa(Lu), в записи /т(/) аргумент uj опускается. Пусть теперь функция / является простой, но несколько более общего вида: ЛМ=?>М1(а,ц(*), G) где (f(uj) — некоторая случайная величина, тогда естественно положить IT(f)=<p(u)\Wb(u)-Wa(w)]. (8) Если функция / является простой вида п-1 /4Н=^оН1{О}(*)+Е^И1(*ь*г+1](*), O^t^T, (9) г=1 где 0 = t\ < t<i < • • • < tn = Т, п ^ 2 и <?о? • • •, фп-i — случайные величины, то снова естественно определить п-1 7о(Я = 0, /*(/) = J2 ^И [Wti+1At(u) ~ WtiAtH] для 0 < t < Т. A0) г=1
Гл. VIII. Стохастический интеграл. Стохастические дифференциальные уравнения 279 Другие варианты простых функций обсуждаются в дополнении к этой главе. Оказывается, чтобы ввести интеграл It (/) для более широкого запаса функций /, да к тому же с "хорошими" свойствами (см. далее лемму 1), на функции cpi(uj) в (9) надо налагать более жесткие условия измеримости. Напомним, что действительная случайная функция / = {ft, t G [0,Т]} называ- называется согласованной с фильтрацией (потоком) EjT = {&t^)te[o,T]i если при каж- каждом t G [О, Т] величина ft является З*^-измеримой. Разумеется, в этом определе- определении вместо естественной фильтрации ?jf можно использовать другую фильтрацию: F = (<^t)tE[o,T] ? а вместо отрезка [О, Т] — иной промежуток, например, [0, оо). Определение 1. Будем говорить, что случайная функция / = {ft, t G [0,T]} является неупреждающей по отношению к потоку ?jf, если она согласована с ?jf и Й§([0,Т]) (8) 3*w-измерима по паре переменных (t,uj) G [0,Т] х П. Класс таких случайных функций обозначим sijp. В случае простых функций / вида (9) их принадлежность классу sUt равносильна тому, что случайные величины (pi(uj) в (9) являются 3*^-измеримыми. Обозначим класс таких простых функций дЩ,. Полезно заметить, что простые функции / допускают, вообще говоря, неоднознач- неоднозначное представление в виде (9). Но как и при определении соответствующих интегра- интегралов J(f) показывается, что значения It (/), t G [0, Т], не зависят от способа представ- представления функции / в виде (9). Следующее предложение описывает основные свойства стохастических интегра- интегралов /*(/)? t G [0, Т], берущихся от простых функций. Лемма 1. Пусть на отрезке [0,Т] заданы простые неупреждающие функции f = ft(u;), t е [О,Г], вида (9) ug — gt(w), t G [0,Г], аналогичной структуры: п-1 ^Ml(tj.,ti+l](t). A1) Пусть все случайные величины (fi(uj) и ^j(uj) имеют конечный второй момент. Тогда стохастические интегралы h{f) u h(g), t G [0,Г], обладают следую- следующими свойствами: (a) (h(f))te[o,T] u (h(g))te[o,T] являются непрерывными квадратично интег- интегрируемыми мартингалами с нулевым средним и EIt(f)It{g) = E^J fs(Lo)gs(Lo)ds\ A2) при всех t G [0,Т]; в частности, E7?(/) = E^*/»ds]. A3) (b) Для каждого t G [О, Т] п. н. Гfs(co)dWs(oo)= Г fs(oo)l@tt](s)dWs(oo), A4) то есть, /о(/) = 0 и при 0 < t ^ Т /*(/)=/т(/1(о,*]) п. п. A5)
280 А. В. Булинский, А. Н. Ширяев Доказательство. Непрерывность процесса (h(f))te[o,T] следует непосредст- непосредственно из определения A0). Так как (п.н.) Е [<Pi(Wti+1 ~ Wti) | &%] = ViE [Wti+1 - Wti | 9%] = 0, A6) то процесс (It(f))te[o,T] является мартингалом, к тому же квадратично интегриру- интегрируемым, поскольку Е [tf{Wu+1 - Wtf] = E [tf = Е [tf E [Wti+1 - Wti]2] = (E^Kti+i - U). D Свойства A2) и A3) легко выводятся с учетом того, что для i < j Е [<fii(Wti+1 ~ Wti)^{Wti+1 - Wtj)} = = E [pi(Wti+1 - Wti)E №j(Wtj+1 - Wtj) | &%]] = 0. Наконец, свойство (b) следует непосредственно из (9) и A0). §4. Введем пространство Ж^, состоящее из квадратично интегрируемых мар- мартингалов М — (Mt)te[o,T]« Иначе говоря, при каждых 0 ^ s ^ t ^ Т п.н., EMt2 < оо. Будем предполагать также, что все траектории рассматриваемых мартингалов М G Ж\ непрерывны справа. В пространстве Ж^ введем скалярное произведение (М, N) = Е MTNT A7) и норму т (ЕМ%I'2. A8) Отметим ряд нужных нам свойств мартингалов из пространства Ж^. Лемма 2. а) Пространство Ж^ является гильбертовым (с введенным ска- скалярным произведением). b) Пусть последовательность мартингалов М^п\ п G N, сходится в Ж^ {по норме || • ||^2 ) к мартингалу М. Тогда Е sup \M[n) -Mt\2 ->0, п^оо. A9) tG[0,T] c) Множество непрерывных мартингалов из Ж^ образует замкнутое под- подпространство (в Ж^)-
Гл. VIII. Стохастический интеграл. Стохастические дифференциальные уравнения 281 Доказательство. Чтобы показать, что пространство Ж2^ является гильберто- гильбертовым, надо убедиться в том, что оно является полным (всякая фундаментальная по- последовательность сходится). Пусть {М(П)}П?^ — фундаментальная последовательность элементов из Ж2^. Ясно, что величины Mj? , n Е N, образуют фундаментальную последователь- последовательность в Ь2(п) — пространстве случайных величин с конечным вторым момен- моментом. В силу полноты этого пространства существует величина Мт такая, что Е|М^п) -Мт\2 ^0,п^оо. Обозначим Mt — Е (Мт \ З*^), t G [О, Т]. Из обычных условий, налагавшихся на фильтрацию (^t^)t^Oj вытекает, что у мартингала М = (Mt)te[o,T] есть версия, непрерывная справа (которую так и будем обозначать М = (Mt)te[o,T])- Следова- Следовательно, фундаментальная последовательность мартингалов М^п\ п Е N, сходится к квадратично интегрируемому мартингалу М = (Mt, &t^)te[o,T] (B смысле Ж^), имеющему непрерывные справа траектории. Согласно неравенству Дуба для квадратично интегрируемых мартингалов М = (Mt)t^T с непрерывными справа траекториями имеем (см., например, [26], а также теорему 5 главы IV) Е sup|Mt|2 ^4E|MT|2. B0) Отсюда и доказанного пункта а) непосредственно следует утверждение Ь). Для доказательства с) предположим, что М^ = (M^n))t^0^T^ n G N, являют- являются непрерывными мартингалами из Ж^ и М = (Mt)t^[o,T] — непрерывный справа мартингал из Ж^, причем Е |М^ — Мт\2 —У 0, п —у оо. Отсюда следует, что найдется подпоследовательность {nf} С {п} такая, что Е [ sup \М[П>) - Mt\2} < оо B1) и, следовательно, в силу неравенства Чебышёва для всякого е > О У" Р{ sup |Mt(n'} - Mt\ > е) < оо. B2) Воспользовавшись леммой Бореля—Кантелли, из B2) заключаем, что с вероятнос- вероятностью единица sup \м[п1) -М*| ->0, п'^оо. B3) По предположению, мартингалы М^ имеют непрерывные траектории, а мартин- мартингал М G Ж^ и, значит, имеет непрерывные справа траектории. Из равномерной п.н. сходимости B3) вытекает, что мартингал М = (Mt)te[o,T] также имеет п.н. не- непрерывные траектории. ? §5. Вернемся к конструкции стохастического интеграла. В пространстве ]), P®mes)
282 А. В. Булинский, А. Н. Ширяев рассмотрим совокупность неупреждающих функций / = /t(c^), t E [0,T], ш E П. Следовательно, для / Е l3p П &4т т f?(u)dt < оо. B4) о Свойство A2) означает, что отображение / н-» /т(/)? гДе / — /t(^)? ? ? [О,Г], cj Е П, есть изометрия подпространства простых функций, принадлежащих «g/j, П L|,, в подпространство Ж^, состоящее из непрерывных квадратично интегрируемых мартингалов. Поэтому отображение / н-» /т(/) может быть распространено на замыкание дЩ, П L\ в L|i. Лемма 3. Замыкание 84^f\L\ (в Lj,) содержит неупреждающие функции f из 8$т П Lji. Доказательство этого утверждения, следующее из метрической теории функ- функций и состоящее в том, что для / Е sUt П L\ найдется последовательность функций /(п) из дЩ, П Lji таких, что Г Jo (и) - fs(u)\2 ds ^у 0, п^оо, B5) можно прочитать, например, в книге [26; гл. IX, § 5] и здесь не воспроизводится. ? §6. Пусть функция / G Lji является простой (/ G &Щр П Lj,)- Каждой та- такой функции был поставлен в соответствие квадратично интегрируемый мартингал /(/) = (/t(f))te[о,т]•> чт0 в более наглядных "интегральных" обозначениях можно записать в виде /•->(/ ДМ^и) . B6) Как уже было отмечено, это отображение является изометрическим отображени- отображением дЩ, П Lji в подпространство пространства М^, состоящее из непрерывных мар- мартингалов. Согласно лемме 3 множество неупреждающих функций / из L^., т.е. множество функций / G ?^т П L|i, входит в замыкание множества sij, П L\ (в L|i). Поэтому каждой такой функции / G &$т П L^. можно поставить в соответствие квадратично интегрируемый мартингал (-^t(/))tG[o,T]j который естественно также обозначать ; Теорема 1. Дл^ всякой функции f G «с/тП!^ стохастический интеграл Ито hU)= [ Jo определен для любых t G [0,Т] (/о(/) = 0)- Процесс (It(f))te[o,T] является непрерывным квадратично интегрируемым мартингалом с нулевым средним. При этом для всех / и g из класса dTf\L\ справедливы соотношения A2)—A4), установленные ранее для простых функций из 8$т П L\.
Гл. VIII. Стохастический интеграл. Стохастические дифференциальные уравнения 283 Доказательство . Возможность определить величины h(f) обеспечивают от- отмеченные выше соображения изометрии. Непрерывность мартингала (h(f))te[o,T] (точнее, существование непрерывной модификации) следует из леммы 2. Свойства A2)—A4) вытекают из их справедливости для функций из sd^ П L\ и (по свойствам изометрии) для функций из замыкания sij, П L^, которое в силу леммы 3 заведомо включает в себя множество six П L^.. ? § 7. Обратимся теперь к определению стохастического интеграла на временном промежутке [0, оо). Пусть si — множество функций / = /t(c^), t G [0, оо), и G П, таких, что при каждом Т > 0 их сужения на [О, Т] х П входят в класс 8$т П L\. Теорема 2. 1) Пусть / Е si. Тогда существует непрерывный квадратич- квадратично интегрируемый мартингал М = (Mt)tе[о,оо) (относительно потока ?w) такой, что Mq = 0 и для всех t > О Mt = h (/) (= j fs M dWs (lo)\ . B7) Кроме того, для f и g из класса si при каждом t > 0 справедливы соотноше- соотношения A2) и A3), а при любом Т > 0 и всех t G [О, Г] имеет место формула A4). Доказательство. Для всякого п G N согласно теореме 1 определены интегралы Г fs(u)dW8(u), te[0,n]. B8) Г Jo При этом в соответствии со свойством A4) имеем М^п = М^п) (Р-п.н.) для вся- всякого t G [0,n — 1]. Определим теперь для всех t G [0, оо) процесс М = (Mt)t^o? полагая Мо = 0 и Mt = Mt^ при ? G (n - 1, n], n G N. Ясно, что так определенный процесс М является мартингалом. Поскольку М(п) — непрерывные мартингалы, то М является также непрерывным процессом за исключением, быть может, тех элементарных исходов а;, которые принадлежат множеству U {uiM^iu) ф М^1}(^)}. Поскольку М^\(и) = М^1}(^) (Р-п.н.), то это множество имеет нулевую вероятность. Тем самым, процесс М имеет (Р-п.н.) непрерывные траектории. (Если на ука- указанном множестве переопределить М, полагая его равным, скажем, нулю, то такой новый процесс будет уже иметь непрерывными все траектории.) Остальные свойства (It(f))t^o очевидны. ? § 8. Обсудим некоторые детали конструкции интеграла Ито. Отправляясь от не- непрерывного квадратично интегрируемого мартингала /*(/)? определенного согласно формуле A0) для простых функций / класса sUt П L^., можно реализовать построе- построение интеграла Ито, не обращаясь к формулам B0) и B1). А именно, наличие непре- непрерывной модификации процесса {/^(/), t G [0,Т]} нетрудно установить с помощью следствия б главы IV. Достаточно рассмотреть фундаментальную в L\ последова- последовательность простых функций fn G srfjinLiji (n G N) и, выбрав должную возрастющую подпоследовательность {п^}, оценить сверху Р( sup \I(fnk+1) ~ I(fnk)\ > k~2) vte[o,T] / для k G N, а затем воспользоваться леммой Б ope ля-Канте л ли.
284 А. В. Булинский, А. Н. Ширяев Свойство неупреждаемости функций /, для которых введен интеграл Ито, явля- является ключевым при обеспечении "хороших" свойств этого интеграла. В то же время имеются близкие понятия измеримости, как показывает следующее определение. Определение 2. Пусть Ft = (&t)te[o,T] — некоторый поток сг-алгебр в веро- вероятностном пространстве (П, 3, Р). Действительная случайная функция X = {Xt, t G [0, Т]} называется прогрессивно измеримой (по отношению к Ft), если для лю- любого t e [О, Г] отображение (s,u) ^ (Xs(lo),lo), где (s,u) Е [0,t] x П, является Й§([0,?]) (8) ^-измеримым. Другими словами, сужение функции X на [О, t] х П должно быть Й§([0, i\) (8) ^-измеримым при каждом t Е [О, Т]. Легко видеть, что любая прогрессивно измеримая по отношению к потоку Ft функция / = ft (и), t Е [0, Т], uj Е П, будет неупреждающей по отношению к этому потоку (в определении 1 вместо FjT берется Ft ). Разумеется, данные выше опреде- определения переносятся на процесс X = {Xt, t ^ 0}. Представляет интерес следующий результат. Теорема 3 (Чжун и Дуб; см., например, [148; с. 5]). Пусть X = {Xt, t ^ 0} — неупреэрсдающий действительный процесс по отношению к фильтрации F = (^t)t^o (т.е. согласованный с F и Й§([0, оо)) 0 3*-измеримый). Тогда X имеет прогрессивно измеримую модификацию. Простое достаточное условие прогрессивной измеримости содержит Теорема 4. Пусть согласованный с фильтрацией ?= (^t)t^o действитель- действительный процесс X = {Xt, t ^ 0} имеет траектории п. н. непрерывные слева на @, оо) (или п.н. непрерывные на [0, оо)). Тогда у процесса X существует про- прогрессивно измеримая модификация с теми же свойствами траекторий. Доказательство, основанное на построении должных аппроксимаций Xпрогрес- Xпрогрессивно измеримыми процессами Хп (п G N) и применении леммы 5 главы I, оставляем в качестве легкого упражнения. Отметим, что вместо естественной фильтрации броуновского движения ?^ при построении интеграла Ито (т. е. при задании It (/), t G [0, Т], формулой A0) для прос- простых функций / вида (9) из класса дЩ, П L\ и распространении этого интеграла на замыкание sij, П L\ в пространстве L\) можно использовать более общую фильт- фильтрацию F = (^t)t^o- В этой связи напомним следующее определение. Определение 3. Говорят, что W = {Wt, t ^} — винеровский процесс относи- относительно фильтрации F = (^t)t^o? если процесс W согласован с F, имеет п.н. непре- непрерывные траектории, W@) = 0 п.н. и для всех 0 ^ s < t < оо W(t) - W(s)±&s, W(t) -W(s) где "_IL" обозначает независимость. Использование более общей фильтрации F, нежели Ww, предоставляет дополни- дополнительные возможности. Например, в случае естественной фильтрации измеримость
Гл. VIII. Стохастический интеграл. Стохастические дифференциальные уравнения 285 случайной величины ( относительно сг-алгебры ^о означала бы, что ( = const п. н. При изучении стохастических дифференциальных уравнений, выбирая общую фильтрацию F (как правило, удовлетворяющую обычным условиям), мы сможем в качестве начальных условий рассматривать не только константы. В частности, ес- если случайная величина ?_11_^ , где 3* = а{3'^ , t ^ 0}, то W будет винеровским процессом относительно потока сг-алгебр 3*t = cr{3'^v, сг{?}}, t ^ 0. Формула A0) показывает, что располагая функцией /, заданной на [0, Т], мы фак- фактически берем интеграл по промежутку @, Т], основываясь на поведении этой функ- функции при t Е @, Т]. Исследуя стохастические дифференциальные уравнения, мы уви- увидим, что естественно использовать процессы, определенные на промежутке измене- изменения ?, включающем левый конец, чтобы задавать начальные условия. § 9. Для того чтобы привести пример вычисления интеграла Ито, установим вспо- вспомогательный результат. Лемма 4. Пусть прогрессивно измеримая по отношению к потоку ?т функ- функция / = f(t,u;), где t G [0,Г], и; G П, является непрерывной в среднем квадра- квадратическом. Тогда п-1 f Jo =l.Lm. k=0 где W — броуновское движение относительно ?т и 1. i. m. (предел в среднем квадратическом) берется при измельчении разбиений вида 0 = ?q < • • • < < tn = 1 , т. е. когда Sn = max (t^ - 4п)) -^ 0, п -> оо. C0) П — 1 ^ч ^ч ^ч Доказательство . Функция fn=^2f Щ? ) 1 Vk •> %+-i] — эт0 пР0Стая Функ~ к=0 ция из sij, П 13р. Для нее согласно A0) п-1 к=0 Кроме того, E|/(t,^)-/n(t,^)|2^T sup Г Jo при Sn —> 0 в силу того, что случайная функция, непрерывная в среднем квадрати- квадратическом на отрезке, будет равномерно непрерывна в среднем квадратическом на этом отрезке. ? Пример 1. Найдем / Wt dWt, где Т > 0. Пользуясь леммой 4, получим </О </О Wt dWt = 1. i. ш. X>(^>) (W(t[%) - w№)) = Ц- - I, C1) где Sn фигурирует в C0).
286 А. В. Булинский, А. Н. Ширяев Подчеркнем, что выбор в B9) значений / в левых концах промежутков (?д., Ц^_^\ играет принципиальную роль. Взяв в рассматриваемом примере значение подынтег- подынтегральной функции в других промежуточных точках (в правых концах промежутков ,получаем ^ ,(п)^f ,(п)^ ,(п)Л W* Т 1 1 m > W(t^ ' ) I W(t^ } ) — W(t{ }) I — — -I C$2) Sn^0 k=Q \ /22 § 10. Пусть W = {Wt, t ^ 0} — винеровский процесс относительно фильтрации F = (^t)t^o и пусть прогрессивно измеримая функция / такова, что при всех t ^ 0 rt Е / f(s)ds < оо, C3) Jo а прогрессивно измеримая функция д: [0, оо) х(]-^1 такова, что \g(s,u)\ds < оо") =1, t>0. C4) Будем говорить, что п.н. непрерывный на [0, оо) действительный процесс X = = {Xt, t ^ 0} допускает "стохастический дифференциал" dXt = /(?, uj) dWt + g(t, uj) dt, C5) если Xo G ^o | Зё(Ж) и с вероятностью 1 сразу для всех t ^ 0 Xt = X0+ / f(s,uj)dWs+ / g(s,uj)ds C6) Jo Jo (винеровский процесс W = {Wt, t ^ 0} согласован с фильтрацией F). В силу нала- налагаемых условий на функции / и g правая часть C6) определена и дает п. н. непрерыв- непрерывный процесс. Определение 4. Если выполнено C6), то говорят, что X = {Xt, t ^ 0} является процессом Ито. Замечание 1. Если действительные процессы X = {Xt, t ^ 0}, U = {Щ, t ^ 0} имеют стохастические дифференциалы, т.е. наряду с C5) dUt = <r(t, uj) dWt + b(t, uj) dt (с аналогичными свойствами измеримости, как и для функций, входящих в C5)), то, очевидно, существует d(Xf -\-Ut), причем d(Xt + Ut) = dXt + dUt = (/ + cr)dWt + (g + b) dt. Разумеется, все сказанное очевидным образом переформулируется на случай, ког- когда t G [u,v], 0 ^ u < v < оо.
Гл. VIII. Стохастический интеграл. Стохастические дифференциальные уравнения 287 §11. Краеугольным камнем всего стохастического исчисления является Теорема 5 (формула Ито). Пусть процесс X = {Xt, t ^ 0} с действительны- действительными значениями имеет стохастический дифференциал C5) (функции fug обла- обладают указанными выше свойствами). Пусть функция Н: [0, оо) xl-^E тако- ва, что существуют непрерывные производные dH/dt, д2Н/дх2. Пусть так- также процесс h = < f(s,Xs)-jr—(s,Xs), s ^ 0 > G si. Тогда процесс Y± = {H(t,Xt), I vx J t ^ 0}, имеет стохастический дифференциал сП[ сП-f 1 сР~T~f dYt = —(t,Xt)dt+ —(t,Xt)dXt + -—(t,Xt){dXtJ, C7) где (dXtJ определяется из C5) no следующему правилу "оперирования с диф- дифференциалами" : dt-dt = dt- dWt = dWt • dt = 0, dWt • dWt = dt. C8) Иначе говоря, п. н. для всех t ^ 0 ,X0) + J f(s,Xs) — (s,Xs)dWs + Этот фундаментальный результат удивителен в том отношении, что в выражении для первого дифференциала возникает вторая производная функции Н. Условие h G si в теореме 5 требуется лишь для того, чтобы был определен сто- стохастический интеграл по винеровскому процессу в C9). В дополнении к этой главе показано как расширить класс функций, для которых определено интегрирование по Ито. Тогда оказывается, что условие Н G С1'2 является достаточным для выпол- выполнения формулы Ито. Точнее говоря, это условие уже обеспечивает существование стохастического дифференциала и справедливость формулы C9), в которой стохас- стохастический интеграл понимается в этом более широком смысле. Мы не будем дока- доказывать приведенную теорему, поскольку даже для более узкого класса функций Н доказательство весьма длинное (см., например, [12; § 12.3]). Кроме того, на сегод- сегодняшний день имеются далеко идущие ее обобщения, например, для процессов, явля- являющихся непрерывными семимартингалами со значениями в Ж171. Мы рекомендуем ознакомиться с теоремами 15.19, 15.21 в [146]. Некоторые обобщения формулы Ито рассматриваются в дополнении к этой главе. § 12. Покажем, как может использоваться формула Ито. Вернемся к примеру 1. Воспользуемся теоремой 5, положив Xt = Wt и H(t,x) = \х2. Очевидно, вс( условия этой теоремы выполнены. Тогда для Y± = H(t, Wt) = 2 W2 имеем Yt = ^dt + ^ dWt + \ ^(dWtJ = 0 + Wt dWt + \{dWtJ = Wt dWt + \ dt.
288 А. В. Булинский, А. Н. Ширяев Следовательно, Таким образом, в соответствии с интегральной формой записи стохастического диф- дифференциала (см. C6)) и так как Wo = 0, получаем / WsdWs + \t, t^O, D0) o z т. е. другим способом приходим к C1). § 13. Прежде чем излагать некоторые общие результаты о стохастических диф- дифференциальных уравнениях, мы обратимся к одному частному, но важному случаю, связанному с описанием движения частицы в жидкой среде. Рассмотрим уравне- уравнение mv = -f3v + "шум", t ^ 0, D1) где т — масса частицы, v — ее скорость, параметр C > 0 характеризует вязкость среды. В соответствии с физическими представлениями в уравнении D1) "сила mv", тормозящая частицу, пропорциональна "скорости v", а наличие "шума" связано с ха- хаотическими соударениями данной частицы с молекулами среды (вследствие тепло- теплового движения последних). Посмотрим как можно интерпретировать уравнение D1), если в качестве шума попытаться использовать, например, "процесс" W, где W = {Wt, t ^ 0} — броу- броуновское движение. Согласно теореме 1 главы III, выписанная выше производная W не существует у траектории процесса W = {Wt, t ^ 0} ни в одной точке t (с вероят- вероятностью единица). Значит, если попытаться как-то осмыслить уравнение mv = -f3v + W, t ^ 0, D2) известное в физике как уравнение Ланжевена, то следует, прежде всего, иметь в виду эту крайнюю нерегулярность в поведении "процесса" W. Придадим строгий смысл этому уравнению. Перепишем его вначале в виде v = av + aW, t ^ 0, D3) где а = -Р/т < 0, а = 1/т > 0. D4) Хорошо известно, что решение обыкновенного дифференциального уравнения (с постоянными коэффициентами а, а) v = av + af, t ^ 0, D5) находится применением к решению однородного уравнения v = av, имеющему вид v(t) = ceat (с = v@)), метода вариации произвольной постоянной, состоящего в том, что решения ищутся в виде v(t) = c(t)eat. В результате находим, что rt = v@)eat + / ea^-u^af(u) du, t > 0. v(t) = v@)eat + / ea^-u)af{u) du, t ^ 0. D6)
Гл. VIII. Стохастический интеграл. Стохастические дифференциальные уравнения 289 (Это решение существует для t Е [О, Т], если, например, / — непрерывная на [О, Т] функция.) Уравнение D5) можно записать в дифференциальной форме dv = avdt + af dt, t ^ 0, D7) являющейся эквивалентной записью интегрального уравнения v(t)=v@)+ / av(s)ds+ / <jf(s)ds, t ^ 0. Jo Jo Это наводит на мысль, что и в "стохастическом" случае уравнение D3) надо пони- понимать именно в интегральном виде: V(t) = V@)+ f aV(s)ds + f Jo Jo t ^ 0, D8) формально записываемом обычно (по аналогии с D7)) в следующей дифференциаль- дифференциальной форме: dV (t) = aV (t) dt + a dWt. D9) Таким образом, мы как бы заменили Wdt на dWt, а это позволило придать смысл rt уравнению D3). В D8) входит стохастический интеграл Ито / a dWs, который ра- J о вен (по определению этого интеграла) aW(t), так как а в рассматриваемом случае есть константа. Тем самым, соотношение D8) принимает вид: V(t) - V@) =a [ V(s) ds + aWt. E0) Сильным решением этого уравнения будем называть такой п.н. непрерывный процесс V = {V(t), t ^ 0}, согласованный с фильтрацией (^t)t^o? чт0 Для каж- каждого t ^ 0 с вероятностью единица левая часть соотношения D8) равна его правой части. Отметим, что обе части указанного уравнения дают п.н. непрерывные про- процессы, поэтому с вероятностью единица совпадение обеих частей уравнения будет сразу для всех t ^ 0. Подобно D6) естественно ожидать, что сильное решение этого уравнения, рас- рассматриваемого как уравнение относительно V = {V(t),t ^ 0}, задается формулой rt / e Jo V(t) = V@)eat + / e^-^odWu, E1) Jo где в правой части фигурирует интеграл Ито (неслучайная функция е~аи G si). Таким образом, / du, "отвечающее" W du, заменяется на dWu. Покажем, что выбрав (в силу теоремы 2) п.н. непрерывный на [0, оо) процесс У, определенный согласно E1), мы действительно получим сильное решение уравне- уравнения D8). Для этого воспользуемся формулой Ито. Положим = Xt= e~audWu, H(t,x)=aeatx, где t ^ 0, x G
290 А. В. Булинский, А. Н. Ширяев Тогда dXt = e~at dWt (Хо = 0 по определению интеграла Ито). Возьмем Ft = = H(t, Xt). В силу C7) имеем dYt = aaeatXt dt + aeat dXt = aaeatXt dt + a dWt E2) (еще раз подчеркнем, что все дифференциальные преобразования имеют смысл толь- только как сокращенная запись соответствующих интегральных соотношений). Снова по формуле Ито легко убедиться, что d(V@)eat) = aeatV@) dt, E3) где V@) является ^о-тлзшеримой величиной. Из E2), E3) и замечания 1 вытека- вытекает D9). Итак, доказана Теорема 6. Для всех t ^ 0 существует сильное решение уравнения Лан- жевена с начальным условием V = V@) Е 3;о\ё%(Ж), которое задается формулой E1). § 14. Рассмотрим некоторые свойства решения E1). Пусть существует Е V@). Поскольку математическое ожидание интеграла Ито равно нулю, из E1) выводим, что EV(t) = eatEV@), E4) т.е. средняя скорость экспоненциально убывает (напомним, что а < 0). Теорема 7. Пусть начальное значение V@) является ^о \ $(Щ -измеримой величиной и V@) ~ N@, а2/Bа)), где а = —а. Тогда сильным решением урав- уравнения Ланжевена является процесс Орнштейна-Уленбека, т. е. гауссовский процесс V = {V(t), t ^ 0}, имеющий нулевое среднее и ковариационную функ- функцию 2 cov(V(s),V(t)) = — е-5"*1, s,t^ 0. E5) Доказательство. Из E4) следует, что Е V(t) = 0 при всех t ^ 0. Покажем, что для s, t ^ 0 E6) где, как обычно, s Л t = min{s, ?}. Действительно, в силу A2) as ft \ a(s-w) jtt/ / a(t-u) jW \ _ & UVV и / ^ UVV u I — ^0 / rsAt ea(s+t) = e e du = - A — e J, (^7) Jo Za а, кроме того, для всех t ^ 0 согласно теореме 2 /•t / / ft \ \ ЕУ@) / ea^-u)dWu = Е (У@)Е( / ea(t-w)^Ww | J^o ]]= 0. E8) Jo \ \Jo )) Если Е (V@)J = , то из E6) получаем E5). 2а
Гл. VIII. Стохастический интеграл. Стохастические дифференциальные уравнения 291 Докажем теперь, что V = {V(t), t ^ 0} — гауссовский процесс. Прежде все- всего заметим, что Wt = Wt — Wo-U-^o ПРИ каждом t ^ 0, а поскольку V@) является ^о-измеримой величиной, то V@)JLWt при любом t ^ 0. Из E1) следует, что доста- достаточно установить при любом к Е 1Чивсех0 ^ t\ < • • • < tk < оо гауссовость вектора (U(ti),..., U(tk)), где U(t) = / ехр{—аи} dWu, t ^ 0. Пользуясь леммой 4, полу- получаем, что для каждого т = 1,..., к величина U(tm) есть предел в Ь2(п) при п —У оо величин Vrn(n) = 3=0 где tj'(n) = jtm/n (n G N). Очевидно, вектор G71 (n),..., щ(п)) имеет многомер- многомерное нормальное распределение. Совершив предельный переход в L2(Q) при п —у оо, приходим к требуемому утверждению. ? § 15. Пусть, по-прежнему, имеется (расширенная Р-нулевыми событиями) фильт- фильтрация F = (^t)t^o и винеровский процесс W = {Wt, t ^ 0} относительно F. Обратимся теперь к более общим уравнениям, нежели D9). Именно, будем рас- рассматривать стохастическое дифференциальное уравнение dXt = b(t, Xt) dt + <r(t, Xt) dWt, 0 ^ t ^ Г, E9) с начальным условием Xq = Z, являющимся ^о-измеримой величиной. Понима- Понимается это уравнение просто как формальная запись стохастического интегрального уравнения Xt=X0+ f b(s,Xs)ds+ f a(s,Xs)dWs, 0 ^ t <: Г, Jo Jo F0) где b и а определены на [0, T] x Ж. При этом считается, что b(s, Xs) и a(s, Xs) — "хорошие" функции, для которых правая часть формулы F0) определена. Определение 5. Сильным решением уравнения F0) на отрезке [0,Т] называ- называется процесс X = {Xt, t G [0, Т]}, имеющий п.н. непрерывные траектории, согласо- согласованный с фильтрацией F и такой, что при подстановке его в левую и правую части формулы F0) при каждом t G [0, Т] получается равенство с вероятностью единица. Как уже говорилось выше, подстановка процесса X в правую часть F0) должна приводить к тому, что п.н. при всех t G [0, Т] определены оба содержащихся в ней интеграла (которые будут задавать непрерывные процессы). Разумеется, вместо от- отрезка [0, Т] можно рассматривать [и, v] или [и, оо). Определение 6. Говорят, что уравнение F0) имеет единственное сильное ре- решение Хна отрезке [0, Г], если из того, что процессы X = {Xt,t G [0,Г]}и U = {Ut, t G [0,Т]} являются (сильными) решениями указанного уравнения с одинаковыми начальными условиями, следует, что U есть модификация X (тем самым непрерыв- непрерывные процессы X vlU будут неразличимыми). Потребуем, чтобы выполнялось следующее условие Липшица: существует кон- константа L = L(T) > 0 такая, что \b(t,x)-b(t,y)\ + \a(t,x)-a(t,y)\ ^ L\x - у\, х,УеЖ, te [0,Г]. F1)
292 А. В. Булинский, А. Н. Ширяев Пусть также для некоторого с = с(Т) > О \b(t,x)\2 + \a(t,x)\2 ^сA + ж2), xGi, te[0,T]. F2) Заметим, что даже при а = 0, т. е. при изучении обыкновенного дифференциально- дифференциального уравнения, без условий такого рода решение может не существовать или сущест- существовать, но быть неединственным. Если, в частности, b(t, х) = b(x), a(t, х) = сг(х), то F1) влечет F2). § 16. Далее нам понадобятся простые вспомогательные утверждения. Лемма 5. Пусть действительный процесс Y = {У5, s Е [0,Т]} прогрессивно измерим на [0,Т] относительно фильтрации ?т = (^s)se[o,T] в &• Пусть дей- действительная функция a{s,x) задана на [0,Т] х!и измерима, т. е. Й§([0,Т]) (8) (8) ё% (Ж)-измерима. Тогда процесс U = {Щ = a(?,lt), t G [0,Т]} прогрессивно измерим (относительно той же фильтрации F71). Доказательство. Покажем Й8([0, t]) ® ^ I ^([0, *]) 0 ^(М)-измеримость ото- отображения (s,o;) ь-)> E,^@;)) для^ G [0,t] ни G П @ ^ t ^ Г). Возьмем 1х е [0, t] и Б G Й8(М), тогда {(s,cj) G [0,t] х П: (в,УвИ) G [0,и] х В} = = {(s,u;) e [0,tAu] х u:Ys(u) е В} Требуемая измеримость получается в силу следствия 1 главы I. Теперь заметим, что для (s,x) G [0, t] х Ж отображение (s,x) н->- a(s,x) является по условию Й§([0, ?]) 0 0 ^(М)-измеримым. Остается учесть, что суперпозиция измеримых отображений будет должным образом измерима. ? Пусть измеримые на [0, Т] х Ж функции Ъ и а удовлетворяют введенным выше усло- условиям F1), F2). Возьмем X® = Z,t G [0, Т], и положим для п ^ 1 f Jo F3) To, что эта рекуррентная процедура законна, обосновывает Лемма 6. Пусть Ъ и а — описанные выше функции. Пусть величина Z яв- является 3*го-измеримой, причем Е Z2 < оо. Тогда при каждом п G N процесс Х(п) _ {х\.п\ t G [0,Т]} корректно определяется формулой F3), прогрессив- прогрессивно измерим (т. е. допускает такую модификацию). При этом sup E |Х^|2 < te[o,T] 00, п G N, и правая часть F3) может быть выбрана п. н. непрерывной на [0,Т].
Гл. VIII. Стохастический интеграл. Стохастические дифференциальные уравнения 293 Доказательство. Функция Ys(uS) = Z(u), где s G [0,T],u; G П, прогрессивно измерима, поскольку Z G ^о | Зё(Ж). По лемме 5 процессы b(s,Z(u)) и a(s,Z(u)) прогрессивно измеримы (s G [О, Г]). В силу F2) sup Ecr2O,Z) ^ c(l + EZ2) < оо. sG[O,T] Поэтомупроцесс{сг(з,^),? G [О, Г]} G ^тПЬ^и / cr(s, Z) dWs при? G [О, Г] может быть выбран, согласно теореме 4, п.н. непрерывным (и прогрессивно измеримым). Учитывая прогрессивную измеримость {b(s, Z"), s G [О, Т]} и условие F2), имеем ?2)I/2<оо. По теореме Фубини интеграл / b(s, Z") (is конечен для п. в. о; и для этих и процесс </О / b(s, Z) (is будет непрерывен по t G [О, Т]. Прогрессивная измеримость указанного ^ о интеграла с переменным верхним пределом следует из теоремы 4. Точно так же объясняется прогрессивная измеримость и п. н. непрерывность пра- правой части F3) в предположении, что процесс {X(n~1\t), t G [О, Г]} прогрессивно измерим и sup E \Xs |2 < оо. Кроме того, sG[O,T] Е {X[n)f < 3 (е Z2 + Е [j* Ь(в, Х^)) z2 + Tc f E(l + \X(sn-^\2)ds + c f + sup E\X(n-V\2)) < oo, sG[O,T] УУ откуда вытекает, что sup E |Xtw |2 < оо. П te[o,T] Лемма 7 (Гронуолл). Пусть у = y(t) — неотрицательная непрерывная на [О, Т] функция, удовлетворяющая неравенству y(t)^co + q f y(s)ds, tG[O,T], F4) Jo с некоторыми константами со ^ 0 и q ^ 0. Тогда y(t)^coeqt при tG[O,T]. F5)
294 А. В. Булинский, А. Н. Ширяев Доказательство. Из F4) в предположении, что co + q y(s) ds > 0, имеем для </О t ^ 0 (в нуле правая производная) (ln(co + W y(s)ds)) = qVt ^q. F6) V V Jo JJ co + qjoy{s)ds Интегрируя от 0 до t, получаем / f* \ ln[co + q y(s)ds) -Inco^qt, t e [Q,T]. \ Jo ) Следовательно, y(s)ds^coeqt, t e [Q,T\. Jo Самостоятельно рассмотрите элементарные случаи, когда проведенные рассужде- рассуждения необходимо уточнить (если со = 0 и y(s) = 0 при s G [0, и], то брать логарифм в F6) нельзя). ? § 17. Следующий результат представляет собой теорему существования и един- единственности решения стохастического дифференциального уравнения. Теорема 8. Пусть измеримые на [0,Т] х Ж функции Ъ и а удовлетворяют условиям F1) и F2). Пусть величина Z является &о-измеримой, причем EZ2 < оо. Тогда существует единственное сильное решение уравнения F0) с ^о-измеримым начальным условием Хо = Z такое, что Xt G L2(Q) для любого t G [0,Т] и функция EXf ограничена на [0,Т]. Доказательство. Воспользуемся методом последовательных приближений, оперируя п. н. непрерывными процессами Х^ = {Х\п , t G [0,Т]}, определенными в F3). Разобьем доказательство на несколько шагов. А. Оценим сверху Е \Х\п —Х^п) |2. Если п = 0, то, учитывая F2), для t G [О, Т] имеем | [ b(s,Z)ds+ f a(s,Z)dW \Jo Jo 2 f b(s,Z)ds) +2E / a(s,Z)dWs о ) \Jo ft / c(l + \Z\2)ds ^ Jo EZ2) ^ Mit, Mi = 2c(T + l)(l + EZ2). F7)
Гл. VIII. Стохастический интеграл. Стохастические дифференциальные уравнения 295 Для п ^ 1 и t Е [О, Т], применяя F1), получаем = Е (f (b(s, iW) - b(s, Xl"-1))) Г L\X^-Xin-^\ds^j +2E Г Г (a(s, *(">) - a(s, X Г (a(s,X^) - a(s,Xin- ds F8) [ Из F7) и F8) по индукции заключаем, что при М = max{Mi, 2L2A + Т)} (n) („ , n = 0,l,..., te[O,T]. F9) При т > п ^ 0 и ? ? [О, Т] в силу F9) т — 1 v(n)i2\l/2 ^ V^ I k=n oo G0) Полнота пространства L2(Q) влечет существование для каждого t G [0, Т] предела в Ь2(п) величин Х^ при п —> оо. Этот предел обозначим Y^ (t G [0, Т]). В. Теперь заметим, что sup |-*Q ^ — Х^ | оценивается сверху величиной 0<t<T sup [ Jo Теорема 2, следствие б главы IV, неравенства F8) и F9) влекут оценки Р( sup |Xt(n+1) -X[n)\ > 2" Р ( SUp >2- Г Jo o2n+2 / р \~(„ х(п') — <т(S Х^п~^ Л ' v' s ' y's fT Mnsn DMT)n+1 'Х) Уо n! dS ^ (n + 1)! G1)
296 А. В. Булинский, А. Н. Ширяев По лемме Боре ля-Канте л ли из полученного неравенства следует, что Р( sup |Xt(n+1) - Х[п)\ > 2~п бесконечно часто) = 0. G2) Поэтому для п.в. uj существует Щ = Nq(uj) такое, что для всех п sup |Xt(n+1)-Xt(n)|^2—. G3) te[o,T] Итак, последовательность п-1 - *«WM), G4) к=0 состоящая из функций, п.н. непрерывных на отрезке [0, Т], с вероятностью 1 равно- равномерно сходится на этом отрезке. Пусть По — множество, где все Х\ непрерыв- непрерывны на [0,Т] и равномерно сходятся; Р(По) — 1- Лля ^ Е По обозначим Xt(uS) = = lim Х).71' (uj). Для uj G Q \ По и ? G [0, T] пусть Xt(о;) = 0. Отсюда вытекает, что п—^оо Xt(uS) является непрерывной функцией на [0, Т] для всех ио. Величина Xt G 3*t \ ё$(Щ в силу леммы б главы I как предел величин Х).п , являющихся ^-измеримыми. Про- Прогрессивная измеримость процесса {Xt, t G Т} вытекает из его непрерывности и со- согласованности с фильтрацией Ft . Очевидно, Yt = Х± п.н. при каждом t G [0,Т], где процесс У = {It, t G [0,Т]} построен в пункте А. С. Покажем, что процесс X = {Xt, t G [0, Т]} есть решение уравнения F0). Из G0) для всех t е [0, Г] и m G N имеем () () Е >fr .л, () с(М,Г). G5) Поэтому и H*tllL2(n) ^ ||^1ь2(п) +с(М,Т) для t G [0,Т]. G6) Пользуясь теоремой Фубини и теоремой Лебега о мажорируемой сходимости, с уче- учетом G5) и G6) получаем, что Е [ (X8-XWJds= f E(Xs-X^Jds^0 при n^oo. G7) В силу F1) и G7) для каждого t G [0, Т] при п —у оо Г (r&X^dWs1^ I a(s,Xs)dWs, Jo Jo f b(s,xW)dsL^P f b(s,Xs)ds. Jo Jo
Гл. VIII. Стохастический интеграл. Стохастические дифференциальные уравнения 297 Таким образом, для каждого t Е [О, Т], переходя к пределу в F3) по подпоследова- подпоследовательности {пш = nm(t)}, получаем F0). D. Докажем единственность решения. Прежде всего покажем, что если процесс X = {Xt, t G [0,Т]} непрерывен п.н. на [0,Т] и sup EX2 < оо, то этот процесс непрерывен в среднем квадратическом на [0, Т]. * ? [° > ^ Дляз,? е [0, Г] имеем Е (Xt - Xsf = Е \ [ Ь(щ Хи) du+ [ а(щ Хи) dWu] <: du < rt rt 2{t- s) I Eb2(u,Xu)du + 2 I E(j2(u,Xu) J s J s sup I [0,Т] Заметим также, что ||-X"t Hz,2m\ является функцией, непрерывной на [0, Т]. Пусть Xt — сильное решение F0) с начальным условием Xq = Z, a Xt — сильное решение F0) с начальным условием Хо = Z (Z и Z удовлетворяют условиям доказываемой теоремы), причем функции ЕХ2 и ЕХ2 ограничены на [0, Т]. Тогда процесс X — X непрерывен в среднем квадратическом на [0,Т] и функция y(t) = = Е (Xt — XtJ непрерывна на [0, t]. Аналогично F8) находим, что E\Xt-Xt\2 ^3E|Z-Z|2 + 3E ( / (b(s,Xs)-b(s,Xs))ds' /о + ЗЕ (J (a(s,X8) - <t(s,) Ce\xs- Jo ^3E|Z- Z\z + 3A + t)L2 E\Xs-Xs\2ds. G8) Таким образом, получаем неравенство F4), в котором co = E\Z - Z\2,q = 3A + T)L2. Теперь нужное нам утверждение о единственности сильного решениявытекает из леммы 7: если Z = Z п.н., то со = 0 в F4) и, следовательно, Е \Xt — Xt\2 = 0 для каждого t G [0,Т]. Принимая во внимание непрерывность траекторий \Xt — Xt\, заключаем, что процессы X и X неразличимы, т. е. P(Xt(u) = Xt(u) для всех t G [0,Г]) = 1. ? Замечание 2. Единственность сильного решения уравнения F0) можно дока- доказать в более широком классе процессов (см. упражнение 21). § 18. Этот раздел посвящен свойству марковости (сильных) решений стохасти- стохастических дифференциальных уравнений. Заметим, что теорема 8 очевидным образом переформулируется на случай, когда вместо [0,Т] рассматривается промежуток [и,Т],0 ^ и < Т < оо, т. е. когда ищется решение уравнения Zt=?+ f b(s,Zs)ds+ [ a(s,Zs)dWs G9)
298 А. В. Булинский, А. Н. Ширяев с начальным условием Zu = ?, измеримым относительно 3*и (здесь, как и ранее, броуновское движение W = (Wt,&t)t^o)- Это решение Zt обозначим ^t(?), te[u,T]. Для процесса X = {Xt, t G [0, Т]}, являющегося решением уравнения F0), имеем rt rt Xt = X0+ b(s,Xs)ds+ a(s,Xs)dWs = Jo Jo f b(s,Xs)ds + Г a(s,Xs)dWs+ f a(s,Xs)dWs = Ju Jo Ju = XU+ f b(s,Xs)ds + f a(s,Xs)dWs. (80) J и Jи Учитывая, что Хи есть 3*u-измеримая величина, ЕХ% < оо, и пользуясь единст- единственностью сильного решения уравнения G9), получаем, что при каждом начальном условии Хо (Хо G &ъ | ЩЖ), EXl < оо) Xt((j) = Zt(Xu,u) п.н. для t^u. (81) Нам понадобится следующее утверждение. Лемма 8. Пусть b = b(s, х) и а = a(s, x) удовлетворяют при (s, x) G [и, Т]х1 условиям теоремы 8. Тогда для любого t G [и,Т] и любой ? G &и (с Е^2 < оо) величина Zt(?,uj) измерима относительно а-алгебры s — a{^j Ws — WUJ s G [^,t]}, расширенной классом нулевых событий. Доказательство. Воспроизводя доказательство теоремы 8, получаем, что Zt((;,uj) есть п.н. предел t G [и,Т], а если п ^ 1, то р , есть п.н. предел величин Z^ (?,о;) при п —> оо, где Z\ (?,u;) = ^ для ))^, t€[u,T\. (82) Нетрудно видеть, что Z\ G s4\u,t\ I ^(^)- Индукция и аналог леммы б (для от- отрезка [и, Т] вместо [0, Т]) обеспечивает si[u,t]-измеримость Z}n^ (?, о;) при всех n ^ 1. Применяя лемму б главы I, получаем требуемое утверждение. ? Теперь нетрудно установить следующий результат. Теорема 9. Пусть выполнены условия теоремы 8. Тогда сильное решение стохастического уравнения E9) является марковским процессом. Доказательство . Достаточно убедиться, что для любых точек unt таких, что O^u^t^Tn произвольной ограниченной борелевской функции /: Ш. —У Ж E(f(Xt)\&u)=E(f(Xt)\Xu). (83) Поскольку Xt{uo) = Zt(Xu( -),uj) п.н. (используются те же обозначения, что в лем- лемме 8), то вместо/(Х^) в (83) можно рассматривать f(Zt(Xu,uj)). По лемме 8 величи- величина f(Zt (Xu, и) является ограниченной и si[u,t]-измеримой. В силу леммы 2 главы VI
Гл. VIII. Стохастический интеграл. Стохастические дифференциальные уравнения 299 можно получить f(Zt(Xu,uj)) как предел п.н., так ив L1(Q, ^, P) конечных линей- линейных комбинаций случайных величин вида г\ — g(Xu)h\ (Wt1 — Wu) • • • h(Wtrn — Wu), где g, /ii,..., hm — ограниченные борелевские функции, и ^ t\ < • • • < tm ^ t, m G N. Вспоминая, что Xu есть 3<u-измеримаявеличина, a Ws — WUJL^U при s ^ u, имеем E (v I &u) = giXJEtniWt! -Wu)--- hm(Wtm - Wu). Очевидно, E G71 Xu) равно тому же выражению. Совершив предельный переход по должной последовательности упомянутых сумм (величин типа 77), приходим к соот- соотношению (83) в общем случае. ? Следствие 1. Процесс Орнштейна-Уленбека является не только гауссов- ским {см. теорему 7), по и марковским процессом. Дополнения и упражнения Из материала, рассмотренного в главе VIII, ясно, что вопрос о возможности ин- интегрирования случайных функций / = f(t,uj) напрямую связан со свойствами их измеримости. Имея это в виду, приведем ряд важных определений. В становлении и развитии "общей теории случайных процессов" большую роль сыграло осознание того, что в дополнение к тройке (П, &, Р) надо еще "подключать" поток сг-алгебр F = (^t)t^o? индексированных временным параметром t из множест- множества М_|_ (или некоторого его подмножества), с интерпретацией ^ как совокупности событий из ^, наблюдаемых до момента времени t (включительно). Глубокие результаты "общей теории случайных процессов" показывают, что все "идет гладко" и сама эта теория принимает весьма стройный вид, если считать, что фильтрация F = (^t)t^o удовлетворяет так называемым "обычным" условиям, т.е. является непрерывной справа (^ — ^t-\--> гДе ^t+ — П ^s, для всех t ^ 0) и ^ s>t ' сг-алгебра & пополнена множествами Р-меры нуль, а каждая сг-алгебра SPt содер- содержит множества из ^ с Р-мерой нуль. Определение 7. Подмножество А из М+ х П называется прогрессивно измери- измеримым (или просто — прогрессивным), если процесс X = 1а (т. е. Xt(uS) = 1а(^), t ^ 0, uj G Q) прогрессивно измерим. Совокупность прогрессивных множеств А образует сг-алгебру, обозначаемую Prog. Понятно, что Prog С 38A+) 0^оо, где ^оо = \/ &f Каждый прогрессивно измеримый процесс, очевидно, адаптиро- ван (т. е. согласован с фильтрацией F). 1. Приведите пример адаптированного, но не прогрессивно измеримого про- процесса. Следующие две сг-алгебры (опциональных и предсказуемых множеств) играют в стохастическом анализе важную роль в самых разнообразных вопросах.
300 А. В. Булинский, А. Н. Ширяев Определение 8. Опциональная сг-алгебра б подмножеств М+ х П есть сг-алгеб- ра, порожденная всеми согласованными процессами X = {Xt,t ^ 0}, рассматривае- рассматриваемыми как отображения (?, и) н-» Xt(uo) из М+ х (] в (М, Й§(М)), траектории которых принадлежат пространству Скорохода D[0, oo). Процессы, которые измеримы отно- относительно сг-алгебры б, называются опциональными. 2. Докажите, что равносильным образом опциональная сг-алгебра б может быть определена как сг-алгебра подмножеств в М_|_ х П, порожденная стохастичес- стохастическими интервалами вида где г (со) — марковские моменты (относительно потока F). Определение 9. Предсказуемая сг-алгебра & подмножеств М+ х Q есть сг-ал- сг-алгебра, порожденная всеми согласованными процессами X = {Xt, t ^ 0}, рассмат- рассматриваемыми как отображения (t,uj) н-» Xt(uS) из М+ х Ив (Ж,Зё(Ж)), траектории которых принадлежат пространству С = С(М+, Ж) непрерывных на М+ действи- действительных функций. Процессы, которые измеримы относительно сг-алгебры &, назы- называются предсказуемыми. 3. Докажите что равносильным образом предсказуемая сг-алгебра & может быть определена как сг-алгебра, порожденная всеми согласованными процес- процессами, траектории которых лишь непрерывны слева на @, оо). Имеются и другие альтернативные (и во многих рассмотрениях более удобные) определения, см., например, [26] и [46]. 4. Проверьте, что сг-алгебра 9> порождается каждым из следующих классов подмножеств: (a) {0} х А с А е ^о и множествами вида (t, оо) хБсБе^,^0; (b) {0} х А с A G ^о и стохастическими интервалами [[0,т]] = {(t,u):0 ^ t ^ т(ии)} или ]]т, оо[[ = {(t,u): г (и) < t < оо}, где т — марковский момент; (c) {0} х А с А е ^о и множествами вида (s, t] х В с Б G ^s, s ^ t. 5. Убедитесь, что между введенными сг-алгебрами имеют место следующие включения: & С ^ С Prog С Й8(М+) (g) J^oo. Напомним, что по теореме 4 всякий согласованный с фильтрацией F = (^t)t^o непрерывный слева или непрерывный процесс является прогрессивно измеримым. 6. Пусть процесс X является прогрессивно измеримым иг — конечный марков- марковский момент. Покажите, что случайная величина Хт будет ^-измеримой (это свойство является едва ли не определяющим целесообразность введения понятия прогрессивной измеримости).
Гл. VIII. Стохастический интеграл. Стохастические дифференциальные уравнения 301 7. Проверьте, что простые (кусочно-непрерывныесправа) функции/ = f(t,uj) вида гп-1 к=0 где величины ?&(о;) — i^-измеримы, 0 = to < • • • < tm = Т, являются опциональными. 8. Покажите, что простые (кусочно-непрерывные слева для ? Е @,Т]) функции / = f(t,u) вида т —1 f(t,u) = ri(u)l где величина 77@;) — <^о-измерима, щ(и) — i^-измеримы, 0 = to < • • • < < tm = Т, являются предсказуемыми. 9. Проведите построение интеграла Ито, изложенное в § 2 и § 3, отправляясь от класса опциональных простых функций, фигурирующих в упражнении 7 (ко- (которым приписывается то же значение интеграла Ито, что и для предсказуе- предсказуемых простых функций из упражнения 8). Можно ли утверждать, что замы- замыкание в 13р данной совокупности простых функций, входящих в L|i, приводит к пространству sUt П L\l. 10. Пусть т — марковский момент такой, что г (и) ^ Т при всех ио G О, (Т — положительная константа). Докажите, что /т(/) = /т(/1(о т])? где ^т(/) := 11. Докажите, что если /: [0, оо) —У Ж есть неслучайная функция из пространст- пространства Скорохода D [0, оо), то процесс It (/), t ^ 0, является гауссовским. Найдите его среднее и ковариационную функцию. Обсудим теперь некоторые обобщения интеграла Ито. Интегрирование можно определить для следующего класса функций J\. Определение 10. Будем говорить, что функция / принадлежит классу Ji, если /: [0, оо) х П —у Ж прогрессивно измерима и at \ f2(s,u)ds < оо = 1, t >0. (84) / Идея построения интеграла It(f) для функции / G J\ состоит в том, чтобы по- подобрать последовательность функций /n, n G N, для которых интеграл It(fn) Уже построен, и таких, что ft 2 Р / (/(s,o;) —/n(s,o;)) ds —> 0 при п —> оо. (85) Отсюда выводится, что последовательность {It (/n) }nGN является фундаменталь- фундаментальной по вероятности. Следовательно, существует случайная величина, обозначаемая
302 А. В. Булинский, А. Н. Ширяев р /t(/), такая, что It(fn) ~^ h(f), п ~* оо. Предел /*(/) записывают, по-прежне- по-прежнему, как / f(s,u)dWs или (/ • W)t- Часто броуновское движение обозначают {Bt, t ^ 0}, и тогда пишут (/ • B)f. 12. Докажите, что существует последовательность {/n, n E N}, описанная выше в (85). Докажите, что если / Е Ji, то существует п.н. непрерывная модифи- модификация процесса /*(/)? t ^ 0. Отметим, что для / Е Ji процесс /*(/) не обязан быть мартингалом, но являет- является локальным мартингалом (т.е. процессом, для которого существует последова- последовательность моментов остановки тп таких, что тп t оо п.н. (п —У оо) и для каждого п "остановленные" процессы Цп (/) := It/\rn (/)? гДе ^ ^ 05 являются мартингалами). 13. Рассмотрим для / Е Ji процесс -77 / /2(s,^)rf4, t^O. (86) Докажите, что (iZt = Ztf{t, и) dWt (по-прежнему, все дифференциальные со- соотношения означают сокращенную запись соответствующих интегральных). Подробнее со стохастическим исчислением можно ознакомиться по книгам [26, 124, 128]. Кратные стохастические интегралы рассматриваются в [163]. Стохастический интеграл по винеровскому процессу (броуновскому движению) позволяет дать "явное" представление фрактального броуновского движения. Определение 11. Случайный процесс X = {Xt, t ^ 0} со значениями в Мт на- называется автомодельным (самоподобным), если для каждого а > 0 можно найти такое Ъ > 0, что Law(Xat, t ^ 0) = LawFXt, t ^ 0). (87) Иначе говоря, для таких процессов изменение временной шкалы (t н-» at) при- приводит к тому же самому результату, что и изменение фазовой шкалы (х н-» Ьж). Если в определении (87) для любого а > 0 параметр Ъ = а^, то случайный про- процесс X называется автомодельным процессом с показателем Харста Н. Вели- Величина D = 1/Н называется статистической фрактальной размерностью случай- случайного процесса X. Напомним, что фрактальное броуновское движение В^н^ = {B^H\t), t ^ 0} для 0 < Н ^ 1 определяется как центрированный гауссовский процесс с ковариаци- ковариационной функцией cov(B^H\s),B^H\t)) = s2H+t2H -\s- t\2H, 8,t>0. При Н = 1/2 процесс .ЕК1/2) = {B^^2\t), t ^ 0} является броуновским движением. 14. Докажите, что В^н^ = {B^H\t), t ^ 0}, где 0 < Н ^ 1, есть автомодельный процесс (с показателем Харста Н). Процессы В^н^ впервые рассматривались А. Н. Колмогоровым в 1940 г. (см. [33]) и были названы им спиралями Винера. Термин фрактальное (или дробное, "frac- "fractional") броуновское движение был введен Б. Мандельброттом и И. Ван Нессом в 1968 г. в работе [164], и там же была получена следующая теорема.
Гл. VIII. Стохастический интеграл. Стохастические дифференциальные уравнения 303 Теорема 10 (Мандельбротт, ВанНесс). Для 0<H<lut^0 фрактальное броуновское движение В^н^ допускает представление где нормирующая константа сн выбрана так, что E(B^H\l)J = 1 (см., на- например, [86; т. 1, с. 281]), a {Ws, s ^ 0} и {Ws, s ^ 0} — независимые стандарт- стандартные винеровские процессы, причем Ws = W-s для s ^ 0. Фрактальное броуновское движение (и его дискретные аналоги) используются во многих важных моделях, в частности, для описания динамики финансовых индексов (см. [86]). Сложность изучения процессов B^H\t) с 0 < Н ^ 1 состоит в том, что за исключением случая Н = 1/2 (броуновского движения) и Н = 1 эти процессы не являются семимартингалами (т. е. не входят в важный класс процессов, для ко- которых развито стохастическое исчисление, см. [46; гл. 4]). Если в представлении (88) вместо Нвзятьгёльдеровскую функцию Ht (т.е. \Ht — Hs\ ^ c\t-s\a,a>0) со значе- значениями в @,1), то получается случайный процесс, называемый мулътифракталъным броуновским движением. 15. Докажите, что для процесса В^н^ при п —у оо Нп := In n > №M>(k/n) - В(н>((к - l)/n) / In(l/n) -)¦ Я п.н. v fe=i V/ Пусть Ji ([О, Т]) —пространство прогрессивно измеримых функций /: [О, Т] х П —у —У М, удовлетворяющих (84) при t G [О, Т]. Следующая теорема, основанная на понятии стохастического интеграла, описы- описывает структуру броуновских функционалов. Теорема 11 (Ито, Кларк). Пусть (&t^)te[o,T] — естественная фильтрация броуновского движения W = {Wt, t G [0,Т]} и X — X(uj) является ^^Г | ё$(Ж)- измеримой случайной величиной. Справедливы следующие утверждения. 1. Если ЕХ2 < оо, то найдется такой стохастический процесс f = {/(s), s G [О,Г]} G dTr\L2T, что = EX f(s,co)dWs п.н. (89) 2. Если Е\Х\ < оо, то представление (89) справедливо с некоторым про- процессом / G Ji([0,T]). 3. Если X — положительная случайная величина (т. е. Р(Х > 0) = 1) и ЕХ < оо, то найдется процесс f G Ji[0,T] такой, что X допускает пред- представление вида X = ZtEX, где Zt определяется согласно (86). 16. Объясните, почему величина X, фигурирующая в теореме 11, есть функ- функционал от броуновского движения, т.е. Х(ио) = g(W(s,uo), 0 ^ s ^ T), где g: C[0,T] ^Rnge Зё(С[О,Т\)\Зё(Ж).
304 А. В. Булинский, А. Н. Ширяев Доказательства теоремы 11 в различных вариантах даны К. Ито, Дж. Кларком, Дж. Дубом и приведены во многих книгах, см., например, [46, 182]. В качестве примера использования представления функционалов от броуновского движения в виде стохастических интегралов приведем следующую экстремальную задачу: среди всех моментов остановки т (относительно естественной фильтрации бро- броуновского движения W = {W(t),t Е [0,1]}) найти такой момент т*, для кото- которого достигается 2 V* = inf E WT — max Ws . Можно представить себе модель, в которой колебания цен на акции в течение единич- единичного периода времени (например, день) описываются броуновским движением (разу- (разумеется, это лишь интерпретация, так как цены не бывают отрицательными) и требу- требуется выбрать момент т* для самой выгодной в среднем квадратическом смысле про- продажи имеющихся акций. Заметим, что WT^ дает смещенную оценку для max Ws, поскольку Е WT^ = 0 и Е max Ws = л/2/тг (поясните в качестве простого упраж- нения). Поэтому можно обратиться к величине V = inf E 2 E WT + а - max Ws тЕ[0,1] Легко проверить (убедитесь в этом), что V = У* - 2/тг. Положим St = max WSJ t e [0,1]. Теорема 12 (Граверсен, Пешкир, Ширяев). Решение приведенной выше зада- задачи дается формулой г* =ini{te [0,1]: St-Wt = z*y/T=t}, где константа z* находится из уравнения здесь Фиф — соответственно функция распределения и плотность стандарт- стандартной нормальной величины. При этом z* = 1.12..., V* = 2Ф(;г*) — 1 = 0.73.... Ключевую роль при доказательстве этого результата играет представление вида max Ws=b+ / f(s,u)dWs, где b = const (укажите эту константу) и f(s,u) = 2{i-$((St-wt)/y/T=t)}, se[o,i], ooen. Широкое использование результатов и методов стохастического анализа связано с возможностью построения теории стохастического интегрирования по процессам более общей структуры, нежели броуновское движение.
Гл. VIII. Стохастический интеграл. Стохастические дифференциальные уравнения 305 Определение 12. Согласованный с фильтрацией F = (^t)t^o процесс А = = {At; t G M+} называется возрастающим, если для п.в. ио имеем Ао(и) = 0, где At (и) — неубывающая функция по t Е [0, оо) и Е At < оо для каждого t Е М+ . Определение 13. Действительный случайный процесс Y = {Yt, t ^ 0} при- принадлежит классу Дирихле (D), если семейство величин {Ут, т Е 5} равномерно интегрируемо, где 5 обозначает совокупность всех конечных марковских моментов (относительно фильтрации F). Справедлив следующий фундаментальный результат (см. [26, т. 1, с. 68]; сравните с теоремой 1 главы IV). Теорема 13 (разложение Дуба-Мейера). Субмартингал X = (Xt, &t)t^o клас- класса (D) с траекториями типа cadlag допускает и притом единственное (с точ- точностью до стохастической неразличимости) разложение вида где А = (At,^t)t^o — возрастающий предсказуемый интегрируемый (ЕАоо, где Аоо = limt-юо At) процесс, а М = (Mt,^t)t^o есть равномерно интегрируемый мартингал. Рассмотрим для простоты непрерывный квадратично интегрируемый мартингал М = (Mt)t^o на фильтрованном вероятностном пространстве (П, ^, (^t)t^o? P) та~ кой, что Mq = 0. Класс таких мартингалов обозначается М^- Будем предполагать, что фильтрация F = (^t)t^o удовлетворяет обычным условиям. Наметим схему построения интеграла по процессу М — (Mt,^t)t^o ? ^2- Пусть (М) = ((M)t)t^o —возрастающий предсказуемый процесс, участвующий в разло- разложении Дуба—Мейера субмартингала Mf, t ^ 0, обладающий тем свойством, что Е(М)оо < оо. Как и в случае броуновского движения, интеграл 1Т(Х)= / Xt((j)dMt(uj), (90) Jo вообще говоря, не может быть определен потраекторно, как интеграл Лебега-Стил- тьеса (можно определить в тривиальном случае Mt = 0 при t ^ 0). Процесс X = {Xt, t ^ 0, uj G Щ назовем простым, если существуют стро- строго возрастающая числовая последовательность {?п}?^0 с to = Ои lim tn = оо, п—^оо а также последовательность действительных случайных величин {/п(и)}^=0 и не- некоторая константа 0 < с < оо, для которых sup |/n(cj)| ^ с при всех и; G П, причем (bH1]W, 0 ^ t < оо. (91) /с=0 Класс простых процессов обозначим J^o и для X G i^o положим п-1 /t(X) = Y^ fk(Mtk+1 - Mtk) + fn(Mt - Mtn) при tn^t< tn+1. (92)
306 А. В. Булинский, А. Н. Ширяев Далее доказывается, что определение (92) можно распространить (снова используя должные аппроксимации простыми процессами) с ??$ на более широкий класс про- процессов. Интересно отметить, что этот класс будет зависеть от того, является ли функция (M)t для п. в. uj абсолютно непрерывной относительно меры Лебега. Де- Детальное построение и дальнейшие обобщения можно почерпнуть в книгах [46, 182]. 17. С помощью формулы Ито докажите, что для детерминированной функции f = /(?) из С1 справедлива формула "интегрирования по частям": f f(s) dWs = f(t)Wt - I f'(s)W8 ds. (93) Постройте пример случайной функции / = /(?, о;), для которой формула (93) не имеет места. Пример 2. Рассмотрим стохастически возмущенное уравнение роста популя- популяции —-— = rXt, r = const. (94) at Точнее говоря, рассмотрим стохастическое дифференциальное уравнение вида dXt = rXt dt + aXt dWt (95) с начальным условием Хо (Хо Е 3* \ ?$(Ж) и Е Xq < оо), где константы а > 0, г Е Ж. Из формулы Ито непосредственно следует, что процесс Xt = х0е(г-ст2/2)*+^ (96) является сильным решением уравнения (95). Поскольку Xq JLW = {Wt, t ^ 0}, то EXt = EXoexp{rt}. (97) Иначе говоря, при Е Хо ф 0 средний рост (убывание, если г < 0) Е Xt будет таким же, как в случае детерминированной модели (94). ? Уравнение (95) играет важную роль в финансовой математике. Его реше- решение (96) часто называют "геометрическим броуновским движением" Локальный снос г — а2/2 характеризует скорость изменения среднего значения процесса X, диффузию а2 в финансовой литературе часто называют волатилъностъю. Види- Видимо, П. Самуэльсон был первым, кто осознал важность геометрического броуновско- броуновского движения для описания эволюции цен, используя для него термин " экономическое броуновское движение". Определения и результаты, излагавшиеся в главе VIII, ценой некоторых усложне- усложнений естественным образом переносятся на многомерный случай. Так, уравнение E9) можно понимать как систему, в которой Xt = (X). ,..., Х^ ), Ь — векторная, а — матричная функции, точнее говоря, о[ •, •): |_U, 1 \ х к —у К , G[ •, •): [и, 1 \ х к —У
Гл. VIII. Стохастический интеграл. Стохастические дифференциальные уравнения 307 W = {Wt, t ^ 0} — m-мерное броуновское движение (относительно фильтрации )- Считаем также, что (99) Интеграл от векторной функции определяется как вектор из интегралов от компо- компонент, а / a(s, Xs) dWs обозначает вектор-функцию с г-й компонентой, равной ^ ro.ti k=1 Определение прогрессивной измеримости очевидным образом переносится на мно- многомерный случай. Теорема 13 остается в силе, если в условиях F1) и F2) модуль Ъ понимать как норму в Мп, а модуль а понимать как норму матрицы, например, счи- п т тать, что \а\2 = X) Z) о\к. i=ik=i При исследовании систем стохастических дифференциальных уравнений важней- важнейшую роль играют многомерные варианты формулы Ито, т.е. формулы, описываю- описывающие стохастический дифференциал процесса H(t, Xf), где Н = H(t, xi,..., жп), а X = (Х^\ ..., Х^) — векторный процесс Ито. Другими словами, X имеет сто- стохастический дифференциал вида dXt = f{t,uo)dWt+g{t,uo)dt, t e [0,oo), A00) где / и д — соответственно матричная и векторная прогрессивно измеримые функ- функции, / = (fik(t,u), г = 1,...,п; к = 1,...,ш); д = (gi(t,u),... ,gn(t,v)), Wt = = (WJ; ,..., Wt ). Иначе говоря, пусть ; +gi(t,u)dt, i = l,...,n, A01) где при любом t > 0, всех г = 1,...,пи& = 1,...,т \gi(s,u)\ds < oo^j =1 и pf/ |/ifc(8,^)|2^<oo>) =1. A02)
308 А. В. Булинский, А. Н. Ширяев Теорема 14. Пусть функция Н: [0, оо) х W1 —у Ж такова, что Н Е С1'2, т. е. существуют непрерывные производные dH/dt и d2H/dxidxj, i,j = l,...,n. Тогда процесс Yt = H(t,Xt), где процесс Ито Xt определяется согласно A00), A02), имеет стохастический дифференциал, который задается формулой г=1 г=1j=l J здесь dX^ и dX\3\ фигурирующие в A01), "перемножаются по правилу": dWt • dWt = Sij dt, dt - dt — dW^ • dt = dt • dW^ = 0. Доказательство формулы Ито см., например, в [26]. По поводу разнообразных обобщений этой формулы см., например, [11, 146]. Приведем здесь лишь один результат в направлении упомянутых обобщений. Пусть X — стандартное (одномерное) броуновское движение W = {Wt, t ^ 0}. Пусть функция F = F{x) является абсолютно непрерывной, F(x)=F@)+ / f(y)dy, Jo причем функция / = f(y) G L2OC(M), т.е. для любого с > 0 f2(y)dy < оо. Определим [/(И^),!^] — квадратическую ковариацию процессов f{W) и W сле- следующим образом: [f(W),W]t = P-lim? (f(Wt(n4m+1)At)-f(Wt(n4m)At))x Здесь P-lim обозначает предел по вероятности, a {t^n\m), m G N} при каждом п G N—римановские последовательности детерминированных моментов t^n\m), т. е. ?(п) (т) ^ t^ (т + 1) для mGN при каждом п е N и t^ (т) -у оо при т -у оо; кроме того, для любого t > 0 (n)(m + l)A?-?(n)(m)At) -У 0, п -У оо. Важно подчеркнуть, что поскольку процесс f(W) не является, вообще говоря, семи- мартингалом, нетривиален факт существования предела по вероятности, задающе- задающего [/(W), W]t- Один из результатов работы [129] состоит именно в доказательстве существования этого предела.
Гл. VIII. Стохастический интеграл. Стохастические дифференциальные уравнения 309 Теорема 15 (Проттер, Фёльмер, Ширяев; [129]). При сделанных выше предпо- предположениях о функции F = F(x) имеет место формула f f(Ws)dWs + hf(W),W]t. A03) Jo z 18. Пусть функция / G С2. Докажите, что [f(W),W]t= f' f\Wa)ds, t>0. Jo Тем самым, формула A03) переходит в формулу Ито. 19. Пусть /(ж) = |ж|. Докажите, что [/(W), W]t = 2Lt@), где Lt@) — локаль- локальное время (П. Леви) броуновского движения в точке 0, определяемое следу- следующим образом: Lt = Hm-L / l{\W8\^e}ds. A04) siO IE Jo Таким образом, в данном случае формула (ЮЗ) переходит в формулу Танака для броуновского движения. Обобщению интеграла Ито на функции многих переменных посвящена моногра- монография [163]. Приведем только один результат этого направления. Теорема 16 (Ито). При всех t > 0 и п G N справедлива формула / ¦ / dWSl---dWSn= —Пп (-1 , A04) где Нп — полином Эрмита степени п, т. е. Нп{х) = {-1)пех I2—~(e~x ^2)j n = 0,1,... . Обратимся теперь к однородной диффузии, т.е. рассмотрим решение уравнения где Wt — ттг-мерный винеровский процесс, а функции Ь: Жп —> Жп и а: Жп - удовлетворяют условиям теоремы 8, которые в данном случае сводятся к одному тре- требованию: существует L > 0 такое, что \b(x)-b(y)\ + \a(x)-a(y)\^L\x-y\, х,у?Шп, A07) / п т \ (для вектора | • | обозначает евклидову норму и \а\2 = ^2 ^2 а2к ), из которого ^ i=ik=i J вытекает, что для некоторого с > 0 \Ъ(х)\2 + \а(х)\2 <: сA + |х|2), хеЖп. A08) Обозначим Xl'x единственное сильное решение уравнения A06) при t^s. 20. Докажите, что для каждого х G W1 определенный выше процесс Х^х, t ^ s, является однородным марковским процессом.
310 А. В. Булинский, А. Н. Ширяев 21. Докажите, что при выполнении условий теоремы 8 единственность сильного решения (на [0,Т]) будет в более широком классе процессов, у которых су- существует Е XI для t G [0, Г]. 22. Покажите, что если условия теоремы 8 выполнены на полуинтервале [0, Т), то ее утверждение также справедливо на этом промежутке, только в этом случае рассматривается класс процессов X, для которых функция ЕХ% бу- будет ограничена на каждом отрезке, вложенном в [0, Т). 23. С помощью формулы Ито докажите, что решением (одномерного) уравнения dXt = ^-^ dt + dWt, te [0, Г), Хо = а, A09) является процесс Xt = аA - t/T) + pt/T + (T-t) [ ^^ . (ПО) Jo T~s 24 (продолжение упражнения 23). Докажите, что Xt —> Р п.н. при t —у Т-. Таким образом, решение уравнения A09) представляет собой броуновский мост над отрезком [0, Т] с закрепленными концами Xq = а и Хт = Р- Стандартный броуновский мост получается при Т = 1на = Р = 0. Из различного рода обобщений теоремы существования и единственности реше- решения стохастического дифференциального уравнения приведем несколько неожидан- неожиданный результат А. К. Звонкина (см., например, [86; с. 322]), утверждающий, что для существования сильного решения стохастического дифференциального уравнения dXt = b(t, Xt) dt + dWt A11) достаточно лишь измеримости по (?, х) и равномерной ограниченности 6(?, х). 25. Докажите, что стохастическое дифференциальное уравнение dXt = <j(Xt)dt + dWt A12) с "плохим" коэффициентом а(х) = sgnx имеет и, притом, единственное сильное решение. Определение 14. Стохастическое дифференциальное уравнение E9) с на- начальным условием /i, где \± — заданная мера на Й§(М), имеет слабое решение на промежутке [0,Т], если найдутся фильтрованное вероятностное пространство (П, ^, (<^t)tE[o,T]j Р)? броуновское движение W = (Wt^t)te[o,T] и непрерывный (п.н.) случайный процесс X = (Xt, &t)te[o,T] такие, что Law(Xo) = \i и Р-п.н. для каждого t > 0 выполнено равенство F0). Разумеется, вместо отрезка [0, Т] можно рассматривать промежутки вида [u,v], где 0 ^ и < v < оо, или [и, v) для 0 ^ и < v ^ оо. Важно подчеркнуть, что в отличие от сильного решения, рассматриваемого на заданном фильтрованном вероятностном пространстве с заданным на нем броунов- броуновским движением, в определении слабого решения такие объекты (вероятностное про- пространство и броуновское движение) не фиксируются, а требуется лишь, чтобы они нашлись. Очевидно, сильное решение является и слабым.
Гл. VIII. Стохастический интеграл. Стохастические дифференциальные уравнения 311 Определение 15. Слабая единственность (слабого или сильного) решения урав- уравнения E9) означает, что у любых двух решений (в рассматриваемом смысле) с оди- одинаковыми начальными условиями совпадают распределения, т.е. совпадают конеч- конечномерные распределения. 26. Докажите, что уравнение dXt = a(Xt) dWu t G [0,1], Xo = 0, A13) где a(x) = sgnx, имеет no крайней мере два решения (но "слабых"). Кроме того, на некоторых вероятностных пространствах у этого уравнения может вовсе и не быть "сильного" решения. Для выполнения последнего упражнения следует убедиться, что процесс Bt= a(Ws)dWs, t G [0,1], a(x)=sgnx, Jo является броуновским движением на отрезке [0,1]. Это утверждение обеспечивает (см., например, [83]) следующая Теорема 17 (П. Леви). Пусть В = (Bt,&t)t^o — Р-п.н. непрерывный квад- квадратично интегрируемый мартингал, заданный на некотором фильтрованном вероятностном пространстве (П, &, (^t)t^o? Р)- Пусть (В%—t,&t)t^o также является мартингалом, т. е. Е(В* -B2S\&S) =t-s, O^s^t. A14) Тогда В = {Bt, t ^ 0} есть стандартное броуновское движение. Заметим, что М. Барлоу [92] доказал, что уравнение A13) может не иметь силь- сильного решения даже в случае непрерывных ограниченных функций а = а(х) > 0. 27. Пусть выполнены условия теоремы 8. Докажите, что решение (слабое или сильное) уравнения E9) слабо единственно. Упражнение 26 показывает, что могут существовать слабые решения, но не быть сильных. Поэтому естественно ожидать существования слабого решения при менее ограничительных условиях на коэффициенты уравнения E9). Один из первых результатов в этом направлении (см. об этом [86; т. 1, с. 325]) формулируется следующим образом. Рассмотрим стохастическое дифференциаль- дифференциальное уравнение dXt = b(Xt) dt + a(Xt) dWt A15) с начальным распределением \i = Law(Xo) таким, что Jr n(dx) = Е |Xq|7 < оо для некоторого ^ > 2. Если коэффициенты Ъ = Ь{х) и а = а(х) являются непрерывными ограниченными функциями, то уравнение A15) имеет слабое решение.
312 А. В. Булинский, А. Н. Ширяев Замечание 3. В предположении ограниченности и невырожденности коэффи- коэффициента а(х) утверждение о существовании и единственности (по распределению) слабого решения уравнения A15) остается в силе, если потребовать лишь огра- ограниченность и измеримость коэффициента Ъ(х). Приведенные результаты о слабых решениях также допускают обобщения на многомерный случай, на случай коэффи- коэффициентов, зависящих от прошлого и т. д. (см., например, [86; т. 1, гл. III]). Важную роль в теории стохастических дифференциальных уравнений (в част- частности, для построения слабых решений) играет теорема Гирсанова об абсолютно непрерывной замене меры, для формулировки которой введем необходимые обозна- обозначения. Пусть (n,^r,(^rt)t^o5P) — некоторое фильтрованное вероятностное пространст- пространство, W = (Wt,^t)t^o — m-мерное броуновское движение, W = (W1,..., W171). Пусть Ь = (bt,^t)t^o, где Ь = (Ь1,..., Ьш) — прогрессивно измеримый т-мерный случайный процесс такой, что р(У ||bs||2ds < оо) =1, te[o,T], A16) здесь ||6,||2 = (б*J + ¦ ¦ ¦ + {bff и Г < оо. Образуем процесс Z = (Zt,^t)te[o,T]j полагая Zt = expU (bs,dWs)-^J \\bs\\2ds\, A17) т где (bs,dWs):= ? bksdW*. k=i Лемма 10 (см., например, [86; т. 2, с. 326]). Если выполнено условие Новикова: \\bs\\2ds\ < оо, A18) то EZt = 1 и введенный в A17) процесс Z = (Zt,&t)te[o,T] будет равномерно интегрируемым мартингалом. В силу положительности Zf (Р-п. н.) и условия Е Zt = 1 на (П, ЗРт) можно задать вероятностную меру Qt, полагая QT(A) = E AAZT), A e &т. (П9) Теорема 18 (Гирсанов). Для введенных выше процессов W и Ъ определим Bt = wt- [ bsds, te[o,T]. Jo Тогда В = (Bt,&t)te[o,T] является т-мерным броуновским движением на фильтрованном пространстве (П, ^т, (&t)te[o,T]i Qt). Эта теорема, как уже было отмечено, дает возможность строить слабые решения стохастических дифференциальных уравнений вида dXt = b(Xt) dt + dWt, t G [0,T], A20) а также более общих уравнений (см., например, [86; гл. VII, § 36], [148; гл. 5]).
Гл. VIII. Стохастический интеграл. Стохастические дифференциальные уравнения 313 Наряду с интегралом Ито в ряде задач полезно обращение к "симметризованным" интегралам Стратоновича (или Фиска-Стратоновича): f f(s,u)odW8(w), A21) Jo где W = {Ws, s ^ 0} — броуновское движение, а функция / входит в определенный класс. Чтобы пояснить суть дела, скажем, что для некоторого класса функций / интеграл A21) строится с помощью интегральных сумм вида N-1 г=0 где 0 = t0 < • • • < tN = t и t* = (U + ?;+i)/2, г = 0,..., N - 1. Доводы в пользу трактовки стохастического дифференциального уравнения как интегрального уравнения на основе интеграла A21) можно прочитать в [169]. В част- частности, замена переменных в интеграле Стратоновича не приводит к появлению до- дополнительных членов второго порядка, возникающих в формуле Ито. Это использу- используется при изучении стохастических дифференциальных уравнений на многообразиях (см. [128]). В то же время при изучении стохастических уравнений вида Xt=X0+ I b(s,Xs)ds+ I a(s,Xs)odWs A22) Jo Jo требуются большие ограничения на коэффициенты, чем при изучении уравнений ви- вида F0). Если функция сг(?, x) дифференцируема по ж, то уравнение A22) равносильно следующему уравнению Ито: (s,Xs)ds + l f a/x(s,Xs)a(s,Xs)ds+ f a(s,Xs)dWs. A23) ^ Jo Jo В частности, если a(s,x) не зависит от ж, то обе интерпретации уравнения F0) сов- совпадают. Подчеркнем также, что интеграл Ито является мартингалом (например, при усло- условиях теоремы 2), а интеграл Стратоновича этим свойством не обладает. Для дальнейшего ознакомления с различными аспектами теории стохастических дифференциальных уравнений можно обратиться, например, к [11,130,154]. Стохас- Стохастические уравнения в частных производных изучаются в [196]. Квантовая диффузия рассматривается, например, в [88]. Заключительная часть этого раздела посвящена некоторым проблемам, относя- относящимся к самостоятельным научным направлениям в рамках общей теории случай- случайных процессов и ее приложений. Исключительно важным направлением является теория фильтрации. Предпо- Предположим, что на полезный сигнал налагается "шум" и требуется выделить (отфильт- (отфильтровать) этот сигнал из зашумленных наблюдений. Формализация так просто описан- описанного явления нетривиальна. Мы обратимся к модельной ситуации, которая привела к построению очень красивой теории.
314 А. В. Булинский, А. Н. Ширяев Пусть интересующий нас n-мерный процесс Xt, t G [0, Т], — полезный, но "скры- "скрытый" сигнал — описывается стохастическим дифференциальным уравнением (систе- (системой) E9) с начальным условием Хо = Z и пусть "наблюдаемый" g-мерный процесс It, t G [0, Т], задается стохастическим уравнением dYt = c(t,Xt)dt + 'y(t,Xt)dBu A24) где с: Mn+1 ->• M9,7:^n+1 ->• M.qXp. Считается, что все рассматриваемые слу- случайные процессы определены на некотором вероятностном пространстве (П, ^, Р), причем р-мерное броуновское движение В = {Bt,t ^ 0}не зависит от т-мерного броуновского движения W = {Wt, t ^ 0}, фигурирующего в E9). Одна из возможных здесь задач состоит в том, чтобы по наблюдениям {Ys, s G [0, ?]} дать оценку величины Xt для t G [0, Г]. Обозначив Жь = cr{Ys, s G [0, ?]}, будем искать оптимальную оценку Xt G J^i | ^(Мп) такую, что t-t/|2: U еЬ2(п,ЖиР)}. A25) Легко видеть, что Xt — Е (Xt | J%)- Мы будем предполагать, что все рассматри- рассматриваемые случайные величины обладают конечным вторым моментом (это относится к компонентам векторов), а также, что выполнены условия, обеспечивающие сущест- существование решений используемых стохастических уравнений. Следующий классический результат (см., например, [169]) для линейных систем явился одним из первых результатов, дающих удобные для практической реализа- реализации формулы, описывающие динамику оптимального фильтра. Теорема 19 (фильтр Кальмана-Бьюси). Пусть dXt =F(t)Xtdt + C(t)dWt, F(t) Glnxn, C(t) GlnXm, dYt =G(t)Xtdt + D(t)dBt, G(t) e RqXn, D(t) Пусть матрица D(t)D(t)* обратима для всех t и норма матрицы ограничена на каждом конечном промежутке изменения t. Тогда решение X = {Xt, t G [0,Г]} задачи A25) удовлетворяет стохастическому дифференци- дифференциальному уравнению dXt = (F- SG^DD*)-^)^ dt + SG^DD*)-1 dYt A26) с начальным условием Хо = ЕХо, а матричная функция S(t) = Е (Xt - Xt)(Xt - Xt)* G Жпхп подчиняется матричному дифференциальному уравнению Риккати ^ = FS + 5F* - S at
Гл. VIII. Стохастический интеграл. Стохастические дифференциальные уравнения 315 Пример 3. Применив теорему 19 к определенному ниже частному случаю ска- скалярных процессов, найдем Xf Пусть dXt=0, т.е. Xt = X0, EXo = 0, EX$ = v, dYt = Xt dt + m dBtj Yo = 0. Уравнение Риккати A27) для S(t) = E (Xt — XtJ приобретает вид откуда mz + vt Для X = {-Xt} из A26) находим, что dXt = ^~-Xt dt + V dYx mz + vt mz + vt Отсюда, используя формулу Ито для диффузионных процессов, находим, что d\ Xtexp<^ т* -\- vt ) mz следовательно, Xt = ^—Yt, t>0. mz + vt Теории фильтрации посвящена, например, книга [160]. В качестве введения в дан- данную область рекомендуем прочитать главу IV в [169]. Мы лишь укажем на еще одно важное обстоятельство, которому уделяется пристальное внимание последние десять лет. Если в упомянутой выше задаче фильтрации "шум" рассматривался как помеха, от которой желательно избавиться, то имеются задачи (относящиеся к анализу нелинейных систем), в которых шум играет полезную роль. Речь идет о том, что некоторые системы под воздействием определенного случайного шума могут приобретать черты устойчивого поведения. В последние годы большое внимание уделяется проблемам стохастической финан- финансовой математики. Здесь для моделей с непрерывным временем вместо разностных уравнений возни- возникают стохастические дифференциальные уравнения. При этом ценой значительных технических усложнений удается получить аналоги результатов, известных для дискретных моделей. Мы не приводим переформулировок "дискретных" определе- определений, данных в дополнении к главе IV, на случай непрерывного времени (см. [86]). Заметим, что широкую известность получили исследования в рамках модели Блэ- ка-Мертона-Шоулса, использующие для описания эволюции цен геометрическое броуновское движение. Работы М. Шоулса и Р. Мертона были отмечены в 1997 г. Нобелевской премией по экономике (Ф. Блэк скончался, а посмертно Нобелевские премии не присуждаются). Итак, если теперь (В, 5)-рынок описывается стохастическими дифференциальны- дифференциальными уравнениями dBt = rBt dt, dSt = St(fidt + adWt), A28) где В = (Bt)t^o — банковский счет, a S = (St)t^o — цены акций, то имеет место следующий замечательный результат.
316 А. В. Булинский, А. Н. Ширяев Теорема 20 (формула Блэка-Шоулса). В модели A28) с постоянными г, /i, a справедливая стоимость стандартного опциона-колл европейского типа с пла- платежной функцией /т = (St — К)+ определяется формулой C(T) = S0$(y+)-Ke-r$(y-), A29) где _ logE0/К) + Т(г± а2/2) У± ~ а Ф — функция распределения стандартной нормальной величины. В частнос- ти, при So = К и г = 0 имеем при этом С(Т) - Кау/Т/Bтт), когда Т -+ 0. Имеются разные подходы к доказательству этой теоремы (см., например, [86]). Отметим, что наряду с мартингальным подходом, опирающимся на теорему Гирса- нова, имеется подход (который и был использован Блэком и Шоулсом), состоящий в получении при определенных условиях для цены совершенного хеджирования Y(t,T) = C[t,T] = inf {ж >0 : 3 тг с Х^ = жиХ? = /т Р-п.н.} так называемого фундаментального уравнения 8Y 8Y 1 2q232Y +S + SY с краевым условием Y(T, S) = (S — К) +. При этом величина С[о т] = С(^) (см. A29)). Полное представление о проблемах стохастической финансовой математики мож- можно получить, ознакомившись с [86, 147, 155].
Приложение 1 Доказательство теоремы Колмогорова Доказательство этой теоремы разобьем на несколько шагов. А. От заданных мер Ptb...,tn перейдем к мерам Qj, индексированным множест- множествами J = {?]_,..., ?п}, положив = Ptu...,tn(Btlx---xBtn), A) где "прямоугольник" Bj = {yeSj:y(tk)eBtk, к = 1,...,п} и Btk eSStk, к = 1,...,п. B) В силу условия 1° (§11 главы I) это определение корректно, т. е. левая часть форму- формулы A) не зависит от порядка нумерации точек множества J (следует лишь не забы- забывать, какое именно множество Btk отвечает точке tj~, где к = 1,..., п). С полукольца введенных "прямоугольников" мера Qj однозначно продолжается по теореме Кара- теодори на порожденную этим полукольцом сг-алгебру, т. е. на ё% j. При этом соглас- согласно условиям 1° и 2° (§ 11 главы I) меры Qj, где J G F(T), удовлетворяют условию согласованности A.44). Таким образом, можем сразу считать, что на пространствах (S j, Sej) имеется се- семейство согласованных мер Qj, J G F{T). Покажем, что существует единственная мера Q на (St, ё%т) с данными проекциями Q j, J G F{T). Легко понять, что это, по сути дела, эквивалентная формулировка теоремы Колмогорова на языке мер. Дей- Действительно, если последнее утверждение будет установлено, то согласно лемме 8 главы I на вероятностном пространстве (St, ^>t-> Q) построим координатным спосо- способом случайную функцию X = {X(t), t G Т} (см. A.42)), для которой Рх = Q- Тогда Qj = Qtt^ j = Рх^т j при каждом J G F{T). Поэтому для любого у казаного выше "прямоугольника" Bj получаем Qj(Bj) = Px = P(X G Tr-^(Bj)) = p(f]{Xtk G Btk}\ 4=i ' Отсюда в силу A) и леммы 2 главы I следует, что X имеет конечномерные распреде- распределения Р* ь..., tn. В. На проблему построения упомянутой в пункте А меры Q удобно взглянуть еще с точки зрения продолжения меры, заданной на системе сг-подалгебр, на
318 А. В. Булинский, А. Н. Ширяев объемлющую их сг-алгебру. Для этого перенесем меру Qu с сг-алгебры S&u на сг-алгебру ?$т,и С ^>т-> где U Е F(T), определив Qu(Cu) = Qu(Cu), где Си и Си связаны соотношением вида A.36). Тогда легко видеть, что условие согласованности мер Qu, U Е F(T), равносильно тому, что сужение меры Qu с сг-алгебры ?$т,и на ё$т,v совпадает с мерой Qy для V С U. Итак, еще одна эквивалентная формулировка теоремы Колмогорова на языке мер состоит в том, что существует единственная мера Q на ё%т такая, что Q = Qu на сг-алгебре ?$т,и для каждого U Е F(T). Следовательно, нам не остается ничего другого, как задать функцию множеств Q на цилиндрической алгебре Чот формулой = Qc/(C), где Се%, U e F(T). C) Другими словами, положим Q(tt^1uB) := Qu(B) для В Е S%u,U ? F(T). Заметим, что определение C) корректно, так как если С G ?&т,и и С G С/, J G F(T), то С G ^t,uuj в силу A.37) и поэтому Очевидно также, что формула C) определяет конечно-аддитивную функцию мно- множеств на алгебре ^т, причем Q(St) = Qu($u) = 1 для любого U G F(T). Если мы теперь докажем счетную аддитивность Q на ^Тч т0 п0 теореме Ка- ратеодори мера Q однозначно продолжится до меры на о\Чот}-, т. е. до меры на ё%т (в силу теоремы б главы I). C. Как известно, на алгебре множеств счетная аддитивность Q равносильна ее конечной аддитивности вместе со свойством непрерывности (в "нуле" 0). Тем самым надо доказать лишь свойство непрерывности Q на Чот, т.е. что Q(Cn) —У 0, если Сп | 0 при п ->• оо f Cn G ^т, CVi+i С Cn, n G N, и П Сп = 0J. Отметим, что указанное свойство непрерывности меры требуется проверять на алгебре 4>т, а не на полукольце "прямоугольников", порождающем эту алгебру. Пусть Сп е $&т,зп, где Jn e F(T), n e N. Без ограничения общности можно сразу считать, что Jn С Jn+b n ^ N. Допустим, что Q{Cn) ^ го для некоторого го > 0 и всех достаточно больших п (тогда и для всех п, поскольку Cn+i С Сп, п G N). Убедимся, что это предположение противоречит условию Сп \. 0 (п —>- оо). Доказательство этого утверждения также разобьем на части. D. Пусть вначале St = [0,1], Set — ^([0,1]) и /?t = Р (евклидова метрика) при всех t G Т. Покажем, что^ можно иметь дело лишь с множествами Сп = Вп, где Вп — компакт в Sjn, 5n = тг^ j 5n, Jn G F(T), n G N. В силу леммы 11 главы I имеем ?$jn = ^(Sjn), а метрика pjn вводится на Sjn по формуле A.39). Для любого Вп G ?%jn по лемме 4 главы I находим в метрическом пространстве ($jn,pjn) замкнутое множество Кп С Вп такое, что QJn(Bn\Kn)<e02 -n-\
Приложение 1. Доказательство теоремы Колмогорова 319 Очевидно, пространство Sjn компактно. Поэтому Кп — компакт в Sjn. При каж- каждом п согласно последнему неравенству и C) Q(Bn \ Кп) = Q(Tr-iJn(Bn \ Кп)) = Qjn(Bn \ Кп) < ео2-п-\ П ^ Введем Ln = f] Hi, где Hi = Ki, i = 1,... ,n. Заметим, что Ln \, 0, поскольку г=1 Ln+i С LnnLn С Нп С Сп. При непрерывном отображении прообраз замкнутого п -1 множества замкнут. Поэтому множество Dn = f] ttj j.Ki (основание цилиндра Ln = tt^j Dn) есть компакт в S jn как пересечение замкнутых множеств с компак- компактом Кп = ttJ 1 j Кп. Учитывая, что Сп С Ci,i = l,...,n, получаем cn П Ш Q(Cn\Ln) = Q[Cnn[ lj Щ) ) =Q( U(Cn\m) ^О(Сг\Яг)< ' г=1 Следовательно, г0 ^ Q(Cn) = Q(bn)+Q(Cn\Ln) ^ Q(Ln)+s0/2,T.e. Q(Ln) ^ го/2 для всех n G N. Поэтому далее Cn = тт^ j Bn, где ??n — компакт в Sjn, Jn С Jn+b Ль G F(T), n G N. oo E. Для множества С/ = (J Jn определим в пространстве Sjj метрику 71=1 pjj(x,y) = 2_^2 1+ / г, D) где жп = х\ ,Уп=у\Т , n G N. Модифицируя доказательство леммы 11 главы I, видим, что метрическое прост- пространство (Sjjjpu) является польским, причем цилиндрическая сг-алгебра ё&и совпа- совпадает с борелевской сг-алгеброй 3e(Sjj)- Кроме того, Sjj — компактно. Множества будут компактами в метрическом пространстве (Su,pu) как замкнутые подмножест- подмножества компакта. Заметим, что Сп = tt^j Вп = ^j^u^u^ ^п = ^т^и^'п^ n ^ N. Поскольку ОО I ОО ч Сп С Сш при п ^ т, то С'п С С^ при п ^ т. Кроме того, f] Сп = тгт ^ ( f] C'n j. 71=1 ' 4l=l ^ Пересечение вложенных компактов в полном пространстве непусто, поэтому ОО ОО Р| С'п ф 0. Следовательно, Р| Сп ф 0 (тгт и отображает St на Sjj)- Пришли п=1 п=1 ОО к противоречию с предположением Р| Сп = 0. 71=1 F. Пусть теперь St G SS([0,1]), ^t = St П Й8([0,1]) и St ^ [0,1] хотя бы для одного t еТ. Положим Lt = [0,1], t G Г. Для J G F(T) введем на 38(Lj), где Lj = J] Lt, меру tGJ Б G 58(Lj). E)
320 А. В. Булинский, А. Н. Ширяев С учетом A.44) легко проверить, что введенные меры Q j (J E F(T)) являются согласованными. Пользуясь этим, а также рассмотренным случаем D, видим, что на пространстве (Ьт,^т), где 5?т = ® ^t и %t — ^([0,1]), существует мера Q ter с проекциями Qj на пространствах (Lj,3e(Lj)),J e F(T). Было бы заманчиво в качестве искомой меры Q взять сужение Q на множество St- Однако так действовать мы не можем, поскольку при несчетном Т множество St не входит в сг-алгебру J^V, на которой задана мера Q. Поэтому возьмем на вероятност- вероятностном пространстве (П,^, Р), где П = Ьт, 3* — !?т, P = Q, случайный процесс X(t,u) = u(t) (см. A.42)) с распределением Р^ = Р. Если St ф [0,1], то выберем точку at G St (St ф 0) и введем на Т х Q функцию (t,u;) e St, at npnX(t,cj) ^ St. По построению X = {X(t), t G Г} является случайным процессом на (П, ^, Р), причем X(t, ш) G St для всех t G Г и ее? G П [X(t, •) е 3 \Set ПРИ каждом t еТ). Очевидно, для каждого ? G Т {и: X(t,u) ф X(t,u)} = {и: u(t) G [0,1] \ St} = Ct. F) В силу E) для элементарного цилиндра Ct имеем P(Ct) = Q{Ct) = Q{t} ({[0,1] \ St} П St) = 0. G) Для прямоугольника Bj вида B), учитывая E) и F), получаем П (х^е Б*л) = р( П ^^)е вч) fe=i j vfe=i y = PjfTr^yBj) = QjEj) = Qj(Bj). Следовательно, Q = Px есть искомая мера на сг-алгебре ё%т — ® $&t- teT G. Объясним, какие изменения следует внести в данное доказательство, чтобы охватить любые борелевские пространства (St,?$t)teT- Лемма 1. Пусть (St,$t)teT u (Tt,?#t)teT — такие семейства измеримых пространств, что (St,^t) ~ (Г*?«^) пРи каждом t G Т. Тогда (Sjj^u) ~ ~ {Yjj^u) для любого U С Т, гдеё%и и sdjj — соответственно цилиндрические а-алгебры подмножеств Su и Гц ("~" обозначает изоморфизм измеримых пространств).
Приложение 1. Доказательство теоремы Колмогорова 321 Доказательство. Пусть отображение ht'.St —> 1\ задает изоморфизм прост- пространств (St, S%t) и (Tt, sit) при t G Т. Для U С Т введем отображение Ни: St/ —>¦ Г с/, положив дляхи = (ж(?), ? G С/) G St/ Ьихи=Уи1 гДе Уи = (у(*)^?и), y(t) = ht(x(t)) еГи t e U. Тогда прообраз элементарного цилиндра в Г и есть элементарный цилиндр в S[/, поэтому hjj G Й§с/ | sijj в силу следствия 1 главы I. Очевидно, Нц есть взаимно-однозначное отображение Su на IV. Те же рассуждения показывают, что 1 . ? Если выполнены условия леммы 1 и на пространствах (St/, Й§с/) заданы согласо- согласованные меры Qu (U G F(T)), то легко проверить, что на пространствах (IV, sijj) возникают согласованные меры Рц = Qjjh^}1 (иначе говоря, Ру — Pjj^irv для V С U С Г (U G F(T)), где "проектирование" П[/5у для семейства пространств имеет тот же смысл, что и отображение к-ц у для пространств Допустим теперь, что на пространстве (Гт, ^т) удалось построить меру Р, имею- имеющую заданные проекции Рц на пространствах (IV, sijj) для U G F(T). Тогда легко убедиться, что на (St, S&t) мера Q = PhT (т.е. Q = P(/i^1)~1) будет иметь проек- проекции Qu на (St/, ^c/), U e F(T). Таким образом, без потери общности можно считать, что St — борелевское под- подмножество отрезка [0,1]и^ = St П Й§([0,1]) для каждого t G Т. Тем самым прихо- приходим к рассмотренным выше случаям Е и F. Итак, теорема Колмогорова полностью доказана. ? Следующий результат поясняет, почему в определении 8 главы I рассматривается борелевское подмножество отрезка [0,1], а не сам этот отрезок. Для этого возьмем двуточечное множество S = {0,1} и превратим его в польское пространство с дис- дискретной метрикой v (j/(x,y) = 0 при ж = yniy(x,y) = 1 при ж ф у). Положим К = SN и введем на К метрику аналогично D). Теорема 1 (см., например, [15; с. 98]). Пусть измеримое пространство (S, Й§) изоморфно борелевскому подмножеству некоторого польского пространства (с о-алгеброй его борелевских подмножеств). Тогда { если S несчетно, (N,?/(N)), если S счетно, (A,..., |S|), «с/A,..., |S|)), если S конечно, здесь si(M) — а-алгебра всех подмножеств множества М, | • | — число эле- элементов конечного множества. Сделаем еще несколько замечаний, относящихся к теореме Колмогорова. Доказа- Доказательство этой теоремы для семейств польских пространств (St, ?%t)teT, основанное на важных результатах функционального анализа (теоремы Стоуна-Вейерштрасса и Рисса-Маркова), можно прочитать в [101]. Ключевая идея здесь состоит в том, что меры порождают линейные функционалы, которые удается должным образом про- продолжить. Кроме того, используется теорема б главы I, позволяющая ограничиться
322 А. В. Булинский, А. Н. Ширяев построением согласованных мер Qc/, индексированных семейством счетных подмно- подмножеств [/сГ, что показывает следующее упражнение. 1. Пусть Qu — меры на пространствах (S[/, 8&и\ заданные для конечных или счетных множеств U С Т (для конечных U меры Qu задавались изначаль- изначально). Пусть условие согласованности A.43) выполняется для всех конечных или счетных V,U С Т (V С U С Т). По теореме б главы I для каждого В е ё%т найдем множество U = U(B) G N(T) такое, что В е 88т,и (т«е. В = тг^ jjBjj , где В и G 8$ и) и определим функцию Q, положив Q(B) = Qu(Bu)- (8) Докажите, что (8) корректным образом задает меру на сг-алгебре 8%т- В заключение этого раздела упомянем также известную теорему Ионеску Тул- ча (см., например, [85; т. 1, с. 318]), позволяющую строить последовательность случайных элементов, имеющих заданные совместные распределения (определяе- (определяемые по семейству вероятностных ядер) в произведениях произвольных измеримых пространств.
Приложение 2 Доказательство теоремы Прохорова А. Пусть {Ра}аеА — плотное семейство мер на 3e(S), где S — метрическое про- пространство. Докажем, что это семейство слабо относительно компактно. Если S — компактное пространство (компакт), то C(S) = Cb(S) и слабая сходи- сходимость вероятностных мер эквивалентна сходимости в слабой *-топологии на сопря- сопряженном пространстве C*(S). При этом совокупность 3^(S) вероятностных мер на 3$(S) является слабо *-замкнутым подмножеством единичного шара пространства C*(S), который, как известно, есть компакт в слабой *-топологии. Следовательно, 3^(S) также является компактом в слабой *-топологии как пересечение замкнутого множества с компактом. Это завершает доказательство первого утверждения тео- теоремы б главы V для компакта S. Теперь поясним сказанное подробнее. По теореме Рисса-Маркова (см. [60; т. 1, с. 124]) каждый неперывный линейный функционал F на C(S), т.е. F Е C*(S), однозначно представляется в виде здесь Q — заряд на Зё (S), т. е. Q = Q+ — Q ~, где Q+ и Q ~ — конечные меры на Зё (S), а (/, Q) = (/, Q+) — (/, Q~). Функционал F идентифицируется с зарядом Q; кроме того, = {Qe C"(S): (/,Q) ^ Одля/ ^ О, / e C(S)h(T,Q> = l}, (l) где 1 — функция, равная единице на множестве S, иначе говоря, 1 = 1$. Для F G G С* (S) имеем ||F||c,(s):=sup{|F(/)|:/GC(S), sup |/(Ж)К l}. B) Очевидно, для Q G 0>(S) и / G C(S) |()||()|()|()| xes xes поэтому 3^(S) — подмножество единичного шара в С* (S), т. е. шара
324 А. В. Булинский, А. Н. Ширяев Проверим *-замкнутость 5^(S). Допустим, что Qa слабо *-сходится к некоторому элементу Q в пространстве C*(S), т.е. </,Q«> -»¦ (/,Q> для любого / е C(S), C) здесь индекс а пробегает некоторое направленное множество. Но тогда Q обладает свойствами, описывающими в A) меры из 3^(S). Подчеркнем, что шар В берется по норме B), по его компактность (теорема Банаха—Алаоглу, см., например, [60; т. 1, с. 133]) устанавливается не по этой норме, а в смысле слабой * -топологии. Итак, утверждение А детально доказано для компактных пространств S. Общий случай сводится к уже рассмотренному. Пусть S есть сг-компактное про- оо странство, т. е. S = (J Sm, где Sm — компакт (m Е N). Тогда S сепарабельно и, т=1 следовательно, гомеоморфно борелевскому подмножеству М компактного простран- пространства V = [0,1]N (см., например, [101; с. 2]). В силу теоремы 2 главы V слабая схо- сходимость мер сохраняется под действием непрерывного отображения. Это же, оче- очевидно, относится и к свойству плотности. Поэтому можно сразу считать меры Ра (a Е А) заданными на (S, 5§(S)) = (М, Зё(М)) и продолженными до плотного семей- семейства мер Qa на (V, 38(V)) по формуле Qa(i):=Pa(inM), AeSS(V), a е А. D) При этом Qa = Pa на 3e(S) С Зё(V). По доказанному, из произвольной последова- последовательности мер Qn (пишем Qn вместо QanM входящих в семейство {Qa}aeA5 можно извлечь подпоследовательность Qn. такую, что Qnj => Q При j -> 00, где Q — некоторая (вероятностная) мера на (У, 3e(V)). Покажем, что Рп. => Р при j —у оо (пишем Рп вместо РаггM где Р = Q|^s., т.е. сужение меры Q на ?$(S). Для каждого т G N найдем в S компакт i^m такой, что Pa(ifm) > 1 — 1/т при всех a G А. E) Тогда по теореме 1 главы V, учитывая, что 3e(S) С 3e(V), имеем для произвольного т G N Q(S) ^ Q(Km) > HmsupQnj(Km) = limsup Рп. (К j j ш) j Следовательно, Q(S) = 1, значит, P(S) = 1. Кроме того, для любого замкнутого множества F С S, снова используя теорему 1 главы V, получаем, что limsup Pn.(F) = limsup Qn.(F) ^ Q(F) = P(F), j j откуда вытекает, что Pnj => P на (S, 3&{S)) при j —> oo.
Приложение 2. Доказательство теоремы Прохорова 325 оо Рассмотрим произвольное метрическое пространство S и So = U Кш Е 5§(S), где Кш фигурирует в E). Тогда Pa(So) = 1 для любого a Е Л. Таким образом, {Ра, a Е Л} есть плотное семейство мер на сг-компактномпростран- сг-компактномпространстве (So, ?$(So)), где Ра обозначает сужение меры Ра на 5§(So). По доказанному из любой последовательности Рп (т.е. Рап) мер, входящих в семейство {Ра}аеА, мож- можно выбрать подпоследовательность Р такую, что Рп. => Р при j —У оо, здесь Р — некоторая (вероятностная) мера на (So, ^S(So)). Введем меру Р на (S, 5S(S)) аналогично D), положив Р(А) :=P(AnS0), AeSS(S). Заметим, что P(S \ So) = 0 и Pa(S \ So) = 0, a Е Л. Следовательно, для любой функции / Е Cb(S) и всех а Е Л имеем где/|5 G Cb(So). Таким образом, Pnj ^> Р при j ->• оо. Искомое утверждение А доказано. В. Докажем теперь, что если S — польское пространство с метрикой р, то слабо относительно компактное семейство мер {Ра}аеЛ является плотным. Из доказательства леммы б главы V видно, что если для любых г, S > О можно указать конечный набор открытых шаров Vs(y<m) = {ж G S: р(х,ут) < 5}, т = 1,..., N (N и у\,..., yjsf зависят от г, S) такой, что ( \ Ра ( U Vsiym) ) > 1 - ? при всех а е Л, F) то семейство {Ра}аел плотно. Покажем, что если условие F) не будет справедливо, то это будет противоречить слабой относительной компактности {Ра}аеЛ- В силу сепарабельности S для каж- каждого S > 0 можно найти покрытие этого пространства шарами вида ViB/m), где ут G S, m G N. Невыполнение F) влечет существование e,S > 0 таких, что для любого n G N найдется мера Рп (т. е. мера ParJ, для которой п Рп(С„)^1-е, где Сп= [j V5{ym). G) т=1 Предположим, что выбрана подпоследовательность \п'} С N такая, что Рп/ => Р при п1 —у оо для некоторой меры Р на (S, S$(S)). Множества Cj открыты, поэтому согласно теореме 1 главы V P(Cj) ^ liminf Pn'(Cj) для каждого j e N.
326 А. В. Булинский, А. Н. Ширяев Заметив, что Cj С Спг при п' ^ j, учитывая G), получаем п' —юо что противоречит тому, что lim Р(С7) = P(S) = 1. Таким образом, утверждение В j->oo доказано. П Замечание 1. Первоначально теорема Прохорова была получена в форме не- необходимых и достаточных условий относительной компактности семейства мер в польских пространствах. Достаточность (т.е. плотность семейства мер обеспечи- обеспечивает его относительную компактность) была установлена для любых метрических пространств Варадарайном. Доказательство этого результата, не опирающееся на теорему Рисса-Маркова (как это делалось выше более подробно, чем в [11]), а ис- использующее редукцию к сг-компактному пространству, затем к пространству М°°, а после этого — к1^ (с применением теоремы Хелли и теоремы Колмогорова о согласованных распределениях) можно прочитать в [2] на с. 58-62.
Приложение 3 Доказательства теорем Линдеберга и Дуба В этом разделе приводятся доказательства многомерной теоремы Линдеберга (теорема 9 главы V) и теоремы Дуба (теорема 13 главы V). Кроме того, даются вспомогательные результаты и упражнения, относящиеся к слабой сходимости случайных векторов со значениями в ^-мерном евклидовом пространстве. § 1. Доказательство теоремы Линдеберга А. Матрица В2 в условии (V.17) будет симметрична и неотрицательно опреде- определена как предел матриц с этими свойствами. Следовательно, можно рассмотреть распределение N@, В2). В силу теоремы 13 главы V достаточно проверить, что при каждом Л Е Шк VsnW ^ехр{-(Б2А,А)/2}, п^оо, A) здесь и далее (•, •) ~ скалярное произведение в Жк , порождающее евклидову норму II ' II•> Vs — характеристическая функция случайного вектора Sn. Независимость ?n,g5 Q = 1? • • • •> тт влечет, что при любых п Е N и Л Е Шк q=l Поэтому вначале обратимся к анализу поведения характеристических функций век- векторов ^п^д. В. Воспользуемся известным неравенством | ехр{ш} - 1 - га - ... - (шO1/^ - 1)!| ^ \а\п/п\, а е М, п G N. B) Применив B) при п = 2 и п = 3, получаем | ехр{ш} - 1 - га + а2/2| ^ min{|a а Отсюда, поскольку |(Л, х}\ ^ ||А|| ||ж|| для Л, х G Шк , находим ехр{г(Л,х)} = 1 + i(\,x) - (Л,хJ/2 + ^((Л,х)), C) где |a;||2,||a;||3} и С(А) = тах{||А||2, ||А||3}. D)
328 А. В. Булинский, А. Н. Ширяев Из C) при всех п е N, q = 1,..., mn, \ е Жк имеем <^(A) = Е exp{i<A,?n,g» = 1 - \{В2п^\\) + Ед((Х^щд)) = 1 + sn,g(A); E) мы воспользовались тем, что Eg((A, ^n,q)) существует в силу D), и учли, что Зафиксируем A Е Жк и покажем, что |zn5g(A)| < 1/2 при всех достаточно больших п и любых q = 1,..., тп. Пусть \\A\\ для матрицы А = (arj)r j=i обозначает ее норму как линейного опе- оператора, т.е. ||А|| = sup \\Ax\\. Для ковариационной матрицы А А, где с = с(к) > О, Тг — это след матрицы, т. е. сумма ее диагональных элементов. Поэтому КВ^Л,Л)К||В^||||Л||2^сТгБ2;д||Л||2. F) Далее, при п G N, q = 1,..., тп и произвольном г > О Tr Blq = E ||?n>g||2 = E ||^,g||2l{||en,g|| < е}+Е ||?п,д|2 2 где функция Линдеберга q=l Следовательно, учитывая условие Линдеберга (V.18), видим, что выполнено усло- услоmax ТгБ2 -)>() при п -»> оо. G) ^q^m Из D)-G) вытекает, что Согласно G) получаем max |^n,g(A)| —У 0 при п —у оо. (8) С. Для idgC, w ф 0, как обычно, полагаем log w = log |г^| + iargw, здесь О ^ argw; < 2тт. Известно, что для z G С 2) = 2 + ftB), где |/i(z)| ^ |z|2 при |z| < 1/2. (9)
Приложение 3. Доказательства теорем Линдеберга и Дуба 329 Поэтому для любого п и некоторого JV(A, n) Е Ъ тп logy>5n(A) = X>S^WA) + 2тггЛГ(А,п). A0) В силу (8)—A0) при всех достаточно больших п имеем тп тп J> ^ 2тггЛГ(А,п). A1) Теперь заметим, что в силу E) тп тп Е *яМ = - о <ВпА, А) + Е Е^(А' ^», A2) 1 1 поскольку 5^ = ^2 Щь^- Учитывая D), находим, что для для всех 0 < г < 1 q=l ^ С(А)(Е ||^,д||31{||^,д|| ^ S} + Е ||^,д||21{||^,д|| > в}). A3) При этом, очевидно, EMn,q\\3l{Mn,q\\^e}^eEUn,q\\2 = eTiBlq. A4) Из A3), A4), условия Линдеберга (V.18), а также соотношения тп J2 ЪВ1д = ТтВ2п^ ТтВ2 при п -+ оо q=l получаем, что гпп ^U)hO, n^oo. A5) Поскольку (В2п\\) ~+ (В2\,\) при п^оо, A6) согласно A2), A5) имеем У^^ з(А) ->• -~{В2\ А) при n ^ оо. A7) Заметим, что для всех достаточно больших п q=l q=l ^шж \{В11д\,\)\{В%\,\)+2Г?1Е\д((\,?п,<1))\\ ^0 при п -»• оо,
330 А. В. Булинский, А. Н. Ширяев где первое неравенство цепочки следует из (9), второе — элементарно вытекает из определения zn^q{\) в E), а стремление к нулю последней оценки обеспечивают со- соотношения F), G), A6)иA5). Из A1), A6) заключаем, что W i/2<B2A,A>5 n ^ оо. П § 2. Обсудим условия теоремы Линдеберга. Условие (V. 17) фактически содержит в себе нормировку исходных слагаемых. На- Например, если В^ > 0, начиная с некоторого п (при к = 1 это тривиальное тре- требование, означающее, что в n-й серии не все слагаемые вырождены), то, полагая ?nq = В~г{;п^д (из В2^ извлекается квадратный корень и берется обратная матри- ца), для Zn = J2 Cn,q находим, что q=l DZn = DiB-'Sn) = B-'DSniB-1)* = I. Иначе говоря, условие (V.I7) становится выполненным для величин Cn,q- 1. Пусть для любого е > О тп а~2 ^ Е (||^n,g||2l(||^n,g|| > е<гп)) ^0 при п -> оо, A8) q=l где а^ = Тг 5^ > 0, а векторы ?п^д центрированы. Докажите, что тогда для введенных выше величин (П:Я выполняется условие Линдеберга (V.18). Заметим, что при к = 1 теорема Линдеберга обычно формулируется с един- единственным условием A8), которое обеспечивает сходимость нормированных сумм Zn к N@,1). 2. Проверьте, что если {Xj, j G N} — последовательность независимых одина- одинаково распределенных случайных векторов в Жк со средним 0 и ковариацион- ковариационной матрицей В2, то при 3. Объясните, почему условие Ляпунова: для некоторого s G B,3] при п -»> оо — влечет условие Линдеберга.
Приложение 3. Доказательства теорем Линдеберга и Дуба 331 Представляет интерес следующий результат. Лемма 1. Пусть для серий независимых {при каждом п) центрированных случайных векторов ?п,д5 1 ^ Q ^ mm n Е N, выполнено условие Линдебер- Линдеберга (V.18). Если тп Sn'-=J2^q^Y, A9) q=l где Y — некоторый случайный вектор, то верно (V.17) и при этом Y ~ Доказательство. Допустим, что последовательность {Ьп , п ^ 1} не имеет предела для некоторого г Е {1, ...,&}, где Ьп — это элемент матрицы ковариа- ций В^, стоящий в r-й строке и j-m столбце. Если последовательность {Ьп } не~ ограничена, то Ь^/ —У оо для некоторой подпоследовательности {п'}. По теореме 2 главы V слабая сходимость векторов влечет слабую сходимость под- векторов (составленных из фиксированной части компонент исходных векторов). По- Поэтому S{nr) А Г(г) при п -^ оо, B0) где верхний индекс г обозначает r-ю компоненту векторов. Мы можем также утверждать, что ^ rwoo, B1) поскольку для центрированных величин ?п нено условие Линдеберга: действительно, при всех достаточно больших п' величины п'Г ^ 1 и тогда ПРИ любом г > О (ll^,gl|{||^,gll > S}) ^0, П7 ^ 00. q=l Воспользуемся (при к = 1) следующим простым фактом. 4. Если ?п —> ( в Mr w числовая последовательность {сп}п^^ такова, что сп ->• 0, то спСп —^ 0 (п ^ оо). Поскольку l/ л/6^J'Г —>- 0, п' —у оо, то видим, что одновременное выполнение B0) и B1) невозможно. Пришли к противоречию, следовательно, последовательность {Ьп 'г , п ^ 1} не может быть неограниченной. Если последовательность {Ьп } ограничена, но не имеет предела, то найдутся подпоследовательности {nf} и {п"} такие, что Ь^'г —> V и Ь^, —>¦ Ь7/, причем
332 А. В. Булинский, А. Н. Ширяев V ф Ъ". Тогда получим, что S^) А N@, V) и S$ A N@, Ъ") в силу теоремы 9 главы V. Однако это невозможно в силу единственности слабого предела (лемма 1 главы V). Таким образом, доказано, что последовательности {Ьп }n^i имеют пРе" делы при каждом г = 1,..., к. По неравенству Коши-Буняковского-Шварца |Ьп | ^ у Ъп'г Ъп3 , поэтому про- проведенные рассуждения показывают, что {Ьп }neN ограничена и при любых г ф j (г, j Е {1,..., к}). Рассмотрев двумерный вектор (Sn , Sj? )? снова придем к проти- противоречию с леммой 1 главы V, если допустим, что не существует предел у о? при п —у оо для каких-либо г ф j (векторы, составленные из части компонент исходных векторов ^n^q также удовлетворяют условию Линдеберга). Таким образом, условия теоремы влекут соотношение (V.17), а это обеспечивает в силу теоремы 8 главы V, что Y ~ N@, В2). ? 5. Постройте пример (при к = 1), показывающий, что если для серий незави- независимых центрированных векторов потребовать выполнения центральной пре- предельной теоремы, т.е. соотношения A9) с Y ~ N@, В2), но без условия Лин- Линдеберга, то соотношение (V.17) не обязано быть справедливым. § 3. Следующий результат объясняет, в каком смысле условие Линдеберга явля- является необходимым для справедливости центральной предельной теоремы. Теорема 1 (Феллер). Пусть ^n,qj I ^ Q ^ шп, п G N, - серии независимых (в п-и серии) центрированных случайных векторов в Жк, удовлетворяющих ус- условию (V.17). Если справедлива центральная предельная теорема, т. е. име- имеет место соотношение (V.19), и при этом удовлетворяется условие G), то выполнено условие Линдеберга (V.18). Таким образом, для упомянутых се- серий условие Линдеберга равносильно одновременному выполнению централь- центральной предельной теоремы и условия "асимптотической малости" слагаемых в форме G). Доказательство теоремы можно провести, обобщая рассуждение, приведенное в [85; т. 1, с. 427] для одномерного случая (к = 1). При этом ключевую роль будет играть следующее упражнение. 6. Пусть ? — центрированный случайный вектор вЕ^с ковариационной матри- матрицей С. Тогда для любого г > 0 справедливо неравенство Е||?Ц21A1?11 > е) ^ ?2(Re^(A) - 1 - <СА,А>/2), где (р — характеристическая функция ? и Л = Л (г) = @,..., у&к/е,..., 0) G Glfe,a единственная ненулевая компонента Л имеет номер jo = jo(s) такой, что &г)= max Еф(||?|| > е). § 4. Доказательство теоремы Дуба (теорема 14 главы V). Нам потребуется несколько вспомогательных утверждений. Введем еще одно условие "бесконечной малости" случайных векторов ^n^q (I ^ q ^ mn, n G N) со значениями в Жк: max ||?n,g|| —> 0? n ^ оо. B2)
Приложение 3. Доказательства теорем Линдеберга и Дуба 333 Лемма 2. Для серий независимых {при каждом п) случайных векторов ^n^q условие B2) равносильно тому, что >я11 > ^) ^ 0 при каждом е > О, когда п ->• оо. B3) q=l Доказательство. Для любого г > 0 и всех n G N имеем max P(||fn,g||>e)^P( max ||fn,g|| > eU X) p(H&mzII > e). B4) Следовательно, B3) влечет B2). В силу независимости ?П)д, g = 1,..., ?тгп, Р( max ||^n,g|| > И = 1 - Д A - Р(||^,,|| > е)). Теперь заметим, что для ап q G М, 1 ^ q ^ mn, n G N, если max |an g| —у 0, то п,д^0 (п-^оо). q=l q=l Поэтому из B2) вытекает B3). ? Лемма 3. Пусть для серий независимых векторов ^n^q выполнено условие B2) и пусть для некоторой константы с > О II&mzKc п.н., gf = l,...,mn, n G N. B5) Тогда серии центрированных векторов r\n,q — ?,n,q — E^n?g, 1 ^ q ^ mn, n G N, удовлетворяют условию Линдеберга (V.17). Доказательство. Для любого v > 0 с учетом B5) имеем max ||Е?те|д|| ^ г/+ с max Отсюда max ||Е?П)д|| —У 0 при п —У оо. B6) По лемме 2, а также в силу B5) и B6), при каждом г > 0 и всех достаточно больших п получаем J2 Е (||7?n,g||2l{||7?n,g|| > ?}) SS BcJ ^ P(||^,g|| > e/2) ^0 (n -»¦ 00). ? B7) g=l g=l
334 А. В. Булинский, А. Н. Ширяев Лемма 4. Пусть серии независимых {при каждом п) случайных векторов ^n:q со значениями вЖк A ^ q ^ mn, n Е N) удовлетворяют условию B2). Если имеет место соотношение A9), то Y ~ Ы(а,В2) для некоторого вектора а и некоторой матрицы В2. Доказательство. Положим^nq = ?n,gl{||?n,g|| ^ с}, где с > 0. Из условия B2) вытекает, что для каждого с > 0 тп ^2 д|| > С} -А 0, П -^ ОО. Поэтому достаточно лишь проверить, что ^2 ?n,q —^ Ы(а,В2) при п —У оо, как g=l показывает (при S = Ж и ^/ = 0) следующее упражнение. 7. Пусть ?, CrnVni ^GN, — случайные элементы на (П, «^ Р) со значениями в се- парабельном метрическом векторном пространстве (S, ^). Пусть (п —у ( и р Р р ?7п —> У при п —У оо, где точка у G S (?7п —^ 2/ означает, что р(г]п,у) —у О при п —у оо). Тогда (п + ?7п —^ С + У-> п ~^ °°- Таким образом, опуская черту над ?,n,q, можем сразу считать, что исходные век- векторы удовлетворяют условию B5) при некотором с > 0. Воспользуемся очень полезным методом симметризации, а именно: рассмот- рассмотрим симметризованные векторы ^n^q = ^'nq — ?<?, где причем ?/п q, ^ g, I ^ q ^ ?тгп, - независимы (n G N). Тогда Sn —У Y при п —У оо, где Sn,Y - симметризованные векторы Sn и Y. При этом для любого г > О Отсюда в силу леммы 2 видим, что ?,n,q также удовлетворяют условию бесконечной малости: max ||?n,g|| —У О ПРИ п —У °°- Поэтому по лемме 3 (с учетом того, что |\?n,q 11 ^ 2с п. н.) для них также справедливо условие Линдеберга: Е ||^n,g||2l{||^n,g|| > в} —>- 0 при каждом г > 0, когда п —У оо. q=l Поскольку векторы ?n Q центрированы, то по лемме 1 существует lim D5n, где D5n — ковариационная матрица Sn. Заметим, что DSn = 2D5n. Следовательно, существует lim DSn = В2. п—юо Тогда по лемме 3 и теореме 9 главы V имеем Sn — Е Sn —У N@,5) при п —У оо.
Приложение 3. Доказательства теорем Линдеберга и Дуба 335 Теперь применим следующий результат. 8. Пусть (п —у (w(n+an —у г), где (nXiV — случайные векторы в Mr (n Е N). Тогда существует lim ап = а (и в силу упражнения 7 тогда ( + а = 77). п—>-оо Таким образом существует lim E Sn, что завершает доказательство леммы. ? п Теперь нетрудно доказать теорему Дуба. Пусть 0 ^ s < t < 00. Разделим отре- п зок [s, ?] точками ?П)д. = s + k(t — s)/n, к = 0,..., п. Очевидно, Xt — Xs = J^ ?n,fc> fc=i где^^ = X(tnjk) — X(tnjk-i) • Непрерывность траекторий влечет (п.н.) равномер- равномерную непрерывность X на [s, t]. Поэтому max ||^n,g|| ->• 0 п.н. при п ->• оо. B8) Кроме того, в силу независимости приращений X видим, что ^n,g5 1 ^ Я. ^ п-> неза- независимы при каждом п. Теперь заметим, что для всех п Е N / ™ \ Law У ?п а = Law(Xt — Xs). \^[ ' / Поэтому с учетом B8) нормальность вектора Xt — Xs вытекает из леммы 4. Отсю- Отсюда и из установленной независимости приращений процесса X следует гауссовость процесса Y = {Ft = Xt — Хо, t ^ 0}. Для завершения доказательства теоремы Дуба положим Mt = E(Xt-X0) и Gt = D(Xt-X0) (все моменты существуют, так как Xt — Х$ — гауссовский вектор). Тогда, очевидно, для 0 ^ s < t < оо имеем Е (Xt — Xs) = Mt — Ms, а в силу независимости прира- приращений процесса X получаем, что D(Xt — Xs) = Gt — Gs. Теперь нам понадобится упражнение 9. Пусть (п —у ( при п -У оо, где Сп, С — случайные векторы в Rk и (п ~ ~ N(an,Gn). Докажите, что тогда существует lim an = а и lim Cn = С п—юо п—^оо (поэлементно), причем ( ~ N(a, G). Отсюда легко вытекает, что функции М: М+ —У Ж171 и G: IR+ —>¦ Mfc являются непрерывными. Теорема Дуба доказана. ? Следствие 1. Если в условиях теоремы 14 главы V величина Xq имеет вырожденное распределение, то и сам процесс X = {Xt, t ^ 0} будет га- уссовским. Если процесс X удовлетворяет условиям теоремы 14 главы V для t G [a,b], то ее утверждение выполняется для процесса {Yt — Xt — Ха, te[a,b]}. Замечание 1. В книге [50] для независимых действительных случайных величин ^n^q (q = 1,... ,тп) исследована слабая сходимость распределений сумм тп Sn = ^2 ?n,q ПРИ п —> оо, когда слагаемые удовлетворяют менее жесткому, чем B2), q=l условию "равномерной малости слагаемых" max P(|?n,g| > ?) ->• 0 для каждого г > 0, когда п -у оо. B9)
336 А. В. Булинский, А. Н. Ширяев Так, при условии B9) класс возможных предельных распределений Sn совпада- совпадает с классом безгранично делимых законов (включающим нормальное распределе- распределение). В [50; гл. 4] можно найти необходимые и достаточные условия сходимости Sn к заданному безгранично делимому закону. Изучение сумм независимых случайных величин без каких-либо условий бесконечной малости слагаемых проводится в моно- монографии [27]. В [157] исследуется поведение сумм случайных элементов со значениями в банаховом пространстве.
Приложение 4 Доказательство теоремы Бохнера—Хинчина Если R@) = 0, то R(t) = 0 для t Е Ж и (VII.36) выполняется при G = 0. Поэтому пусть далее Д@) т^ 0. В силу следствия 3 главы VII функция R = Д(?) является непрерывной на Ж. А. Предположим вначале, что R Е L1(M) = L1(M, ^(M), mes), где mes — мера Лебега. Докажем, что тогда / R(t) dt ^ 0. A) J — оо В силу непрерывности и неотрицательной определенности функции R получаем, что при всех N > 0 > 0 B) (здесь учтено, что при составлении интегральных сумм можно выбирать разбиение квадрата [—JV, TV]2 на одинаковые квадраты со стороной TV/п и брать любые проме- промежуточные точки ц? G [kN/n, (к + lOV/n], к = —п,..., п — 1). ( ^ d(t,s) 1\ Совершив замену переменных t — s = u,t + s = v якобиан — = - , имеем V d(u,v) 2) 1 rN rN — I / R(t-s)dtds = i r />2iV p2N-u rO ru+2N \ = — / duR(u) dv+ duR(u) dv\ = 4^ I ^0 Ju-2N J-2N J-U-2N J /°° / u\\ f°° ( 1 - 2^1 l{-2N,2N}(u)R(u) du^ I R(u) du C) при N —У оо. Законность предельного перехода в C) обеспечивает теорема Лебега о мажорируемой сходимости (R G L1(M)). Таким образом, B) и C) влекут A). В. Положим 1 Г°° f(x) = — / e~ltxR(t) dt, хеЖ. D) 27Г J-oo
338 А. В. Булинский, А. Н. Ширяев Поскольку R Е L1(M), видим, что / - ограниченная непрерывная функция на Ж. По теореме 4 главы II неотрицательно определенную функцию R = R(t), где t G 1, можно считать ковариационной функцией некоторого центрированного комплексно- значного процесса X = {X(t), t G 1}. Заметим, что для любого х Е М, если R(t) = = ЕХ(Ь)Х@),то Еe-isxX(s)e-itxX(t) = e'^^-^R^ - t). Поэтому e~ltxR{t) — ковариационная функция центрированного стационарного про- процесса e~itxX(t) (напомним, что EX(t) = 0), причем \e~itxR(t)\ = |Д(*)|. Следова- Следовательно, по доказанному f(x) ^ 0 для любого xGl. C. Убедимся, что |Д(*)| ^ Д@) для всех t G М. E) Очевидно, Д@) = Е |Х@)|2 ^ 0. Кроме того, по неравенству Коши-Буняковского- Шварца |Д(*)| ^ \\X(t)\\ \\X@)\\ (здесь || • || — норма в L2(ft, ^, P)), но ||Х(?)||2 = = EX(i)X(i) = Д@), t e R. Поэтому E) справедливо. D. Рассмотрим в Ь2(Ж) = L2(M, Й§(М), mes) преобразование Фурье F[h](t)= / eitxh{x) dx. J — OO Известно (см., например, [60; т. 2, с. 20]), что F взаимно-однозначно отображает Ь2(Ж) на 1/2(М), причем для п. в. х (по мере Лебега) h(x) = — / e-itxF[h](t)dt и для h, g G Ь2(Ж) выполнено равенство Парсеваля ,g) = (F[h],F\g]), F) где \ •, • / — скалярное произведение в . Функция R G Z/2(M), так как R G L1(M) и выполнено E). Следовательно, гоо R(t) = eltxf(x)dx п. в. по мере Лебега. G) J — оо Е. Покажем, что последнее равенство верно, на самом деле, при всех t G ffiL Для этого проверим, что / G L1(M). Пользуясь тем, что характеристическая функция величины ? ~ N@, г2), г > 0, известна (см. (П.7) при п = 1), согласно F) имеем _^_ Г00 (ежJ Г00 е 2е2 / е 2—f(x)dx= / ——R(t)dt. (8) J-oo ./-оо вл/27Г
Приложение 4. Доказательство теоремы Бохнера-Хинчина 339 В силу непрерывности функции R правая часть (8) стремится к Д@) при г —> О, поскольку е 2е2 /(sv/2tt ) представляет собой ^-образное семейство функций (аргу- (аргумента t). По теореме о монотонной сходимости получаем, что Г. f(x)dx = R@). (9) Функция R непрерывна на всей оси, а правая часть формулы G) также непрерывна на М, поскольку / Е L1(M). Следовательно, равенство G) справедливо при каждом t G Ж. F. Пусть теперь R необязательно принадлежит L1 (Ж). Рассмотрим в L1 (Ж) функ- функцию R?{t) = R{t)e 2~, где параметр г > 0. Это непрерывная неотрицательно определенная функция, поскольку R?(t) = Eelt?^R(t), где ? ~ N@,1) (а именно, приняли во внимание, что при каждых е и и функция elt?^R(t) неотрицательно опре- определена). Тогда по доказанному R?(t)/R@) - характеристическая функция некото- некоторого вероятностного распределения, т. е. Re(t)/R@)= eitxf?(x)dx, j f?(x)dx = l, fe(xJ 0. J — oo J — oo Но если характеристические функции сходятся поточечно к функции, непрерывной в нуле, то предельная функция является характеристической (см. теорему 12 гла- главы V). Итак, R(i)/R@) — характеристическая функция, т.е. R(t)/R@) = / eitxdF(x), J — oo где F — некоторая функция распределения, и представление (VII.36) установлено с мерой G(dX) = R@)F(dX) (F(dX) — мера, отвечающая функции распределения F(x)). ? Замечание 1. Определение 9 главы I вводит характеристическую функцию ме- меры Q, заданной на сг-алгебре ё$(Жп). А.Н. Колмогоров (см., например, [33; т. 1]) предложил понятие характеристического функционала меры Q, заданной на боре- левской сг-алгебре банахова пространства В: ж*) = Г eix*^Q(dx), х* е В*. /в Отметим, что обобщение теоремы Бохнера-Хинчина на характеристические функ- функционалы не является тривиальным.
Приложение 5 Доказательство теоремы Колмогорова—Сегё Покажем, как для стационарного в широком смысле процесса X = {Xt,t Е Z } за- задача нахождения ошибки линейного прогноза за один шаг переформулируется в ап- проксимационную задачу классической теории функций. Доказательство теоремы 16 главы VII разобьем на отдельные шаги. А. Согласно определению ошибки прогноза величина 6A) может быть найдена из формулы S2 A) = lim 7n, где п—>-оо 7n = inf k=i A) а нижняя грань берется по всевозможным значениям ai,..., ап Е С. Ясно, что {in}n^i — невозрастающая последовательность. В силу теоремы 5 главы VII пространства Н(Х) и L2([-tt, тг], Й§([—тг, тг]), Q) изо- изоморфны, где Q — спектральная мера процесса X, имеющая плотность д по мере Ле- Лебега. При этом для каждого t G Z JtX X е [-7Г,7Г]. Поэтому для /(А) = 2тгд(А), где A G [—тг, тг], имеем п 2 -, р7Т akXt_k = — / b0 + helA ¦ bne inX\2 /(A) dX, B) здесь bo = 1, bk = — а/с5 & — 1, • • • ,n. Следовательно, = inf< — C) где нижняя грань берется по функциям h(X) = |Р(егЛ)|2, для которых Р = Р(^) — любой полином (z G С) с Р@) = 1, а среднее -4(г;) функции v определено перед формулировкой теоремы 16 главы VII. В. Вычислим правую часть формулы C), предположив, что In/ e L1 = L1([-Tr,Tr],^([-Tr,Tr]),mes), где mes — мера Лебега.
Приложение 5. Доказательство теоремы Колмогорова-Сегё 341 Допустим вначале, что все нули полинома P(z) лежат вне круга \z\ ^ 1. Тогда по теореме о среднем для гармонических функций \nh) = ±- Г \nh(\)d\=±- f 27Г J-7T 27Г J\Z\ = 1 \Z\ и, значит, G(h) = exp{A(\nh)} = 1. Если неотрицательная функция v = г;(A), A Е [—тг,тг], такова, что функции v и In г' Е L1, то из неравенства Иенсена следует известное неравенство между средним арифметическим и средним геометрическим: A(v) 2 G(v). D) Поскольку G(h) = 1 и In / G L1, имеем A(hf) > G(hf) = G(h)G(f) = G(f). E) Итак, если все нули полинома P(z) лежат вне единичного круга, то с учетом C) получаем, что S2(l) ^ G(f). C. Рассмотрим теперь случай, когда все нули полинома P(z) по модулю меньше п или больше единицы. Ясно, что |P(z)|2 = b Y[ \z — Zk\2, где b > 0 и z^ ф О, к = 1,..., п, так как Р@) = 1. к=1 Если существуют корни zj~ такие, что \zj^\ < 1, то можно перейти к новому полино- полиному P(z), для всех корней которого 2^ имеем |2^| > 1, и при этом |Р(егЛ)|2 = |Р(егЛ)|2 при любом A G [-7Г, тг], а также Р@) = 1. Действительно, если z = егЛ, \zk\ < 1, то \z — Zk\ = \zk\ \z — z"^ |, где точка (~Zk)~1 лежит вне единичного круга. Тем самым, в этом случае снова S2(l) ^ G(f). D. Допустим, наконец, что |;г:/с|^1,& = 1,...,п,и что среди корней z^ найдутся такие, для которых \z\^\ = 1. При любом е > 0 выберем полином P^(z) той же степени п, но у которого все нули ^ лежат строго вне единичного круга (если \z\z\ = 1, то берем корень ;?& достаточно близким к^,но так, чтобы \z\~\ > 1) и при этом ^ Г |/(А)|||Р^(егЛ)|2-|Р(егЛ)|2|^А<г, Р@) = 1. F) 27Г J-7T Проводя оценку E) для функции h(X) = |Р^е^(егЛ)|2, с учетом F) получаем, что и в этом случае S2(l) ^ G(f). E. Теперь докажем, что S2(l) ^ G(f), если In/ G L1. Известно (см., напри- например, [53; т. 2, с. 92]), что любой неотрицательный тригонометрический полином (от A G [—тг, тг]) может быть записан как |Р(егЛ)|2 для некоторого полинома Р = P(z). Это замечание и теорема Вейерштрасса показывают, что в соотношении C) величи- величина S(lJ не изменится, если нижнюю грань брать по всем неотрицательным непре- непрерывным на отрезке [—тг, тг] функциям /i, для которых G(h) = 1. Пусть вначале /(А) ^ а > 0 для всех A G [—тг,тг]. G)
342 А. В. Булинский, А. Н. Ширяев Тогда, очевидно, G(f) ^ а. Для любого г > 0 можно найти тригонометрический полином Л(егЛ), Л Е [—тт, тг], такой, что для г (Л) = \Я(егХ)\2 верны оценки г(А)^а при Ае[-тг,тг], ^- Г \г(Х) - f(X)\d\ < е. (8) 27Г J-7T Заметим, что при 0 < х < у л / у — х\ у — х \ъу - \ъх = In 1 + < X ) X Поэтому из G) и (8) вытекает, что In r G L1. Кроме того, Для функции ho(X) = G(r), Л G [—тг, тг], имеем г(Л) В силу (8) j 2 а<1 + . г(Л) а Из A0) и A1) получаем, что Й2A) ^ G(f) при выполнении условия G). F. Избавимся от предположения G). С этой целью для а > 0 введем функции fa (А) = /(Л) + а, Л G [—тг, тг]. Убедимся в том, что G(fa) -> G(/) при а ^ 0 + . A2) Очевидно, In fa G I/1 при любом а > 0 и Г ln/(A)dA ^ Г ln/a(A)dA. A3) J —тг «/ —тг Для произвольного v > 0 положим В (г/) = {A G [—тт,тт]: /(А) > г/}. Аналогично (9) при всех а > 0 и г/ > 0 находим (К / ln/a(A)dA- / ln/(A)dA^—. JB{u) JB{u) v Если v + a < 1, то Г ln/a(A)dA^ / ln/a(A)^A^ / in/(A)dA + —. J-7T JB{v) JB{v) V Следовательно, при каждом v G @,1)
Приложение 5. Доказательство теоремы Колмогорова-Сегё 343 limsup / In fa(X) d\ ^ [ In f(X)dX. A4) a->0+ J-7T JB{v) Из A3) и A4) заключаем, что если In / G L1, то A(ln/a)->A(ln/) при a^0+, A5) откуда следует свойство A2), поскольку G(v) = ехр{АAпг>)}. G. При а > 0 обозначим 7а(^) решение задачи A) для процесса {Xt(a), t E Z}, имеющего спектральную плотность /a(A), A Е [—тг,тг]. Из формулы B) нетрудно видеть, что гуп(а) ¦!¦ 7п Для каждого n G N при а —у 0+. Следовательно, обозначив #2A) = lim 7п(й) 5 получаем, что п—>>оо <^A) ^ ?2A) при всех a > 0. A6) Согласно пункту Е имеем 6^A) ^ G(fa). Отсюда, из A6) и A2) вытекает, что 2 Н. Пусть, наконец, In / ^ L1 (т. е. не существует конечный интеграл от этой функ- функции). Тогда АAп /) = Г In /(A) dA = -оо, A7) J — 7Г поскольку функция (In/(А)) + = 1п/(АI{/(А) ^ 1} ^ /(А) при всех A G [-тг,тг], причем / интегрируема на отрезке [—тг, тг]. Таким образом, Для любого а > 0 имеем In fa G L1 и согласно пп. Е и G 52A) ^ 6l(l) ^ G{fa). Поэтому осталось проверить соотношение A2). В рассматриваемом нами случае (когда In/ ^ L1) соотношение A4) также верно. Поэтому limsup / ln/a(A)dA ^ /{/(A) > 0}ln/(A)dA. A8) Если интеграл в правой части A8) равен —оо, то A5) доказано, значит, справедливо и A2). Пусть теперь интеграл в правой части A8) конечен. Тогда в силу A7) имеем /(А) = 0 (и In/(А) = — оо) на множестве А положительной меры Лебега. Итак, для всех 0 < а < 1 Г (In /a(A))+ d\ < Г (/(А) + a) d\ < Г /(A) dX + 2тг а, J —тг J —тг «/ —тг (In fa(X))- dX [ {f(\)=O} Переходя к пределу при а —у 0+, из последних соотношений снова получаем A5), значит, и A2). Теорема полностью доказана. ? Замечание. Мы установили, что условие In/ ^ L1 равносильно тому, что 5A) = 0. В силу теоремы 11 главы VII тем самым получен критерий сингулярности стационарного процесса, имеющего спектральную плотность д (/(А) = 2тгд(А), AG [-7Г,7Г]).
Приложение 6 Доказательство строго марковского свойства семейства броуновских движений § 1. Результаты этого раздела мы используем далее в приложении 7 для полу- получения в вероятностных терминах решения задачи Дирихле. При этом будет удобно оперировать семейством броуновских движений, выходящих из различных точек пространства Жш в начальный момент t = 0. Сейчас наша главная цель — полу- получить вариант (теорема 2) строго марковского свойства этого семейства, обобщаю- обобщающий строго марковское свойство винеровского процесса, доказанное в § б главы III. Введем необходимые обозначения. Пусть стандартное m-мерное броуновское движение W = {Wt = (Wt , ...,VFt ), t ^ 0} непосредственно задано, т.е. Wt(uj) = u{t) для ?^0исс?еП = С([0,оо),Мт), на пространство непрерывных на [0, оо) функций со значениями в Ж171 снабжено метрикой (V.46) равномерной сходимости на компактах (где Кп = [0,n], n Е N). Нетрудно убедиться, что борелевская сг-алгебра Й§(С([О, оо), Ж171)) совпадает с цилиндрической сг-алгеброй 3 этого пространства. В качестве меры Р на (П, 3) возьмем распределение броуновского движе- HUtfW. Заметим, что Р(Со([0, oo),Mm)) = 1, где Со ([0, оо),Мт)—подпространство С([0, оо), Жш), состоящее из функций, принимающих в точке t = 0 значение О G Ж171. Как обычно, считаем вероятностное пространство (П, 3, Р) пополненным, а ес- естественную фильтрацию F = (^t)t^o броуновского движения W — расширенной классом JV множеств Р-нулевой вероятности. Введем на (П, 3) семейство мер Р*(С):=Р(С-х), A) где х G Жш, С G ^ а С - х = {у( •) - х: у( •) G С}. Очевидно, Р = Р°. Лемма 1. Для каждого х G Ж171 процесс W = {Wt, t ^ 0} является на про- пространстве (П,^, Рх) броуновским движением относительно фильтрации F, начинающимся из точки х G Ж171 в момент t = 0; его траектории Рх-п.н. непрерывны и {относительно меры Рх) 1) И^о = х; 2) Wt — Ws-L^s, т. е. Wt — Ws и 3е's независимы при 0 ^ s < t; 3) Wt —Ws ~ N@, (t - s)I) при 0 ^ s < t, где I — единичная матрица т-го порядка.
Приложение 6. Доказательство строго марковского свойства 345 Доказательство. Легко видеть, что Рх есть распределение вероятностей на сг-алгебре 3 процесса Wx = {Wf, t ^ 0}, где W?(u) = х + Wt(u), xGlm,tH, uj G П. Другими словами, P^(C) = P(^gC) для CeJ^. B) Требуемые свойства 1), 2) и 3) вытекают непосредственно из соответствующих свойств броуновского движения W на (П, 3*, Р). ? §2. Нам понадобится ряд вспомогательных результатов о процессах Wx = = {VFX(?),? ^ 0},ж Е Мт. Прежде всего, рассмотрим интегрирование по введенным мерам Рж, ж е Ж171. Лемма 2. Усреднение EXY (по мере Рх) ограниченной действительной слу- случайной величины Y является ?$(ЖШ) | ?$ (Ж)-измеримой функцией от х Е Ж171. В частности, функция РХ(А) измерима по х при каждом A G 3*. Доказательство . Пользуясь леммой б главы I, нетрудно видеть, что множества А из ^, для которых функция Рх (А) измерима, образуют А-систему @. Обозначим ё совокупность цилиндров из 3 вида С — {Wt\ ? ^Ъ • • • •> Wtn G Вп}, где ti G [0, оо), 5^ — параллелепипеды в Ж171 ,г = 1,...,пип G N. Этот класс замкнут относительно операции пересечения. По теореме 3 главы VI процесс Wx = {Wx, ? ^ 0} является марковским на (П, ^, Р) для любого х G Mm. При этом, очевидно, для каждого х G Ж171 естественная фильтрация процесса Wx совпадает с F. Пример 2 главы VI показывает, что переходная функция процесса Wx на (П, 3, Р) задается формулой P(x,t,B) = { v ' Jb C) при? = О, где ж G Mm ,БЕЙ (Mm), | • | — евклидова норма в Жш ж5х- мера Дирака, сосредо- сосредоточенная в точке х. Пользуясь формулами B), (VI.34) и C), получаем, что ё С @. По теореме о монотонных классах ^ = cr{(f}c®,a так как @ С 3, то заключаем, что 3 = 3). Величину У можно представить, как равномерный предел (см. A.4)) линейных комбинаций индикаторов событий из 3, поэтому утверждение об измеримости функ- функции EXY вытекает из леммы б главы I. ? Лемма 3. При каждых xGMm,0O^^G ё%(Жш) Px(Wt eB\3s) = P(W3,t- s, B) (Px-n. н.), D) где переходная функция в правой части D) задается формулой C).
346 А. В. Булинский, А. Н. Ширяев Доказательство . Учитывая crjH^} | ^(М)-измеримость правой части D), нуж- нужно лишь установить, что для любого A Е 3 s имеет место равенство Px(An{Wt eB}) = ExP(Ws,t-s,B)lA. E) Множества А из 3, для которых верно E), образуют А-систему @. Покажем, что в эту систему входит тг-система 8, состоящая из множеств вида A = {wtl евъ ..., wtn евп}, где (К h < -" < tn <с s,#b...,?n (), С этой целью прежде всего отметим, что если функция h: Q —> Ж ограничена и 3 | Зё(Ж)-измерима, то для любого х Е Ж171 Exh(W) = Eh(Wx), F) здесь Е обозначает интегрирование по мере Р = Р°. В самом деле, при h = 1с, где С Е 3, это верно в силу B), и свойство F) следует из того, что h можно получить как равномерный предел линейных комбинаций рассмотренных индикаторов. Из F) вытекает, что для введенных цилиндров А формула E), которую надо про- проверить, перепишется в следующем виде: P(Axn{W?eB}) = EP(W?,t-s,B)lAx, G) где Ах = {Wxx е В\, ..., Wxn е Вп} е 3^х. Равенство G) справедливо, поскольку {Wx, t ^ 0} при каждом х G Mm есть однородный марковский процесс с переходной функцией C). По теореме о монотонных классах заключаем, что 3е's — &{$} = @. П Для случайного вектора V и действительной случайной величины Y полагаем EVY := H(V), где Н(х) = EXY (вид Н(х) зависит от величины У, которая предпо- предполагается интегрируемой по любой мере Рж, х G Ж171). Лемма 4. Пусть W — броуновское движение на пространстве (О,,3, Рж), описанное в лемме 1. При каждыхх G Mm, s,u ^ 0 и всех ограниченныхЗё(Ж171)- измеримых функциях / Ex(f(Ws+u) | &8) = Ew'f(Wu) (Рж-п. «.)• (8) Доказательство. Положим в формуле D) s = 0, t = ix и возьмем математичес- математическое ожидание Ех от обеих частей. Тогда получим для любых и ^ 0, В G ?$(Мт) и PX(WU eB) = ExP(W0,u,B) = P(x,u,B), (9) где учтено, что Wo = х Рж-п.н. Если/(^) = lB(z),B e $(Жш)иг G Мт,тоиз D) находим: Ex(f(Ws+u) | 3s) = PX(WS+U eB\3s)= P(W8,u,B) (Рж-п.н.). С другой стороны, с учетом (9) имеем EX1B(WU) = PX(WU eB)= P(x,u,B),
Приложение 6. Доказательство строго марковского свойства 347 что доказывает формулу (8) для индикаторных функций /. В общем случае требуе- требуемое свойство (8) следует из доказанного, поскольку произвольная ограниченная дей- действительная измеримая функция / равномерно аппроксимируется линейными комби- комбинациями индикаторов. ? § 3. Теперь введем семейство броуновских движений (сравните с дополнением к главе VI). Для s ^ 0 положим 3*^s = a{W(t), t ^ s}. Пусть при каждых s ^ 0 и х е Жш на сг-алгебре &^s задана вероятностная мера Рв'ж. Определение 1. Броуновским семейством с переходной функцией C) назо- назовем непосредственно заданный на П = С([0, оо), Ж171) процесс W = {W(t),t ^ 0} такой, что {W(t),t ^ s} при любых s ^ Оиж G Ж171 является однородным марков- марковским процессом (с переходной функцией C)) на пространстве (П, i^s, Ps'xM выходя- выходящим из точки х в момент s. Иначе говоря, при любых sH,zG Ж171 выполняются условия: 1°. PS>X(WS = х) = 1 для каждых s ^ Оиж G Mm; 2°. Ps>x(Wt Е B\&[SjU]) = P(Wu,t-u,B) Р8'ж-п.н. для каждых s ^ 0, х G Жш и В G^(Mm),s ^u^t; здесь &[SiU] =a{Wv,v G [s,u]}. Покажем, как легко построить такое семейство. Для и ^ 0 определим оператор сдвига ви: П —у П, положив Fии)(')=и(' + и), где и; G П. A0) Если А С П, то пусть #WA := {^^a;:^; G А}, а для функции Y(lj), заданной на П, пусть (OuY)(uj) := F(^wcj), cj G П. Для системы si подмножеств ft определим #~1?/ := {в~1А: A G si}. Мы пишем один и тот же символ ви для аналогичных действий на разные объекты, чтобы не усложнять обозначений. Лемма 5. Для всех u^OuO^s^t ^[] = ^[]' A1) при t = оо считаем &[s,t] := &>з и Доказательство. В силу A0) для всех u,t ^ Оиш Е О, имеем @uWt)(u;) = Wt@uu;) = Wt(u(- + и)) = u(t + и) = Wt+uM- A2) Для г; G [s + гх, t + гл] и Б G ^(Mm) имеем {Wv G В} = 0-Чи: Wv_ttM G В} G в^^]. Поскольку cr-алгебра &[s+u,t+u] порождается множествами {Wv G В}, где г; G G [s + u,t + и], получаем, что ^[5+г^+гб] С ^1^[s,t]- С другой стороны, для ft G [s, t] и В е ё$(Жш) имеем ^"Ч^ ?В} = {и: Wh{0uu) еВ} = {и: Wh+U(u;) G В} G ^[e+t4,t+u].
348 А. В. Булинский, А. Н. Ширяев В силу следствия 1 главы I 6и что завершает доказательство леммы. ? Таким образом, по лемме 5 для каждого С Е 3*^s существует A Е ^^о — & такое, что С — 0~1А. Теперь положим Р5>Ж(С) :=РХ(А), где О 0, хеГ. A3) Это определение корректно, поскольку 6S отображает П на П для каждого s ^ О (если С = 0~гА, то А определено однозначно: А = 0sC). Кроме того, посколь- поскольку взятие прообраза сохраняет теоретико-множественные операции, Ps'x есть мера на ^^s- Учитывая A3) и лемму 4, легко видеть, что свойства 1° и 2° при этом вы- выполнены. Имея в виду A3), будем обозначать наше однородное броуновское семейство (с пе- переходной функцией C)) (Wt, Px)t^o,xewn • Еще раз подчеркнем: мы располагаем од- одним процессом W, а варьируем сг-алгебры и заданные на них меры, что соответству- соответствует интуитивному представлению о возможности "выпускать" броуновское движение в любой момент s ^ 0 из произвольной точки х G Ж171. Определение 2. Однородное марковское семейство (процесс) с переходной функ- функцией Р(х, ?, В) называется феллеровским, если для каждых t ^ 0, х G Mm имеет место слабая сходимость P(z,t,')=>P(x,t,-) при z^x. A4) Таким образом, это определение связано лишь с переходной функцией, а не с самим семейством или процессом и означает, что для любых t ^ 0 и / G C^ (Mm) при z —У х (z,x G Мт) справедливо соотношение (Ttf)(z) := f f(y)P(z,t,dy) -»• / f(y)P(x,t,dy) = (Ttf)(x). A5) Следовательно, Cfr(Mm) — это инвариантное подпространство оператора Tt при каждом t ^ 0. Пользуясь формулой C) и теоремой Лебега о мажорируемой сходимости, получа- получаем, что броуновское семейство (Wt, Рх)г^о,хешт является феллеровским. § 4. Для изучения функционалов от броуновского движения и в других вопросах требуются различные формы строго марковского свойства (см. далее §6). Прежде всего, обобщим лемму 4 на более "богатые" сг-алгебры ^s+ = f] &s+e- Лемма 6. Для каждого х G Mm, всех s,u ^ 0 и / G Ex(f(Ws+u) | ^s+) = EW*/(WU) P*-n. n. A6)
Приложение 6. Доказательство строго марковского свойства 349 Доказательство. В силу (8) для всех s, и ^ 0, г > О, х е Мт и / е Сь(Мт) Ex(f(Ws+u+?) | &s+?) = Ew^f(Wu) Рж-п.н. Учитывая непрерывность траекторий W = {Wt, t ^ 0} и теорему 16 главы IV (вмес- (вместо г I 0 можно взять еп -| 0), находим, что Рж-п.н. Ex(f(Ws+u+e)\&s+?)^Ex(f(Ws+u)\&s+) при е4.0. Принимая во внимание, что на (П, ^, Рх) процесс W = {Wt, t ^ 0} является марков- марковским с переходной функцией C), имеем [ f(y) P(x, и, dy) = (Tuf)(x). В силу феллеровости броуновского семейства для каждого х G Mm получаем Ezf(Wu) -+ Exf(Wu) при z^x. Поэтому для всех ио G П w w при ?4 0. П Следующий шаг в обобщении марковского свойства состоит в переходе от сг-ал- гебры &*3+ в A6) к использованию сг-алгебры, порожденной марковским моментом т относительно фильтрации (^rs+)s^o- А именно, положим J^T+ = {Асп:Ап{т ^t} e &t+ для любого t ^ 0}. Далее будем считать WT и Wu+T, где и ^ 0, равными нулю в Мт на множестве {cj G П: г (о;) = оо}. Теорема 1. Пусть Рх(т < оо) = 1 при всех х G Mm, где г — марковский момент относительно фильтрации (^+)t^o- Тогда для каждых х G Мт, и ^ 0 и любой ограниченной функции /: Мт —>¦ М, / G Рж-п.«. A7) Доказательство. Введем моменты тп = ([2пт] + lJ~n, n G N, обозначая [ • ] — целую часть числа. Очевидно, тп (со) \^ т(а;)прип —>- оодлявсехо; G П. Кроме того, тп есть марковский момент относительно фильтрации (^t)t^o ПРИ любом п G N (см. доказательство леммы 7 главы IV). Покажем, что при каждых х G Im,n G N, w ^ 0и/G Сь(Мт) ) Р*-п.н. A8) Правая часть A8) является crjWV^} | 08(Ж)-измеримой, следовательно, §<Тп измеримой, а значит, и ^Тп+ \ 08{Ш)-измеримой. Поэтому требуется лишь прове- проверить, что для любого A G ^тп+ ExlAf(WTn+u) = Ex
350 А. В. Булинский, А. Н. Ширяев Применяя лемму б и учитывая, что А П {тп = к2~п} Е ^k2~n-\- для всех к, п Е N, а также пользуясь счетной аддитивностью интеграла Лебега, при каждом х Е Ж171 получаем оо ExlAf(WTn+u) = Y,Ea k=i ) = ExlAEWmf(Wu). к=1 Итак, A8) установлено. Совершим предельный переход в A8). В силу феллеровос- ти броуновского семейства EWrn f(Wu) —У EWt f(Wu) при п —> оо для всех uj E ? {т < оо}. По теореме 16 главы IV имеем Рж-п.н. при п —у оо. / 71=1 Проверим, что оо ^т+ = f| ^Тп+. A9) 71=1 Покажем, что ^т+ С З'тп+ ПРИ любом n G N. Действительно, если A G ^v+5 то А П {г ^ t} G J^t+ для каждого t ^ 0. Тогда А П {гп ^ t} = А П {г ^ t} П П{гп ^ t} G ^t+ (учли, что {гп ^ t} G ^t+ и {тп ^ t} С {г ^ t} при любом оо t ^ 0). Следовательно, ^т+ С П ^Tn+- Для получения обратного включения 71=1 воспользуемся тем, что А е &т+ <^> Ап{т <t} e &t Для любого t ^ 0. Пусть А е ^тп+ при всех п е N. Тогда А П {тп < t} e &t Для всех п е N и t ^ 0. Поэтому • оо \ оо \J{rn<t}) = \J(An{rn<t})e&t, 71=1 ^ 71=1 что завершает доказательство A9). Тем самым, A7) доказано для любой непрерыв- непрерывной и ограниченной функции /. Класс множеств С из ?$(Мт), для которых A7) выполнено при / = 1с, образует А-систему. Любое замкнутое множество В G ^(Ж171) входит в эту систему, поскольку 1в можно получить (см. доказательство теоремы 4 главы III) как предел убываю- убывающей последовательности непрерывных "шапочек" /п, для которых A7) установлено. Пересечение конечного числа замкнутых множеств замкнуто, и замкнутые множес- множества порождают ?$(Мт). По теореме о монотонных классах соотношение A7) будет справедливо для / = 1с, где С — любое борелевское множество в Ж171. Доказатель- Доказательство теоремы 1 завершается, если учесть, что ограниченную измеримую функцию / можно представить как равномерный предел последовательности линейных комби- комбинаций индикаторов борелевских множеств. ?
Приложение 6. Доказательство строго марковского свойства 351 Формула A6) получается, в частности, из A7) при т = s, поскольку в этом случае Для доказательства так называемого строго марковского свойства броунов- броуновского семейства (далее теорема 2) нам потребуется еще один вспомогательный ре- результат. Лемма 7. Пусть Рх(т < оо) = 1 для всех х Е Mm, где т — марковский момент относительно потока (&t+)t^o- Тогда при каждом х Е Ж171 для любых п е N, uk ^ 0 и ограниченных функций fk: Ж171 -> М, Д е ^(Мт) |Й8(М), fe = 1,... ,п, имеем Ex(f1(WT+Ul)---fn(WT+Un)\$T+) = EwTf1(WUl)---fn(WUn) Рх-п.н. B0) Доказательство леммы ведется по индукции. Прип = 1 результат дает теоре- теорема 1. Покажем, как от п = 1 перейти к п = 2 (переход от произвольного п к п + 1 аналогичен). Пусть для определенности 0 ^ и\ ^ U2. Тогда Рж-п.н. Ex(fi(WT+ul)f2(WT+U2)\&T+) = = Ex(Ex(h(WT+ul)f2(WT+U2)\&iT+ul)+)\&T+) = = Ex(h{WT+ul)Ex{f2{WT+U2) |^(T+Ul)+) |^T+) = Последнее равенство в этой цепочке вытекает из следующих рассуждений. При лю- любом z G Mm Ez{f1(WUl)Ex(f2(WU2)\^Ul+))=Ez(f1(WUl)f2(WU2)). В формуле A7) при каждом х G Мт и г = s в качестве варианта Ex(f(Ws+u) I ^s+) можно выбрать E^S/(WW) при всех и; G П. Поэтому = Ezfi(Wul)f2(WU2). D § 5. Теперь мы располагаем всем необходимым, чтобы получить основной резуль- результат приложения 6. Введем оператор сдвига на случайную величину т G [0, оо). Для отображения Y(u), заданного на П, положим (втУ)(со):=?(вт(ш)со), соеП, B1) где 6uuj определено в A0). На множестве Рж-меры нуль, где т = оо, всюду считаем WT = 0 G Mm и 6Th(W) = 0 (здесь h: п -> R и п = С([0, оо), Ж171). Для броуновского семейства (Wt, Px)t^o, жемт следующий результат дает форму строго марковского свойства. Нетрудно показать, что приводимая ниже теорема обобщает теорему 4 главы III, описывающую более простой вариант строгой марковости винеровского процесса.
352 А. В. Булинский, А. Н. Ширяев Теорема 2. Пусть Рх(т < оо) = 1 при всех х Е Мт, где т — марковский момент относительно фильтрации Ct+)t^o- Тогда для каждого х Е Ж171 и любого ограниченного &< \ ё&(Ж) -измеримого функционала h: п —У Ж Ex@Th(W) | ^Т+) = Ew-h(W) Px-n. н. B2) Доказательство. В силу B1) получаем (напомним, что процесс W непосредст- непосредственно задан и & = SB (С ([0, оо), Ж171)) eT{u)u)) = h(eT^ B3) По лемме 7 формула B2) верна для h = 1с, где цилиндр C = {Wtl еви ..., wtneBn}, (К h < -" < tn,B^...,Bn e По теореме о монотонных классах получаем, что B2) справедливо для h = 1а, где А — произвольное множество из 3*. Теперь заметим, что функция (функционал) h равномерно приближается линейными комбинациями индикаторов событий из 3. Это завершает доказательство теоремы. ? Замечание 1. Свойство (IV.2) и^т | ё$(Ж)-измеримость EWr/(W), разумеется, влекут, что в B2) вместо ^т+ можно брать 3Т. Кроме того, очевидно, марковский момент т относительно "более бедной" фильтрации (^t)t^o всегда является марков- марковским и относительно "более богатой" фильтрации (j^+)t^o- Формула B2) обобща- обобщается и на 3 | ё%(Ж)-измеримые (необязательно ограниченные) функционалы /i, для которых Ex\h(W)\ < оо при всех х е Ж171. B4) Для этого достаточно рассмотреть ограниченные функционалы вида hN(u) := h(u)l{\h(u;)\ 4: N}, воспользоваться доказанной для них формулой B2) и учесть, что Ддг (^) —> h{uo) при N —У оо для каждого ио G П. § 6. Еще одну полезную форму строго марковского свойства броуновского движе- движения содержит следующее упражнение. 1. Докажите, что теорема 1 остается справедливой, если вместо константы и ^ О взять неотрицательную случайную величину г] G ^т+ | Зё(Ж) такую, что Рх(т] < оо) = 1 при всех х G Ж171. Точнее говоря, для каждого х G Ж171 имеет место равенство E*(f(WT+ri) | ^т+) = (ТУ)(]?Т) Рж-п.н., B5) где (Т77f)(WT) понимается как выражение, получаемое при подстановке t = t](uj) и z = Wt^)(uj) в формулу f(y)P(z,t,dy); переходная функция Р(г, t, 5) задается формулой C).
Приложение 6. Доказательство строго марковского свойства 353 2. Левая часть формулы B5) получилась подстановкой г) вместо и в левую часть формулы A7). Однако если и в правую часть A7) подставить г\ вместо и, взяв EWt f(Wrj) (т.е. вычислить ф(г) = Ezf(Wrj) и затем взять ф(\?т)), то полу- получится, вообще говоря, некорректное равенство. Проверьте это. Пример 1. Покажем, как с помощью соотношения B5) (приттг = 1) можно найти распределение величины та = inf{t ^ 0: Wt = а}, где а > 0 (другие два способа ре- решения этой задачи для броуновского движения, начинающегося из нуля, изложены в § 10 главы III и дополнении к главе V). Пусть в B5) т = та и т] = (t - т) V 0, где t > 0. Тогда Рх(т < оо) = 1 для всех х Е Ш. и, очевидно, 77 Е ^т+ | Й§(М), а также Рх(г] < оо) = 1 при всех xGl. Кроме того, /(И/т+Г7Iа = f(Wt)lA Для А = {т < t} ? &Т+- Поэтому, согласно B5), для всех ж?Ми/ = 1в, где В G Зё(Ж), имеем ExlAlB(Wt) = ЕЖ1А(ГВ)(И^Т). B6) Здесь левая часть есть Рх (г < t, Wt G В). Поскольку WT = а (Рж-п.н.) для каждого xGi, имеем (TnB)(WT) = (Гв)(а) = Pia^B). Тем самым, правая часть B6) равна {r<t} Для каждого и; из множества {uj: rj(u) > 0} плотность нормальной величины N(a,r)(uj)) симметрична относительно вертикальной прямой, проходящей через точку а. Поэтому Jx(a,t,B) = Jx(a,t,2a-B), где множество 2а — В есть отражение множества В относительно точки а. Итак, Рх(т <t,WteB) = Px(r <t,Wte2a- В). B7) Возьмем х < а, В = (-оо, а) и t > 0. Тогда {Wt > а} С {г < ?} и в силу B7) Рж(т < ?, VFt < а) = Рх{т <t,Wt>a) = Px{Wt > а). Поэтому с учетом того факта, что Рх (Wt = а) = 0 при всех a,xGl, получаем Рж(г < t) = Рж(т < ?, Wt < а) + Рж(т <t,Wt^a) = = Рж(т < ?, Wt < а) + Рж(т < t, Wt > a) = Г00 1 = 2Px(Wt > а) = 2 / ^ Ja \2ъ Выше предполагалось, что а > 0 w x < а. В качестве упражнения получите ответ для любых а, х G М. ?
Приложение 7 Вероятностное решение задачи Дирихле § 1. Свойства многомерного броуновского движения, изложенные в приложении б, дают возможность получить (см. далее теорему 1) в вероятностных терминах реше- решение классической задачи Дирихле. Напомним постановку этой задачи. В области G С Ж171 требуется найти гармоническую функцию д, совпадающую на границе об- области 8G с заданной измеримой и ограниченной функцией /, здесь / Е ё% (8G) \ ?$ (Ж), 8G — замкнутое множество в Ж71. Другими словами, ищется функция д = g(xi,..., жт), удовлетворяющая уравне- уравнению Ад = О A) и граничному условию где, как обычно, А — оператор Лапласа. Займемся подготовительной работой. Определим момент rdG первого выхода m-мерного броуновского движения W = {Wt, t ^ 0} на границу 8G: TdG=ini{t^0:WtedG}. C) Как и в приложении б, мы считаем броуновское движение W непосредственно за- заданным на пространстве п = С([0, оо), Ж171) с сг-алгеброй борелевских множеств 3* этого пространства. По-прежнему, Р-распределение процесса W, F — естествен- естественная фильтрация. О вероятностном пространстве (П, ^, Р) и фильтрации F делаются обычные предположения. По теореме 3 главы III величина Tqq есть марковский момент относительно фильт- фильтрации (^t)t^o- Положим v = Tqq и введем функционал h(W) = f(Wv). D) Заметим, что Wv G 8G в силу непрерывности процесса W = {Wt,t ^ 0}. Поскольку Wv G &и | SS(dG), ^С^и/G SS(dG) \ 8&(Ж), то h - ограниченный 3 \ Й8(М)-из- Й8(М)-измеримый функционал. Для каждой точки х открытого множества G возьмем открытый шар Vr(x) = {y:\y-x\KR}
Приложение 7. Вероятностное решение задачи Дирихле 355 радиуса R > 0 с центром в ж, который содержится в G, и рассмотрим шар Vr(x) = — {У'Лу ~ х\ ^ г}> где г < R. Аналогично C) определим rdVr(x) = inf{i ^ 0: Wt e dVr(x)}, E) и пусть далее т = т^у /у По теореме 3 главы III этот момент т также является марковским моментом относительно фильтрации (^t)t^o- Нам понадобятся операторы сдвига вида вт, введенные в § 5 приложения б, а так- также меры Рж, определенные в § 1 того же приложения. Будем далее считать, что если функция Y(uS) не определена вне множества D, то Y(ojId '•= 0 дляои ? D. Лемма 1. Для каждого х Е Mm и любого г > 0 такого, что [Vr(x)] С G, имеем 0Tf(Wu)lD = f(Wu)lD (Рх-п. п.), F) где D = {v<oo},v = rdG, r = rdVr{x). Доказательство . Пусть пх = х + Со([0, оо), Мт), где х е Мт. Заметим, что РХ(ПХ) = 1 при всех х G Mm. Чтобы выйти на <9G, траектория, начавшаяся в точ- точке ж, должна сначала выйти на границу шара Vr(x). Поэтому, если uj G D П Qx, то непрерывность траекторий броуновского движения влечет неравенство т(ш) < v(uj) (< оо). G) Напомним, что WuFtuj) = Wu-\-t{^) для всех ix,t ^ 0. Возьмем произвольную точку cj G D П Пж. Пусть для нее г (со) = t. В силу F.21) получаем равенство eTf{Wv){w) = f{Wv{etuj){9tu:)) = f(Wt+v(etbj)(oo)). (8) Теперь остается лишь показать, что для рассматриваемой точки uj t + v@tou) = 1/(и). (9) Из G) имеем v(uS) > t. Кроме того, Ws(uo) G G при s < t, и t + i/@t^) = * + inf{^ ^ 0: VFw(i9tcc;) G dG] = = t + Ы{и > 0: Vrt+W(cj) G dG} = inf{s ^ 0: Ws(u) G 9G} = i/(o;). Тем самым, соотношение (9) установлено, что вместе с (8) доказывает свойство F). ? § 2. Перейдем непосредственно к решению задачи Дирихле. Теорема 1. Пусть область G С Мт такова, что PX(TdG < °°) = 1 пРи каждом xGlm. A0) Тогда для ограниченной и измеримой функции f на dG решение задачи Дирих- Дирихле A) с граничным условием B) существует и дается следующей "вероят- "вероятностной" формулой g(x) = Exf(WTgG), xeGUOG. A1)
356 А. В. Булинский, А. Н. Ширяев Доказательство. Положим v — rdG. Поскольку для ж Е 0G имеем Рж-п.н. v = 0 и Wo = ж, то граничное условие B) выполнено для указанной функции д. Обратимся к проверке свойства A). Учитывая лемму 3 приложения б и то, что f(Wu) есть ограниченная случайная величина (см. рассуждения после формулы D)), видим, что функция д(х), задаваемая формулой A1), определена при всех х Е Ж171, является ограниченной и ^(Ж171) | ?$(Ж)-измеримой. В силу A0), F) и F.22) для любой точки х Е G имеем Exf(Wv) = Ехвт/A?„) = EXEW'f(Wv), A2) здесь и далее придерживаемся соглашения, что если действительная функция Y(uS) не определена вне множества D = {v < оо}, то EXY := Ex(Y1d) (когда последний интеграл существует), a YIjj понимается как Y на множестве {у < оо} и как 0 на множестве {у = оо}. Пусть Qx = Рх (WT)~1 — распределение случайной величины WT на dVr (x), ког- когда WT рассматривается как случайная величина на пространстве (П, ^, Рх). Вос- Воспользовавшись формулой A.23), согласно которой H(Y) = f H(y)PxY-\dy) Js ' ' IS для Y: п -» S, Y € & \ ®{S), S G ЩШ) и Я: 5 -» M, Я ? 5§E) | 5§(М), из A2) и A1) имеем д{х) = [ Eyf(Wu) Qx(dy) = [ д(у) Qx(dy). A3) JdVr(x) JdVr(x) Докажем, что введенное распределение Qx — равномерное распределение на сфе- сфере dVr(x). Учитывая F.1) и F.2), можно ограничиться рассмотрением лишь точки х = 0. Кроме того, достаточно проверить (поясните в качестве простого упраж- упражнения), что Q°U~1 = Q° для любой ортогональной матрицы U порядка т. Мера Q°U~1 есть распределение случайной величины U(Wa), заданной на (П, ^, Р), где а = rdVr(oy Заметим, что U(Wa) = (UW)p, здесь/3 = inf{t ^ 0: (UW)t G dVr@)}. Это вытекает из того, что при ортогональном преобразовании евклидовы длины век- векторов сохраняются. Остается принять во внимание, что {(UW)t, t ^ 0} также явля- является стандартным броуновским движением на (П, ^, Р). Последнее следует из фор- формулы (П.З) и того, что UU* = /, где / — единичная матрица ттг-го порядка. Таким образом, ограниченная измеримая функция д: G —> Ж для каждого х G G Ж171 и любого шара Vr(x), содержащегося в области G, удовлетворяет в силу A3) соотношению 1 д^ (я1 ( \\ I fj,r(dVr(x)) JdVr{x) где \ir — мера Лебега на dVr (x). Из представления A4) вытекает, что функция д яв- является гармонической в области G, иначе говоря, Дд = 0. Для полноты изложения доказательство этого утверждения, следующее рассуждениям из [24], дается ниже. § 3. Получим интегральное условие гармоничности функции д в области G С Ж171. Лемма 2. Пусть д{х) есть ограниченная измеримая функция такая, что для любого х G G и каждого шара Vr(x) С G выполнено свойство A4). Тогда функция g является гармонической в области G.
Приложение 7. Вероятностное решение задачи Дирихле 357 Доказательство. Для г > 0 рассмотрим гладкую функцию, заданную на Ж171 формулой Гф)ехр{1/(И2-е2)}, \х\<е, v?(x) = < 10, \х\>е, где | • | — евклидова норма в Ж171, а с(е) выбирается так, чтобы / ve(x)dx = c(e) f Srexp{l/(r2 -s2)}dr = 1, A5) JW71 JO здесь Sr = 2гт~1тгт/2/Г(т/2) — площадь сферы радиуса г в Мт. Определим так- также функцию д? как свертку функций д и v?, т. е. пусть 9е(х)= [ v?(x-y)g(y)dy, г>0, хеГ, A6) где функция ^ считается доопределенной нулем вне области G. Легко видеть, что при каждом г > 0 функция де бесконечно дифференцируема в Ж171 [д? G Соо(Жт)). Пусть G(e) = {х е G: V?(x) С G}, где г > 0. Каждое такое множество открыто и G{e) t G при г | 0, т.е. |J С(г) = G и G(e') С С(г) при г < г'. Из A6), учитывая A5) и A4), находим, что для е > 0 и ж G = с(е) / / д(х + 2:) ехр{1/(г2 - г2)} /ir(^) dr = io Jdvr(o) = c(e) ехр{1/(г2-г2)} / g(x + z) fir(dz) dr = Jo Jdvr(o) = g(x)c(e) Г Sr exp{l/(r2 - e2)} dr = g(x). Jo Следовательно, g(x) = g?(x) для г > Оиж G G(e). Поэтому^ G C°°(G(e)) для каждого г > 0, значит, g G C°°(G). Зафиксируем произвольное s > 0 и ж G G(s). Для z G Уг@) по формуле Тейлора / ч / ч v^ oqix) где sup R(x, z) = o(e2) при г —у 0. zev?(o) Заметим, что для k,j = l,...,mw.e>0 / zk fi?(dz) = 0, / zkzj fi?(dz) = 0 при к ф j. JdVJO) JdV,(O)
358 А. В. Булинский, А. Н. Ширяев Поэтому 1 , ( , д[х + z) @) 1 дх\ dz) + — R(x,z)dz. A7) Учитывая, что для х Е G(e) 1 f — / R(x, z) dz = o(e2) при г -»> О ^e JdVJO) ldv?(o) 1 _ ' т получаем в силу A7), что для х Е G{e) 2т Отсюда вытекает, что Ад = 0 для всех х Е G. ? Итак, лемма 2, следовательно, и теорема 1 доказаны. ? § 4. Полезно сопоставить доказанную теорему 1 с результатами следующих уп- упражнений. 1. Приведите пример области G такой, что задача Дирихле A) с граничным условием / = 0 на 8G имеет неограниченное решение. 2. Докажите, что при т ^ 2 броуновское движение, начинающееся в точке х Е Br@) \ {0}, выйдет (с вероятностью 1) из этого шара с выколотым центром, не заходя при этом в точку 0. Тем самым, ограниченное решение задачи Дирихле A), B) в области G = {у Е Mm : 0 < \у\ < г} будет определяться лишь значениями функции / на сфере \у\ = г. Замечание 1. Если G — ограниченная область в Мт, то условие A0) всегда выполнено. Это следует, например, из того, что каждая компонента процесса W удовлетворяет закону повторного логарифма (см. теорему 8 главы III). Далее, ес- если G — ограниченная область, то 8G есть компакт, как замкнутое и ограниченное множество. Поэтому, если функция / для такой области G непрерывна на dG, то она автоматически ограничена. В силу замечания 1 приложения б вместо условия ограниченности / в условиях теоремы 1 можно потребовать, чтобы Ex\f(WTdG)\ < оо для всех х е Ж171. A8) § 5. Рассмотрим вопрос о единственности решения задачи Дирихле. Теорема 2. Пусть выполнены условия теоремы 1. Тогда каждое ограни- ограниченное решение задачи A) с граничным условием B) дается формулой A1).
Приложение 7. Вероятностное решение задачи Дирихле 359 Доказательство. Пусть д — ограниченное гармоническое в области G С Мт решение и д dQ =/. Пусть Gn = I х G G: inf \х — у\ > 1/п \. Для ж G Gn, n G N, ? ^ О рассмотрим марковский момент /3n(x,t) = ? Л Td(GnnvnO)) относительно фильтрации (^rt)t^o, где Гм =inf{t^O: W* G М} для замкнутого множества М С Mm, a Vn(x) = {у ? Мт : |^/ - х\ < п). По формуле Ито с E = /3n(x,t) можно получить, что g(W0) = g(W0) + JT Г ^^ dW^ + \Г Ag(Ws) ds. A9) JZ^ JO Oxk * JO Второй интеграл в правой части A9) равен нулю, поскольку Ws G G для s ^ C. Взяв математическое ожидание Ех от обеих частей A9), видим, что p) = Exg(W0) = д(х). Для каждого х G G, совершив предельный переход по t (t —у оо), а затем по п (п —у оо), имеем 9(WMx,t)) ^ g(WTeG) Рж-п.н., и в силу теоремы Лебега о мажорируемой сходимости что доказывает представление функции д в виде A1), а, значит, и единственность решения задачи Дирихле. ? § 6. Обратимся к поведению решения задачи Дирихле вблизи границы области G. Для открытого множества G в Ж171 введем марковский момент vG := inf{? > 0: Wt G G}, где G = Rrn\G. Определение 1. Точкам G dG называется регулярной, если Pz(vG = 0) = 1. Остальные точки границы G называются нерегулярными. Заметим, что определение регулярности является локальным в том смысле, что точка z G dG регулярна тогда и только тогда, когда эта точка регулярна как точка границы G П Vr(z) для некоторого г > 0, где Vr(z) = {у G Mm : |з/ — г| < г}. 3. Докажите, что если т = 1, то любая точка границы dG является регуляр- регулярной. Теорема 3 ([148; с. 145]). Пусть т ^ 2 и z G <9G. Тогда следующие условия эквивалентны: 1) z — регулярная точка границы 8G области G; 2) для любой непрерывной и ограниченной функции f: dG —У Ж lim Exf(WT ) = f(z); 3x^z dG 3) для любого е > О
360 А. В. Булинский, А. Н. Ширяев Простое достаточное условие регулярности точек границы содержит упражнение 4. Пусть z Е 8G и для некоторого г > 0 найдется конус с вершиной в точке z, содержащийся в R171 \ (G П Vr(z)). Тогда z — регулярная точка. Напомним, что конус с вершиной в нуле есть множество С(у,в) = {хеГ: (х,у) ^ \х\ • Mcos0}, где y^0eRm, 0Е[О,тг]. В упомянутом выше условии конус с вершиной в точке z — это множество z + C(y,в),гдеу^0иве@,тг). В связи с условием A0) в теореме 1 представляет интерес упражнение 5 ([148; с. 253]). Положим v(x) = Px(t-q = оо). Докажите, что v(x) — гар- гармоническая функция в области G, и если z — регулярная точка границы, то lim g(x) = 0. В частности, если любая точка границы является регуляр- G3x-*z ной, то функция Xv + g(x), где g(x) = Exf(WT-)l{T_<oc}, при каждом A G 1 является решением задачи Дирихле A), B). Кроме того, любое ограниченное решение этой задачи имеет указанный вид. § 7. Следующие два результата относятся к диффузионным процессам, вообще говоря, более общим, чем броуновское движение. Предварительно напомним определение генератора. Определение 2. Генератором (или инфинитезималъным оператором) полу- полугруппы (Xt)t^o? заданной на банаховом пространстве 5, называется оператор А, дей- действующий следующим образом: A/= s-lim Г^1^1 t на тех элементах / G 5, для которых указанный предел (по норме) существует. Теорема 4 ([169; с. 95]). Пусть X = {Xt, t ^ 0} — процесс в Мт, являющийся решением стохастического дифференциального уравнения dXt = b(Xt) dt + a(Xt) dWu Xo = x G Mm. B0) Пусть А — генератор полугруппы, задаваемой формулой (Ttf)(x) = E*f(Xt), где Ех подчеркивает, что Хо = х. Тогда в область определения О)а оператора А входят все функции / из С^(Мт) {непрерывные ограниченные функции, имеющие непрерывные и ограниченные производные первого и второго порядков) и для / G Ц
Приложение 7. Вероятностное решение задачи Дирихле 361 Замечание 2. Поскольку броуновское движение W = {Wt, t ^ 0} является ре- решением уравнения dXt = dWt (здесь Ъ = 0 и а = / — единичная матрица ттг-го порядка), то теорема 4 показывает, что для броуновского движения генератор соот- соответствующей полугруппы является расширением оператора А/2, заданного на функ- функциях из С2 (Ж171). Теорема 5 (формула Дынкина). Пусть выполнены условия предыдущей тео- теоремы иг — марковский момент такой, что Ет < оо. Пусть f есть ограни- ограниченная функция из 3)а- Тогда при каждом х Е Ж171 B1) Пример 1. Пусть W = {Wt? t ^ 0} — броуновское семейство в Мт (??г ^ 2). Найдем математическое ожидание Ежт, где т — тду ^ — момент первого выхода броуновского движения из шара радиуса г с центром в нуле (|ж| ^ г). Положим тп = тЛп,п G N. Возьмем функцию / G С%(Ж171) такую, что f (у) = \у\2 при \у\ < г. Тогда в силуB1) и замечания 2 для х G Vr@) VTn) = f(x) + E Следовательно, Exf(WTn) = \х\2 + ЕХ( Г" mds) = \х\2 + тЕхтп. Учитывая, что \WTn\ ^ г для каждого п G N, получаем Ежтп ^ (г2 — \х\2)/тп. Очевидно, что Рж-п.н. г < оо и гп / т при п —У оо. Поэтому Exf(WTn)^E*f(WT)=r2 при 7W<x) И Ежт = lim Ехтп = (г2 - |ж|2)/ш. 4. Пусть х fi Vr@). Найдите вероятность Рх(т < оо), где т — момент первого достижения броуновским движением (в Ж171, тп ^ 2) шара Vr@M г > 0. § 8. Обсудим некоторые обобщения задачи Дирихле A)-B). Пусть на С2 (Ж171) задан полу эллиптический оператор где полуэллиптичность означает, что при каждом х G Ж171 собственные значения симметричной матрицы а = (а^) неотрицательны (эллиптичность означает по- поло эрсительность всех собственных значений).
362 А. В. Булинский, А. Н. Ширяев Для области G С Ж171 и непрерывной функции/: 8G —> Мпопытаемся по аналогии с задачей Дирихле для оператора Лапласа найти так называемое L-гармоническое расширение /, т. е. найти такую функцию д, что Lg = 0 в G B2) И lim д(х) = f(z) для всех "регулярных" точек z G dG. B3) "Вероятностный подход" к решению этой задачи состоит в том, что с операто- оператором L связывается соответствующий m-мерный диффузионный процесс X = {Х^, t ^ 0}, т.е. процесс, являющийся решением уравнения B0), где W = {Wt, t ^ 0} — броуновское движение в Ж171, а матрица а = (a^j) такова, что аа* = а. Можно показать, что генератор А такого процесса X = {Xt, t ^ 0} совпадает на С^(ЖШ) с оператором L. Поэтому по аналогии с теоремой 1 было бы естественно обратиться к функции д(х) = Еж/(ХТас) B4) и посмотреть, будет ли она удовлетворять соотношениям B2) и B3). Для того чтобы реализовать этот план, надо прежде всего позаботиться о су- существовании решения уравнения B0). Для этого достаточно, например, чтобы ко- коэффициенты Ъ и а удовлетворяли при некотором М > 0 условию Липшица \Ъ(х) - Ъ(у)\ + \а(х) - <т(у)\ ^ М\х - 2/|, х,у G Mm, B5) где норма | • | понимается соответствующим образом для векторов и для матриц. При этом вместо условий на а можно потребовать следующее: все элементы мат- матрицы а входят в класс G^(Мт). Чтобы функция д в B4) имела смысл, естественно также предполагать, что Px(rdG < оо) = 1 для всех х G G, B6) где TdG — момент первого выхода процесса X — {Xt, t ^ 0} на границу области G. (Определение "регулярности" точек границы, данное выше для броуновского дви- движения, сохраняется и для диффузионных процессов.) К сожалению, в отличие от броуновского движения (когда L = А/2) указан- указанный выше подход в общем случае не приводит к решению задачи B2)-B3). Тем не менее, при некотором видоизменении этой задачи, изложенные выше идеи позво- позволяют получить "вероятностное" решение так называемой "стохастической задачи Дирихле" Определение 3. Локально ограниченная измеримая функция g называется X-гармонической в области G С Ж171 для диффузионного процесса X, если 9(х) = ?.хди при всех х G G и всех ограниченных открытых множеств U Э ж, U С G.
Приложение 7. Вероятностное решение задачи Дирихле 363 Пусть / = f(x) — некоторая ограниченная измеримая функция, определенная для х Е 3G. Будем говорить, что функция д = д{х), х Е G, является решением "стохастической задачи Дирихле", если д является Х-гармонической функцией в G B7) и lim g(Xt) = f(XTQG) Рж-п.н. для х Е G. B8) Теорема 6 (см. [169; с. 139]). Пусть / — ограниченная измеримая функция на 8G, где область G С Ж171. Тогда существует решение стохастической зада- задачи Дирихле B7)-B8), которое задается формулой B4). Решение стохастичес- стохастической задачи Дирихле единственно в том смысле, что если g — ограниченная на G функция, удовлетворяющая B7) и B8), то g задается формулой B4). Приведем один результат, показывающий, что при определенных условиях ре- решение стохастической задачи Дирихле будет являться также решением ис- исходной задачи Дирихле B2)-B3). Для целого неотрицательного к и а Е @,1] пусть Ск+а (G) обозначает множество действительных функций, заданных в области G С Ж171 таких, что существуют все их частные производные до порядка к включительно и к-я производная удовлетво- удовлетворяет условию Гёльдера с показателем а. По-прежнему будем предполагать выпол- выполненными упоминавшиеся условия, обеспечивающие существование решения уравне- уравнения B0), в том числе условие B5). Теорема 7 ([169; с. 141]). Пусть L — равномерно эллиптический оператор в области G, т. е. для х Е G все собственные значения матрицы (а^-(ж)) отде- отделены от нуля некоторым положительным числом. Пусть / — ограниченная непрерывная на 8G функция такая, что / Е C2+a(G) при некотором а Е @,1). Тогда формула B4) дает решение задачи Дирихле B2)-B3). Броуновское движение и диффузионные процессы позволяют получать "вероят- "вероятностные" решения не только задачи Дирихле, но и многих других классических за- задач теории дифференциальных уравнений с частными производными (задача Коши, задача Стефана и др.). Приведем один известный результат, относящийся к этому направлению. Теорема 8 (формула Фейнмана-Каца). Пусть функция / E С^Ж171) и u(t,x) = Ex(^f(Xt)exp{- где q — ограниченная непрерывная функция. Тогда ди -=Au-qu, где А — генератор диффузионного процесса X = {Xt, t ^ 0}. Подробные сведения о затронутых выше проблемах, а также о вероятностном подходе к другим уравнениям в частных производных см., например, в [5, 95, 148, 169].
Приложение 8 Большие уклонения § 1. Во многих интересных задачах приходится рассматривать семейства очень маловероятных событий. Так, в § 19 главы IV мы видели, что при определен- определенных условиях вероятность разорения компании, имеющей начальный капитал уо, убывает (по у о) экспоненциально быстро. Оказывается, в ряде случаев удается идентифицировать показатель экспоненты, задающий скорость убывания (по неко- некоторому параметру) изучаемой вероятности. Термин большие уклонения возник в связи с изучением сумм независимых слагаемых. Если Xi,X2,... — после- последовательность независимых одинаково распределенных центрированных величин и Sn = Х\ + • • • + Хп (п Е N), то большими уклонениями называют события, состоящие в отклонении Sn от нуля (т. е. от среднего) на величину порядка п. Иначе говоря, в отличие от нормальных уклонений, т.е. уклонений порядка л/п, фигу- фигурирующих в центральной предельной теореме, речь идет о вероятностях событий вида {Sn ^ хп}, {Sn ^ —хп}, {\Sn\ ^ хп}, где х > 0. Так, классическая теорема Крамера (см., например, [115]) описывает при определенных условиях на производя- производящую функцию моментов Ф(t) = Е exp{?Xi},rae t G М, асимптотическое поведение*) A/п) log P(Sn ^ жп)прип —у оо для х > 0. Эквивалентным образом можно сказать, что изучается A/п) log P(Sn/n G F), где замкнутое множество F есть полупрямая [ж, оо). Разумеется, можно ставить аналогичную задачу о вероятности попадания в множество В С Ш. сумм Sn при нормировках, отличных от 1/п. Ответ будет зависеть от выбора нормировки, от распределения слагаемых и от множества В. Излагаемый ниже подход к исследованию логарифмической асимптотики ве- вероятностей семейств определенных событий проводится в рамках общей теории функциональных предельных теорем: будут рассматриваться семейства мер в мет- метрических пространствах. Вводимая далее функция уклонений играет ключевую роль при описании упомянутой асимптотики. При этом вместо сумм случайных величин можно изучать семейства (должным образом нормированных) случайных процессов. Мы также покажем, что функциональный закон повторного логарифма в форме Штрассена теснейшим образом связан с распределением вероятностей процессов {y/eW(t), t G [0,Г]} при г -У 0, где W — винеровский процесс. Для этого будет установлена теорема Шильдера. В свою очередь, для ее доказательства потребуются свойства функции уклонений, а также вспомогательные результаты, относящиеся к поведению m-мерного винеровского процесса. В заключение этого раздела кратко рассматриваются и некоторые другие обобщения закона повторного логарифма. ^В этом разделе натуральный логарифм будем обозначать log.
Приложение 8. Большие уклонения 365 § 2. Всюду далее S — польское пространство с метрикой р. Напомним Определение 1. Функция /: S —У [—оо, оо] называется полунепрерывной снизу в точке х Е S, если для любой последовательности {xn}n^i точек из S такой, что хп —У х при п —У оо, верно неравенство liminf /On) ^ /(ж). п—ь-оо Легко видеть, что это свойство равносильно соотношению lim inf f(y) = /(ж), где Уе(ж) = {у е S:p(y,x) <s},s> 0. Говорят, что функция /: S —> [—оо, оо] полунепрерывна снизу, если она полуне- полунепрерывна снизу в каждой точке х Е S. Нетрудно показать, что / полунепрерывна снизу тогда и только тогда, когда для любого cG 1 множество {х: f(x) ^ с} явля- является замкнутым. Определение 2. Функция /: S —У [0, оо] называется функцией уклонений, если 1)/^оо; 2) / полунепрерывна снизу; 3) для любого с G [0, оо) множество {х : 1{х) ^ с} есть компакт. Заметим, что свойство 2) вытекает из 3) и включается в определение лишь по тра- традиции. Определение 3. Семейство (вероятностью) мер {Ре}е>о, заданных на (S, удовлетворяет принципу больших уклонений с функцией уклонений /, если 1°. lim sup e log P? (F) ^ —I(F) для любого замкнутого множества F, 2°. liminf slog Pe(G) ^ -/(G) для любого открытого множества G, где := inf /(ж), Б С S. В частности, если для Б G inf I(x) = inf I(x), во v ; xe[B] v ; где 5° и [5] обозначают соответственно внутренность и замыкание множества 5, то из 1° и 2° вытекает, что Ре(Я) =ехр{--(/(?)+оA)I при е|0. Таким образом, если 0 < 1(В) < оо, то экспоненциально быстрое убывание Р? (В) при е \. О фактически описывает функция ехр{—1(В)/е}. Определение 3 родственно определению слабой сходимости вероятностных мер (см. теорему 1 главы V). Заметим также, что используется модификация опреде- определения 3, в которой вместо множителя е фигурирует скорость v(e), т.е. некоторая положительная функция v{e) —У 0 при е \. 0. Приведем аналог понятия плотности семейства мер.
366 А. В. Булинский, А. Н. Ширяев Определение 4. Семейство мер {Pn}n^i называется экспоненциально плот- плотным, если для любого М > 0 существует компакт Км С S такой, что Hmsup-logPn(S \КМ) ^ -М. п—Ьоо ТЬ Нетрудно видеть, что если семейство мер {Pn}n^i удовлетворяет принципу боль- больших уклонений (в определении 3 вместо г берется 1/п и вместо Р? — мера Рп), то оно экспоненциально плотно. Следующие два результата выявляют связь принципа больших уклонений для се- семейства мер с исследованием предельного поведения интегралов по этим мерам от определенных семейств функций. Первый из упомянутых результатов (см., напри- например, [115; с. 32]) называют леммой Варадана. Теорема 1 (Варадан). Пусть семейство мер {Pn}n^i удовлетворяет прин- принципу больших уклонений (с функцией уклонений I). Пусть функция Н: S —У Ж является непрерывной и ограниченной сверху. Тогда lim - log f enH^ Pn(dx) = sup(tf (ж) - I(x)). Результат, обратный к лемме Варадана, содержит Теорема 2 (Брик, [104]). Пусть семейство мер {Pn}n^i экспоненциально плотно. Пусть для всех Я Е Cb(S) ЛП(Я) := — log / епН^ Pn(dx) ->• Л(Я) G Ж при п ->• оо. ™ is Тогда {Pn}n^i удовлетворяет принципу больших уклонений с функцией укло- уклонений I(x)= sup (Я(ж)-Л(Я)). #<EC&(S) О больших уклонениях семейства мер в топологических пространствах см., на- например, [114]. § 3. Чтобы привести содержательный пример функции уклонений (а также семей- семейства мер, удовлетворяющих принципу больших уклонений), который будет исполь- использован далее, нам понадобится простое утверждение. Для В С S и S > 0 пусть, как обычно, В5 обозначает ^-окрестность множества В, т.е. В5 = {х е S: р(х,В) < S}, здесь р(х, В) = inf{p(x,y): у G В}. Лемма 1. Если I — функция уклонений и F — замкнутое подмножество S, то I(F5) -+ I(F) при 510.
Приложение 8. Большие уклонения 367 Доказательство. Очевидно, I(F5) = inf I(x) <С inf I(x) = /(F), причем функция I(FS) не возрастает на [0, оо]. Допустим, I(F) < оо. Тогда для лю- любого п е N найдется точка хп е F1/71 такая, что I(xn) ^ /(F1/71) + 1/n ^ /(F) + 1. Из свойства 3) в определении 2 вытекает, что последовательность {xn}n^i принад- принадлежит некоторому компакту V в S. Следовательно, существует х Е V и подпосле- подпоследовательность {п7} такая, что хп. —У х при j —>¦ оо. Поэтому liminf 1{хп.) ^ /(ж). J j —^ оо 3 Кроме того, ж Е F, поскольку жп. Е F 'пэ для^' Е N. Таким образом, I(F) ^I(x) ^liminf/Ц.) : = liminf I(F5) ^ limsupiXF*) ^ I(F). A) Пусть теперь I(F) = оо, т.е. /(ж) = оо для каждого ж Е F. Допустим, что {/(F1/n)}n^i не стремится к оо при п —у оо. Тогда найдутся М > 0 и последо- последовательность {^j}j^i такие, что /(.F1/7^) ^ М при всех j G N. Дальнейшие рассуж- рассуждения совершенно аналогичны оценкам в A). Лемма доказана. ? §4. Введем на пространстве S = С([0,Т]; Ж171) функцию (функционал) и \ _ ; g / 1^1 "^ если ср — абсолютно непрерывная функция, <р@) =0, оо в противном случае, B) здесь | • | для вектора в Ж171 обозначает евклидову норму, ф{Ь) = (ф^^),..., (^^m^(t)), а абсолютная непрерывность вектор-функции ср означает, что при t G [0,Т], к = t Лемма 2. Введенная функция I является функцией уклонений на S. Доказательство. Условие 1) определения 3, очевидно, выполнено. Проверим справедливость условия 3). Докажем, что множество Sm = {^ G S: I(cp) ^ M}, где М ^ 0, есть компакт в S. Для ср G Sm и s, t G [О, Т], воспользовавшись неравенством Коши-Буняковского-Шварца, имеем I A \ip(t) -4>{s)\ = / ф(и)о1и at \ 1/2 |^M|2di/J |t-.|1/2^BM|t-.|I/2, C) где интеграл от вектор-функции, как обычно, берется покомпонентно. Следовательно, \(f(t)\ ^ BMTI/2 при t G [0,Т]. Таким образом, множество состоит из функций, являющихся равномерно ограниченными и равностепенно
368 А. В. Булинский, А. Н. Ширяев непрерывными. По теореме Арцела-Асколи замыкание множества 5м в S есть ком- компакт. Осталось проверить, что 5м замкнуто. Для ip Е S и п Е N введем функционалы п П )|2. D) к = 1 Возьмем замкнутое множество S'0) = {чр Е S: ip@) = 0} и положим ( sup Jn(ip), y> J(<p) := { ^N E) I oo, <^GS\S(°). Из D) вытекает, что G(cp) = sup Jn((p) является полунепрерывной снизу функцией на S (как верхняя грань непрерывных функций). Имеем J(ip) = G(cp) для ср G S^0^ и J{ip) = оо для ф G S\S^0^. Поэтому, очевидно, J — полунепрерывная снизу функция на S^0). Таким образом, для справедливости свойства 3) достаточно доказать, что J = I на S. F) Вначале убедимся, что если 1{ф) < оо, то J(ip) < оо (ср G S^0)). Аналогично C) получаем Гкт/п п " Гкт = w 22 / 2 / ^W du = l J(k-l)T/n 2 G) Следовательно, в рассматриваемом случае J(<p) ^ /(^) < оо. Пусть теперь J(<p) < оо, где ср G S^0^. Далее срп (п G N) обозначает кусочно линейную функцию, имеющую узлы интерполяции (кТ/п, ip(kT/n)), к = 0,..., п. Очевидно, Пусть ф G Co°([0,T];Mm). Учитывая равномерную сходимость ipn к ip при п —>- оо, имеем / Jo = lim / где (•, •) — скалярное произведение в Mm. Формула интегрирования по частям дает г / Jo = - Hm Принимая во внимание (8), получаем т [ а
Приложение 8. Большие уклонения 369 здесь || • ||L2 — норма в пространстве L2[0,T], состоящем из (классов эквивалент- эквивалентных) вектор-функций со значениями в W71, имеющих компоненты, интегрируемые в квадрате по мере Лебега. Итак, на пространстве Cq°([0, Т]; Mm), всюду плотном в L2[0, T], возник ограниченный линейный функционал / o = / (<p(t)Mt))dt, (9) Jo который по теореме Хана-Банаха (см., например, [35; гл. IV, § 4, п. 3]) продолжается на L2[0,T] с сохранением нормы. Согласно лемме Рисса (см., например, [60; т. 1, с. 57]) т (ct(i), ip(i)) dt о для некоторого элемента а е L2[0,T]. Поскольку L2[0, Г] С L^O, T], то, интегрируя по частям, для ф е Cq° ([0, Г]; Ж171) получаем = - (/ a(u)du,il>(t))dt. A0) ^о \Jo I Сопоставляя (9) и A0), заключаем, что = - f a{u)du, te [0,T]. Jo Следовательно, функция if абсолютно непрерывна и 1{ф) < оо. Итак, мы доказали, что {if G S: I{if) < 00} = {if G S: J((f) < 00}. Множество ?f — {if G S: I (if) < 00} называется пространством Камерона- Мартина. Проверим, что I(if) = J(if) для if G 5f}. Пусть if G C°°([0,T]; Mm). Тогда sup |(/3^'^(t)| = Cj < 00, j = 1,... ,m. Поэто- tG[0,T] му, пользуясь формулой Тейлора, получаем n m т / \ n V^ V^l Поскольку 0 e C([0, Г]; Em), то ^^|2^i / |^)|2di<oo, n^^. A1) Из E), (8) и A1) вытекает, что J{<p)=I{<p) для ^eC°°([0,T];]Rm), <^@)=0. A2)
370 А. В. Булинский, А. Н. Ширяев Если (р е У, то для любого г > 0 найдется функция (р^ е С°°([0, Г]; Ж171) такая, ^?)@) =0и Пользуясь свойством нормы (в L2[0, T]), имеем поэтому с учетом G) и (8) находим, что Отсюда легко выводим, что Таким образом, для (/?^Ув силу A3)—A4) приходим к оценке что завершает доказательство совпадения / и J на У. Лемма доказана. ? §5. В этом параграфе устанавливаются некоторые свойства m-мерного вине- ровского процесса W = {W(t), t ^ 0}, заданного на вероятностном пространстве Лемма 3. Для любых j,v > 0 верно неравенство Р( sup |VF(t)| ^ A5) Доказательство. Для# G Mm введем процесс Легко проверить, что {X^(t), t ^ 0} является мартингалом относительно естествен- естественной фильтрации винеровского процесса W. Для любого Л > 0 имеем P( sup (W(t), ^te[o,v] sup exp\x(W(t)J0)-lx2\0\2t}^exp\x1-l\2\0\2v}) = e[o,v] I 2 J I 2 J/ te[o sup tG[0,v]
Приложение 8. Большие уклонения 371 Пусть \в\ = 1. Тогда по следствию 5 главы IV последняя вероятность не превосходит где учтено, что минимум функции д(Х) = — Л7 + \^?v достигается в точке Л = j/v. Если 0^ обозначает единичный вектор к-и. координатной оси (к = 1,..., т), то т / ч sup \W(t)\ ^7) ^ 5^Р( sup |(W(?)A)| ^-^=) *G[O,v] У г._-, \tG[O,v] V771/ V-1 = ± tG[O,v] 2?тг sup Р( sup (W(t),O) ^ —= ) ^ 2me~^mv . П q. \0\-\ \ + a\Ci пЛ \/m t Лемма 4. Пусть детерминированная вектор-функция a G Сг1([0,Т]; Ж171). Тогда Г Г (a(u),dW(u)) = (a(T),W(T))- (a(u),W(u))du. A6) ./о Доказательство. Очевидно, достаточно рассмотреть случай т = 1. В силу леммы 4 главы VIII a(u)dW(u) = l.i.m. V a(tk)(W(tk+1) - W(tk)), A7) где точки tfc = kT/n, к = 0,..., п (п G N). Имеем п—1 п—1 J] a(*fc)(W(*fc+i) " ^(*fc)) = a(T)W(T) - J2 W(tk)(a(tk+1) - a(tk)) = k=0 fe=0 = a(T)W(T) -J2 W(tk)a(u) du. 1„ r\ J t U Учитывая независимось приращений винеровского процесса, видим, что е([ a(u)dW(u)-a(T)W(T)+ f a(u)W(u)du\ = = lim E( V / k+\w(u) - W(tk))a(u)du] = n—юо \^~^ Ji J = lim >ЛЕ( / (W(u)-W(tk))a(u)du lim VE (W(u) - W(tk)J du / a(uJ du = ^ /•** + ! /•** + ! 9 = lim > / (u — tk)du / d(u) du = n^oo ^^ I. . k=OJtk Jtk = Ит У fa+i tk? f+ h{uJdu= lim ^! [a(uJdu = 0. ?
372 А. В. Булинский, А. Н. Ширяев Нам понадобится также частный случай теоремы Гирсанова (теорема 18 гла- главы VIII). Лемма 5. Пусть детерминированная вектор-функция a Е С([0, Т]; Ж171). Тог- Тогда процесс B(t) = W(t) - f a(u) du, t e [О, Г], Jo A8) является винеровским процессом на вероятностном пространстве (П,^, Р), где Е1АХТ, Ае&, A9) Иначе говоря, ЫРw dP/dP = ХТ. Доказательство. Достаточно установить, что при любых п ^ 1, 0 = < ... < tn =ТиАь...,Ап еЖш п~г п~г Г 1 1),\n)}= Д expj-- B0) где Е обозначает усреднение по мере Р. Левую часть формулы B0) с учетом A9) запишем в виде f ^z- ехр< i у = exp{- *П fc=O l x ) ( ^ if E exp (*fe+i) |a(«)| -W(tk), 71- 2 du — iy Г ftk+1 , [It ^a{u) + 1 Afc+i> " ( ^Ц a i\k+iidW( ltk (u) du B1) ) Пользуясь соотношением A7), получаем, что & = /" "+1(a(«),rf^(«)) ~ n(o, f k+1 \a(u)\2du], B2) •i.m.Cfc,r, B3) -t —^, j = 0,..., r. Jtk где j=o
Приложение 8. Большие уклонения 373 Для любых и, v Е Ш. имеем 2а2 B4) поскольку этот интеграл равен е 2 Eetriv, где r\ ~ 1\1(сг2/м, сг2). Теперь заметим, что |Е е (Е exp{2|Cfe| + 2|Cfc - С учетом B2) и B3), принимая во внимание гауссовость величин Ос,г5 нетрудно по- получить, что Е exp{0J = lim E exp{(kr}. г—юо Пользуясь B4), находим П&=1 Е ехр{?&} и заключаем, что выражение B1) совпада- совпадает с правой частью формулы B0). Это и требовалось показать. ? §6. Пусть теперь W = {W(t), t G [0,Т]} есть m-мерный винеровский процесс, заданный на некотором вероятностном пространстве (П,^, Р) и отрезке [0,Т]. Для г > 0 на борелевской сг-алгебре пространства S = С([0, Г]; Мт), снабженного згф-нормой || • ||, введем меры Q?(B) = Be B5) Теорема 3 (Шильдер). Семейство мер {Q?}s>o удовлетворяет принципу больших уклонений с функцией уклонений I вида B). Доказательство. Проверим соотношение 1° определения 3. Возьмем любое замкнутое множество F С S. Пусть ipn для ip G S по-прежнему обозначает кусочно линейную функцию с узлами (кТ/п, ср(кТ/п)), к = 0,..., п. При каждых п G N и 5 > 0 имеем F = {if G F: У - <р - <рп\\ ^ 5}. Очевидно, где FcAn(S)UCn(S), 5, CnE) = & G S: B6) АпE) = {^ G S: ipn G F5} Положим L5 = тт{/(^5), 1/ё}. Тогда Q?(^ G S: I(<pn) 2 LS) = k = w\ — \-w П n ) = -!, BL8)/e 2 2 Г(^
374 А. В. Булинский, А. Н. Ширяев здесь учтено B5) и то, что ^ ^ W ( Ц?- J — W (^ ~п ' J имеет хи-квадрат распре- распределение с числом степеней свободы пт. Для z > 0 и \i Е Ш. получаем J(z, ц) = [°° и»е-% du = 2^e"f + 2/i [°° и^е'? du <: 2^e"t + —J(z, /i). J % z Поэтому J(z,/i) ^ 4zMe~2 при каждом \i Gln^> 4/i. Следовательно, при каждых n G N, S > 0 и всех достаточно малых s > 0 справедливо неравенство elogQ?(,4n(<5)) ^ -Ls+enmlog(BLs)/e)+elog4-elogB^r(^\\ B8) Отсюда -L5. B9) В силу леммы 3 при всех nGN, 5>0иг>0 получаем — + t) - w(—) U ?= n J \nj\ 2-fe max sup S \2 n Выберем n = n(S, T, m) так, чтобы S2n/A6mT) > L$. Тогда -2L6. C0) Принимая во внимание, что для любых а, Ъ > 0 log(a + 6) ^ logBmax{a,6}) = log 2 + max{loga,logfr}, из B6), B9) и C0) получаем при всех 8 > 0 оценку limsupelogQe(F) ^ -L^. C1) В силу леммы 1 из C1) вытекает требуемое свойство 1°. Возьмем открытое множество G С S и проверим свойство 2° определения 3. Если I(G) = оо, то 2°, очевидно, справедливо. Поэтому далее I(G) < oo. Для любого a > 0 найдется такая функция ^a GG, что /(</>«) < /(G) + а. C2) Поскольку G — открытое множество, то VTa (фа) С G для некоторого га > 0, где Vr(^>) = {<?> G S: ||у — f/'H < г}. Для каждого/? € @,а) нетрудно выбрать функцию <^ е С2([О, Г]; Жт) с <^@) = 0 так, чтобы J1/2^ - у») < /З/л/2. Тогда + Р. C3)
Приложение 8. Большие уклонения 375 Кроме того, для t e [О, Г] Поэтому / \фа(и)-ф Jo Таким образом, при /Зу/Т < га/2 имеем Vs(y>) С G для 8 < га/2. Следовательно, Qe(G) ^ Qe(Vi(^)) = P(V?W е V*(?>)) = P(\\W - tp/V^W < S/Vi). C4) Заметим, что ft ip(t) = / ф(и) du. Jo Поэтому по лемме 5, используя A6) и A8), правую часть формулы C4) можно пере- переписать в виде C5) <:h\\W\\, C6) Далее, в силу A6) {ф{и),(Ш{и)) (ф(и), W(u)) du где Л=A sup te[o,T] Из C5) и C6) получаем оценки Qe(G) 2 ехр{ Таким образом, Тогда < 5/у/ё). e\ogP(\\W\\ < lim inf e log Q? (G) ^ -I(<p). C7) Учитывая C2), C3) и C7), приходим к требуемому свойству 2°. Теорема установ- установлена. ? § 7. В этом параграфе доказывается функциональный закон повторного лога- логарифма в форме Штрассена. Для этого потребуется вспомогательный результат. Пусть W = {W(?), t ^ 0} есть m-мерный винеровский процесс, заданный на неко- некотором вероятностном пространстве (П, ^, Р). Введем семейство случайных функций W(r)t) fn(t)= r-\/,, nen, te[o,T], C8) где ф{1) = ф{2) = 1 и ф(п) = v^loglogn, п ^ 3, а вектор-функция умножается на скаляр покомпонентно. Для изучения предельных точек этого семейства понадобится следующая лемма.
376 А. В. Булинский, А. Н. Ширяев Лемма 6. Пусть детерминированная вектор-функция g Е С([0, оо); Ж171) и ^y ie[0'T]' пек C9) Пусть множество А С A,оо) и точка 1 принадлежит замыканию А. Предпо- ложим, что для любого A Е Л limsup^nr(A),y) = 0, D0) г—юо где V — некоторый компакт в пространстве S = C([0,T]; Мт), пг(Л) = [Аг], г Е N, [ • ] — целая часть числа, p(f, V) = inf{p(f, h): h G У}, /? — равномерная метрика в S. Тогда lim p@n,VO = O D1) n>oo n—>-oo г/ множество функций {gn}n^i предкомпактно в S. Доказательство. Если V — компакт в S, то по теореме Арцела-Асколи сущест- существует М > 0 такое, что sup 11/11 ^ М D2) fev и найдется функция А: [0, Т] —у [0, оо) такая, что Д (s) ^ 0 при s ^ 0 и sup \f(t) - f(s)\ <: A(t -s), 0 ^ s,t <: Г, D3) / здесь, как и ранее, || • || есть згф-нормав S. Возьмем A G Лип G N. Найдем г = г(А,п) такое, что пг (А) < п ^ nr+i(A) =: TV, где TV = N(X, п) и п0(А) = 0. Тогда \\9п-9м\\= sup [ gN(nt/N) T 9N(nt/N) - gN(t) ? te[o,T] h(AT\ \ -9N(t)\. D4) пф(п) tG[0,T] Из D0) вытекает, что при каждом A G Л для любого е > 0 найдутся функции vnr{\) ? ^ такие, что ? при г ^ JV0(?,A). D5) Отсюда и из D2) следует, что при всех достаточно больших N \\gN\\ ^М + е. Кроме того, при всех п > JVi = Ni (A) кад-ц. D6)
Приложение 8. Большие уклонения 377 Теперь заметим, что согласно D5) и D3) при всех достаточно больших п имеем sup \gN (nt/N) - gN(t) | <C sup \gN(nt/N) - gN(t) ± vN(nt/N) ± vN (t) \ ^ te[o,T] te[o,T] ^ 2e + sup te[o,T] поскольку nt Выбрав A E Л достаточно близким к единице, получим, что справедливо D1). Возьмем любую подпоследовательность {gnj}5 гае {fij} С N. По доказанному найдутся hn. G V такие, что p(gnj •> hnj) —> О ПРИ j —> оо. Можно извлечь подпосле- подпоследовательность {n'j} С {fij} такую, что hn/ —у h G У, j —> оо. Тогда gn/ —у h при j -у оо. П Теорема 4 (Штрассен). С вероятностью единица для семейства случайных функций вида C8) справедливы соотношения: 1) lim p(fn, К) = 0, где К — шар Штрассена, т. е. п—юо К = У е S: /fa) ^ 1/2} = L G У: У |y.(«)|2d« ^ lj, а функция I определена в B); 2) дл^ любой функции / G К и п. в. w G П найдутся последовательности {rij(u;)} С N такие, что .lim p(fnj^)J) = 0. Иначе говоря, почти наверное семейство функций {/п}п^1 предкомпактно в S г/ совокупность предельных точек этого семейства совпадает с множест- множеством К. Доказательство. Дляй > 0 рассмотрим множество 5A+E)/2 = {^ G S: < A + S)/2}. Если (^ G 5A+E)/2 и (^ ^ К, то существует г G [О, Г] такое, что f Jo Возьмем функцию Тогда ф G К и arT \l/2 \ф{ц)\2йи) (T-z)^2<VST. D7)
378 А. В. Булинский, А. Н. Ширяев Итак, для л/^Г-окрестности шара К имеем К с 5A+5)/2 С К^\ D8) Для замкнутого множества F = S \ К*5Т по теореме Шильдера limsupelogQe(F) ^ - т. е. при любом 7 > 0 и всех достаточно малых ? > О J D9) здесь Q? — распределение в S процесса л/sW, где W = {W(t),t e [О, Г]}. В силу D8) имеем F С S^i+s)/2 = S \ S(i+$y2- Поэтому E0) Таким образом, при j = S/4 из D9) и E0) имеем Заметим, что распределение случайной функции fn вида C8) есть Q?n, где еп — = 1/Ф2(п). Следовательно, для любого Л > 1 и пг(Л) = [Лг], г G N, получаем Я(А) t K4^) ^ E (logn^A))-1-^ < оо, Ar° ^ 3. По лемме Б оре ля-Канте л ли для п. в. и G П имеем /Пг(л) ? К^тприг > Мо(о;,А, Иначе говоря, п. н. limsup^(/nr(A),K) = 0. В силу леммы б утверждение 1) теоремы Штрассена доказано. Докажем утверждение 2). Достаточно рассматривать лишь / G К, для которых /(/) < 1/2. Действительно, если /(/) = 1/2, то для 5 G @,1) возьмем точку и G [О, Т] такую, что [U\f(s)\2ds = l-6. Jo Тогда аналогично D7) находим, что sup \f(t) — h(t) \ ^ \/~5T для функции h такой, te[o,T] что h(t) = f(t) при t G [0, и] и h(t) = f(u) при t G [гх, Г]. Для последовательности {kr}r^i, где А: ^ 2, имеем II/*- - /11[о,т] < ||/И1[о,т//ё] + ||/||[o,T/fe] + ИЛ- - f\\[T/k,T], E1)
Приложение 8. Большие уклонения 379 здесь ||д||[а5ь] для д е С ([О, Г]; Ж171) и [a, b] С [О, Г] обозначает норму в пространстве С ([a, b]; W71) сужения функции д на отрезок [a, b]. Очевидно, для любого г > 0 и любой функции / Е К имеем ||/||[о,т//с] ^ V^/k < ? при к > Т/г2. В силу доказанного утверждения 1) для почти всех и Е П и каждого к ^ 2 в К найдутся функции gfcr (?) (зависящие и от о;) такие, что Wfkr ~9кг\\[о,т] <? при r>M0(s,k,uj) Поэтому при к > Т/г2 и г > М0(г, к, и) \\1кг\\[о,т/к] ^ \\9кг\\[о,т/к] + ИЛ»- -Pfcr||[0,T] Для /с ^ 2 и ? G [О, Т] положим E2) о, te[o,T/k], f(t)-f(T/k), te(T/k,T\. Введем случайные функции te [ , t G (T/k,T]. Тогда Очевидно, при всех достаточно больших к Далее, для п. в. со G л/кф(кт 2e)/Vk, так как ф(кг 1)/ф(кг) —>- 1 при г —>¦ оо, а в силу E2) с вероятностью единица ||/fcr-i ||[о,т] < 1 + ? Для всех достаточно больших г. При каждом к ^ 2 события {||#г,/с ~ /(* ? ^)||[т//с,т] < ^11? ^ ^ N, независимы в совокупности, поскольку они определяются приращениями винеровского процесса на непересекающихся промежутках времени. Используя марковское свойство m-мерного винеровского процесса (совершенно аналогичное теореме 2 главы III), получаем для S > О VtG[O, W(tkr) -ip(t,k)
380 А. В. Булинский, А. Н. Ширяев здесь v = ГA - 1/fc), (p(t, к) = f(t + jr) - /(^) при t e [0, и], открытое множество G = {^ G C([0,t;];Rm): ||^( •) - <р( • ,*011[о,«] < (^амераС^ t G [0, ?;]}), где гп = 1/ф2(п),п еК Заметим, что / \<p(t,k)\2dt ^ 1-a, aG@,l). ./о По теореме Шильдера (для мер Q^ на пространстве С([0, v]; W71) liminf elogQW(G) ^ -I{v)(G), где /(v) — функция уклонений. Поэтому для любого j > 0 и всех достаточно малых г xpj- —(l-a + 27) Следовательно, при j = а/4 Поэтому г=1 Согласно лемме Боре ля-Канте л ли для п. в. cj G П найдется последовательность {rj (и)} такая, что \\9rj(u;),k ~ 1(-,Щ[Т/к,Т] < S- Учитывая проведенные выше оценки слагаемых, фигурирующих в правой части фор- формулы E1), приходим к искомому утверждению 2). ? §8. Рассмотрим некоторые другие обобщения закона повторного логарифма. Прежде всего, по аналогии с принципом инвариантности (теорема 8 главы V) введем в пространстве С[0,1] случайные ломаные, построенные следующим образом по последовательности независимых одинаково распределенных величин Х\, Х2,..., имеющих нулевое среднее и единичную дисперсию. Пусть (к/'п, Sn/'(у/п ф(п))) — узлы интерполяции кусочно линейной (случайной) функции hn (t), где, по-прежнему, Sn =Xi + ... + Xn,0(l) =фB) = 1,ф(п) = V2 Иначе говоря, при построении случайных ломаных hn(t) вместо нормировки у/п, отвечающей центральной предельной теореме, использована нормировка, присущая закону повторного логарифма.
Приложение 8. Большие уклонения 381 С помощью теоремы 13 главы III устанавливается следующий результат. Теорема 5 (Штрассен). Для введенного семейства случайных ломаных {hn}n^i справедливы все утверждения теоремы 4, относившиеся к семейству случайных функций {/n}n^i5 построенных по винеровскому процессу. 1. Используя теорему 5, получите теорему Хартмана—Винтнера: Пусть Х\,Х2, - - - — независимые одинаково распределенные величи- величины с EXi = 0, EXf = 1. Тогда с вероятностью единица множество предельных точек {Sn/y/2n log log n }п^з совпадает с отрезком [—1,1] (здесь Sn = Хг + --- + Хп). 2. Докажите, что если {Х^}, {Sk} — те же, что в упражнении 1, и /(?) — дей- действительная интегрируемая по Риману функция на [0,1], то п. н. п /АЛ /Г1 \ limsupBn3loglogn)-1/2 V/ - )Sk= / F2{u)du) 1/2 где F(u) = / f(t)dt, ue [0,1]. J и Определение 5. Детерминированная положительная функция Ф(?) /* оо при t —У оо (? Е [0, оо)) называется верхней функцией винеровского процесса W = = {W(t), t ^ 0}, если для почти всех lj G 1) существует N = N(uj) > 0 такое, что W(t, uS) ^ Ф(?) при всех t ^ N. Если для почти всех cj G П функция Ф не является верхней, то она называется нижней функцией. 3. Может ли быть так, что свойство, заложенное в определение верхней (нижней) функции винеровского процесса выполнено для точек и G А, где О < ?{А) < 1? Приведем знаменитый интегральный критерий Колмогорова-Петровского (см., например, [31]); говорят также: интегральный критерий Колмогорова-Пет- ровского-Эрдёша-Феллера, имея в виду и аналог упомянутого результата для флуктуации последовательности частных сумм независимых слагаемых). Теорема 6. Пусть (f(t), t > 0, — неубывающая функция такая, что cp(t) —у оо при t —У оо. Тогда функция Ф(?) = уД(р(?) будет верхней функцией винеровского процесса, если 1{ф) < оо; и — нижней, если 1{ф) = оо, где = Г ^Г exp{-<p(tJ/2} dt. E3) Эта теорема дает более тонкий результат, нежели обычный закон повторного ло- логарифма: вычисление 1(ср) показывает, что функции cp{t) = A + e)y/2t\og\ogt яв- являются верхними или нижними соответственно, когда е > 0 и s ^ 0 (теорема 8 главы III не охватывает случай е = 0).
382 А. В. Булинский, А. Н. Ширяев В связи с теоремой 4 естественно возникает вопрос: каково множество предельных точек более общего, нежели C8), семейства Fn(t)= { / , tG [0,1], neN, E4) где if — та же функция, что и в теореме б? Ответ дается с помощью функционала, обобщающего E3). При этом оказывается, что скорость сходимости к шару Штрас- сена необычайно чувствительна к выбору нормировки и даже добавление константы к используемой нормировке ip может изменить для семейства E4) (степенной) поря- порядок скорости сходимости к шару Штрассена (см. [9] и там же ссылки). Имеется много направлений, в которых происходило и происходит развитие зако- закона повторного логарифма. Например, о функциональном законе повторного логариф- логарифма в топологиях, отличных от равномерной, см. [48]. Упомянем также исследования, относящиеся к функциональному закону повторного логарифма для эмпирических процессов, для гауссовских процессов более общих, чем броуновское движение, для процессов частных сумм со значениями в абстрактных пространствах (в том числе для суперпроцессов), а также случайных полей.
Заключительные замечания Не расширяя чрезмерно список литературы, невозможно перечислить все замеча- замечательные книги, направленные на изложение вопросов, относящихся даже к "ядру" теории случайных процессов. Для удобства читателей из общего списка цитируе- цитируемых работ выделим учебную литературу по теории вероятностей: [4, 18, 34, 63, 68, 78, 85, 101, 106, 123, 131, 143, 189], а также — по теории случайных процессов: [12, 17, 25, 30, 32, 39, 42, 61, 72, 77, 83, 93, 100, 103, ПО, 125, 135, 142, 146, 149, 177, 185, 192]. Отдельно укажем на энциклопедию [57], справочники [37, 56], а также книгу контрпримеров [75] и задачник [55]. Отметим, что многие из приведенных выше книг посвящены одновременно и теории вероятностей и теории случайных процессов. Имеется ряд направлений современной теории случайных процессов и ее прило- приложений, с которыми будет легче познакомиться, завершив изучение обязательных ос- основ. Перечислим некоторые из этих направлений, не задаваясь целью упорядочить их, следуя моде и т. п. Прежде всего заметим, что наряду с аксиоматикой Колмогорова существуют дру- другие подходы к построению теории вероятностей. Известен подход, основанный на алгоритмическом понятии сложности (см., например, А.Н. Ширяев "Дополнение" к [34]). С другой стороны, большое внимание в настоящее время уделяется кванто- квантовой вероятности (в том числе квантовым случайным процессам), см., например, [88, 165,167,171]. Статистика случайных процессов (или анализ "временных рядов") — направле- направление исключительно важное, как для теории, так и для приложений. Об исследова- исследованиях в этой области можно прочитать в [6, 80, 134, 137, 160, 183, 191]. При этом от- отметим проблемы численных методов в теории случайных процессов, см., например, [102,144]. Обширное и традиционное направление — построение моделей, описывающих различные варианты зависимых наблюдений. В данном курсе рассматривались гауссовские, марковские процессы, а также мартингалы. Описание зависимости событий, сг-алгебр, систем случайных величин может производиться и в других терминах (перемешивание, ассоциированность, семиинварианты и т.д.) При тех или иных формах зависимости устанавливаются результаты как классического направления теории предельных теорем, так и нетрадиционного; см., например, [8, 28, 98,105,117,118,137,162,168,198]. Во многих вопросах существенную роль играют события малой вероятности. Об экстремумах случайных процессов см., например, [38, 45, 52]. С теорией больших уклонений можно ознакомиться, обратившись, например, к [114,115, 116,127,194]. Много интересных задач находится на стыке различных дисциплин. В качестве яркого иллюстрирующего примера была рассмотрена задача Дирихле. О различ-
384 А. В. Булинский, А. Н. Ширяев ных проблемах анализа и вероятности см., например, [95, 108, 121]. Прослеживается постоянная тенденция максимального обобщения результатов классической теории вероятностей благодаря рассмотрению случайных элементов со значениями в бесконечномерных пространствах, см., например, [140, 157]. Отдельного упоминания заслуживает огромное поле деятельности, предоставля- предоставляемое стохастической финансовой математикой. Еще раз сошлемся на книги [86, 147], где имеется обширная библиография. Кроме того, упомянем вероятностные задачи теории страхования и актуарной математики. По-прежнему, физика, химия, биология, другие естественные и гуманитарные на- науки, а также техника постоянно дают новые модели, требующие глубокого вероят- вероятностного анализа. В этой связи укажем, например, на исследования [10, 132, 176].
Список литературы 1. Афанасьева Л. Г., Булинская Е. В. Случайные процессы в теории массового обслуживания и управления запасами. - М.: Изд-во МГУ, 1980. 2. Биллингсли П. Сходимость вероятностных мер. -М.: Наука, 1977. 3. Боровков А. А. Математическая статистика. -М.: Наука, 1984. 4. Боровков А. А. Теория вероятностей. -3-е изд. -М.; Новосибирск: Эдитори- ал УРСС; Ин-т математики, 1999. 5. Бородин А. Н., Салминен П. Справочник по броуновскому движению. Факты и формулы. - СПб.: Лань, 2000. 6. Бриллинджер Д. Временные ряды. Обработка данных и теория. -М.: Мир, 1980. 7. Булинский А. В. Предельные теоремы для случайных процессов и полей. - М.: Изд-во МГУ, 1981. 8. Булинский А. В. Предельные теоремы в условиях слабой зависимости. - М.: Изд-во МГУ, 1989. 9. Булинский А. В., Лифшиц М. А. Скорость сходимости в функциональном за- законе повторного логарифма при нестандартных нормирующих множителях // УМН. - 1995. - Т. 50, № 5. - С. 83-102. 10. Ван Кампен Н. Г. Стохастические процессы в физике и химии. - М.: Высшая школа, 1990. 11. Ватанабэ С, Икэда Н. Стохастические дифференциальные уравнения и диф- диффузионные процессы. -М.: Наука, 1986. 12. Вентцелъ А. Д. Курс теории случайных процессов. - 2-е изд. - М.: Наука, 1996. 13. Вентцелъ Е. С, Овчаров Л. А. Теория случайных процессов и ее инженерные приложения. -М.: Наука, 1991. 14. Гапошкин В. Ф. Критерии усиленного закона больших чисел для классов стационарных в широком смысле процессов и однородных случайных полей // ТВП. - 1977. - Т. 22, № 2. - С. 295-319. 15. Георги Х.-О. Гиббсовские меры и фазовые переходы. -М.: Мир, 1992. 16. Гихман И. И., Скороход А. В. Теория случайных процессов. Т. 1-3. - М.: Наука, 1971,1973,1975. 17. Гихман И. И., Скороход А. В. Введение в теорию случайных процессов. - 2-е изд. -М.: Наука, 1977. 18. Гнеденко Б. В. Курс теории вероятностей, -б-еизд. -М.: Наука, 1988. 19. Гнеденко Б. В., Коваленко И. Н. Введение в теорию массового обслужива- обслуживания. - 2-е изд. - М.: Наука, 1987.
386 А. В. Булинский, А. Н. Ширяев 20. Гнеденко Б. В., Колмогоров А.Н. Предельные распределения для сумм неза- независимых случайных величин. — М.—Л.: Гостехиздат, 1949. 21. Добрушин Р. Л. Пример счетного однородного марковского процесса, все состояния которого являются мгновенными // ТВП. — 1956. — Т. 1, № 4. — С.481-485. 22. Добрушин Р. Л. Задание системы случайных величин при помощи условных распределений // ТВП. - 1970. - Т. 15, № 3. - С. 469-497. 23. Дынкин Е. Б. Марковские процессы. -М.: Физматгиз, 1963. 24. Дынкин Е. Б., Юшкевич А. А. Теоремы и задачи о процессах Маркова. - М.: Наука, 1967. 25. Дуб Дж. Л. Вероятностные процессы. -М.: ИЛ, 1956. 26. Жакод Ж., Ширяев А. Н. Предельные теоремы для случайных процессов. Т. 1, 2. - М.: Физмат лит, 1994. 27. Золотарев В.М. Современная теория суммирования независимых случайных величин. -М.: Наука, 1986. 28. Ибрагимов И. А., Линник Ю. В. Независимые и стационарно связанные вели- величины. -М.: Наука, 1965. 29. Ибрагимов И. А., Розанов Ю. А. Гауссовские случайные процессы. -М.: На- Наука, 1970. 30. Ито К. Вероятностные процессы. Вып. 1, 2. -М.: ИЛ, I960, 1963. 31. Ито К., Маккин Г. Диффузионные процессы и их траектории. - М.: Мир, 1968. 32. Карлин С. Основы теории случайных процессов. -М.: Мир, 1971. 33. Колмогоров А.Н. Математика и механика. Избранные труды. Т. 1. - М.: Наука, 1985. 34. Колмогоров А. Н. Основные понятия теории вероятностей. - 3-е изд. - М.: Фазис, 1998. 35. Колмогоров А.Н., Фомин СВ. Элементы теории функций и функционально- функционального анализа. - 7-е изд. - М.: ФИЗМАТЛИТ, 2004. 36. Корнфелъд И. П., Синай Я. Г., Фомин С В. Эргодическая теория. - М.: На- Наука, 1980. 37. Королюк В. С (ред.) Справочник по теории вероятностей и математической статистике. —Киев: Науковадумка, 1978. 38. Крамер Г., Лидбеттер М. Стационарные случайные процессы. - М.: Мир, 1969. 39. Крылов Н. В. Введение в теорию случайных процессов. Части 1 и 2. - М.: Изд-во МГУ, 1986,1987. 40. Крылов Н. В. Введение в стохастическое исчисление // Стохастическое исчис- исчисление / Под ред. Ю.В. Прохорова, А.Н. Ширяева. -М.: ВИНИТИ, 1989. (Се- (Серия соврем, проблемы матем. Фундаментальные направления. Т. 45. Гл. 1.) 41. Куратовский К. Топология. Т. 1, 2. -М.: Мир, 1966. 42. Ламперти Дэрс. Случайные процессы. -Киев: Вища школа, 1983. 43. Леей П. Стохастические процессы и броуновское движение. -М.: Наука, 1972. 44. Леоненко Н.Н., Иванов А. В. Статистический анализ случайных полей. - Киев: Вища школа, 1986.
Список литературы 387 45. Лидбеттер М., Линдгрен Г., Ротсен X. Экстремумы случайных последова- последовательностей и процессов. -М.: Мир, 1989. 46. Липцер Р. А., Ширяев А.Н. Теория мартингалов. -М.: Наука, 1986. 47. Лиггетт Т. Марковские процессы с локальным взаимодействием. -М.: Мир, 1989. 48. Лифшиц М. А. Гауссовские случайные функции. - Киев: TBiMC, 1995. 49. Малышев В. А., Минлос Р. А. Гиббсовские случайные поля. - М.: Наука, 1985. 50. Петров В. В. Предельные теоремы для сумм независимых случайных величин. -М.: Наука, 1987. 51. Пешкир Г., Ширяев А.Н. НеравенстваХинчинаимартингальноерасширение сферы их действия // УМН. - 1995. - Т. 50, № 5. - С. 3-62. 52. Питербарг В. И. Асимптотические методы в теории гауссовских процессов и полей. - М.: Изд-во МГУ, 1988. 53. Полиа Г., Сегё Г. Задачи и теоремы из анализа. Т. 1, 2. -М.: Наука, 1988. 54. Престон К. Гиббсовские состояния на счетных множествах. -М.: Мир, 1977. 55. Прохоров А. В., Ушаков В. Г., Ушаков П. Г. Задачи по теории вероятностей: Основные понятия. Предельные теоремы. Случайные процессы. -М.: Наука, 1986. 56. Прохоров Ю. В., Розанов Ю. А. Теория вероятностей: Основные понятия. Предельные теоремы. Случайные процессы. - 3-е изд. - М.: Наука, 1987. 57. Вероятность и математическая статистика. Энциклопедия / Под ред. Ю. В. Прохорова. - М.: Большая Российская энциклопедия, 1999. 58. Пугачев В. С. Теория случайных функций и ее применение к задачам автома- автоматического управления. -М.: Физматгиз, I960. 59. Ревюз Д. Цепи Маркова. -М.: РФФИ, 1997. 60. Рид М., Саймон В. Методы современной математической физики. Т. 1-4. - М.: Мир, 1977-1982. 61. Розанов Ю. А. Введение в теорию случайных поцессов. - М.: Наука, 1982. 62. Розанов Ю.А. Марковские случайные поля. -М.: Наука, 1981. 63. Розанов Ю.А. Теория вероятностей, случайные процессы и математическая статистика. -М.: Наука, 1985. 64. Розанов Ю. А. Стационарные случайные процессы. - 2-е изд. - М.: Наука, 1990. 65. Саханенко А. И. О сильном принципе инвариантности для сумм независимых случайных векторов // Труды Ин-та математики им. С. Л. Соболева. — Ново- Новосибирск, 1980. 66. Севастьянов В. А. Эргодическая теорема для марковских процессов и ее при- приложение к телефонным системам с отказами // ТВП. - 1957. - Т. 2, № 1. - С. 106-116. 67. Севастьянов Б. А. Ветвящиеся процессы. -М.: Наука, 1971. 68. Севастьянов Б. А. Курс теории вероятностей и математической статистики. -М.: Наука, 1982. 69. Синай Я. Г. Теория фазовых переходов. Строгие результаты. - М.: Наука, 1980. 70. Синай Я. Г. Введение в эргодическую теорию. - 2-е изд. - М.: Фазис, 1996.
388 А. В. Булинский, А. Н. Ширяев 71. Скороход А. В. Исследования по теории случайных процессов. -Киев: Изд-во Киевского ун-та, 1961. 72. Скороход А. В. Элементы теории вероятностей и случайных процессов. - Ки- Киев: Вища школа, 1980. 73. Скороход А. В. Случайные процессы с независимыми приращениями. - 2-е изд. -М.: Наука, 1986. 74. Спицер Ф. Принципы случайного блуждания. -М.: Мир, 1969. 75. Стоянов И. Контрпримеры в теории вероятностей. -М.: Факториал, 1999. 76. Тихонов В. И., Миронов М. А. Марковские процессы. -М.: Сов. радио, 1987. 77. Тутубалин В.Н. Теория вероятностей и случайных процессов. — М.: Изд-во МГУ, 1992. 78. Феллер В. Введение в теорию вероятностей и ее приложения. Т. 1, 2. - М.: Мир, 1984. 79. Ферник К. Регулярность траекторий гауссовских случайных функций // Слу- Случайные процессы. Выборочные функции и пересечения. - М.: Мир, 1978. - С. 63-132. 80. Хеннан Э. Многомерные временные ряды. -М.: Мир, 1974. 81. Хида Т. Броуновское движение. -М.: Наука, 1987. 82. Чэрсун Кай-лай. Однородные цепи Маркова. -М.: Мир, 1964. 83. Ширяев А.Н. Случайные процессы (лекции для студентов 3 курса). — М.: Изд-во МГУ, 1972. 84. Ширяев А.Н. Статистический последовательный анализ. - 2-е изд. -М.: На- Наука, 1976. 85. Ширяев А.Н. Вероятность. Т. 1, 2. -3-еизд. -М.: МЦНМО, 2004. 86. Ширяев А.Н. Основы стохастической финансовой математики. Т. 1,2. -М.: Фазис, 1998, 2004. 87. Яглом А. М. Корреляционная теория стационарных случайных функций. -Л.: Гидрометеоиздат, 1987. 88. Accardi L., Frigerio A., Lewis J. Т. Quantum stochastic processes//Publ. Res. Inst. Math. Sci., Kyoto. - 1982. - V. 18, № 1. - P. 97-133. 89. Adler R. J. The Geometry of Random Fields. - Chichester: Wiley, 1981. 90. Alexander K. S., Pyke R. A uniform central limit theorem for set-indexed par- partial-sum processes with finite variance // Ann. Probab. - 1986. - V. 14, № 2. - P.582-597. 91. Barbour A. D. Stein's method for diffusion approximations // Probab. Theory Related Fields. - 1990. - V. 84, № 3. - P. 297-322. 92. Barlow M. One-dimensional stochastic differential equations with no strong solu- solution // J. London Math. Soc. B). - 1982. - V. 26, № 2. - P. 335-347. 93. Bartlett M. S. An Introduction to Stochastic Processes, with Special Reference to Methods and Applications. - Cambridge Univ. Press, 1978. 94. Bass R. F. Law of the iterated logarithm for set-indexed partial sum processes with finite variance // Z. Wahr. verw. Geb. - 1985. - Bd. 70, H. 4. - S. 591-608. 95. Bass R. F. Probabilistic Techniques in Analysis. - New York: Springer-Verlag, 1995.
Список литературы 389 96. Bass R. F., Руке R. A strong law oflarge numbers for partial-sum processes in- indexed by sets // Ann. Probab. - 1984. - V. 12, № 1. - P. 268-271. 97. Bass R. F., Руке R. A central limit theorem for ®(A)-valued processes // Sto- Stochastic Process Appl. - 1987. - V. 24, № 1. - P. 109-131. 98. Berkes L, Morrow G. J. Strong invariance principles for mixing random fields // Z. Wahr. verw. Geb. - 1981. - Bd. 57, H. 1. - S. 15-37. 99. Bertoin J. Levy Processes. - Cambridge: Cambridge Univ. Press, 1996. 100. Bhattacharya R.N., Waymire E. С Stochastic Processes with Applications. - New York: Wiley, 1990. 101. Borkar V. S. Probability Theory. An Advanced Course. - New York: Springer- Verlag, 1995. 102. Bouleau N., Lepingle D. Numerical Methods for Stochastic Processes. - New York: Wiley, 1994. 103. Brzezniak Z., Zastawniak T. Basic Stochastic Processes. -London: Springer-Ver- lag, 1999. 104. Bryc W. Large deviations by the asymptotic value method // Diffusion Processes and Related Problems in Analysis. V. 1 / Ed. M. Pinsky. - Boston: Birkhauser, 1990.-P. 447-472. 105. Bulinski A. V., Keane M. S. Invariance principle for associated random fields // J. Math. Sci. - 1996. - V. 81, № 5. - P. 2905-2911. 106. Chow Y. S., Teicher H. Probability Theory: Independence. Interchangeability. Martingales. - 3-rd ed. - New York: Springer-Verlag, 1997. 107. Chung K. L. Lectures from Markov Processes to Brownian Motion. - New York: Springer-Verlag, 1982. 108. Chung K. L. Green, Brown, and Probability. - River Edge: World Scientific, 1995. 109. Csorg'o M., Revesz P. Strong Approximations in Probability and Statistics. - New York: Academic Press, 1981. 110. Cox D. R., Miller H. D. The Theory of Stochastic Processes. - London: Chapman & Hall, 1980. 111. Daley D. J., Vere- Jones D. An Introduction to the Theory of Point Processes. - New York: Springer-Verlag, 1988. 112. Daniell P. J. Integrals in an infinite number of dimensions // Ann. of Math. B). 1918-1919. - V.20. - P. 281-288. 113. Dawson D. A. Measure-valued Markov processes. Ecole d'Ete de Probabilites de Saint-Flour, XXI, Lect. Notes Math. V. 1541. Springer, 1991. 114. Dembo A., Zeitouni 0. Large Deviations Techniques and Applications. -2-nded. - New York: Springer-Verlag, 1998. 115. Den Hollander F. Large Deviations. - Providence, RI: Amer. Math. Soc, 2000. 116. Deuschel J.-D., Stroock D. W. Large Deviations. -Boston: Academic Press, 1989. 117. Dobrushin R. L., Major P. Non-central limit theorems for non-linear functionals of Gaussian fields // Z. Wahr. verw. Geb. - 1979. - Bd. 50, № 1. - S. 27-52. 118. Doukhan P. Mixing. Properties and Examples. - LNS, 85. - Springer-Verlag, 1994. 119. Dozzi M. Stochastic Processes with a Multidimensional Parameter. - Harlow: Longman, 1989.
390 А. В. Булинский, А. Н. Ширяев 120. Dudley R. М. A course on empirical processes. - Lect. Notes Math. V. 1097. - Berlin: Springer, 1984. - P. 1-142. 121. Dudley R. M. Real Analysis and Probability. - Pacific Grove, С A: Wadsworth & Brooks / Cole Advanced Books & Software, 1989. 122. Dudley R. M., Philipp W. Invariance principles for sums of Banach-space valued random elements and empirical processes // Z. Wahr. verw. Geb. -1983. - Bd. 62, H. 4.-S. 509-552. 123. Durrett R. Probability: Theory and Examples. - 2-nd ed. - Belmont, С A: Duxbury Press, 1996. 124. Durrett R. Stochastic Calculus: A Practical Introduction. -Boca Raton, FL: CRC Press, 1996. 125. Durrett R. Essentials of Stochastic Processes. - New York: Springer, 1999. 126. Dynkin E. B. Superprocesses and partial differential equations // Ann. Probab. - 1993. - V. 21, № 3. - P. 1185-1262. 127. Ellis R. S. Entropy, Large Deviations and Statistical Mechanics. - New York: Springer- Verlag, 1985. 128. Emery M. Stochastic Calculus in Manifolds. - Berlin: Springer-Verlag, 1989. 129. Fdllmer H., Protter Ph., Shiryaev A. N. Quadratic covariation and an extention oflto's formula//Bernoulli. -1995. -V. 1,№ 1/2. -P. 149-169. 130. Freedman D. Brownian Motion and Diffusion. - 2-nd ed. - New York: Springer- Verlag, 1983. 131. Fristedt В., Gray L. A Modern Approach to Probability Theory. - Boston, MA: Birkhauser, 1997. 132. Gardiner С W. Handbook of Stochastic Methods for Physics, Chemistry and the Natural Sciences. - Berlin: Springer-Verlag, 1983. 133. Gnedenko B. V., Korolev V. Yu. Random Summation. Limit Theorems and Ap- Applications. - Boca Raton, FL: CRC Press, 1996. 134. Grenander U. Abstract Inference. - New York: Wiley, 1981. 135. Grimmett G. R., Stirzaker D. R. Probability and Stochastic Processes. - 2-nd ed. - Oxford: Clarendon Press, 1992. 136. Grimmett G. R., Stirzaker D. R. Probability and Random Processes: Problems and Solutions. - Oxford: Clarendon Press, 1992. 137. Guyon X. Random Fields on a Network. Modeling, Statistics, and Applications. -New York: Springer-Verlag, 1995. 138. Hall P., Heyde С. С Martingale Limit Theorem and its Applications. -New York: Academic Press, 1980. 139. Harris Т. Е. The Theory of Branching Processes. - New York: Dover, 1989. 140. Hoffmann-J0rgensen J. Stochastic Processes on Polish Spaces. - Aarhus: Aarhus Univ. Math. Inst., 1991. 141. Hughes B.D. Random Walks and Random Enviroments. Vol. 1, 2. - Oxford: Clarendon Press, 1995,1996. 142. Iranpour R., Chacon P. Basic Stochastic Processes. - New York: Macmillan, 1988. 143. Jacod J., Protter Ph. Probability Essentials.-Berlin: Springer-Verlag, 2000. 144. Janicki A., Weron A. Simulation and Chaotic Behavior of a-stable Stochastic Processes. - New York: M. Dekker, 1994.
Список литературы 391 145. Kallenberg О. Random Measures. - 3-rd ed. - London: Academic Press, 1983. 146. Kallenberg 0. Foundations of Modern Probability. - New York: Springer, 1997. 147. Karatzas L, Shreve S. E. Methods of Mathematical Finance. -New York: Colum- Columbia Univ. Press, 1995. 148. Karatzas L, Shreve S. E. Brownian Motion and Stochastic Calculus. - 2-nd ed. - New York: Springer-Verlag, 1991. 149. Karlin S., Taylor H. M. A Second Course in Stochastic Processes. - New York- London: Academic Press, 1981. 150. Kindermann R., Snell J.L. Markov Random Fields and their Applications. - Providence, RI: Amer. Math. Soc, 1980. 151. Kingman J. F. C. Poisson Processes. - Oxford: Clarendon Press, 1993. 152. Komlos J., Major P., Tushnddy G. An approximation of partial sums of indepen- independent RV's and the sample DF. I, II // Z. Wahr. verw. Geb. - 1975. - Bd. 32. - S. 111-131; - 1976. - Bd. 34. - S. 33-58. 153. Kono N. Recent development on random fields and their sample paths. Part I: sample path continuity of random fields // Soochow J. Math. - 1990. - V. 16, № 2.-P. 123-161. 154. Krylov N. V. Introduction to the Theory of Diffusion Processes. - Providence, RI: Amer. Math. Soc, 1995. 155. Lamberton D., Lapeyre B. Introduction to Stochastic Calculus Applied to Fi- Finance. - London: Chapman & Hall, 1996. 156. Last G., Brandt A. Marked Point Processes on the Real Line: The Dynamic Ap- Approach. - New York: Springer-Verlag, 1995. 157. Ledoux M., Talagrand M. Probability in Banach Spaces. - Berlin: Springer-Ver- Springer-Verlag, 1991. 158. Levy P. Theorie de l'addition des variables aleatoires. - Paris: Gauthier-Villars, 1937. 159. Lindvall T. Lectures on coupling method. - New York: Wiley, 1992. 160. Liptser R. S., Shiryaev A. N. Statistics of Random Processes. V. 1: General The- Theory. V. 2: Applications. -Berlin: Springer-Verlag, 2001. 161. Lomnicki Z., Ulam S. Sur la theorie de la mesure dans les espaces combinatoires et son application au calcul des probabilites. I: Variables independentes // Fund. Math. - 1934. - V. 23. - P. 237-278. 162. McLeish D. L. Dependent central limit theorems and invariance principles // Ann. Probab. - 1974. - V. 2. - P. 620-628. 163. Major P. Multiple Wiener-Ito integrals // Lect. Notes Math. - 1981. - V. 849. - P. 1-127. 164. Mandelbrot В. В., Van Ness J. W. Fractional Brownian motions, fractional noises and applications. - SIAM Rev. - 1968. - V. 10. - P. 422-437. 165. Meyer P.-A. Quantum Probability for Probabilists // Lecture Notes in Math. V. 1538. - Berlin: Springer-Verlag, 1993. 166. Molchanov S. A. Ideas in the Theory of Randon Media // Acta Appl. Math. - 1991. - V. 22. № 2-3. - P. 139-282. 167. Nelson E. Construction of quantum fields from Markoff fields // J. Funct. Anal. - 1973.-V. 12.-P. 97-112.
392 А. В. Булинский, А. Н. Ширяев 168. Newman С. М., Wright A.L. A invariance principle for certain dependent se- sequences // Ann. Probab. - 1981. - V. 9, № 4. - P. 671-675. 169. 0ksendal B. Stochastic Differential Equations. - 4-th ed. - Berlin: Springer-Ver- lag, 1995. 170. Ortega J., Wschebor M. On the increments of the Wiener process // Z. Wahr. verw. Geb. - 1984. - Bd. 65, H. 3. - S. 329-339. 171. Parthasarathy K. R. An Introduction to Quantum Stochastic Calculus. - Basel: Birkhauser, 1992. 172. Pollard D. Convergence of Stochastic Processes. - New York: Springer-Verlag, 1984. 173. Protter P. Stochastic Integration and Differential Equations. A new Approach. - Berlin: Springer-Verlag, 1990. 174. Руке R. The Haar-function constraction of Brownian motion indexed by sets // Z. Wahr. verw. Geb. - 1983. - Bd. 64, H. 4. - S. 523-539. 175. Rachev S. T. Probability Metrics and the Stability of Stochastic Models. - Chich- ester: Wiley, 1991. 176. Ramakrishnan A. Stochastic Processes in Physics and Astronomy. -Madras: Inst. of Math. Sci., 1978. 177. Rao M. M. Stochastic Processes. General Theory. - Dortrecht: Kluwer Academic Publishers, 1995. 178. Reiss R.-D. A Course on Point Processes. - New York: Springer-Verlag, 1993. 179. Resnick S. L. Adventures in Stochastic Processes. - Boston, MA: Birkhauser, 1992. 180. Renter G. E. H. Denumerable Markov processes and the associated contraction semigroups on I // Acta Math. - 1957. - V. 97, № 1-2. - P. 1-46. 181. Revesz P. On the increments of the Wiener and related processes // Ann. Probab. - 1982. - V. 10, № 3. - P. 613-622. 182. Revus D., Yor M. Continuous Martingales and Brownian Motion. - 2-nd ed. - Berlin: Springer-Verlag, 1994. 183. Ripley B. D. Statistical inference for spatial processes. - Cambridge: Cambridge Univ. Press, 1988. 184. Rogers L. С G., Williams D. Diffusion, Markov Processes, and Martingales. - V. 1. 2-nd ed. - Chichester: Wiley, 1994. - V. 2. - New York: Wiley, 1987. 185. Rosenblatt M. Random Processes. - New York: Springer-Verlag, 1974. 186. Rosenblatt M. Stationary Sequences and Random Fields. - Boston, MA: Birkhau- Birkhauser, 1985. 187. Samorodnitsky G., Taqqu M. S. Stable Non-Gaussian Random Processes. - New York: Chapman & Hall, 1994. 188. Strassen V. An invariance principle for the law of the iterated logarithm // Z. Wahr. verw. Geb. - 1964. - Bd. 3. - S. 211-226. 189. Stroock D. W. Probability Theory. An analytic view. - Cambridge: Cambridge Univ. Press, 1993. 190. Thorisson H. Coupling, Stationarity, and Regeneration. - Berlin: Springer, 2000. 191. Tong H. Nonlinear Time Series. A Dynamical System Approach. - Oxford: Cla- Clarendon Press, 1990.
Список литературы 393 192. Todorovic P. An Introduction to Stochastic Processes and Their Applications. - New York: Springer-Verlag, 1992. 193. Van der Vaart A. W., Wdiner J. A. Weak Convergence and Empirical Processes. - New York: Springer-Verlag, 1996. 194. Varadhan S. R. S. Large Deviations and Applications. - Philadelphia, PA: Soc. for Industrial and Appl. Math., 1984. 195. Walsh J. B. Martingales with a multidimensional parameter and stochastic inte- integrals in the plane - Lect. Notes Math. V. 1215. - 1986. 196. Walsh J. B. An introduction to stochastic partial differential equations. - Lect. Notes Math. V. 1180. - Berlin: Springer-Verlag, 1985. - P. 265-439. 197. Williams D. Probability and Martingales. - Cambridge: Cambridge Univ. Press, 1991. 198. Yu H. A strong invariance principle for associated sequences // Ann. Probab. - 1996. - V. 24, № 4. - P. 2079-2097.
Алфавитный указатель (Б,5)-рынок 143 Cadlag функция 69 ©-система 15 L-гармоническое расширение / 362 L2-nponecc 53, 267 - комплекснозначный 234 Q-решение 220 [/-статистика 137 Х-гармоническая функция 362 (ж,/,ЛГ)-хедж 146 ^-импульс 273 Л-система 14 тг-система 14 ^-измеримость 13 & | ^-измеримое отображение 13 сг-алгебра 20 - борелевская 13 - 8 расщепляет ^и^2 224 - инвариантных множеств 274 - опциональная 300 - перестановочных событий 139 - предсказуемая 300 - расширенная 16 - событий, наблюдаемых до случайного момента т 81 - цилиндрическая 24, 33 сг-конечная мера 22 s-сеть 72 е-энтропия множества 72 Абсолютно непрерывная мера ц относительно меры v 28 Автомодельный случайный процесс 302 Адаптированный процесс 299 Актив безрисковый 143 - рисковый 143 Алгебра цилиндрическая 31 Аналог закона Эрдёша-Реньи 71 Арифметическое среднее A(v) 257 Банковский счет 143 Барлоу М. 311 Безгранично делимые законы 336 Безрисковый актив 143 Белый шум 244, 267, 268 Блуждание случайное 21 — возвратное 65 , критерий Чжуна-Фукса 65 — в Rm простейшее симметричное 67 —, симметризация 66 — транзиентное (невозвратное) 65 —, эффективная размерность 66 Блуждания случайные в случайной среде 68 Большие уклонения 364 Борелевская сг-алгебра 13 Борелевское пространство 26 Броуновский мост 163, 310 Броуновское движение 49 — - Леви 106 — семейство 347 Ван Несс Й. 302 Варадарайн 326 Вариант закона больших чисел 240 Вариационный ряд 162 Ведущая мера 23 Векторный процесс 267 Величина "перескока" за уровень t 214 Венгерский метод 175 Вероятностная мера 13 Вероятность потери требования в стационарном режиме 211 Версия процесса 35 Верхняя функция 381 Ветвящийся процесс Гальтона— Ватсона 125 Винеровская мера 63 Винеровский процесс 49 Возвратное случайное блуждание 65
394 А. В. Булинский, А.Н. Ширяев Волатильность 306 Воспроизводящее ядро гильбертова пространства 55 Выборочная функция 19 Гарсиа А. 275 Гауссовская случайная функция действительная 51 комплексная 50 Гауссовский (нормальный) случайный вектор 50 Геометрическое броуновское движение 306 — среднее G(v) 257 Движение броуновское 49 — в точке 0, локальное время 309 — геометрическое 306 — - Леви 106 — многомерное 50 — мультифрактальное 303 — фрактальное, представление 302 — фрактальное (дробное) 105 — экономическое 306 Движения броуновские, семейство 344, 347 Дворецкий А. 78 Демимартингал 143 Детерминированный процесс 249 Диаграмма переходов 208 Дискретизация 19 Дискретный аналог формулы Танака 114 Дисперсионная (ковариационная) матрица 158 Дифференцирование в среднем квадратическом 271 Диффузия 306 Доминирующая мера 176 Дробовый шум 216 Дробь Ляпунова 178 Дуб Дж. 304 Дэвис Б. 97, 136 Единственность сильного решения 291 Естественная фильтрация 78 Задача Дирихле 354 — восстановления полугруппы по ее генератору 219 — о разорении 119 Закон арксинуса 97 Закон больших чисел, вариант 240 равномерный усиленный 42 Закон больших чисел усиленный (Колмогоров) 140 - нуля или единицы, Хьюитт— Сэвидж 139 , Колмогоров 87 - повторного логарифма (Хинчин) 91 локальный 93 функциональный в форме Штрассена 375 Законы безгранично делимые 336 Измеримое пространство 13 Измеримый случайный процесс 37 Изоморфизм измеримых пространств 26 Изотропное случайное поле 268 Импульсная переходная функция 273 Инвариантная мера 201 Инвариантное множество 274 Инвестиционная стоимостью платежного обязательства 146 - стратегия 144 Индикатор множества 17 Интеграл Ито 282 - по ортогональной случайной мере 229 - Стратоновича 313 - Хеллингера 177 Интегральный критерий Колмогорова- Петровского-Эрдёша-Феллера 381 Интегрирование в среднем квадратическом случайных процессов 272 Интенсивности вероятностей переходов 218 Инфинитезимальная матрица 204 Ито К. 304 Какутани С. 78 Капитал портфеля 144 Каплинг-метод 178 Като Т. 220 Квадратическая вариация процесса 115 - ковариация 308 - характеристика мартингала 115 Квадратично интегрируемый мартингал 115 Кларк Дж. 304 Класс Р-нулевых множеств 16 - Дирихле 305 - множеств, конечно-аппроксимируемый относительно меры Q 41 -, определяющий (слабую) сходимость 170
Алфавитный указатель 395 Класс, определяющий меру 169 Ковариационная функция 53, 234 Колмогоров А.Н. 271, 302 Комлош И. 175 Компенсатор 115 Комплекснозначный L -процесс 234 Конечно-аппроксимируемый относительно меры Q класс множеств 41 Конечномерные распределения 25 - марковской цепи 189 Консервативная цепь 204 Контракт с опционом 145 Координатное отображение 24 Координатный процесс 34 Коэффициент нагрузки 211 - передачи 273 Крамер Г. 254, 270 Кратные стохастические интегралы 302 Критерий Пойа 67 - согласия Колмогорова 161 - Чжуна-Фукса возвратности случайного блуждания 65 Кузнецов СЕ. 193 Ледерман В. 220 Лемма Боре ля-Канте лли 61 - Варадана 366 - Дуба о числе пересечений 122 - Рисса 369 - Улама 169 Линейный прогноз 248 Логарифм отношения правдоподобия 67 Локальная абсолютная непрерывность меры Q относительно меры Р 111 Локально компактное хаусдорфово пространство 173 - конечная мера 48 Локальное время броуновского движения в точке 0 309 Локальный закон повторного логарифма 93 - мартингал (субмартингал, супермартингал) 142 - снос 306 Мажорирующая вероятностная мера 74 Майор П. 175 Максимальное неравенство 158 Максимальные L^-неравенства (Дуб) 121 - и минимальные вероятностные неравенства (Дуб) 120 Мандельбротт Б. 302 Маркированный точечный случайный процесс 216 Марковская переходная функция 192 - полугруппа 175 - случайная функция 224 Марковский момент 79 - процесс 180, 185 Марковское свойство винеровского процесса 78 - семейство 223 Мартингал 109, 185 - Леви 111 - с многопараметрическим индексом 143 Мартингал-разность 110 Мартингальная вероятностная мера 144 Мартингальное преобразование 111 Матрица дисперсионная (ковариацион- (ковариационная) 158 - инфинитезимальная 204 -, неотрицательно определенная 50 - полустохастическая 220 Матричное дифференциальное уравнение Риккати 314 Мера 13 - ведущая 23 - вероятностная 13 - винеровская, Винера 63 - доминирующая 176 - инвариантная 201 - локально конечная 48 - мажорирующая вероятностная 74 - мартингальная вероятностная 144 - //, абсолютно непрерывная относительно меры v 28 - ортогональная случайная 226 - пребывания процесса 65 - пуассоновская случайная 23, 43 - Радона 215 - сг-конечная 22 - сингулярная 176 - случайная 179 - спектральная 239, 240 - структурная 226, 240, 269 - считающая 40 - точечная без кратных точек 215 - случайная 215 - эмпирическая 22 Метод Крамера-Уолда 170 Метрика Ки Фан 177 - Леви-Прохорова 168 Миксингал 143
396 А. В. Булинский, А.Н. Ширяев Минимальная s-сеть 72 Многомерная центральная предельная теорема (Линдеберг) 157 Многомерное броуновское движение 50 Многомерный вариант теоремы Гливенко-Кантелли 41 Множество возвратности 65 — в среднем 65 — достижимости 65 Модель Блэка-Мертона-Шоулса 315 — страхования Крамера-Лундберга 22 Модификация процесса 35 Модуль непрерывности функции 154 Момент остановки 79 — первого выхода из множества 97 — достижения множества 97 — последнего пребывания в множестве 97 Мост броуновский 163, 310 Мультифрактальное броуновское движение 303 Начальное распределение 189 Независимые случайные величины 20 Независящая от сг-алгебры случайная функция 78 Необходимое и достаточное условие непрерывности действительной гауссовской случайной функции 75 ограниченности действительной гауссовской случайной функции 75 Неотрицательная определенность матрицы 50 Неотрицательно определенная действительная функция 52 — комплекснозначная функция 52 — матричная функция множества 269 Непосредственно заданный процесс 34 Непрерывность процесса в среднем квадратическом 242 Непрерывный цилиндр 33 Неравенства Хинчина и Марцинкевича- Зигмунда 135 Неравенство Коши-Буняковского- Шварца 338 — Оттавиани 69 Неразличимые процессы 36 Нерегулярная точка границы 359 Несобственное решение 220 Нижняя функция 381 Нормальные уклонения 364 Нормальный случайный вектор 50 Носитель меры 66 Обновляющий процесс 254, 270 Обобщенная вторая производная 272 - обратная функция 39 Образ меры 18 Обращенная версия теоремы Леви 139 Обращенный мартингал (субмартингал, супермартингал) 137 Обычные условия 142 Однородная диффузия 309 - марковская цепь 196 Однородное изотропное векторное поле 269 Однородный марковский процесс 195 Оператор Гильберта-Шмидта 260 - Фредгольма 259 - сдвига 251, 347 - на случайную величину 351 Опцион 145 - американского типа 145 - европейского типа 145 - колл (опцион на продажу) 146 - пут (опцион на покупку) 146 Опциональная сг-алгебра 300 - случайная мера 226 Опциональный момент 128 - процесс 300 Ортогональные приращения 257 Основная теорема теории восстановле- восстановления 42 - финансовой математики 144 Основное свойство стохастического интеграла 233 Остановленный процесс 302 Отраженный процесс 85 Оценка Бартлета 248 Ошибка линейного прогноза 249 Парадокс времени ожидания 214 Параметр Харста 105 Перестановочная последовательность случайных величин 137 Переходная функция 193 Переходные вероятности 188 Периодограмма 247 Планк М. 222 Платежное обязательство 145 Плотное семейство мер 154 Плотность меры /и, по мере v 28 Поглощающее состояние 218 Подкласс арбитражных портфелей 144 Поле дробового шума 265
Алфавитный указатель 397 Поле однородное изотропное векторное 269 - случайное 19 - изотропное 268 - Винера-Ченцова 106 - Леви-Шемберга 106 Положительно определенная матричная функция 267 Полосовой фильтр 274 Полукольцо подмножеств 225 Полунепрерывность снизу 365 Полустохастическая матрица 220 Полуэллиптический оператор 361 Польское пространство 26 Пополнение исходного вероятностного пространства 16 Портфель 144 - хеджирующий 145 Поток сг-алгебр (фильтрация) 78 Предсказуемая а-алгебра 300 - последовательность 111 Предсказуемый процесс 300 Представление фрактального броуновского движения 302 Пример Т. Арака 179 Принцип инвариантности Донскера- Прохорова 157 - отражения 85 Прогрессивная измеримость 299 Продолжение меры 16 Продолжения функций Pij(s,t) 189 Производная Радона-Никодима 176 Прореженный процесс 217 Простая функция 17, 229, 301 Простейшее симметричное случайное блуждание в Rm 67 Пространства Скорохода D([0, l]q) и D([0,oo)q) 174 Пространство С(Т, S) 33 Пространство борелевское 26 - вероятностное, расширение 37 —, стандартное расширение 37 - гильбертово, воспроизводящее ядро 55 - измеримое 13 - исходное вероятностное, пополнение 16 - Камерона-Мартина 369 - локально компактное хаусдорфово 173 - польское 26 - Скорохода ?>[0,1] 40 - Скорохода ?>[0, оо) 69 - состояний 187 Пространство фазовое 187 - фильтрованное вероятностное 110 Процесс автомодельный случайный 302 - авторегрессии — скользящего среднего 274 - адаптированный 299 - Бесселя размерности m 136 - векторный 267 -, версия 35 - восстановления 21 - Гальтона—Ватсона ветвящийся 125 - винеровский 49 —, марковское свойство 78 —, явная конструкция 58 - детерминированный 249 - измеримый случайный 37 -Ито 286 -, квадратическая вариация 115 - Кокса 216 - координатный 34 - Леви 70 - маркированный точечный случайный 216 - марковский 180, 185 -, мера пребывания 65 -, модификация 35 - заданный непосредственно 34 - Орнштейна-Уленбека 107, 290 -, непрерывность в среднем квадратическом 242 - обновляющий 254, 270 - однородный марковский 195 - опциональный 300 - остановленный 302 - отраженный 85 - предсказуемый 300 - прореженный 217 - пуассоновский 48, 191 - регулярный 249 - рождения и гибели 221 - независимыми приращениями 46, 186 - пуассоновский 48, 191 - , явная конструкция 48 - сепарабельный случайный 44 траекториями ограниченной вариации 143 - стационарными приращениями 105 - сингулярный 249 -, скачок в фиксированной точке 70 - скользящего среднего 244 - случайный 19
398 А. В. Булинский, А.Н. Ширяев Процесс случайный, интегрирование в среднем квадратическом 272 -, спектральное представление стационарных (в широком смысле) процессов 240 - стандартный пуассоновский с постоянной интенсивностью 48 - стационарный в узком смысле 200, 274 - стационарный в широком смысле 238 - стохастически непрерывный случайный 45 - точечный случайный 179, 215 - частных сумм 22 - чисто дискретный 68 - недетерминированный 249 - эргодический 274 Процессы неразличимые 36 - с векторными значениями 267 - стохастически эквивалентные 35 в широком смысле 36 - эквивалентные 35 Прямоугольник 25 Пуассоновская случайная мера 23, 43 Пуассоновский процесс 48, 191 Равенство Парсеваля 59, 338 Равномерная метрика 33 Равномерно интегрируемое семейство действительных случайных величин 98 - эллиптический оператор 363 Равномерный усиленный закон больших чисел 42 Разбиение п 188 Разложение Вольда 252, 256 -Дуба 112 - Дуба-Мейера 305 - Крикеберга 135 - Рисса 135 Распределение случайного элемента 18 Расстояние Какутани-Хеллингера 177 - по вариации 176 Расширение вероятностного пространства 37 Расширенная сг-алгебра 16 Реализация 19 Регулярная точка границы 359 - условная вероятность 193 Регулярный процесс 249 Результат Винера и Пэли 95 - Дадли 74 - Звонкина 310 Результат Колмогорова 87 - П. Леви 90 - Харли 95 - Хинчина 90 Римановская последовательность 308 Рисковый актив 143 Ройтер Г. 220 Самофинансируемый класс портфелей 144 Самуэльсон П. 306 Свойство автомодельности 64 - локальности условного математического ожидания 116 - марковости (сильных) решений стохастических дифференциальных уравнений 297 - регулярности меры 15 Сглаженные оценки спектральной плотности 247 Семейство броуновских движений 344, 347 Семимартингал 143, 303 Сепарабельный случайный процесс 44 Сильное решение 289, 291 Сильный принцип инвариантности 175 Симметризация случайного блуждания 66 Симметризованные векторы 334 Симметричное устойчивое распределе- распределение 66 Сингулярная мера 176 Сингулярный процесс 249 Система массового обслуживания 209 с отказами 209 Скачок процесса в фиксированной точке 70 Скорость 365 Слабая единственность решения 311 - сходимость мер 147 Слабое решение 310 Следствие Башелье 89 -Дуба 131 Случайная величина 13 - мера 179 - последовательность 19 - функция 18 Случайное блуждание 21 Случайное поле 19 - Винера-Ченцова 106 - Леви-Шемберга 106
Алфавитный указатель 399 Случайные блуждания в случайной среде 68 Случайный вектор 13 - процесс 19 - элемент 13 Смешанные моменты 238 Собственное решение 220 События из "прошлого" и "будущего" 181 Совершенное хеджирование 316 Спектральная мера 239, 240 Спектральная плотность 243 Спектральное представление стационарных (в широком смысле) процессов 240 Спектральные окна 247 Спирали Винера 302 Справедливая цена опциона 146 Среднее арифметическое А(у) 257 - геометрическое G(v) 257 Стандартная однородная цепь Маркова 201 - полугруппа, порожденная однородной цепью Маркова 201 Стандартное расширение вероятностного пространства 37 Стандартный пуассоновский процесс с постоянной интенсивностью 48 Статистическая фрактальная размерность 302 Статистический последовательный анализ В а льда 67 Статистическое оценивание спектральной плотности 246 Стационарное распределение 199 Стационарный процесс в узком смысле 200 Стационарным процесс в широком смысле 238 Стохастическая задача Дирихле 362 - полугруппа 196 - последовательность 116 - экспонента 143 Стохастически возмущенное уравнение роста популяции 306 - непрерывный случайный процесс 45 - эквивалентные процессы 35 - в широком смысле процессы 36 Стохастические интервалы 300 Стохастический базис 110 - дифференциал 286 Стохастическое дифференциальное уравнение с начальным условием 291 Строго марковское свойство 83 Структурная мера 226, 240, 269 Субординатор 70 Супергармоническая функция 135 Схема построения интеграла по процессу М еМ% 305 Сходимость по распределению 150 Счет банковский 143 Считающая мера 40 Талагран М. 75 Телеграфная волна (сигнал) 213, 264 Телескопическое свойство условного математического ожидания 109 Теорема А. Д. Александрова 148 - Ароншайна 55 - Арцела-Асколи 155 - Банаха-Алаоглу 324 - Биркгофа-Хинчина 274 - Блэкуэлла 42 - и Дубинса 140 - Боровкова 178 - Бохнера-Хинчина 242 - Буркхольдера 136 - Буркхольдера-Дэвиса-Ганди 136 - Гапошкина 266 - Герглотца 239 - Гильберта-Шмидта 260 - Гирсанова 312, 372 - Даниеля 27 - Даниеля-Колмогорова 27 - Де ла Балле Пуссена 98 - Дуба 64, 112, 121, 123, 135, 171 - Ионеску Тулча 322 -Ито 309 - -, Кларка 303 - Карунена 235 - Карунена-Лоэва 261 - Колмогорова 26, 72, 161, 256 - Колмогорова-Сегё 257 - Котельникова-Шеннона 270 - Крамера 243, 364 - Ледермана-Ройтера 222 - Линдеберга 158 - Ломницкого-Улама 21 - Мандельбротта, Ван Несса 303 - Мерсера 261 - о возвратности 66 - дихотомии 65 - монотонных классах 15
400 А. В. Булинский, А.Н. Ширяев Теорема о полноте E, 5)-рынка 145 - свободном выборе или об остановке (Дуб) 116 Теорема П. Леви 68, 128, 311 - Парзена 57 - Проттера, Фельмера, Ширяева 309 - Прохорова 154, 156, 173 - Пуассона 176 - Пэли-Винера-Зигмунда 76 - Радона-Никодима 108 - Рисса-Маркова 323 - Скорохода 104, 165 - Стоуна 271 - Феллера 332 - Хана-Банаха 369 - Хартмана-Винтнера 381 - Хинчина 97 -Шеффе 177 - Ширяева 304 - Штрассена 175, 177, 377, 381 Теория массового обслуживания 209 - фильтрации 313 Тождества Вальда 99 Топология равномерной сходимости на компактах 173 Точечная мера без кратных точек 215 - случайная мера 215 Точечный случайный процесс 179, 215 Траектория 19 Транзиентное (невозвратное) случайное блуждание 65 Тушнади Г. 175 Убывающее семейство сг-алгебр 137 Уравнение Колмогорова-Чепмена 192, 195 - Ланжевена 288 - Маркова 189 Усиленный закон больших чисел (Колмогоров) 140 Условие Линдеберга 156 - Липшица 291 - Новикова 312 - асимптотической малости 328 - бесконечной малости 178 - подчиненности 252 - равномерной малости 335 Условия симметрии и согласованности мер 27, 29 - согласованности проекций меры 35 Условное математическое ожидание 108 Устойчивое состояние 218 Фазовое пространство 187 Феллер В. 220 Феллеровское однородное марковское семейство 348 Ферник К. 75 Физически осуществимый фильтр 244 Фильтр 244, 273 Фильтр Кальмана—Бьюси 314 Фильтрация 78 Фильтрованное вероятностное пространство 110 Фильтрованный стохастический эксперимент 110 Фоккер 222 Формула Блэка-Шоулса 316 - Дынкина 361 - замены переменных в интеграле Лебега 28 -Ито 287 - Леви 70 - Леви-Хинчина 71 - обращения 28 - Танака 309 —, дискретный аналог 114 - Фейнмана-Каца 363 - Хинчина 69 Формулы Эрланга 209 Фрактальное (дробное) броуновское движение 105 Фундаментальное уравнение 316 Фундаментальный результат математической теории страхования 131 Функции класса Харли Жч 257 Функции pij(s,t), продолжение 189 - Хаара 58 - Шаудера 59 Функциональный закон повторного логарифма в форме Штрассена 375 Функция Cadlag 69 - Х-гармоническая 362 - верхняя 381 - восстановления 42 - выборочная 19 - гауссовская действительная случайная 51 - комплексная случайная 50 - действительная гауссовская случайная, необходимое и достаточное условие непрерывности 75 , ограниченности 75
Алфавитный указатель 401 Функция импульсная переходная 273 - ковариационная 53, 234 - Линдеберга 328 - марковская переходная 192 - случайная 224 - матричная положительно определенная 267 - меры характеристическая 27 - множества матричная, неотрицательно определенная 269 -, модуль непрерывности 154 - неотрицательно определенная действительная 52 комплекснозначная 52 - нижняя 381 - обобщенная обратная 39 - переходная 193 - простая 17, 229, 301 - распределения эмпирическая 40 - случайного вектора характеристичес- характеристическая 28 - случайная, независящая от а-алгебры 78 - супергармоническая 135 - уклонений 365 Функционал меры характеристичес- характеристический 171 Характеристическая функция меры 27 - случайного вектора 28 Характеристический функционал меры 171 Хеджирование 145 Хеджирующий портфель 145 Цилиндр 31 Цилиндрическая а-алгебра 24, 33 - алгебра 31 Частотная характеристика 273 Чисто дискретный процесс 68 - недетерминированный процесс 249 Шаг распределения 42 Шум белый 244, 267, 268 - дробовый 216 —, поле 265 Эквивалентные процессы 35 Экономическое броуновское движение 306 Экспоненциально плотное семейство мер 366 Элементарный цилиндр 23 Эмпирическая мера 22 - функция распределения 40 Эргодическая теорема 196 Эргодический процесс 274 Эргодичность процесса X в L2(H) 241 Эрдеш П. 78 Этемади Н. 140 Эффективная размерность случайного блуждания 66 Явная конструкция винеровского процесса 58 - пуассоновского процесса 48 Ядро интегрального оператора 259 Ядро Фейера 247 Цепь Маркова 188
Учебное издание БУЛИНСКИЙ Александр Вадимович ШИРЯЕВ Альберт Николаевич ТЕОРИЯ СЛУЧАЙНЫХ ПРОЦЕССОВ Редактор И. Л. Легостаева Оригинал-макет: А.Д. Изаак Оформление переплета: А.А. Логунов ЛР №071930 от 06.07.99. Подписано в печать 09.04.03. Формат 70x100/16. Бумага офсетная. Печать офсетная. Усл. печ. л. 32,41. Уч.-изд. л. 35,6. Заказ № Издательская фирма «Физико-математическая литература» МАИК «Наука/Интерпериодика» 117997, Москва, ул. Профсоюзная, 90 E-mail: fizmat@maik.ru, fmlsale@maik.ru; http://www.fml.ru Отпечатано с готовых диапозитивов в ПФ «Полиграфист» 160001, г. Вологда, ул. Челюскинцев, 3 Тел.: (8172) 72-55-31, 72-61-75, факс: (8172) 72-60-72 E-mail: form.pfp@votel.ru http://www.vologda/~pfpv ISBN 5-9221-0335-0 9 78592203350